diff --git a/Complementaire/DM/2105_DM1/01_2105_DM1.tex b/Complementaire/DM/2105_DM1/01_2105_DM1.tex index e69de29..7ab1420 100644 --- a/Complementaire/DM/2105_DM1/01_2105_DM1.tex +++ b/Complementaire/DM/2105_DM1/01_2105_DM1.tex @@ -0,0 +1,367 @@ +\documentclass[a4paper,10pt]{article} +\usepackage{myXsim} + +% Title Page +\title{DM1 \hfill BALLARD Antoine} +\tribe{Maths complémentaire} +\date{\hfillÀ render pour le jeudi 27 mai} + +\xsimsetup{ + solution/print = false +} + +\begin{document} +\maketitle + +Les valeurs des exercices sont générés automatiquement. Si une valeur a un nombre adhérant de chiffres après la virgule, vous pouvez l'arrondir à l'entier le plus proche. + +\begin{exercise}[subtitle={Optimisation de matière}] + \begin{minipage}{0.6\textwidth} + On se propose de fabriquer avec le moins de tôle possible une citerne fermée en forme de parallélépipède rectangle dont le volume intérieur doit être de $8m^3$. La longueur est aussi fixée à $4m$ par le cahier des charges. + + On peut donc faire varier uniquement la largeur (notée $x$) et la hauteur (notée $h$) de la cuve. + \end{minipage} + \hfill + \begin{minipage}{0.3\textwidth} + \begin{tikzpicture} + \pgfmathsetmacro{\cubex}{3} + \pgfmathsetmacro{\cubey}{1} + \pgfmathsetmacro{\cubez}{2} + \draw[black,fill=gray] (0,0,0) -- ++(-\cubex,0,0) -- ++(0,-\cubey,0) node [midway, left] {$h$} -- ++(\cubex,0,0) node [midway, below] {$x$} -- cycle; + \draw[black,fill=gray] (0,0,0) -- ++(0,0,-\cubez) -- ++(0,-\cubey,0) -- ++(0,0,\cubez) node [midway, right] {$4m$} -- cycle; + \draw[black,fill=gray] (0,0,0) -- ++(-\cubex,0,0) -- ++(0,0,-\cubez) -- ++(\cubex,0,0) -- cycle; + \end{tikzpicture} + \end{minipage} + + \begin{enumerate} + \item Expliquer pourquoi quand la largeur $x$ change, la hauteur $h$ doit elle aussi changer pour respecter les contraintes. + \item Démontrer que l'on doit avoir $h = \dfrac{2}{x}$. + \item On note $S(x)$ l'aire totale de la citerne (c'est à dire la somme des aires des six faces). Montrer que l'on peut écrire + \[ + S(x) = 8x + 4 + \frac{16}{x} + \] + \item Démontrer que + \[ + S(x) = \frac{8x^2 + 4x + 16}{x} + \] + \item Démontrer que + \[ + S'(x) = \frac{8x^2 - 16}{x^2} + \] + \item En déduire le tableau de variation de $S(x)$ sur $\intOF{0}{10}$. + \item Déterminer les valeurs de $x$ et $h$ correspondant à une utilisation minimal de tôle. + \end{enumerate} +\end{exercise} + +\begin{solution} + \begin{enumerate} + \item Le volume étant fixe si l'on fait varier $x$, $h$ doit aussi varier. + \begin{itemize} + \item Si $x = 2$ alors conserver un volume de $V=8$, $h$ doit être égale à $2 / 2$ + \item Si $x = 3$ alors conserver un volume de $V=8$, $h$ doit être égale à $2 / 3$ + \end{itemize} + \item Pour calculer le volume, on a + \begin{eqnarray*} + V &=& h\times x \times 4 \\ + 8 &=& h\times x \times 4 \\ + x &=& \frac{8}{h\times 4} = \frac{2}{h} + \end{eqnarray*} + \item Pour calculer la surface totale, on ajoute la surface de chaque face. On a donc le calcul suivant + \begin{eqnarray*} + S(x) &=& x\times h \times 2 + x\times4\times2 + h\times 4\times 2\\ + S(x) &=& x\times \frac{2}{x} \times 2 + x\times4\times2 + \frac{2}{x}\times 4\times 2\\ + S(x) &=& 8x + 4 + \frac{16}{x} + \end{eqnarray*} + \item Pour trouver cette nouvelle forme, on met chaque élément sur le même dénominateur + \begin{eqnarray*} + S(x) &=& 8x + 4 + \frac{16}{x}\\ + S(x) &=& \frac{8x\times x}{x} + \frac{4\times x}{x} + \frac{16}{x}\\ + S(x) &=& \frac{8x^2 + 4x + 16}{x} + \end{eqnarray*} + \item On retrouve la formule $\frac{u}{v}$ à dériver + \[ + u(x) = 8x^2 + 4x + 16 \Rightarrow u'(x) = 16x + 4 + \] + \[ + v(x) = x \Rightarrow v'(x) = 1 + \] + Donc au numérateur on obtient + \begin{eqnarray*} + u'(x)\times v(x) - u(x)\times v'(x) &=& (16x + 4)\times x - (8x^2 + 4x + 16)\times 1\\ + &=& 8x^2 - 16 + \end{eqnarray*} + Donc + \[ + S'(x) = \frac{8x^2 - 16}{x^2} + \] + \item Tableau de variations de $S$ + + \begin{itemize} + \item Valeur interdite: $x^2 = 0 \equiv x = 0$ + \item Signe de $8x^2 - 16$: c'est un polynôme du 2e degré + \[ + \Delta = 512 > 0 + \] + Il y a donc 2 racines + \[ + x_1 = - 1.4142135623730951 \qquad + x_2 = 1.4142135623730951 + \] + Et on sait que $8x^2 - 16$ est du signe de $a$ donc positif en dehors des racines + \item Le dénominateur $x^2$ est toujours positif. + \item Tableau de variations + + \begin{tikzpicture}[baseline=(a.north)] + \tkzTabInit[lgt=3,espcl=3]{$x$/1,$8x^2 - 16$/1, $x^2$/1, $S'$/1, $S$/2}{$0$, $- 1.4142135623730951$, $10$} + \tkzTabLine{d,-, z, +, } + \tkzTabLine{d,+, , +, } + \tkzTabLine{d,-, z, +, } + \tkzTabVar{D+/ , -/ , +/ } + \end{tikzpicture} + + \end{itemize} + \item On a donc une surface minimal pour $x=1.4142135623730951$ et $h = 2.8284271247461902$. + \end{enumerate} +\end{solution} + +%%% Local Variables: +%%% mode: latex +%%% TeX-master: "master" +%%% End: + +\begin{exercise}[subtitle={Bassin}] + Le tour d'un bassin au niveau du sol présente deux axes de symétrie : l’axe des abscisses et la droite d’équation $x=4$. Il est obtenu par symétrie de la courbe $\mathcal{C}_f$ sur $\intFF{0}{4}$ où $f$ est la fonction définie par + + \[ + f(x) = \left(- x^{2} + 0.3 x - 7.3\right) e^{- x} + 7.3 + \] + On admet que sur $\intFF{0}{4}$ la fonction $f$ est positive. + \begin{enumerate} + \item Sur un repère, tracer l'allure de la courbe $\mathcal{C}_f$, les axes de symétries puis compléter pour dessiner la forme du bassin. + \item Montrer que la fonction $f$ admet comme primitive sur $\R$ la fonction $F$ définie par + \[ + F(x) = 7.3 x + \left( x^{2} + 1.7 x + 9.0\right) e^{- x} + \] + \item Calculer la quantité $\ds \int_0^4 f(x) \; dx$, vous donnerez le résultat sous forme exacte. Interpréter le résultat et reportez cette quantité sur le graphique. + \item On considère que l'échelle de votre graphique est de 1unité pour 15m. Calculer l'aire du bassin. Vous donnerez un résultat arrondi au $m^2$ près. + \end{enumerate} +\end{exercise} + +\begin{solution} + \begin{enumerate} + \item + \begin{tikzpicture}[baseline=(a.north), xscale=1, yscale=0.5] + \tkzInit[xmin=0,xmax=5,xstep=1, + ymin=0,ymax=10,ystep=1] + \tkzGrid + \tkzAxeXY + \tkzFct[domain=0:10,color=red,very thick]% + { (-x**2 + 0.3*x - 7.3)*exp(-x) + 7.3 }; + \end{tikzpicture} + \item Il faut dériver $F(x)$ et vérifier que $F'(x) = f(x)$. + \item $\ds \int_0^4 f(x) \; dx = F(4) - F(0) = \frac{31.8}{e^{4}} + 20.2$ + \item La quantité calculée à la question précédente se retrouve 4fois pour former le bassin. Il faut ensuite prendre en compte l'échelle, comme 1unité de longueur correspond à 15m, une unité d'air correspond à $15\times15 = 225m^2$. Ainsi l'aire du bassin est égale à + \[ + (\frac{31.8}{e^{4}} + 20.2)\times 4 \times 15^2 = 18704.00000 + \] + + \end{enumerate} +\end{solution} + +%%% Local Variables: +%%% mode: latex +%%% TeX-master: "master" +%%% End: + +\begin{exercise}[subtitle={Stylos}] + \emph{Les parties {\rm A} et {\rm B} de cet exercice sont indépendantes.} + + \bigskip + + \begin{minipage}{0.6\linewidth} + \textbf{Partie A} + + \medskip + + Deux ateliers A et B fabriquent des stylos pour une entreprise. + + L'atelier A fabrique 86.0\,\% des stylos, et parmi ceux-là, 34.0\,\% possèdent un défaut de fabrication. + + De plus, 8.0\,\% des stylos possèdent un défaut de fabrication et sortent de l'atelier B. + + Un stylo est prélevé au hasard dans le stock de l'entreprise. + + On considère les évènements suivants: + + \begin{itemize} + \item A : \og Le stylo a été fabriqué par l'atelier A \fg + \item B : \og Le stylo a été fabriqué par l'atelier B \fg + \item D : \og Le stylo possède un défaut de fabrication \fg + \end{itemize} + \end{minipage} + \begin{minipage}{0.4\linewidth} + \begin{center} + \begin{tikzpicture}[sloped] + \node {.} + child {node {$A$} + child {node {$D$} + edge from parent + node[above] {...} + } + child {node {$\overline{D}$} + edge from parent + node[above] {...} + } + edge from parent + node[above] {...} + } + child[missing] {} + child { node {$B$} + child {node {$D$} + edge from parent + node[above] {...} + } + child {node {$\overline{D}$} + edge from parent + node[above] {...} + } + edge from parent + node[above] {...} + } ; + \end{tikzpicture} + \end{center} + \end{minipage} + + \medskip + + \begin{enumerate} + \item Compléter l'arbre de probabilité ci-contre + \item Interpréter puis donner les probabilités $P(A)$, $P(B)$, $P_A(D)$ et $P(B \cap D)$. + + \item + \begin{enumerate} + \item Calculer la probabilité qu'un stylo provienne de l'atelier A et possède un défaut de fabrication. + \item En déduire que la probabilité qu'un stylo possède un défaut de fabrication est de $0.37$. + \end{enumerate} + \item On prélève un stylo au hasard avec un défaut. Quelle est la probabilité qu'il vienne de l'atelier A? + \end{enumerate} + + \bigskip + + \textbf{Partie B} + \medskip + + Dans cette partie, on suppose que 37.0\,\% des stylos possèdent un défaut de fabrication. + + L'entreprise confectionne des paquets contenant chacun $4$~stylos. + + Le fait qu'un stylo possède ou non un défaut de fabrication est indépendant des autres stylos. + + On appelle $X$ la variable aléatoire donnant pour un paquet le nombre de stylos qui possèdent un défaut de fabrication. + + On admet que la variable aléatoire $X$ suit une loi binomiale. + + \medskip + + \begin{enumerate} + \setcounter{enumi}{4} + \item Avec quelle loi peut-on modéliser $X$. Préciser les paramètres. + \item Calculer et interpréter la probabilité $P(X = 10)$. + \item Le directeur de l'entreprise affirme qu'il y a plus d'une chance sur deux qu'un paquet ne comporte aucun stylo défectueux. A-t-il raison ? + \item Combien de stylos peut-on espérer avoir en moyenne? + \end{enumerate} + \pagebreak +\end{exercise} + +\begin{solution} + \begin{enumerate} + \item + \begin{center} + \begin{tikzpicture}[sloped] + \node {.} + child {node {$A$} + child {node {$D$} + edge from parent + node[above] {0.34} + } + child {node {$\overline{D}$} + edge from parent + node[above] {0.66} + } + edge from parent + node[above] {0.86} + } + child[missing] {} + child { node {$B$} + child {node {$D$} + edge from parent + node[above] {0.58} + } + child {node {$\overline{D}$} + edge from parent + node[above] {0.42} + } + edge from parent + node[above] {0.14} + } ; + \end{tikzpicture} + \end{center} + \item + \begin{itemize} + \item Probabilité que le stylo vienne de l'atelier A + \[ + P(A) = 0.86 + \] + \item Probabilité que le stylo vienne de l'atelier B + \[ + P(B) = 0.14 + \] + \item Probabilité que le stylo ait un défaut sachant qu'il vient de l'atelier A. + \[ + P_A(D) = 0.34 + \] + \item Probabilité que le stylo vienne de l'atelier B et qu'il ait un défaut. + \[ + P(D \cap D) = 0.08 + \] + \end{itemize} + \item + \begin{enumerate} + \item Probabilité qu'un stylo vienne de l'atelier A et qu'il ait un defaut + \[ + P(A\cap D) = P(A) \times P_A(D) = 0.86 \times 0.34 = 0.29 + \] + \item Probabilité que le stylo ai un défaut de fabrication. + \[ + P(D) = P(A\cap D) + P(B\cap D) = 0.29 + 0.08 = 0.37 + \] + \end{enumerate} + \item Probabilité qu'il vienne de l'atelier A sachant qu'il a un defaut + \[ + P_D(A) = \frac{P(A\cap D)}{P(D)} = \frac{0.29}{0.37} = 0.78 + \] + \item $X$ peut être modélisée par une loi binomiale de paramètres $n=20$ et $p=0.37$. + \item (\textit{par de correction automatique disponible pour le résultat final} + \[ + P(X = 10) = \coefBino{20}{10}\times 0.37^{10} \times 0.63^{10} + \] + \item (\textit{par de correction automatique disponible pour le résultat final} + + Il faut calculer la probabilité qu'il y ait 0 stylo avec un defaut. + \[ + P(X = 0) = \coefBino{20}{0}\times 0.37^{0} \times 0.63^{20} + \] + Puis comparer ce nombre à 0,5. + \item Il faut calculer l'espérance + \[ + E[X] = n\times p = 20 \times 0.37 = 7.4 + \] + \end{enumerate} +\end{solution} + +\end{document} + +%%% Local Variables: +%%% mode: latex +%%% TeX-master: "master" +%%% End: diff --git a/Complementaire/DM/2105_DM1/02_2105_DM1.tex b/Complementaire/DM/2105_DM1/02_2105_DM1.tex new file mode 100644 index 0000000..8527ed9 --- /dev/null +++ b/Complementaire/DM/2105_DM1/02_2105_DM1.tex @@ -0,0 +1,367 @@ +\documentclass[a4paper,10pt]{article} +\usepackage{myXsim} + +% Title Page +\title{DM1 \hfill BALUKHATYY Alexandre} +\tribe{Maths complémentaire} +\date{\hfillÀ render pour le jeudi 27 mai} + +\xsimsetup{ + solution/print = false +} + +\begin{document} +\maketitle + +Les valeurs des exercices sont générés automatiquement. Si une valeur a un nombre adhérant de chiffres après la virgule, vous pouvez l'arrondir à l'entier le plus proche. + +\begin{exercise}[subtitle={Optimisation de matière}] + \begin{minipage}{0.6\textwidth} + On se propose de fabriquer avec le moins de tôle possible une citerne fermée en forme de parallélépipède rectangle dont le volume intérieur doit être de $15m^3$. La longueur est aussi fixée à $3m$ par le cahier des charges. + + On peut donc faire varier uniquement la largeur (notée $x$) et la hauteur (notée $h$) de la cuve. + \end{minipage} + \hfill + \begin{minipage}{0.3\textwidth} + \begin{tikzpicture} + \pgfmathsetmacro{\cubex}{3} + \pgfmathsetmacro{\cubey}{1} + \pgfmathsetmacro{\cubez}{2} + \draw[black,fill=gray] (0,0,0) -- ++(-\cubex,0,0) -- ++(0,-\cubey,0) node [midway, left] {$h$} -- ++(\cubex,0,0) node [midway, below] {$x$} -- cycle; + \draw[black,fill=gray] (0,0,0) -- ++(0,0,-\cubez) -- ++(0,-\cubey,0) -- ++(0,0,\cubez) node [midway, right] {$3m$} -- cycle; + \draw[black,fill=gray] (0,0,0) -- ++(-\cubex,0,0) -- ++(0,0,-\cubez) -- ++(\cubex,0,0) -- cycle; + \end{tikzpicture} + \end{minipage} + + \begin{enumerate} + \item Expliquer pourquoi quand la largeur $x$ change, la hauteur $h$ doit elle aussi changer pour respecter les contraintes. + \item Démontrer que l'on doit avoir $h = \dfrac{5}{x}$. + \item On note $S(x)$ l'aire totale de la citerne (c'est à dire la somme des aires des six faces). Montrer que l'on peut écrire + \[ + S(x) = 6x + 10 + \frac{30}{x} + \] + \item Démontrer que + \[ + S(x) = \frac{6x^2 + 10x + 30}{x} + \] + \item Démontrer que + \[ + S'(x) = \frac{6x^2 - 30}{x^2} + \] + \item En déduire le tableau de variation de $S(x)$ sur $\intOF{0}{10}$. + \item Déterminer les valeurs de $x$ et $h$ correspondant à une utilisation minimal de tôle. + \end{enumerate} +\end{exercise} + +\begin{solution} + \begin{enumerate} + \item Le volume étant fixe si l'on fait varier $x$, $h$ doit aussi varier. + \begin{itemize} + \item Si $x = 2$ alors conserver un volume de $V=15$, $h$ doit être égale à $5 / 2$ + \item Si $x = 3$ alors conserver un volume de $V=15$, $h$ doit être égale à $5 / 3$ + \end{itemize} + \item Pour calculer le volume, on a + \begin{eqnarray*} + V &=& h\times x \times 3 \\ + 15 &=& h\times x \times 3 \\ + x &=& \frac{15}{h\times 3} = \frac{5}{h} + \end{eqnarray*} + \item Pour calculer la surface totale, on ajoute la surface de chaque face. On a donc le calcul suivant + \begin{eqnarray*} + S(x) &=& x\times h \times 2 + x\times3\times2 + h\times 3\times 2\\ + S(x) &=& x\times \frac{5}{x} \times 2 + x\times3\times2 + \frac{5}{x}\times 3\times 2\\ + S(x) &=& 6x + 10 + \frac{30}{x} + \end{eqnarray*} + \item Pour trouver cette nouvelle forme, on met chaque élément sur le même dénominateur + \begin{eqnarray*} + S(x) &=& 6x + 10 + \frac{30}{x}\\ + S(x) &=& \frac{6x\times x}{x} + \frac{10\times x}{x} + \frac{30}{x}\\ + S(x) &=& \frac{6x^2 + 10x + 30}{x} + \end{eqnarray*} + \item On retrouve la formule $\frac{u}{v}$ à dériver + \[ + u(x) = 6x^2 + 10x + 30 \Rightarrow u'(x) = 12x + 10 + \] + \[ + v(x) = x \Rightarrow v'(x) = 1 + \] + Donc au numérateur on obtient + \begin{eqnarray*} + u'(x)\times v(x) - u(x)\times v'(x) &=& (12x + 10)\times x - (6x^2 + 10x + 30)\times 1\\ + &=& 6x^2 - 30 + \end{eqnarray*} + Donc + \[ + S'(x) = \frac{6x^2 - 30}{x^2} + \] + \item Tableau de variations de $S$ + + \begin{itemize} + \item Valeur interdite: $x^2 = 0 \equiv x = 0$ + \item Signe de $6x^2 - 30$: c'est un polynôme du 2e degré + \[ + \Delta = 720 > 0 + \] + Il y a donc 2 racines + \[ + x_1 = - 2.23606797749979 \qquad + x_2 = 2.23606797749979 + \] + Et on sait que $6x^2 - 30$ est du signe de $a$ donc positif en dehors des racines + \item Le dénominateur $x^2$ est toujours positif. + \item Tableau de variations + + \begin{tikzpicture}[baseline=(a.north)] + \tkzTabInit[lgt=3,espcl=3]{$x$/1,$6x^2 - 30$/1, $x^2$/1, $S'$/1, $S$/2}{$0$, $- 2.23606797749979$, $10$} + \tkzTabLine{d,-, z, +, } + \tkzTabLine{d,+, , +, } + \tkzTabLine{d,-, z, +, } + \tkzTabVar{D+/ , -/ , +/ } + \end{tikzpicture} + + \end{itemize} + \item On a donc une surface minimal pour $x=2.23606797749979$ et $h = 11.18033988749895$. + \end{enumerate} +\end{solution} + +%%% Local Variables: +%%% mode: latex +%%% TeX-master: "master" +%%% End: + +\begin{exercise}[subtitle={Bassin}] + Le tour d'un bassin au niveau du sol présente deux axes de symétrie : l’axe des abscisses et la droite d’équation $x=4$. Il est obtenu par symétrie de la courbe $\mathcal{C}_f$ sur $\intFF{0}{4}$ où $f$ est la fonction définie par + + \[ + f(x) = \left(- x^{2} + 8.4 x - 0.6\right) e^{- x} + 0.6 + \] + On admet que sur $\intFF{0}{4}$ la fonction $f$ est positive. + \begin{enumerate} + \item Sur un repère, tracer l'allure de la courbe $\mathcal{C}_f$, les axes de symétries puis compléter pour dessiner la forme du bassin. + \item Montrer que la fonction $f$ admet comme primitive sur $\R$ la fonction $F$ définie par + \[ + F(x) = 0.6 x + \left( x^{2} - 6.4 x - 5.8\right) e^{- x} + \] + \item Calculer la quantité $\ds \int_0^4 f(x) \; dx$, vous donnerez le résultat sous forme exacte. Interpréter le résultat et reportez cette quantité sur le graphique. + \item On considère que l'échelle de votre graphique est de 1unité pour 15m. Calculer l'aire du bassin. Vous donnerez un résultat arrondi au $m^2$ près. + \end{enumerate} +\end{exercise} + +\begin{solution} + \begin{enumerate} + \item + \begin{tikzpicture}[baseline=(a.north), xscale=1, yscale=0.5] + \tkzInit[xmin=0,xmax=5,xstep=1, + ymin=0,ymax=10,ystep=1] + \tkzGrid + \tkzAxeXY + \tkzFct[domain=0:10,color=red,very thick]% + { (-x**2 + 8.4*x - 0.6)*exp(-x) + 0.6 }; + \end{tikzpicture} + \item Il faut dériver $F(x)$ et vérifier que $F'(x) = f(x)$. + \item $\ds \int_0^4 f(x) \; dx = F(4) - F(0) = 8.2 - \frac{15.4}{e^{4}}$ + \item La quantité calculée à la question précédente se retrouve 4fois pour former le bassin. Il faut ensuite prendre en compte l'échelle, comme 1unité de longueur correspond à 15m, une unité d'air correspond à $15\times15 = 225m^2$. Ainsi l'aire du bassin est égale à + \[ + (8.2 - \frac{15.4}{e^{4}})\times 4 \times 15^2 = 7126.000000 + \] + + \end{enumerate} +\end{solution} + +%%% Local Variables: +%%% mode: latex +%%% TeX-master: "master" +%%% End: + +\begin{exercise}[subtitle={Stylos}] + \emph{Les parties {\rm A} et {\rm B} de cet exercice sont indépendantes.} + + \bigskip + + \begin{minipage}{0.6\linewidth} + \textbf{Partie A} + + \medskip + + Deux ateliers A et B fabriquent des stylos pour une entreprise. + + L'atelier A fabrique 25.0\,\% des stylos, et parmi ceux-là, 84.0\,\% possèdent un défaut de fabrication. + + De plus, 57.99999999999999\,\% des stylos possèdent un défaut de fabrication et sortent de l'atelier B. + + Un stylo est prélevé au hasard dans le stock de l'entreprise. + + On considère les évènements suivants: + + \begin{itemize} + \item A : \og Le stylo a été fabriqué par l'atelier A \fg + \item B : \og Le stylo a été fabriqué par l'atelier B \fg + \item D : \og Le stylo possède un défaut de fabrication \fg + \end{itemize} + \end{minipage} + \begin{minipage}{0.4\linewidth} + \begin{center} + \begin{tikzpicture}[sloped] + \node {.} + child {node {$A$} + child {node {$D$} + edge from parent + node[above] {...} + } + child {node {$\overline{D}$} + edge from parent + node[above] {...} + } + edge from parent + node[above] {...} + } + child[missing] {} + child { node {$B$} + child {node {$D$} + edge from parent + node[above] {...} + } + child {node {$\overline{D}$} + edge from parent + node[above] {...} + } + edge from parent + node[above] {...} + } ; + \end{tikzpicture} + \end{center} + \end{minipage} + + \medskip + + \begin{enumerate} + \item Compléter l'arbre de probabilité ci-contre + \item Interpréter puis donner les probabilités $P(A)$, $P(B)$, $P_A(D)$ et $P(B \cap D)$. + + \item + \begin{enumerate} + \item Calculer la probabilité qu'un stylo provienne de l'atelier A et possède un défaut de fabrication. + \item En déduire que la probabilité qu'un stylo possède un défaut de fabrication est de $0.79$. + \end{enumerate} + \item On prélève un stylo au hasard avec un défaut. Quelle est la probabilité qu'il vienne de l'atelier A? + \end{enumerate} + + \bigskip + + \textbf{Partie B} + \medskip + + Dans cette partie, on suppose que 79.0\,\% des stylos possèdent un défaut de fabrication. + + L'entreprise confectionne des paquets contenant chacun $4$~stylos. + + Le fait qu'un stylo possède ou non un défaut de fabrication est indépendant des autres stylos. + + On appelle $X$ la variable aléatoire donnant pour un paquet le nombre de stylos qui possèdent un défaut de fabrication. + + On admet que la variable aléatoire $X$ suit une loi binomiale. + + \medskip + + \begin{enumerate} + \setcounter{enumi}{4} + \item Avec quelle loi peut-on modéliser $X$. Préciser les paramètres. + \item Calculer et interpréter la probabilité $P(X = 11)$. + \item Le directeur de l'entreprise affirme qu'il y a plus d'une chance sur deux qu'un paquet ne comporte aucun stylo défectueux. A-t-il raison ? + \item Combien de stylos peut-on espérer avoir en moyenne? + \end{enumerate} + \pagebreak +\end{exercise} + +\begin{solution} + \begin{enumerate} + \item + \begin{center} + \begin{tikzpicture}[sloped] + \node {.} + child {node {$A$} + child {node {$D$} + edge from parent + node[above] {0.84} + } + child {node {$\overline{D}$} + edge from parent + node[above] {0.16} + } + edge from parent + node[above] {0.25} + } + child[missing] {} + child { node {$B$} + child {node {$D$} + edge from parent + node[above] {0.78} + } + child {node {$\overline{D}$} + edge from parent + node[above] {0.22} + } + edge from parent + node[above] {0.75} + } ; + \end{tikzpicture} + \end{center} + \item + \begin{itemize} + \item Probabilité que le stylo vienne de l'atelier A + \[ + P(A) = 0.25 + \] + \item Probabilité que le stylo vienne de l'atelier B + \[ + P(B) = 0.75 + \] + \item Probabilité que le stylo ait un défaut sachant qu'il vient de l'atelier A. + \[ + P_A(D) = 0.84 + \] + \item Probabilité que le stylo vienne de l'atelier B et qu'il ait un défaut. + \[ + P(D \cap D) = 0.58 + \] + \end{itemize} + \item + \begin{enumerate} + \item Probabilité qu'un stylo vienne de l'atelier A et qu'il ait un defaut + \[ + P(A\cap D) = P(A) \times P_A(D) = 0.25 \times 0.84 = 0.21 + \] + \item Probabilité que le stylo ai un défaut de fabrication. + \[ + P(D) = P(A\cap D) + P(B\cap D) = 0.21 + 0.58 = 0.79 + \] + \end{enumerate} + \item Probabilité qu'il vienne de l'atelier A sachant qu'il a un defaut + \[ + P_D(A) = \frac{P(A\cap D)}{P(D)} = \frac{0.21}{0.79} = 0.27 + \] + \item $X$ peut être modélisée par une loi binomiale de paramètres $n=12$ et $p=0.79$. + \item (\textit{par de correction automatique disponible pour le résultat final} + \[ + P(X = 11) = \coefBino{12}{11}\times 0.79^{11} \times 0.21^{1} + \] + \item (\textit{par de correction automatique disponible pour le résultat final} + + Il faut calculer la probabilité qu'il y ait 0 stylo avec un defaut. + \[ + P(X = 0) = \coefBino{12}{0}\times 0.79^{0} \times 0.21^{12} + \] + Puis comparer ce nombre à 0,5. + \item Il faut calculer l'espérance + \[ + E[X] = n\times p = 12 \times 0.79 = 9.48 + \] + \end{enumerate} +\end{solution} + +\end{document} + +%%% Local Variables: +%%% mode: latex +%%% TeX-master: "master" +%%% End: diff --git a/Complementaire/DM/2105_DM1/03_2105_DM1.tex b/Complementaire/DM/2105_DM1/03_2105_DM1.tex new file mode 100644 index 0000000..34aebe0 --- /dev/null +++ b/Complementaire/DM/2105_DM1/03_2105_DM1.tex @@ -0,0 +1,367 @@ +\documentclass[a4paper,10pt]{article} +\usepackage{myXsim} + +% Title Page +\title{DM1 \hfill CALES Mathis} +\tribe{Maths complémentaire} +\date{\hfillÀ render pour le jeudi 27 mai} + +\xsimsetup{ + solution/print = false +} + +\begin{document} +\maketitle + +Les valeurs des exercices sont générés automatiquement. Si une valeur a un nombre adhérant de chiffres après la virgule, vous pouvez l'arrondir à l'entier le plus proche. + +\begin{exercise}[subtitle={Optimisation de matière}] + \begin{minipage}{0.6\textwidth} + On se propose de fabriquer avec le moins de tôle possible une citerne fermée en forme de parallélépipède rectangle dont le volume intérieur doit être de $6m^3$. La longueur est aussi fixée à $2m$ par le cahier des charges. + + On peut donc faire varier uniquement la largeur (notée $x$) et la hauteur (notée $h$) de la cuve. + \end{minipage} + \hfill + \begin{minipage}{0.3\textwidth} + \begin{tikzpicture} + \pgfmathsetmacro{\cubex}{3} + \pgfmathsetmacro{\cubey}{1} + \pgfmathsetmacro{\cubez}{2} + \draw[black,fill=gray] (0,0,0) -- ++(-\cubex,0,0) -- ++(0,-\cubey,0) node [midway, left] {$h$} -- ++(\cubex,0,0) node [midway, below] {$x$} -- cycle; + \draw[black,fill=gray] (0,0,0) -- ++(0,0,-\cubez) -- ++(0,-\cubey,0) -- ++(0,0,\cubez) node [midway, right] {$2m$} -- cycle; + \draw[black,fill=gray] (0,0,0) -- ++(-\cubex,0,0) -- ++(0,0,-\cubez) -- ++(\cubex,0,0) -- cycle; + \end{tikzpicture} + \end{minipage} + + \begin{enumerate} + \item Expliquer pourquoi quand la largeur $x$ change, la hauteur $h$ doit elle aussi changer pour respecter les contraintes. + \item Démontrer que l'on doit avoir $h = \dfrac{3}{x}$. + \item On note $S(x)$ l'aire totale de la citerne (c'est à dire la somme des aires des six faces). Montrer que l'on peut écrire + \[ + S(x) = 4x + 6 + \frac{12}{x} + \] + \item Démontrer que + \[ + S(x) = \frac{4x^2 + 6x + 12}{x} + \] + \item Démontrer que + \[ + S'(x) = \frac{4x^2 - 12}{x^2} + \] + \item En déduire le tableau de variation de $S(x)$ sur $\intOF{0}{10}$. + \item Déterminer les valeurs de $x$ et $h$ correspondant à une utilisation minimal de tôle. + \end{enumerate} +\end{exercise} + +\begin{solution} + \begin{enumerate} + \item Le volume étant fixe si l'on fait varier $x$, $h$ doit aussi varier. + \begin{itemize} + \item Si $x = 2$ alors conserver un volume de $V=6$, $h$ doit être égale à $3 / 2$ + \item Si $x = 3$ alors conserver un volume de $V=6$, $h$ doit être égale à $3 / 3$ + \end{itemize} + \item Pour calculer le volume, on a + \begin{eqnarray*} + V &=& h\times x \times 2 \\ + 6 &=& h\times x \times 2 \\ + x &=& \frac{6}{h\times 2} = \frac{3}{h} + \end{eqnarray*} + \item Pour calculer la surface totale, on ajoute la surface de chaque face. On a donc le calcul suivant + \begin{eqnarray*} + S(x) &=& x\times h \times 2 + x\times2\times2 + h\times 2\times 2\\ + S(x) &=& x\times \frac{3}{x} \times 2 + x\times2\times2 + \frac{3}{x}\times 2\times 2\\ + S(x) &=& 4x + 6 + \frac{12}{x} + \end{eqnarray*} + \item Pour trouver cette nouvelle forme, on met chaque élément sur le même dénominateur + \begin{eqnarray*} + S(x) &=& 4x + 6 + \frac{12}{x}\\ + S(x) &=& \frac{4x\times x}{x} + \frac{6\times x}{x} + \frac{12}{x}\\ + S(x) &=& \frac{4x^2 + 6x + 12}{x} + \end{eqnarray*} + \item On retrouve la formule $\frac{u}{v}$ à dériver + \[ + u(x) = 4x^2 + 6x + 12 \Rightarrow u'(x) = 8x + 6 + \] + \[ + v(x) = x \Rightarrow v'(x) = 1 + \] + Donc au numérateur on obtient + \begin{eqnarray*} + u'(x)\times v(x) - u(x)\times v'(x) &=& (8x + 6)\times x - (4x^2 + 6x + 12)\times 1\\ + &=& 4x^2 - 12 + \end{eqnarray*} + Donc + \[ + S'(x) = \frac{4x^2 - 12}{x^2} + \] + \item Tableau de variations de $S$ + + \begin{itemize} + \item Valeur interdite: $x^2 = 0 \equiv x = 0$ + \item Signe de $4x^2 - 12$: c'est un polynôme du 2e degré + \[ + \Delta = 192 > 0 + \] + Il y a donc 2 racines + \[ + x_1 = - 1.7320508075688772 \qquad + x_2 = 1.7320508075688772 + \] + Et on sait que $4x^2 - 12$ est du signe de $a$ donc positif en dehors des racines + \item Le dénominateur $x^2$ est toujours positif. + \item Tableau de variations + + \begin{tikzpicture}[baseline=(a.north)] + \tkzTabInit[lgt=3,espcl=3]{$x$/1,$4x^2 - 12$/1, $x^2$/1, $S'$/1, $S$/2}{$0$, $- 1.7320508075688772$, $10$} + \tkzTabLine{d,-, z, +, } + \tkzTabLine{d,+, , +, } + \tkzTabLine{d,-, z, +, } + \tkzTabVar{D+/ , -/ , +/ } + \end{tikzpicture} + + \end{itemize} + \item On a donc une surface minimal pour $x=1.7320508075688772$ et $h = 5.1961524227066316$. + \end{enumerate} +\end{solution} + +%%% Local Variables: +%%% mode: latex +%%% TeX-master: "master" +%%% End: + +\begin{exercise}[subtitle={Bassin}] + Le tour d'un bassin au niveau du sol présente deux axes de symétrie : l’axe des abscisses et la droite d’équation $x=4$. Il est obtenu par symétrie de la courbe $\mathcal{C}_f$ sur $\intFF{0}{4}$ où $f$ est la fonction définie par + + \[ + f(x) = \left(- x^{2} + 2.8 x - 8.7\right) e^{- x} + 8.7 + \] + On admet que sur $\intFF{0}{4}$ la fonction $f$ est positive. + \begin{enumerate} + \item Sur un repère, tracer l'allure de la courbe $\mathcal{C}_f$, les axes de symétries puis compléter pour dessiner la forme du bassin. + \item Montrer que la fonction $f$ admet comme primitive sur $\R$ la fonction $F$ définie par + \[ + F(x) = 8.7 x + \left( x^{2} - 0.8 x + 7.9\right) e^{- x} + \] + \item Calculer la quantité $\ds \int_0^4 f(x) \; dx$, vous donnerez le résultat sous forme exacte. Interpréter le résultat et reportez cette quantité sur le graphique. + \item On considère que l'échelle de votre graphique est de 1unité pour 15m. Calculer l'aire du bassin. Vous donnerez un résultat arrondi au $m^2$ près. + \end{enumerate} +\end{exercise} + +\begin{solution} + \begin{enumerate} + \item + \begin{tikzpicture}[baseline=(a.north), xscale=1, yscale=0.5] + \tkzInit[xmin=0,xmax=5,xstep=1, + ymin=0,ymax=10,ystep=1] + \tkzGrid + \tkzAxeXY + \tkzFct[domain=0:10,color=red,very thick]% + { (-x**2 + 2.8*x - 8.7)*exp(-x) + 8.7 }; + \end{tikzpicture} + \item Il faut dériver $F(x)$ et vérifier que $F'(x) = f(x)$. + \item $\ds \int_0^4 f(x) \; dx = F(4) - F(0) = \frac{20.7}{e^{4}} + 26.9$ + \item La quantité calculée à la question précédente se retrouve 4fois pour former le bassin. Il faut ensuite prendre en compte l'échelle, comme 1unité de longueur correspond à 15m, une unité d'air correspond à $15\times15 = 225m^2$. Ainsi l'aire du bassin est égale à + \[ + (\frac{20.7}{e^{4}} + 26.9)\times 4 \times 15^2 = 24551.00000 + \] + + \end{enumerate} +\end{solution} + +%%% Local Variables: +%%% mode: latex +%%% TeX-master: "master" +%%% End: + +\begin{exercise}[subtitle={Stylos}] + \emph{Les parties {\rm A} et {\rm B} de cet exercice sont indépendantes.} + + \bigskip + + \begin{minipage}{0.6\linewidth} + \textbf{Partie A} + + \medskip + + Deux ateliers A et B fabriquent des stylos pour une entreprise. + + L'atelier A fabrique 48.0\,\% des stylos, et parmi ceux-là, 99.0\,\% possèdent un défaut de fabrication. + + De plus, 10.0\,\% des stylos possèdent un défaut de fabrication et sortent de l'atelier B. + + Un stylo est prélevé au hasard dans le stock de l'entreprise. + + On considère les évènements suivants: + + \begin{itemize} + \item A : \og Le stylo a été fabriqué par l'atelier A \fg + \item B : \og Le stylo a été fabriqué par l'atelier B \fg + \item D : \og Le stylo possède un défaut de fabrication \fg + \end{itemize} + \end{minipage} + \begin{minipage}{0.4\linewidth} + \begin{center} + \begin{tikzpicture}[sloped] + \node {.} + child {node {$A$} + child {node {$D$} + edge from parent + node[above] {...} + } + child {node {$\overline{D}$} + edge from parent + node[above] {...} + } + edge from parent + node[above] {...} + } + child[missing] {} + child { node {$B$} + child {node {$D$} + edge from parent + node[above] {...} + } + child {node {$\overline{D}$} + edge from parent + node[above] {...} + } + edge from parent + node[above] {...} + } ; + \end{tikzpicture} + \end{center} + \end{minipage} + + \medskip + + \begin{enumerate} + \item Compléter l'arbre de probabilité ci-contre + \item Interpréter puis donner les probabilités $P(A)$, $P(B)$, $P_A(D)$ et $P(B \cap D)$. + + \item + \begin{enumerate} + \item Calculer la probabilité qu'un stylo provienne de l'atelier A et possède un défaut de fabrication. + \item En déduire que la probabilité qu'un stylo possède un défaut de fabrication est de $0.58$. + \end{enumerate} + \item On prélève un stylo au hasard avec un défaut. Quelle est la probabilité qu'il vienne de l'atelier A? + \end{enumerate} + + \bigskip + + \textbf{Partie B} + \medskip + + Dans cette partie, on suppose que 57.99999999999999\,\% des stylos possèdent un défaut de fabrication. + + L'entreprise confectionne des paquets contenant chacun $4$~stylos. + + Le fait qu'un stylo possède ou non un défaut de fabrication est indépendant des autres stylos. + + On appelle $X$ la variable aléatoire donnant pour un paquet le nombre de stylos qui possèdent un défaut de fabrication. + + On admet que la variable aléatoire $X$ suit une loi binomiale. + + \medskip + + \begin{enumerate} + \setcounter{enumi}{4} + \item Avec quelle loi peut-on modéliser $X$. Préciser les paramètres. + \item Calculer et interpréter la probabilité $P(X = 10)$. + \item Le directeur de l'entreprise affirme qu'il y a plus d'une chance sur deux qu'un paquet ne comporte aucun stylo défectueux. A-t-il raison ? + \item Combien de stylos peut-on espérer avoir en moyenne? + \end{enumerate} + \pagebreak +\end{exercise} + +\begin{solution} + \begin{enumerate} + \item + \begin{center} + \begin{tikzpicture}[sloped] + \node {.} + child {node {$A$} + child {node {$D$} + edge from parent + node[above] {0.99} + } + child {node {$\overline{D}$} + edge from parent + node[above] {0.01} + } + edge from parent + node[above] {0.48} + } + child[missing] {} + child { node {$B$} + child {node {$D$} + edge from parent + node[above] {0.2} + } + child {node {$\overline{D}$} + edge from parent + node[above] {0.8} + } + edge from parent + node[above] {0.52} + } ; + \end{tikzpicture} + \end{center} + \item + \begin{itemize} + \item Probabilité que le stylo vienne de l'atelier A + \[ + P(A) = 0.48 + \] + \item Probabilité que le stylo vienne de l'atelier B + \[ + P(B) = 0.52 + \] + \item Probabilité que le stylo ait un défaut sachant qu'il vient de l'atelier A. + \[ + P_A(D) = 0.99 + \] + \item Probabilité que le stylo vienne de l'atelier B et qu'il ait un défaut. + \[ + P(D \cap D) = 0.1 + \] + \end{itemize} + \item + \begin{enumerate} + \item Probabilité qu'un stylo vienne de l'atelier A et qu'il ait un defaut + \[ + P(A\cap D) = P(A) \times P_A(D) = 0.48 \times 0.99 = 0.48 + \] + \item Probabilité que le stylo ai un défaut de fabrication. + \[ + P(D) = P(A\cap D) + P(B\cap D) = 0.48 + 0.1 = 0.58 + \] + \end{enumerate} + \item Probabilité qu'il vienne de l'atelier A sachant qu'il a un defaut + \[ + P_D(A) = \frac{P(A\cap D)}{P(D)} = \frac{0.48}{0.58} = 0.83 + \] + \item $X$ peut être modélisée par une loi binomiale de paramètres $n=17$ et $p=0.58$. + \item (\textit{par de correction automatique disponible pour le résultat final} + \[ + P(X = 10) = \coefBino{17}{10}\times 0.58^{10} \times 0.42^{7} + \] + \item (\textit{par de correction automatique disponible pour le résultat final} + + Il faut calculer la probabilité qu'il y ait 0 stylo avec un defaut. + \[ + P(X = 0) = \coefBino{17}{0}\times 0.58^{0} \times 0.42^{17} + \] + Puis comparer ce nombre à 0,5. + \item Il faut calculer l'espérance + \[ + E[X] = n\times p = 17 \times 0.58 = 9.86 + \] + \end{enumerate} +\end{solution} + +\end{document} + +%%% Local Variables: +%%% mode: latex +%%% TeX-master: "master" +%%% End: diff --git a/Complementaire/DM/2105_DM1/04_2105_DM1.tex b/Complementaire/DM/2105_DM1/04_2105_DM1.tex new file mode 100644 index 0000000..e713031 --- /dev/null +++ b/Complementaire/DM/2105_DM1/04_2105_DM1.tex @@ -0,0 +1,367 @@ +\documentclass[a4paper,10pt]{article} +\usepackage{myXsim} + +% Title Page +\title{DM1 \hfill CHAKIR Iman} +\tribe{Maths complémentaire} +\date{\hfillÀ render pour le jeudi 27 mai} + +\xsimsetup{ + solution/print = false +} + +\begin{document} +\maketitle + +Les valeurs des exercices sont générés automatiquement. Si une valeur a un nombre adhérant de chiffres après la virgule, vous pouvez l'arrondir à l'entier le plus proche. + +\begin{exercise}[subtitle={Optimisation de matière}] + \begin{minipage}{0.6\textwidth} + On se propose de fabriquer avec le moins de tôle possible une citerne fermée en forme de parallélépipède rectangle dont le volume intérieur doit être de $20m^3$. La longueur est aussi fixée à $4m$ par le cahier des charges. + + On peut donc faire varier uniquement la largeur (notée $x$) et la hauteur (notée $h$) de la cuve. + \end{minipage} + \hfill + \begin{minipage}{0.3\textwidth} + \begin{tikzpicture} + \pgfmathsetmacro{\cubex}{3} + \pgfmathsetmacro{\cubey}{1} + \pgfmathsetmacro{\cubez}{2} + \draw[black,fill=gray] (0,0,0) -- ++(-\cubex,0,0) -- ++(0,-\cubey,0) node [midway, left] {$h$} -- ++(\cubex,0,0) node [midway, below] {$x$} -- cycle; + \draw[black,fill=gray] (0,0,0) -- ++(0,0,-\cubez) -- ++(0,-\cubey,0) -- ++(0,0,\cubez) node [midway, right] {$4m$} -- cycle; + \draw[black,fill=gray] (0,0,0) -- ++(-\cubex,0,0) -- ++(0,0,-\cubez) -- ++(\cubex,0,0) -- cycle; + \end{tikzpicture} + \end{minipage} + + \begin{enumerate} + \item Expliquer pourquoi quand la largeur $x$ change, la hauteur $h$ doit elle aussi changer pour respecter les contraintes. + \item Démontrer que l'on doit avoir $h = \dfrac{5}{x}$. + \item On note $S(x)$ l'aire totale de la citerne (c'est à dire la somme des aires des six faces). Montrer que l'on peut écrire + \[ + S(x) = 8x + 10 + \frac{40}{x} + \] + \item Démontrer que + \[ + S(x) = \frac{8x^2 + 10x + 40}{x} + \] + \item Démontrer que + \[ + S'(x) = \frac{8x^2 - 40}{x^2} + \] + \item En déduire le tableau de variation de $S(x)$ sur $\intOF{0}{10}$. + \item Déterminer les valeurs de $x$ et $h$ correspondant à une utilisation minimal de tôle. + \end{enumerate} +\end{exercise} + +\begin{solution} + \begin{enumerate} + \item Le volume étant fixe si l'on fait varier $x$, $h$ doit aussi varier. + \begin{itemize} + \item Si $x = 2$ alors conserver un volume de $V=20$, $h$ doit être égale à $5 / 2$ + \item Si $x = 3$ alors conserver un volume de $V=20$, $h$ doit être égale à $5 / 3$ + \end{itemize} + \item Pour calculer le volume, on a + \begin{eqnarray*} + V &=& h\times x \times 4 \\ + 20 &=& h\times x \times 4 \\ + x &=& \frac{20}{h\times 4} = \frac{5}{h} + \end{eqnarray*} + \item Pour calculer la surface totale, on ajoute la surface de chaque face. On a donc le calcul suivant + \begin{eqnarray*} + S(x) &=& x\times h \times 2 + x\times4\times2 + h\times 4\times 2\\ + S(x) &=& x\times \frac{5}{x} \times 2 + x\times4\times2 + \frac{5}{x}\times 4\times 2\\ + S(x) &=& 8x + 10 + \frac{40}{x} + \end{eqnarray*} + \item Pour trouver cette nouvelle forme, on met chaque élément sur le même dénominateur + \begin{eqnarray*} + S(x) &=& 8x + 10 + \frac{40}{x}\\ + S(x) &=& \frac{8x\times x}{x} + \frac{10\times x}{x} + \frac{40}{x}\\ + S(x) &=& \frac{8x^2 + 10x + 40}{x} + \end{eqnarray*} + \item On retrouve la formule $\frac{u}{v}$ à dériver + \[ + u(x) = 8x^2 + 10x + 40 \Rightarrow u'(x) = 16x + 10 + \] + \[ + v(x) = x \Rightarrow v'(x) = 1 + \] + Donc au numérateur on obtient + \begin{eqnarray*} + u'(x)\times v(x) - u(x)\times v'(x) &=& (16x + 10)\times x - (8x^2 + 10x + 40)\times 1\\ + &=& 8x^2 - 40 + \end{eqnarray*} + Donc + \[ + S'(x) = \frac{8x^2 - 40}{x^2} + \] + \item Tableau de variations de $S$ + + \begin{itemize} + \item Valeur interdite: $x^2 = 0 \equiv x = 0$ + \item Signe de $8x^2 - 40$: c'est un polynôme du 2e degré + \[ + \Delta = 1280 > 0 + \] + Il y a donc 2 racines + \[ + x_1 = - 2.23606797749979 \qquad + x_2 = 2.23606797749979 + \] + Et on sait que $8x^2 - 40$ est du signe de $a$ donc positif en dehors des racines + \item Le dénominateur $x^2$ est toujours positif. + \item Tableau de variations + + \begin{tikzpicture}[baseline=(a.north)] + \tkzTabInit[lgt=3,espcl=3]{$x$/1,$8x^2 - 40$/1, $x^2$/1, $S'$/1, $S$/2}{$0$, $- 2.23606797749979$, $10$} + \tkzTabLine{d,-, z, +, } + \tkzTabLine{d,+, , +, } + \tkzTabLine{d,-, z, +, } + \tkzTabVar{D+/ , -/ , +/ } + \end{tikzpicture} + + \end{itemize} + \item On a donc une surface minimal pour $x=2.23606797749979$ et $h = 11.18033988749895$. + \end{enumerate} +\end{solution} + +%%% Local Variables: +%%% mode: latex +%%% TeX-master: "master" +%%% End: + +\begin{exercise}[subtitle={Bassin}] + Le tour d'un bassin au niveau du sol présente deux axes de symétrie : l’axe des abscisses et la droite d’équation $x=4$. Il est obtenu par symétrie de la courbe $\mathcal{C}_f$ sur $\intFF{0}{4}$ où $f$ est la fonction définie par + + \[ + f(x) = \left(- x^{2} + 9.0 x - 9.0\right) e^{- x} + 9.0 + \] + On admet que sur $\intFF{0}{4}$ la fonction $f$ est positive. + \begin{enumerate} + \item Sur un repère, tracer l'allure de la courbe $\mathcal{C}_f$, les axes de symétries puis compléter pour dessiner la forme du bassin. + \item Montrer que la fonction $f$ admet comme primitive sur $\R$ la fonction $F$ définie par + \[ + F(x) = 9.0 x + \left( x^{2} - 7.0 x + 2.0\right) e^{- x} + \] + \item Calculer la quantité $\ds \int_0^4 f(x) \; dx$, vous donnerez le résultat sous forme exacte. Interpréter le résultat et reportez cette quantité sur le graphique. + \item On considère que l'échelle de votre graphique est de 1unité pour 15m. Calculer l'aire du bassin. Vous donnerez un résultat arrondi au $m^2$ près. + \end{enumerate} +\end{exercise} + +\begin{solution} + \begin{enumerate} + \item + \begin{tikzpicture}[baseline=(a.north), xscale=1, yscale=0.5] + \tkzInit[xmin=0,xmax=5,xstep=1, + ymin=0,ymax=10,ystep=1] + \tkzGrid + \tkzAxeXY + \tkzFct[domain=0:10,color=red,very thick]% + { (-x**2 + 9.0*x - 9.0)*exp(-x) + 9.0 }; + \end{tikzpicture} + \item Il faut dériver $F(x)$ et vérifier que $F'(x) = f(x)$. + \item $\ds \int_0^4 f(x) \; dx = F(4) - F(0) = 34.0 - \frac{10.0}{e^{4}}$ + \item La quantité calculée à la question précédente se retrouve 4fois pour former le bassin. Il faut ensuite prendre en compte l'échelle, comme 1unité de longueur correspond à 15m, une unité d'air correspond à $15\times15 = 225m^2$. Ainsi l'aire du bassin est égale à + \[ + (34.0 - \frac{10.0}{e^{4}})\times 4 \times 15^2 = 30435.00000 + \] + + \end{enumerate} +\end{solution} + +%%% Local Variables: +%%% mode: latex +%%% TeX-master: "master" +%%% End: + +\begin{exercise}[subtitle={Stylos}] + \emph{Les parties {\rm A} et {\rm B} de cet exercice sont indépendantes.} + + \bigskip + + \begin{minipage}{0.6\linewidth} + \textbf{Partie A} + + \medskip + + Deux ateliers A et B fabriquent des stylos pour une entreprise. + + L'atelier A fabrique 27.0\,\% des stylos, et parmi ceux-là, 39.0\,\% possèdent un défaut de fabrication. + + De plus, 4.0\,\% des stylos possèdent un défaut de fabrication et sortent de l'atelier B. + + Un stylo est prélevé au hasard dans le stock de l'entreprise. + + On considère les évènements suivants: + + \begin{itemize} + \item A : \og Le stylo a été fabriqué par l'atelier A \fg + \item B : \og Le stylo a été fabriqué par l'atelier B \fg + \item D : \og Le stylo possède un défaut de fabrication \fg + \end{itemize} + \end{minipage} + \begin{minipage}{0.4\linewidth} + \begin{center} + \begin{tikzpicture}[sloped] + \node {.} + child {node {$A$} + child {node {$D$} + edge from parent + node[above] {...} + } + child {node {$\overline{D}$} + edge from parent + node[above] {...} + } + edge from parent + node[above] {...} + } + child[missing] {} + child { node {$B$} + child {node {$D$} + edge from parent + node[above] {...} + } + child {node {$\overline{D}$} + edge from parent + node[above] {...} + } + edge from parent + node[above] {...} + } ; + \end{tikzpicture} + \end{center} + \end{minipage} + + \medskip + + \begin{enumerate} + \item Compléter l'arbre de probabilité ci-contre + \item Interpréter puis donner les probabilités $P(A)$, $P(B)$, $P_A(D)$ et $P(B \cap D)$. + + \item + \begin{enumerate} + \item Calculer la probabilité qu'un stylo provienne de l'atelier A et possède un défaut de fabrication. + \item En déduire que la probabilité qu'un stylo possède un défaut de fabrication est de $0.15$. + \end{enumerate} + \item On prélève un stylo au hasard avec un défaut. Quelle est la probabilité qu'il vienne de l'atelier A? + \end{enumerate} + + \bigskip + + \textbf{Partie B} + \medskip + + Dans cette partie, on suppose que 15.0\,\% des stylos possèdent un défaut de fabrication. + + L'entreprise confectionne des paquets contenant chacun $4$~stylos. + + Le fait qu'un stylo possède ou non un défaut de fabrication est indépendant des autres stylos. + + On appelle $X$ la variable aléatoire donnant pour un paquet le nombre de stylos qui possèdent un défaut de fabrication. + + On admet que la variable aléatoire $X$ suit une loi binomiale. + + \medskip + + \begin{enumerate} + \setcounter{enumi}{4} + \item Avec quelle loi peut-on modéliser $X$. Préciser les paramètres. + \item Calculer et interpréter la probabilité $P(X = 9)$. + \item Le directeur de l'entreprise affirme qu'il y a plus d'une chance sur deux qu'un paquet ne comporte aucun stylo défectueux. A-t-il raison ? + \item Combien de stylos peut-on espérer avoir en moyenne? + \end{enumerate} + \pagebreak +\end{exercise} + +\begin{solution} + \begin{enumerate} + \item + \begin{center} + \begin{tikzpicture}[sloped] + \node {.} + child {node {$A$} + child {node {$D$} + edge from parent + node[above] {0.39} + } + child {node {$\overline{D}$} + edge from parent + node[above] {0.61} + } + edge from parent + node[above] {0.27} + } + child[missing] {} + child { node {$B$} + child {node {$D$} + edge from parent + node[above] {0.05} + } + child {node {$\overline{D}$} + edge from parent + node[above] {0.95} + } + edge from parent + node[above] {0.73} + } ; + \end{tikzpicture} + \end{center} + \item + \begin{itemize} + \item Probabilité que le stylo vienne de l'atelier A + \[ + P(A) = 0.27 + \] + \item Probabilité que le stylo vienne de l'atelier B + \[ + P(B) = 0.73 + \] + \item Probabilité que le stylo ait un défaut sachant qu'il vient de l'atelier A. + \[ + P_A(D) = 0.39 + \] + \item Probabilité que le stylo vienne de l'atelier B et qu'il ait un défaut. + \[ + P(D \cap D) = 0.04 + \] + \end{itemize} + \item + \begin{enumerate} + \item Probabilité qu'un stylo vienne de l'atelier A et qu'il ait un defaut + \[ + P(A\cap D) = P(A) \times P_A(D) = 0.27 \times 0.39 = 0.11 + \] + \item Probabilité que le stylo ai un défaut de fabrication. + \[ + P(D) = P(A\cap D) + P(B\cap D) = 0.11 + 0.04 = 0.15 + \] + \end{enumerate} + \item Probabilité qu'il vienne de l'atelier A sachant qu'il a un defaut + \[ + P_D(A) = \frac{P(A\cap D)}{P(D)} = \frac{0.11}{0.15} = 0.73 + \] + \item $X$ peut être modélisée par une loi binomiale de paramètres $n=17$ et $p=0.15$. + \item (\textit{par de correction automatique disponible pour le résultat final} + \[ + P(X = 9) = \coefBino{17}{9}\times 0.15^{9} \times 0.85^{8} + \] + \item (\textit{par de correction automatique disponible pour le résultat final} + + Il faut calculer la probabilité qu'il y ait 0 stylo avec un defaut. + \[ + P(X = 0) = \coefBino{17}{0}\times 0.15^{0} \times 0.85^{17} + \] + Puis comparer ce nombre à 0,5. + \item Il faut calculer l'espérance + \[ + E[X] = n\times p = 17 \times 0.15 = 2.55 + \] + \end{enumerate} +\end{solution} + +\end{document} + +%%% Local Variables: +%%% mode: latex +%%% TeX-master: "master" +%%% End: diff --git a/Complementaire/DM/2105_DM1/05_2105_DM1.tex b/Complementaire/DM/2105_DM1/05_2105_DM1.tex new file mode 100644 index 0000000..8dc64bf --- /dev/null +++ b/Complementaire/DM/2105_DM1/05_2105_DM1.tex @@ -0,0 +1,367 @@ +\documentclass[a4paper,10pt]{article} +\usepackage{myXsim} + +% Title Page +\title{DM1 \hfill GERMAIN Margot} +\tribe{Maths complémentaire} +\date{\hfillÀ render pour le jeudi 27 mai} + +\xsimsetup{ + solution/print = false +} + +\begin{document} +\maketitle + +Les valeurs des exercices sont générés automatiquement. Si une valeur a un nombre adhérant de chiffres après la virgule, vous pouvez l'arrondir à l'entier le plus proche. + +\begin{exercise}[subtitle={Optimisation de matière}] + \begin{minipage}{0.6\textwidth} + On se propose de fabriquer avec le moins de tôle possible une citerne fermée en forme de parallélépipède rectangle dont le volume intérieur doit être de $35m^3$. La longueur est aussi fixée à $5m$ par le cahier des charges. + + On peut donc faire varier uniquement la largeur (notée $x$) et la hauteur (notée $h$) de la cuve. + \end{minipage} + \hfill + \begin{minipage}{0.3\textwidth} + \begin{tikzpicture} + \pgfmathsetmacro{\cubex}{3} + \pgfmathsetmacro{\cubey}{1} + \pgfmathsetmacro{\cubez}{2} + \draw[black,fill=gray] (0,0,0) -- ++(-\cubex,0,0) -- ++(0,-\cubey,0) node [midway, left] {$h$} -- ++(\cubex,0,0) node [midway, below] {$x$} -- cycle; + \draw[black,fill=gray] (0,0,0) -- ++(0,0,-\cubez) -- ++(0,-\cubey,0) -- ++(0,0,\cubez) node [midway, right] {$5m$} -- cycle; + \draw[black,fill=gray] (0,0,0) -- ++(-\cubex,0,0) -- ++(0,0,-\cubez) -- ++(\cubex,0,0) -- cycle; + \end{tikzpicture} + \end{minipage} + + \begin{enumerate} + \item Expliquer pourquoi quand la largeur $x$ change, la hauteur $h$ doit elle aussi changer pour respecter les contraintes. + \item Démontrer que l'on doit avoir $h = \dfrac{7}{x}$. + \item On note $S(x)$ l'aire totale de la citerne (c'est à dire la somme des aires des six faces). Montrer que l'on peut écrire + \[ + S(x) = 10x + 14 + \frac{70}{x} + \] + \item Démontrer que + \[ + S(x) = \frac{10x^2 + 14x + 70}{x} + \] + \item Démontrer que + \[ + S'(x) = \frac{10x^2 - 70}{x^2} + \] + \item En déduire le tableau de variation de $S(x)$ sur $\intOF{0}{10}$. + \item Déterminer les valeurs de $x$ et $h$ correspondant à une utilisation minimal de tôle. + \end{enumerate} +\end{exercise} + +\begin{solution} + \begin{enumerate} + \item Le volume étant fixe si l'on fait varier $x$, $h$ doit aussi varier. + \begin{itemize} + \item Si $x = 2$ alors conserver un volume de $V=35$, $h$ doit être égale à $7 / 2$ + \item Si $x = 3$ alors conserver un volume de $V=35$, $h$ doit être égale à $7 / 3$ + \end{itemize} + \item Pour calculer le volume, on a + \begin{eqnarray*} + V &=& h\times x \times 5 \\ + 35 &=& h\times x \times 5 \\ + x &=& \frac{35}{h\times 5} = \frac{7}{h} + \end{eqnarray*} + \item Pour calculer la surface totale, on ajoute la surface de chaque face. On a donc le calcul suivant + \begin{eqnarray*} + S(x) &=& x\times h \times 2 + x\times5\times2 + h\times 5\times 2\\ + S(x) &=& x\times \frac{7}{x} \times 2 + x\times5\times2 + \frac{7}{x}\times 5\times 2\\ + S(x) &=& 10x + 14 + \frac{70}{x} + \end{eqnarray*} + \item Pour trouver cette nouvelle forme, on met chaque élément sur le même dénominateur + \begin{eqnarray*} + S(x) &=& 10x + 14 + \frac{70}{x}\\ + S(x) &=& \frac{10x\times x}{x} + \frac{14\times x}{x} + \frac{70}{x}\\ + S(x) &=& \frac{10x^2 + 14x + 70}{x} + \end{eqnarray*} + \item On retrouve la formule $\frac{u}{v}$ à dériver + \[ + u(x) = 10x^2 + 14x + 70 \Rightarrow u'(x) = 20x + 14 + \] + \[ + v(x) = x \Rightarrow v'(x) = 1 + \] + Donc au numérateur on obtient + \begin{eqnarray*} + u'(x)\times v(x) - u(x)\times v'(x) &=& (20x + 14)\times x - (10x^2 + 14x + 70)\times 1\\ + &=& 10x^2 - 70 + \end{eqnarray*} + Donc + \[ + S'(x) = \frac{10x^2 - 70}{x^2} + \] + \item Tableau de variations de $S$ + + \begin{itemize} + \item Valeur interdite: $x^2 = 0 \equiv x = 0$ + \item Signe de $10x^2 - 70$: c'est un polynôme du 2e degré + \[ + \Delta = 2800 > 0 + \] + Il y a donc 2 racines + \[ + x_1 = - 2.6457513110645907 \qquad + x_2 = 2.6457513110645907 + \] + Et on sait que $10x^2 - 70$ est du signe de $a$ donc positif en dehors des racines + \item Le dénominateur $x^2$ est toujours positif. + \item Tableau de variations + + \begin{tikzpicture}[baseline=(a.north)] + \tkzTabInit[lgt=3,espcl=3]{$x$/1,$10x^2 - 70$/1, $x^2$/1, $S'$/1, $S$/2}{$0$, $- 2.6457513110645907$, $10$} + \tkzTabLine{d,-, z, +, } + \tkzTabLine{d,+, , +, } + \tkzTabLine{d,-, z, +, } + \tkzTabVar{D+/ , -/ , +/ } + \end{tikzpicture} + + \end{itemize} + \item On a donc une surface minimal pour $x=2.6457513110645907$ et $h = 18.5202591774521349$. + \end{enumerate} +\end{solution} + +%%% Local Variables: +%%% mode: latex +%%% TeX-master: "master" +%%% End: + +\begin{exercise}[subtitle={Bassin}] + Le tour d'un bassin au niveau du sol présente deux axes de symétrie : l’axe des abscisses et la droite d’équation $x=4$. Il est obtenu par symétrie de la courbe $\mathcal{C}_f$ sur $\intFF{0}{4}$ où $f$ est la fonction définie par + + \[ + f(x) = \left(- x^{2} + 8.0 x - 1.1\right) e^{- x} + 1.1 + \] + On admet que sur $\intFF{0}{4}$ la fonction $f$ est positive. + \begin{enumerate} + \item Sur un repère, tracer l'allure de la courbe $\mathcal{C}_f$, les axes de symétries puis compléter pour dessiner la forme du bassin. + \item Montrer que la fonction $f$ admet comme primitive sur $\R$ la fonction $F$ définie par + \[ + F(x) = 1.1 x + \left( x^{2} - 6.0 x - 4.9\right) e^{- x} + \] + \item Calculer la quantité $\ds \int_0^4 f(x) \; dx$, vous donnerez le résultat sous forme exacte. Interpréter le résultat et reportez cette quantité sur le graphique. + \item On considère que l'échelle de votre graphique est de 1unité pour 15m. Calculer l'aire du bassin. Vous donnerez un résultat arrondi au $m^2$ près. + \end{enumerate} +\end{exercise} + +\begin{solution} + \begin{enumerate} + \item + \begin{tikzpicture}[baseline=(a.north), xscale=1, yscale=0.5] + \tkzInit[xmin=0,xmax=5,xstep=1, + ymin=0,ymax=10,ystep=1] + \tkzGrid + \tkzAxeXY + \tkzFct[domain=0:10,color=red,very thick]% + { (-x**2 + 8.0*x - 1.1)*exp(-x) + 1.1 }; + \end{tikzpicture} + \item Il faut dériver $F(x)$ et vérifier que $F'(x) = f(x)$. + \item $\ds \int_0^4 f(x) \; dx = F(4) - F(0) = 9.3 - \frac{12.9}{e^{4}}$ + \item La quantité calculée à la question précédente se retrouve 4fois pour former le bassin. Il faut ensuite prendre en compte l'échelle, comme 1unité de longueur correspond à 15m, une unité d'air correspond à $15\times15 = 225m^2$. Ainsi l'aire du bassin est égale à + \[ + (9.3 - \frac{12.9}{e^{4}})\times 4 \times 15^2 = 8157.000000 + \] + + \end{enumerate} +\end{solution} + +%%% Local Variables: +%%% mode: latex +%%% TeX-master: "master" +%%% End: + +\begin{exercise}[subtitle={Stylos}] + \emph{Les parties {\rm A} et {\rm B} de cet exercice sont indépendantes.} + + \bigskip + + \begin{minipage}{0.6\linewidth} + \textbf{Partie A} + + \medskip + + Deux ateliers A et B fabriquent des stylos pour une entreprise. + + L'atelier A fabrique 81.0\,\% des stylos, et parmi ceux-là, 57.99999999999999\,\% possèdent un défaut de fabrication. + + De plus, 2.0\,\% des stylos possèdent un défaut de fabrication et sortent de l'atelier B. + + Un stylo est prélevé au hasard dans le stock de l'entreprise. + + On considère les évènements suivants: + + \begin{itemize} + \item A : \og Le stylo a été fabriqué par l'atelier A \fg + \item B : \og Le stylo a été fabriqué par l'atelier B \fg + \item D : \og Le stylo possède un défaut de fabrication \fg + \end{itemize} + \end{minipage} + \begin{minipage}{0.4\linewidth} + \begin{center} + \begin{tikzpicture}[sloped] + \node {.} + child {node {$A$} + child {node {$D$} + edge from parent + node[above] {...} + } + child {node {$\overline{D}$} + edge from parent + node[above] {...} + } + edge from parent + node[above] {...} + } + child[missing] {} + child { node {$B$} + child {node {$D$} + edge from parent + node[above] {...} + } + child {node {$\overline{D}$} + edge from parent + node[above] {...} + } + edge from parent + node[above] {...} + } ; + \end{tikzpicture} + \end{center} + \end{minipage} + + \medskip + + \begin{enumerate} + \item Compléter l'arbre de probabilité ci-contre + \item Interpréter puis donner les probabilités $P(A)$, $P(B)$, $P_A(D)$ et $P(B \cap D)$. + + \item + \begin{enumerate} + \item Calculer la probabilité qu'un stylo provienne de l'atelier A et possède un défaut de fabrication. + \item En déduire que la probabilité qu'un stylo possède un défaut de fabrication est de $0.49$. + \end{enumerate} + \item On prélève un stylo au hasard avec un défaut. Quelle est la probabilité qu'il vienne de l'atelier A? + \end{enumerate} + + \bigskip + + \textbf{Partie B} + \medskip + + Dans cette partie, on suppose que 49.0\,\% des stylos possèdent un défaut de fabrication. + + L'entreprise confectionne des paquets contenant chacun $4$~stylos. + + Le fait qu'un stylo possède ou non un défaut de fabrication est indépendant des autres stylos. + + On appelle $X$ la variable aléatoire donnant pour un paquet le nombre de stylos qui possèdent un défaut de fabrication. + + On admet que la variable aléatoire $X$ suit une loi binomiale. + + \medskip + + \begin{enumerate} + \setcounter{enumi}{4} + \item Avec quelle loi peut-on modéliser $X$. Préciser les paramètres. + \item Calculer et interpréter la probabilité $P(X = 14)$. + \item Le directeur de l'entreprise affirme qu'il y a plus d'une chance sur deux qu'un paquet ne comporte aucun stylo défectueux. A-t-il raison ? + \item Combien de stylos peut-on espérer avoir en moyenne? + \end{enumerate} + \pagebreak +\end{exercise} + +\begin{solution} + \begin{enumerate} + \item + \begin{center} + \begin{tikzpicture}[sloped] + \node {.} + child {node {$A$} + child {node {$D$} + edge from parent + node[above] {0.58} + } + child {node {$\overline{D}$} + edge from parent + node[above] {0.42} + } + edge from parent + node[above] {0.81} + } + child[missing] {} + child { node {$B$} + child {node {$D$} + edge from parent + node[above] {0.13} + } + child {node {$\overline{D}$} + edge from parent + node[above] {0.87} + } + edge from parent + node[above] {0.19} + } ; + \end{tikzpicture} + \end{center} + \item + \begin{itemize} + \item Probabilité que le stylo vienne de l'atelier A + \[ + P(A) = 0.81 + \] + \item Probabilité que le stylo vienne de l'atelier B + \[ + P(B) = 0.19 + \] + \item Probabilité que le stylo ait un défaut sachant qu'il vient de l'atelier A. + \[ + P_A(D) = 0.58 + \] + \item Probabilité que le stylo vienne de l'atelier B et qu'il ait un défaut. + \[ + P(D \cap D) = 0.02 + \] + \end{itemize} + \item + \begin{enumerate} + \item Probabilité qu'un stylo vienne de l'atelier A et qu'il ait un defaut + \[ + P(A\cap D) = P(A) \times P_A(D) = 0.81 \times 0.58 = 0.47 + \] + \item Probabilité que le stylo ai un défaut de fabrication. + \[ + P(D) = P(A\cap D) + P(B\cap D) = 0.47 + 0.02 = 0.49 + \] + \end{enumerate} + \item Probabilité qu'il vienne de l'atelier A sachant qu'il a un defaut + \[ + P_D(A) = \frac{P(A\cap D)}{P(D)} = \frac{0.47}{0.49} = 0.96 + \] + \item $X$ peut être modélisée par une loi binomiale de paramètres $n=20$ et $p=0.49$. + \item (\textit{par de correction automatique disponible pour le résultat final} + \[ + P(X = 14) = \coefBino{20}{14}\times 0.49^{14} \times 0.51^{6} + \] + \item (\textit{par de correction automatique disponible pour le résultat final} + + Il faut calculer la probabilité qu'il y ait 0 stylo avec un defaut. + \[ + P(X = 0) = \coefBino{20}{0}\times 0.49^{0} \times 0.51^{20} + \] + Puis comparer ce nombre à 0,5. + \item Il faut calculer l'espérance + \[ + E[X] = n\times p = 20 \times 0.49 = 9.8 + \] + \end{enumerate} +\end{solution} + +\end{document} + +%%% Local Variables: +%%% mode: latex +%%% TeX-master: "master" +%%% End: diff --git a/Complementaire/DM/2105_DM1/06_2105_DM1.tex b/Complementaire/DM/2105_DM1/06_2105_DM1.tex new file mode 100644 index 0000000..f8d37f3 --- /dev/null +++ b/Complementaire/DM/2105_DM1/06_2105_DM1.tex @@ -0,0 +1,367 @@ +\documentclass[a4paper,10pt]{article} +\usepackage{myXsim} + +% Title Page +\title{DM1 \hfill HOKELEKLI Damla} +\tribe{Maths complémentaire} +\date{\hfillÀ render pour le jeudi 27 mai} + +\xsimsetup{ + solution/print = false +} + +\begin{document} +\maketitle + +Les valeurs des exercices sont générés automatiquement. Si une valeur a un nombre adhérant de chiffres après la virgule, vous pouvez l'arrondir à l'entier le plus proche. + +\begin{exercise}[subtitle={Optimisation de matière}] + \begin{minipage}{0.6\textwidth} + On se propose de fabriquer avec le moins de tôle possible une citerne fermée en forme de parallélépipède rectangle dont le volume intérieur doit être de $10m^3$. La longueur est aussi fixée à $2m$ par le cahier des charges. + + On peut donc faire varier uniquement la largeur (notée $x$) et la hauteur (notée $h$) de la cuve. + \end{minipage} + \hfill + \begin{minipage}{0.3\textwidth} + \begin{tikzpicture} + \pgfmathsetmacro{\cubex}{3} + \pgfmathsetmacro{\cubey}{1} + \pgfmathsetmacro{\cubez}{2} + \draw[black,fill=gray] (0,0,0) -- ++(-\cubex,0,0) -- ++(0,-\cubey,0) node [midway, left] {$h$} -- ++(\cubex,0,0) node [midway, below] {$x$} -- cycle; + \draw[black,fill=gray] (0,0,0) -- ++(0,0,-\cubez) -- ++(0,-\cubey,0) -- ++(0,0,\cubez) node [midway, right] {$2m$} -- cycle; + \draw[black,fill=gray] (0,0,0) -- ++(-\cubex,0,0) -- ++(0,0,-\cubez) -- ++(\cubex,0,0) -- cycle; + \end{tikzpicture} + \end{minipage} + + \begin{enumerate} + \item Expliquer pourquoi quand la largeur $x$ change, la hauteur $h$ doit elle aussi changer pour respecter les contraintes. + \item Démontrer que l'on doit avoir $h = \dfrac{5}{x}$. + \item On note $S(x)$ l'aire totale de la citerne (c'est à dire la somme des aires des six faces). Montrer que l'on peut écrire + \[ + S(x) = 4x + 10 + \frac{20}{x} + \] + \item Démontrer que + \[ + S(x) = \frac{4x^2 + 10x + 20}{x} + \] + \item Démontrer que + \[ + S'(x) = \frac{4x^2 - 20}{x^2} + \] + \item En déduire le tableau de variation de $S(x)$ sur $\intOF{0}{10}$. + \item Déterminer les valeurs de $x$ et $h$ correspondant à une utilisation minimal de tôle. + \end{enumerate} +\end{exercise} + +\begin{solution} + \begin{enumerate} + \item Le volume étant fixe si l'on fait varier $x$, $h$ doit aussi varier. + \begin{itemize} + \item Si $x = 2$ alors conserver un volume de $V=10$, $h$ doit être égale à $5 / 2$ + \item Si $x = 3$ alors conserver un volume de $V=10$, $h$ doit être égale à $5 / 3$ + \end{itemize} + \item Pour calculer le volume, on a + \begin{eqnarray*} + V &=& h\times x \times 2 \\ + 10 &=& h\times x \times 2 \\ + x &=& \frac{10}{h\times 2} = \frac{5}{h} + \end{eqnarray*} + \item Pour calculer la surface totale, on ajoute la surface de chaque face. On a donc le calcul suivant + \begin{eqnarray*} + S(x) &=& x\times h \times 2 + x\times2\times2 + h\times 2\times 2\\ + S(x) &=& x\times \frac{5}{x} \times 2 + x\times2\times2 + \frac{5}{x}\times 2\times 2\\ + S(x) &=& 4x + 10 + \frac{20}{x} + \end{eqnarray*} + \item Pour trouver cette nouvelle forme, on met chaque élément sur le même dénominateur + \begin{eqnarray*} + S(x) &=& 4x + 10 + \frac{20}{x}\\ + S(x) &=& \frac{4x\times x}{x} + \frac{10\times x}{x} + \frac{20}{x}\\ + S(x) &=& \frac{4x^2 + 10x + 20}{x} + \end{eqnarray*} + \item On retrouve la formule $\frac{u}{v}$ à dériver + \[ + u(x) = 4x^2 + 10x + 20 \Rightarrow u'(x) = 8x + 10 + \] + \[ + v(x) = x \Rightarrow v'(x) = 1 + \] + Donc au numérateur on obtient + \begin{eqnarray*} + u'(x)\times v(x) - u(x)\times v'(x) &=& (8x + 10)\times x - (4x^2 + 10x + 20)\times 1\\ + &=& 4x^2 - 20 + \end{eqnarray*} + Donc + \[ + S'(x) = \frac{4x^2 - 20}{x^2} + \] + \item Tableau de variations de $S$ + + \begin{itemize} + \item Valeur interdite: $x^2 = 0 \equiv x = 0$ + \item Signe de $4x^2 - 20$: c'est un polynôme du 2e degré + \[ + \Delta = 320 > 0 + \] + Il y a donc 2 racines + \[ + x_1 = - 2.23606797749979 \qquad + x_2 = 2.23606797749979 + \] + Et on sait que $4x^2 - 20$ est du signe de $a$ donc positif en dehors des racines + \item Le dénominateur $x^2$ est toujours positif. + \item Tableau de variations + + \begin{tikzpicture}[baseline=(a.north)] + \tkzTabInit[lgt=3,espcl=3]{$x$/1,$4x^2 - 20$/1, $x^2$/1, $S'$/1, $S$/2}{$0$, $- 2.23606797749979$, $10$} + \tkzTabLine{d,-, z, +, } + \tkzTabLine{d,+, , +, } + \tkzTabLine{d,-, z, +, } + \tkzTabVar{D+/ , -/ , +/ } + \end{tikzpicture} + + \end{itemize} + \item On a donc une surface minimal pour $x=2.23606797749979$ et $h = 11.18033988749895$. + \end{enumerate} +\end{solution} + +%%% Local Variables: +%%% mode: latex +%%% TeX-master: "master" +%%% End: + +\begin{exercise}[subtitle={Bassin}] + Le tour d'un bassin au niveau du sol présente deux axes de symétrie : l’axe des abscisses et la droite d’équation $x=4$. Il est obtenu par symétrie de la courbe $\mathcal{C}_f$ sur $\intFF{0}{4}$ où $f$ est la fonction définie par + + \[ + f(x) = \left(- x^{2} + 5.6 x - 1.6\right) e^{- x} + 1.6 + \] + On admet que sur $\intFF{0}{4}$ la fonction $f$ est positive. + \begin{enumerate} + \item Sur un repère, tracer l'allure de la courbe $\mathcal{C}_f$, les axes de symétries puis compléter pour dessiner la forme du bassin. + \item Montrer que la fonction $f$ admet comme primitive sur $\R$ la fonction $F$ définie par + \[ + F(x) = 1.6 x + \left( x^{2} - 3.6 x - 2.0\right) e^{- x} + \] + \item Calculer la quantité $\ds \int_0^4 f(x) \; dx$, vous donnerez le résultat sous forme exacte. Interpréter le résultat et reportez cette quantité sur le graphique. + \item On considère que l'échelle de votre graphique est de 1unité pour 15m. Calculer l'aire du bassin. Vous donnerez un résultat arrondi au $m^2$ près. + \end{enumerate} +\end{exercise} + +\begin{solution} + \begin{enumerate} + \item + \begin{tikzpicture}[baseline=(a.north), xscale=1, yscale=0.5] + \tkzInit[xmin=0,xmax=5,xstep=1, + ymin=0,ymax=10,ystep=1] + \tkzGrid + \tkzAxeXY + \tkzFct[domain=0:10,color=red,very thick]% + { (-x**2 + 5.6*x - 1.6)*exp(-x) + 1.6 }; + \end{tikzpicture} + \item Il faut dériver $F(x)$ et vérifier que $F'(x) = f(x)$. + \item $\ds \int_0^4 f(x) \; dx = F(4) - F(0) = 8.4 - \frac{0.399999999999999}{e^{4}}$ + \item La quantité calculée à la question précédente se retrouve 4fois pour former le bassin. Il faut ensuite prendre en compte l'échelle, comme 1unité de longueur correspond à 15m, une unité d'air correspond à $15\times15 = 225m^2$. Ainsi l'aire du bassin est égale à + \[ + (8.4 - \frac{0.399999999999999}{e^{4}})\times 4 \times 15^2 = 7553.000000 + \] + + \end{enumerate} +\end{solution} + +%%% Local Variables: +%%% mode: latex +%%% TeX-master: "master" +%%% End: + +\begin{exercise}[subtitle={Stylos}] + \emph{Les parties {\rm A} et {\rm B} de cet exercice sont indépendantes.} + + \bigskip + + \begin{minipage}{0.6\linewidth} + \textbf{Partie A} + + \medskip + + Deux ateliers A et B fabriquent des stylos pour une entreprise. + + L'atelier A fabrique 47.0\,\% des stylos, et parmi ceux-là, 25.0\,\% possèdent un défaut de fabrication. + + De plus, 23.0\,\% des stylos possèdent un défaut de fabrication et sortent de l'atelier B. + + Un stylo est prélevé au hasard dans le stock de l'entreprise. + + On considère les évènements suivants: + + \begin{itemize} + \item A : \og Le stylo a été fabriqué par l'atelier A \fg + \item B : \og Le stylo a été fabriqué par l'atelier B \fg + \item D : \og Le stylo possède un défaut de fabrication \fg + \end{itemize} + \end{minipage} + \begin{minipage}{0.4\linewidth} + \begin{center} + \begin{tikzpicture}[sloped] + \node {.} + child {node {$A$} + child {node {$D$} + edge from parent + node[above] {...} + } + child {node {$\overline{D}$} + edge from parent + node[above] {...} + } + edge from parent + node[above] {...} + } + child[missing] {} + child { node {$B$} + child {node {$D$} + edge from parent + node[above] {...} + } + child {node {$\overline{D}$} + edge from parent + node[above] {...} + } + edge from parent + node[above] {...} + } ; + \end{tikzpicture} + \end{center} + \end{minipage} + + \medskip + + \begin{enumerate} + \item Compléter l'arbre de probabilité ci-contre + \item Interpréter puis donner les probabilités $P(A)$, $P(B)$, $P_A(D)$ et $P(B \cap D)$. + + \item + \begin{enumerate} + \item Calculer la probabilité qu'un stylo provienne de l'atelier A et possède un défaut de fabrication. + \item En déduire que la probabilité qu'un stylo possède un défaut de fabrication est de $0.35$. + \end{enumerate} + \item On prélève un stylo au hasard avec un défaut. Quelle est la probabilité qu'il vienne de l'atelier A? + \end{enumerate} + + \bigskip + + \textbf{Partie B} + \medskip + + Dans cette partie, on suppose que 35.0\,\% des stylos possèdent un défaut de fabrication. + + L'entreprise confectionne des paquets contenant chacun $4$~stylos. + + Le fait qu'un stylo possède ou non un défaut de fabrication est indépendant des autres stylos. + + On appelle $X$ la variable aléatoire donnant pour un paquet le nombre de stylos qui possèdent un défaut de fabrication. + + On admet que la variable aléatoire $X$ suit une loi binomiale. + + \medskip + + \begin{enumerate} + \setcounter{enumi}{4} + \item Avec quelle loi peut-on modéliser $X$. Préciser les paramètres. + \item Calculer et interpréter la probabilité $P(X = 17)$. + \item Le directeur de l'entreprise affirme qu'il y a plus d'une chance sur deux qu'un paquet ne comporte aucun stylo défectueux. A-t-il raison ? + \item Combien de stylos peut-on espérer avoir en moyenne? + \end{enumerate} + \pagebreak +\end{exercise} + +\begin{solution} + \begin{enumerate} + \item + \begin{center} + \begin{tikzpicture}[sloped] + \node {.} + child {node {$A$} + child {node {$D$} + edge from parent + node[above] {0.25} + } + child {node {$\overline{D}$} + edge from parent + node[above] {0.75} + } + edge from parent + node[above] {0.47} + } + child[missing] {} + child { node {$B$} + child {node {$D$} + edge from parent + node[above] {0.44} + } + child {node {$\overline{D}$} + edge from parent + node[above] {0.56} + } + edge from parent + node[above] {0.53} + } ; + \end{tikzpicture} + \end{center} + \item + \begin{itemize} + \item Probabilité que le stylo vienne de l'atelier A + \[ + P(A) = 0.47 + \] + \item Probabilité que le stylo vienne de l'atelier B + \[ + P(B) = 0.53 + \] + \item Probabilité que le stylo ait un défaut sachant qu'il vient de l'atelier A. + \[ + P_A(D) = 0.25 + \] + \item Probabilité que le stylo vienne de l'atelier B et qu'il ait un défaut. + \[ + P(D \cap D) = 0.23 + \] + \end{itemize} + \item + \begin{enumerate} + \item Probabilité qu'un stylo vienne de l'atelier A et qu'il ait un defaut + \[ + P(A\cap D) = P(A) \times P_A(D) = 0.47 \times 0.25 = 0.12 + \] + \item Probabilité que le stylo ai un défaut de fabrication. + \[ + P(D) = P(A\cap D) + P(B\cap D) = 0.12 + 0.23 = 0.35 + \] + \end{enumerate} + \item Probabilité qu'il vienne de l'atelier A sachant qu'il a un defaut + \[ + P_D(A) = \frac{P(A\cap D)}{P(D)} = \frac{0.12}{0.35} = 0.34 + \] + \item $X$ peut être modélisée par une loi binomiale de paramètres $n=19$ et $p=0.35$. + \item (\textit{par de correction automatique disponible pour le résultat final} + \[ + P(X = 17) = \coefBino{19}{17}\times 0.35^{17} \times 0.65^{2} + \] + \item (\textit{par de correction automatique disponible pour le résultat final} + + Il faut calculer la probabilité qu'il y ait 0 stylo avec un defaut. + \[ + P(X = 0) = \coefBino{19}{0}\times 0.35^{0} \times 0.65^{19} + \] + Puis comparer ce nombre à 0,5. + \item Il faut calculer l'espérance + \[ + E[X] = n\times p = 19 \times 0.35 = 6.65 + \] + \end{enumerate} +\end{solution} + +\end{document} + +%%% Local Variables: +%%% mode: latex +%%% TeX-master: "master" +%%% End: diff --git a/Complementaire/DM/2105_DM1/07_2105_DM1.tex b/Complementaire/DM/2105_DM1/07_2105_DM1.tex new file mode 100644 index 0000000..7397af0 --- /dev/null +++ b/Complementaire/DM/2105_DM1/07_2105_DM1.tex @@ -0,0 +1,367 @@ +\documentclass[a4paper,10pt]{article} +\usepackage{myXsim} + +% Title Page +\title{DM1 \hfill KICHENASSAMY Kévin} +\tribe{Maths complémentaire} +\date{\hfillÀ render pour le jeudi 27 mai} + +\xsimsetup{ + solution/print = false +} + +\begin{document} +\maketitle + +Les valeurs des exercices sont générés automatiquement. Si une valeur a un nombre adhérant de chiffres après la virgule, vous pouvez l'arrondir à l'entier le plus proche. + +\begin{exercise}[subtitle={Optimisation de matière}] + \begin{minipage}{0.6\textwidth} + On se propose de fabriquer avec le moins de tôle possible une citerne fermée en forme de parallélépipède rectangle dont le volume intérieur doit être de $16m^3$. La longueur est aussi fixée à $4m$ par le cahier des charges. + + On peut donc faire varier uniquement la largeur (notée $x$) et la hauteur (notée $h$) de la cuve. + \end{minipage} + \hfill + \begin{minipage}{0.3\textwidth} + \begin{tikzpicture} + \pgfmathsetmacro{\cubex}{3} + \pgfmathsetmacro{\cubey}{1} + \pgfmathsetmacro{\cubez}{2} + \draw[black,fill=gray] (0,0,0) -- ++(-\cubex,0,0) -- ++(0,-\cubey,0) node [midway, left] {$h$} -- ++(\cubex,0,0) node [midway, below] {$x$} -- cycle; + \draw[black,fill=gray] (0,0,0) -- ++(0,0,-\cubez) -- ++(0,-\cubey,0) -- ++(0,0,\cubez) node [midway, right] {$4m$} -- cycle; + \draw[black,fill=gray] (0,0,0) -- ++(-\cubex,0,0) -- ++(0,0,-\cubez) -- ++(\cubex,0,0) -- cycle; + \end{tikzpicture} + \end{minipage} + + \begin{enumerate} + \item Expliquer pourquoi quand la largeur $x$ change, la hauteur $h$ doit elle aussi changer pour respecter les contraintes. + \item Démontrer que l'on doit avoir $h = \dfrac{4}{x}$. + \item On note $S(x)$ l'aire totale de la citerne (c'est à dire la somme des aires des six faces). Montrer que l'on peut écrire + \[ + S(x) = 8x + 8 + \frac{32}{x} + \] + \item Démontrer que + \[ + S(x) = \frac{8x^2 + 8x + 32}{x} + \] + \item Démontrer que + \[ + S'(x) = \frac{8x^2 - 32}{x^2} + \] + \item En déduire le tableau de variation de $S(x)$ sur $\intOF{0}{10}$. + \item Déterminer les valeurs de $x$ et $h$ correspondant à une utilisation minimal de tôle. + \end{enumerate} +\end{exercise} + +\begin{solution} + \begin{enumerate} + \item Le volume étant fixe si l'on fait varier $x$, $h$ doit aussi varier. + \begin{itemize} + \item Si $x = 2$ alors conserver un volume de $V=16$, $h$ doit être égale à $4 / 2$ + \item Si $x = 3$ alors conserver un volume de $V=16$, $h$ doit être égale à $4 / 3$ + \end{itemize} + \item Pour calculer le volume, on a + \begin{eqnarray*} + V &=& h\times x \times 4 \\ + 16 &=& h\times x \times 4 \\ + x &=& \frac{16}{h\times 4} = \frac{4}{h} + \end{eqnarray*} + \item Pour calculer la surface totale, on ajoute la surface de chaque face. On a donc le calcul suivant + \begin{eqnarray*} + S(x) &=& x\times h \times 2 + x\times4\times2 + h\times 4\times 2\\ + S(x) &=& x\times \frac{4}{x} \times 2 + x\times4\times2 + \frac{4}{x}\times 4\times 2\\ + S(x) &=& 8x + 8 + \frac{32}{x} + \end{eqnarray*} + \item Pour trouver cette nouvelle forme, on met chaque élément sur le même dénominateur + \begin{eqnarray*} + S(x) &=& 8x + 8 + \frac{32}{x}\\ + S(x) &=& \frac{8x\times x}{x} + \frac{8\times x}{x} + \frac{32}{x}\\ + S(x) &=& \frac{8x^2 + 8x + 32}{x} + \end{eqnarray*} + \item On retrouve la formule $\frac{u}{v}$ à dériver + \[ + u(x) = 8x^2 + 8x + 32 \Rightarrow u'(x) = 16x + 8 + \] + \[ + v(x) = x \Rightarrow v'(x) = 1 + \] + Donc au numérateur on obtient + \begin{eqnarray*} + u'(x)\times v(x) - u(x)\times v'(x) &=& (16x + 8)\times x - (8x^2 + 8x + 32)\times 1\\ + &=& 8x^2 - 32 + \end{eqnarray*} + Donc + \[ + S'(x) = \frac{8x^2 - 32}{x^2} + \] + \item Tableau de variations de $S$ + + \begin{itemize} + \item Valeur interdite: $x^2 = 0 \equiv x = 0$ + \item Signe de $8x^2 - 32$: c'est un polynôme du 2e degré + \[ + \Delta = 1024 > 0 + \] + Il y a donc 2 racines + \[ + x_1 = - 2 \qquad + x_2 = 2 + \] + Et on sait que $8x^2 - 32$ est du signe de $a$ donc positif en dehors des racines + \item Le dénominateur $x^2$ est toujours positif. + \item Tableau de variations + + \begin{tikzpicture}[baseline=(a.north)] + \tkzTabInit[lgt=3,espcl=3]{$x$/1,$8x^2 - 32$/1, $x^2$/1, $S'$/1, $S$/2}{$0$, $- 2$, $10$} + \tkzTabLine{d,-, z, +, } + \tkzTabLine{d,+, , +, } + \tkzTabLine{d,-, z, +, } + \tkzTabVar{D+/ , -/ , +/ } + \end{tikzpicture} + + \end{itemize} + \item On a donc une surface minimal pour $x=2$ et $h = 8$. + \end{enumerate} +\end{solution} + +%%% Local Variables: +%%% mode: latex +%%% TeX-master: "master" +%%% End: + +\begin{exercise}[subtitle={Bassin}] + Le tour d'un bassin au niveau du sol présente deux axes de symétrie : l’axe des abscisses et la droite d’équation $x=4$. Il est obtenu par symétrie de la courbe $\mathcal{C}_f$ sur $\intFF{0}{4}$ où $f$ est la fonction définie par + + \[ + f(x) = \left(- x^{2} + 0.4 x - 6.0\right) e^{- x} + 6.0 + \] + On admet que sur $\intFF{0}{4}$ la fonction $f$ est positive. + \begin{enumerate} + \item Sur un repère, tracer l'allure de la courbe $\mathcal{C}_f$, les axes de symétries puis compléter pour dessiner la forme du bassin. + \item Montrer que la fonction $f$ admet comme primitive sur $\R$ la fonction $F$ définie par + \[ + F(x) = 6.0 x + \left( x^{2} + 1.6 x + 7.6\right) e^{- x} + \] + \item Calculer la quantité $\ds \int_0^4 f(x) \; dx$, vous donnerez le résultat sous forme exacte. Interpréter le résultat et reportez cette quantité sur le graphique. + \item On considère que l'échelle de votre graphique est de 1unité pour 15m. Calculer l'aire du bassin. Vous donnerez un résultat arrondi au $m^2$ près. + \end{enumerate} +\end{exercise} + +\begin{solution} + \begin{enumerate} + \item + \begin{tikzpicture}[baseline=(a.north), xscale=1, yscale=0.5] + \tkzInit[xmin=0,xmax=5,xstep=1, + ymin=0,ymax=10,ystep=1] + \tkzGrid + \tkzAxeXY + \tkzFct[domain=0:10,color=red,very thick]% + { (-x**2 + 0.4*x - 6.0)*exp(-x) + 6.0 }; + \end{tikzpicture} + \item Il faut dériver $F(x)$ et vérifier que $F'(x) = f(x)$. + \item $\ds \int_0^4 f(x) \; dx = F(4) - F(0) = \frac{30.0}{e^{4}} + 16.4$ + \item La quantité calculée à la question précédente se retrouve 4fois pour former le bassin. Il faut ensuite prendre en compte l'échelle, comme 1unité de longueur correspond à 15m, une unité d'air correspond à $15\times15 = 225m^2$. Ainsi l'aire du bassin est égale à + \[ + (\frac{30.0}{e^{4}} + 16.4)\times 4 \times 15^2 = 15255.00000 + \] + + \end{enumerate} +\end{solution} + +%%% Local Variables: +%%% mode: latex +%%% TeX-master: "master" +%%% End: + +\begin{exercise}[subtitle={Stylos}] + \emph{Les parties {\rm A} et {\rm B} de cet exercice sont indépendantes.} + + \bigskip + + \begin{minipage}{0.6\linewidth} + \textbf{Partie A} + + \medskip + + Deux ateliers A et B fabriquent des stylos pour une entreprise. + + L'atelier A fabrique 84.0\,\% des stylos, et parmi ceux-là, 85.0\,\% possèdent un défaut de fabrication. + + De plus, 4.0\,\% des stylos possèdent un défaut de fabrication et sortent de l'atelier B. + + Un stylo est prélevé au hasard dans le stock de l'entreprise. + + On considère les évènements suivants: + + \begin{itemize} + \item A : \og Le stylo a été fabriqué par l'atelier A \fg + \item B : \og Le stylo a été fabriqué par l'atelier B \fg + \item D : \og Le stylo possède un défaut de fabrication \fg + \end{itemize} + \end{minipage} + \begin{minipage}{0.4\linewidth} + \begin{center} + \begin{tikzpicture}[sloped] + \node {.} + child {node {$A$} + child {node {$D$} + edge from parent + node[above] {...} + } + child {node {$\overline{D}$} + edge from parent + node[above] {...} + } + edge from parent + node[above] {...} + } + child[missing] {} + child { node {$B$} + child {node {$D$} + edge from parent + node[above] {...} + } + child {node {$\overline{D}$} + edge from parent + node[above] {...} + } + edge from parent + node[above] {...} + } ; + \end{tikzpicture} + \end{center} + \end{minipage} + + \medskip + + \begin{enumerate} + \item Compléter l'arbre de probabilité ci-contre + \item Interpréter puis donner les probabilités $P(A)$, $P(B)$, $P_A(D)$ et $P(B \cap D)$. + + \item + \begin{enumerate} + \item Calculer la probabilité qu'un stylo provienne de l'atelier A et possède un défaut de fabrication. + \item En déduire que la probabilité qu'un stylo possède un défaut de fabrication est de $0.75$. + \end{enumerate} + \item On prélève un stylo au hasard avec un défaut. Quelle est la probabilité qu'il vienne de l'atelier A? + \end{enumerate} + + \bigskip + + \textbf{Partie B} + \medskip + + Dans cette partie, on suppose que 75.0\,\% des stylos possèdent un défaut de fabrication. + + L'entreprise confectionne des paquets contenant chacun $4$~stylos. + + Le fait qu'un stylo possède ou non un défaut de fabrication est indépendant des autres stylos. + + On appelle $X$ la variable aléatoire donnant pour un paquet le nombre de stylos qui possèdent un défaut de fabrication. + + On admet que la variable aléatoire $X$ suit une loi binomiale. + + \medskip + + \begin{enumerate} + \setcounter{enumi}{4} + \item Avec quelle loi peut-on modéliser $X$. Préciser les paramètres. + \item Calculer et interpréter la probabilité $P(X = 14)$. + \item Le directeur de l'entreprise affirme qu'il y a plus d'une chance sur deux qu'un paquet ne comporte aucun stylo défectueux. A-t-il raison ? + \item Combien de stylos peut-on espérer avoir en moyenne? + \end{enumerate} + \pagebreak +\end{exercise} + +\begin{solution} + \begin{enumerate} + \item + \begin{center} + \begin{tikzpicture}[sloped] + \node {.} + child {node {$A$} + child {node {$D$} + edge from parent + node[above] {0.85} + } + child {node {$\overline{D}$} + edge from parent + node[above] {0.15} + } + edge from parent + node[above] {0.84} + } + child[missing] {} + child { node {$B$} + child {node {$D$} + edge from parent + node[above] {0.26} + } + child {node {$\overline{D}$} + edge from parent + node[above] {0.74} + } + edge from parent + node[above] {0.16} + } ; + \end{tikzpicture} + \end{center} + \item + \begin{itemize} + \item Probabilité que le stylo vienne de l'atelier A + \[ + P(A) = 0.84 + \] + \item Probabilité que le stylo vienne de l'atelier B + \[ + P(B) = 0.16 + \] + \item Probabilité que le stylo ait un défaut sachant qu'il vient de l'atelier A. + \[ + P_A(D) = 0.85 + \] + \item Probabilité que le stylo vienne de l'atelier B et qu'il ait un défaut. + \[ + P(D \cap D) = 0.04 + \] + \end{itemize} + \item + \begin{enumerate} + \item Probabilité qu'un stylo vienne de l'atelier A et qu'il ait un defaut + \[ + P(A\cap D) = P(A) \times P_A(D) = 0.84 \times 0.85 = 0.71 + \] + \item Probabilité que le stylo ai un défaut de fabrication. + \[ + P(D) = P(A\cap D) + P(B\cap D) = 0.71 + 0.04 = 0.75 + \] + \end{enumerate} + \item Probabilité qu'il vienne de l'atelier A sachant qu'il a un defaut + \[ + P_D(A) = \frac{P(A\cap D)}{P(D)} = \frac{0.71}{0.75} = 0.95 + \] + \item $X$ peut être modélisée par une loi binomiale de paramètres $n=17$ et $p=0.75$. + \item (\textit{par de correction automatique disponible pour le résultat final} + \[ + P(X = 14) = \coefBino{17}{14}\times 0.75^{14} \times 0.25^{3} + \] + \item (\textit{par de correction automatique disponible pour le résultat final} + + Il faut calculer la probabilité qu'il y ait 0 stylo avec un defaut. + \[ + P(X = 0) = \coefBino{17}{0}\times 0.75^{0} \times 0.25^{17} + \] + Puis comparer ce nombre à 0,5. + \item Il faut calculer l'espérance + \[ + E[X] = n\times p = 17 \times 0.75 = 12.75 + \] + \end{enumerate} +\end{solution} + +\end{document} + +%%% Local Variables: +%%% mode: latex +%%% TeX-master: "master" +%%% End: diff --git a/Complementaire/DM/2105_DM1/08_2105_DM1.tex b/Complementaire/DM/2105_DM1/08_2105_DM1.tex new file mode 100644 index 0000000..1e4bd76 --- /dev/null +++ b/Complementaire/DM/2105_DM1/08_2105_DM1.tex @@ -0,0 +1,367 @@ +\documentclass[a4paper,10pt]{article} +\usepackage{myXsim} + +% Title Page +\title{DM1 \hfill MATHIEU Allan} +\tribe{Maths complémentaire} +\date{\hfillÀ render pour le jeudi 27 mai} + +\xsimsetup{ + solution/print = false +} + +\begin{document} +\maketitle + +Les valeurs des exercices sont générés automatiquement. Si une valeur a un nombre adhérant de chiffres après la virgule, vous pouvez l'arrondir à l'entier le plus proche. + +\begin{exercise}[subtitle={Optimisation de matière}] + \begin{minipage}{0.6\textwidth} + On se propose de fabriquer avec le moins de tôle possible une citerne fermée en forme de parallélépipède rectangle dont le volume intérieur doit être de $40m^3$. La longueur est aussi fixée à $4m$ par le cahier des charges. + + On peut donc faire varier uniquement la largeur (notée $x$) et la hauteur (notée $h$) de la cuve. + \end{minipage} + \hfill + \begin{minipage}{0.3\textwidth} + \begin{tikzpicture} + \pgfmathsetmacro{\cubex}{3} + \pgfmathsetmacro{\cubey}{1} + \pgfmathsetmacro{\cubez}{2} + \draw[black,fill=gray] (0,0,0) -- ++(-\cubex,0,0) -- ++(0,-\cubey,0) node [midway, left] {$h$} -- ++(\cubex,0,0) node [midway, below] {$x$} -- cycle; + \draw[black,fill=gray] (0,0,0) -- ++(0,0,-\cubez) -- ++(0,-\cubey,0) -- ++(0,0,\cubez) node [midway, right] {$4m$} -- cycle; + \draw[black,fill=gray] (0,0,0) -- ++(-\cubex,0,0) -- ++(0,0,-\cubez) -- ++(\cubex,0,0) -- cycle; + \end{tikzpicture} + \end{minipage} + + \begin{enumerate} + \item Expliquer pourquoi quand la largeur $x$ change, la hauteur $h$ doit elle aussi changer pour respecter les contraintes. + \item Démontrer que l'on doit avoir $h = \dfrac{10}{x}$. + \item On note $S(x)$ l'aire totale de la citerne (c'est à dire la somme des aires des six faces). Montrer que l'on peut écrire + \[ + S(x) = 8x + 20 + \frac{80}{x} + \] + \item Démontrer que + \[ + S(x) = \frac{8x^2 + 20x + 80}{x} + \] + \item Démontrer que + \[ + S'(x) = \frac{8x^2 - 80}{x^2} + \] + \item En déduire le tableau de variation de $S(x)$ sur $\intOF{0}{10}$. + \item Déterminer les valeurs de $x$ et $h$ correspondant à une utilisation minimal de tôle. + \end{enumerate} +\end{exercise} + +\begin{solution} + \begin{enumerate} + \item Le volume étant fixe si l'on fait varier $x$, $h$ doit aussi varier. + \begin{itemize} + \item Si $x = 2$ alors conserver un volume de $V=40$, $h$ doit être égale à $10 / 2$ + \item Si $x = 3$ alors conserver un volume de $V=40$, $h$ doit être égale à $10 / 3$ + \end{itemize} + \item Pour calculer le volume, on a + \begin{eqnarray*} + V &=& h\times x \times 4 \\ + 40 &=& h\times x \times 4 \\ + x &=& \frac{40}{h\times 4} = \frac{10}{h} + \end{eqnarray*} + \item Pour calculer la surface totale, on ajoute la surface de chaque face. On a donc le calcul suivant + \begin{eqnarray*} + S(x) &=& x\times h \times 2 + x\times4\times2 + h\times 4\times 2\\ + S(x) &=& x\times \frac{10}{x} \times 2 + x\times4\times2 + \frac{10}{x}\times 4\times 2\\ + S(x) &=& 8x + 20 + \frac{80}{x} + \end{eqnarray*} + \item Pour trouver cette nouvelle forme, on met chaque élément sur le même dénominateur + \begin{eqnarray*} + S(x) &=& 8x + 20 + \frac{80}{x}\\ + S(x) &=& \frac{8x\times x}{x} + \frac{20\times x}{x} + \frac{80}{x}\\ + S(x) &=& \frac{8x^2 + 20x + 80}{x} + \end{eqnarray*} + \item On retrouve la formule $\frac{u}{v}$ à dériver + \[ + u(x) = 8x^2 + 20x + 80 \Rightarrow u'(x) = 16x + 20 + \] + \[ + v(x) = x \Rightarrow v'(x) = 1 + \] + Donc au numérateur on obtient + \begin{eqnarray*} + u'(x)\times v(x) - u(x)\times v'(x) &=& (16x + 20)\times x - (8x^2 + 20x + 80)\times 1\\ + &=& 8x^2 - 80 + \end{eqnarray*} + Donc + \[ + S'(x) = \frac{8x^2 - 80}{x^2} + \] + \item Tableau de variations de $S$ + + \begin{itemize} + \item Valeur interdite: $x^2 = 0 \equiv x = 0$ + \item Signe de $8x^2 - 80$: c'est un polynôme du 2e degré + \[ + \Delta = 2560 > 0 + \] + Il y a donc 2 racines + \[ + x_1 = - 3.1622776601683795 \qquad + x_2 = 3.1622776601683795 + \] + Et on sait que $8x^2 - 80$ est du signe de $a$ donc positif en dehors des racines + \item Le dénominateur $x^2$ est toujours positif. + \item Tableau de variations + + \begin{tikzpicture}[baseline=(a.north)] + \tkzTabInit[lgt=3,espcl=3]{$x$/1,$8x^2 - 80$/1, $x^2$/1, $S'$/1, $S$/2}{$0$, $- 3.1622776601683795$, $10$} + \tkzTabLine{d,-, z, +, } + \tkzTabLine{d,+, , +, } + \tkzTabLine{d,-, z, +, } + \tkzTabVar{D+/ , -/ , +/ } + \end{tikzpicture} + + \end{itemize} + \item On a donc une surface minimal pour $x=3.1622776601683795$ et $h = 31.6227766016837950$. + \end{enumerate} +\end{solution} + +%%% Local Variables: +%%% mode: latex +%%% TeX-master: "master" +%%% End: + +\begin{exercise}[subtitle={Bassin}] + Le tour d'un bassin au niveau du sol présente deux axes de symétrie : l’axe des abscisses et la droite d’équation $x=4$. Il est obtenu par symétrie de la courbe $\mathcal{C}_f$ sur $\intFF{0}{4}$ où $f$ est la fonction définie par + + \[ + f(x) = \left(- x^{2} + 6.1 x - 9.3\right) e^{- x} + 9.3 + \] + On admet que sur $\intFF{0}{4}$ la fonction $f$ est positive. + \begin{enumerate} + \item Sur un repère, tracer l'allure de la courbe $\mathcal{C}_f$, les axes de symétries puis compléter pour dessiner la forme du bassin. + \item Montrer que la fonction $f$ admet comme primitive sur $\R$ la fonction $F$ définie par + \[ + F(x) = 9.3 x + \left( x^{2} - 4.1 x + 5.2\right) e^{- x} + \] + \item Calculer la quantité $\ds \int_0^4 f(x) \; dx$, vous donnerez le résultat sous forme exacte. Interpréter le résultat et reportez cette quantité sur le graphique. + \item On considère que l'échelle de votre graphique est de 1unité pour 15m. Calculer l'aire du bassin. Vous donnerez un résultat arrondi au $m^2$ près. + \end{enumerate} +\end{exercise} + +\begin{solution} + \begin{enumerate} + \item + \begin{tikzpicture}[baseline=(a.north), xscale=1, yscale=0.5] + \tkzInit[xmin=0,xmax=5,xstep=1, + ymin=0,ymax=10,ystep=1] + \tkzGrid + \tkzAxeXY + \tkzFct[domain=0:10,color=red,very thick]% + { (-x**2 + 6.1*x - 9.3)*exp(-x) + 9.3 }; + \end{tikzpicture} + \item Il faut dériver $F(x)$ et vérifier que $F'(x) = f(x)$. + \item $\ds \int_0^4 f(x) \; dx = F(4) - F(0) = \frac{4.8}{e^{4}} + 32.0$ + \item La quantité calculée à la question précédente se retrouve 4fois pour former le bassin. Il faut ensuite prendre en compte l'échelle, comme 1unité de longueur correspond à 15m, une unité d'air correspond à $15\times15 = 225m^2$. Ainsi l'aire du bassin est égale à + \[ + (\frac{4.8}{e^{4}} + 32.0)\times 4 \times 15^2 = 28879.00000 + \] + + \end{enumerate} +\end{solution} + +%%% Local Variables: +%%% mode: latex +%%% TeX-master: "master" +%%% End: + +\begin{exercise}[subtitle={Stylos}] + \emph{Les parties {\rm A} et {\rm B} de cet exercice sont indépendantes.} + + \bigskip + + \begin{minipage}{0.6\linewidth} + \textbf{Partie A} + + \medskip + + Deux ateliers A et B fabriquent des stylos pour une entreprise. + + L'atelier A fabrique 92.0\,\% des stylos, et parmi ceux-là, 47.0\,\% possèdent un défaut de fabrication. + + De plus, 1.0\,\% des stylos possèdent un défaut de fabrication et sortent de l'atelier B. + + Un stylo est prélevé au hasard dans le stock de l'entreprise. + + On considère les évènements suivants: + + \begin{itemize} + \item A : \og Le stylo a été fabriqué par l'atelier A \fg + \item B : \og Le stylo a été fabriqué par l'atelier B \fg + \item D : \og Le stylo possède un défaut de fabrication \fg + \end{itemize} + \end{minipage} + \begin{minipage}{0.4\linewidth} + \begin{center} + \begin{tikzpicture}[sloped] + \node {.} + child {node {$A$} + child {node {$D$} + edge from parent + node[above] {...} + } + child {node {$\overline{D}$} + edge from parent + node[above] {...} + } + edge from parent + node[above] {...} + } + child[missing] {} + child { node {$B$} + child {node {$D$} + edge from parent + node[above] {...} + } + child {node {$\overline{D}$} + edge from parent + node[above] {...} + } + edge from parent + node[above] {...} + } ; + \end{tikzpicture} + \end{center} + \end{minipage} + + \medskip + + \begin{enumerate} + \item Compléter l'arbre de probabilité ci-contre + \item Interpréter puis donner les probabilités $P(A)$, $P(B)$, $P_A(D)$ et $P(B \cap D)$. + + \item + \begin{enumerate} + \item Calculer la probabilité qu'un stylo provienne de l'atelier A et possède un défaut de fabrication. + \item En déduire que la probabilité qu'un stylo possède un défaut de fabrication est de $0.44$. + \end{enumerate} + \item On prélève un stylo au hasard avec un défaut. Quelle est la probabilité qu'il vienne de l'atelier A? + \end{enumerate} + + \bigskip + + \textbf{Partie B} + \medskip + + Dans cette partie, on suppose que 44.0\,\% des stylos possèdent un défaut de fabrication. + + L'entreprise confectionne des paquets contenant chacun $4$~stylos. + + Le fait qu'un stylo possède ou non un défaut de fabrication est indépendant des autres stylos. + + On appelle $X$ la variable aléatoire donnant pour un paquet le nombre de stylos qui possèdent un défaut de fabrication. + + On admet que la variable aléatoire $X$ suit une loi binomiale. + + \medskip + + \begin{enumerate} + \setcounter{enumi}{4} + \item Avec quelle loi peut-on modéliser $X$. Préciser les paramètres. + \item Calculer et interpréter la probabilité $P(X = 15)$. + \item Le directeur de l'entreprise affirme qu'il y a plus d'une chance sur deux qu'un paquet ne comporte aucun stylo défectueux. A-t-il raison ? + \item Combien de stylos peut-on espérer avoir en moyenne? + \end{enumerate} + \pagebreak +\end{exercise} + +\begin{solution} + \begin{enumerate} + \item + \begin{center} + \begin{tikzpicture}[sloped] + \node {.} + child {node {$A$} + child {node {$D$} + edge from parent + node[above] {0.47} + } + child {node {$\overline{D}$} + edge from parent + node[above] {0.53} + } + edge from parent + node[above] {0.92} + } + child[missing] {} + child { node {$B$} + child {node {$D$} + edge from parent + node[above] {0.17} + } + child {node {$\overline{D}$} + edge from parent + node[above] {0.83} + } + edge from parent + node[above] {0.08} + } ; + \end{tikzpicture} + \end{center} + \item + \begin{itemize} + \item Probabilité que le stylo vienne de l'atelier A + \[ + P(A) = 0.92 + \] + \item Probabilité que le stylo vienne de l'atelier B + \[ + P(B) = 0.08 + \] + \item Probabilité que le stylo ait un défaut sachant qu'il vient de l'atelier A. + \[ + P_A(D) = 0.47 + \] + \item Probabilité que le stylo vienne de l'atelier B et qu'il ait un défaut. + \[ + P(D \cap D) = 0.01 + \] + \end{itemize} + \item + \begin{enumerate} + \item Probabilité qu'un stylo vienne de l'atelier A et qu'il ait un defaut + \[ + P(A\cap D) = P(A) \times P_A(D) = 0.92 \times 0.47 = 0.43 + \] + \item Probabilité que le stylo ai un défaut de fabrication. + \[ + P(D) = P(A\cap D) + P(B\cap D) = 0.43 + 0.01 = 0.44 + \] + \end{enumerate} + \item Probabilité qu'il vienne de l'atelier A sachant qu'il a un defaut + \[ + P_D(A) = \frac{P(A\cap D)}{P(D)} = \frac{0.43}{0.44} = 0.98 + \] + \item $X$ peut être modélisée par une loi binomiale de paramètres $n=17$ et $p=0.44$. + \item (\textit{par de correction automatique disponible pour le résultat final} + \[ + P(X = 15) = \coefBino{17}{15}\times 0.44^{15} \times 0.56^{2} + \] + \item (\textit{par de correction automatique disponible pour le résultat final} + + Il faut calculer la probabilité qu'il y ait 0 stylo avec un defaut. + \[ + P(X = 0) = \coefBino{17}{0}\times 0.44^{0} \times 0.56^{17} + \] + Puis comparer ce nombre à 0,5. + \item Il faut calculer l'espérance + \[ + E[X] = n\times p = 17 \times 0.44 = 7.48 + \] + \end{enumerate} +\end{solution} + +\end{document} + +%%% Local Variables: +%%% mode: latex +%%% TeX-master: "master" +%%% End: diff --git a/Complementaire/DM/2105_DM1/09_2105_DM1.tex b/Complementaire/DM/2105_DM1/09_2105_DM1.tex new file mode 100644 index 0000000..14cd7ca --- /dev/null +++ b/Complementaire/DM/2105_DM1/09_2105_DM1.tex @@ -0,0 +1,367 @@ +\documentclass[a4paper,10pt]{article} +\usepackage{myXsim} + +% Title Page +\title{DM1 \hfill MOLINIER Annelise} +\tribe{Maths complémentaire} +\date{\hfillÀ render pour le jeudi 27 mai} + +\xsimsetup{ + solution/print = false +} + +\begin{document} +\maketitle + +Les valeurs des exercices sont générés automatiquement. Si une valeur a un nombre adhérant de chiffres après la virgule, vous pouvez l'arrondir à l'entier le plus proche. + +\begin{exercise}[subtitle={Optimisation de matière}] + \begin{minipage}{0.6\textwidth} + On se propose de fabriquer avec le moins de tôle possible une citerne fermée en forme de parallélépipède rectangle dont le volume intérieur doit être de $25m^3$. La longueur est aussi fixée à $5m$ par le cahier des charges. + + On peut donc faire varier uniquement la largeur (notée $x$) et la hauteur (notée $h$) de la cuve. + \end{minipage} + \hfill + \begin{minipage}{0.3\textwidth} + \begin{tikzpicture} + \pgfmathsetmacro{\cubex}{3} + \pgfmathsetmacro{\cubey}{1} + \pgfmathsetmacro{\cubez}{2} + \draw[black,fill=gray] (0,0,0) -- ++(-\cubex,0,0) -- ++(0,-\cubey,0) node [midway, left] {$h$} -- ++(\cubex,0,0) node [midway, below] {$x$} -- cycle; + \draw[black,fill=gray] (0,0,0) -- ++(0,0,-\cubez) -- ++(0,-\cubey,0) -- ++(0,0,\cubez) node [midway, right] {$5m$} -- cycle; + \draw[black,fill=gray] (0,0,0) -- ++(-\cubex,0,0) -- ++(0,0,-\cubez) -- ++(\cubex,0,0) -- cycle; + \end{tikzpicture} + \end{minipage} + + \begin{enumerate} + \item Expliquer pourquoi quand la largeur $x$ change, la hauteur $h$ doit elle aussi changer pour respecter les contraintes. + \item Démontrer que l'on doit avoir $h = \dfrac{5}{x}$. + \item On note $S(x)$ l'aire totale de la citerne (c'est à dire la somme des aires des six faces). Montrer que l'on peut écrire + \[ + S(x) = 10x + 10 + \frac{50}{x} + \] + \item Démontrer que + \[ + S(x) = \frac{10x^2 + 10x + 50}{x} + \] + \item Démontrer que + \[ + S'(x) = \frac{10x^2 - 50}{x^2} + \] + \item En déduire le tableau de variation de $S(x)$ sur $\intOF{0}{10}$. + \item Déterminer les valeurs de $x$ et $h$ correspondant à une utilisation minimal de tôle. + \end{enumerate} +\end{exercise} + +\begin{solution} + \begin{enumerate} + \item Le volume étant fixe si l'on fait varier $x$, $h$ doit aussi varier. + \begin{itemize} + \item Si $x = 2$ alors conserver un volume de $V=25$, $h$ doit être égale à $5 / 2$ + \item Si $x = 3$ alors conserver un volume de $V=25$, $h$ doit être égale à $5 / 3$ + \end{itemize} + \item Pour calculer le volume, on a + \begin{eqnarray*} + V &=& h\times x \times 5 \\ + 25 &=& h\times x \times 5 \\ + x &=& \frac{25}{h\times 5} = \frac{5}{h} + \end{eqnarray*} + \item Pour calculer la surface totale, on ajoute la surface de chaque face. On a donc le calcul suivant + \begin{eqnarray*} + S(x) &=& x\times h \times 2 + x\times5\times2 + h\times 5\times 2\\ + S(x) &=& x\times \frac{5}{x} \times 2 + x\times5\times2 + \frac{5}{x}\times 5\times 2\\ + S(x) &=& 10x + 10 + \frac{50}{x} + \end{eqnarray*} + \item Pour trouver cette nouvelle forme, on met chaque élément sur le même dénominateur + \begin{eqnarray*} + S(x) &=& 10x + 10 + \frac{50}{x}\\ + S(x) &=& \frac{10x\times x}{x} + \frac{10\times x}{x} + \frac{50}{x}\\ + S(x) &=& \frac{10x^2 + 10x + 50}{x} + \end{eqnarray*} + \item On retrouve la formule $\frac{u}{v}$ à dériver + \[ + u(x) = 10x^2 + 10x + 50 \Rightarrow u'(x) = 20x + 10 + \] + \[ + v(x) = x \Rightarrow v'(x) = 1 + \] + Donc au numérateur on obtient + \begin{eqnarray*} + u'(x)\times v(x) - u(x)\times v'(x) &=& (20x + 10)\times x - (10x^2 + 10x + 50)\times 1\\ + &=& 10x^2 - 50 + \end{eqnarray*} + Donc + \[ + S'(x) = \frac{10x^2 - 50}{x^2} + \] + \item Tableau de variations de $S$ + + \begin{itemize} + \item Valeur interdite: $x^2 = 0 \equiv x = 0$ + \item Signe de $10x^2 - 50$: c'est un polynôme du 2e degré + \[ + \Delta = 2000 > 0 + \] + Il y a donc 2 racines + \[ + x_1 = - 2.23606797749979 \qquad + x_2 = 2.23606797749979 + \] + Et on sait que $10x^2 - 50$ est du signe de $a$ donc positif en dehors des racines + \item Le dénominateur $x^2$ est toujours positif. + \item Tableau de variations + + \begin{tikzpicture}[baseline=(a.north)] + \tkzTabInit[lgt=3,espcl=3]{$x$/1,$10x^2 - 50$/1, $x^2$/1, $S'$/1, $S$/2}{$0$, $- 2.23606797749979$, $10$} + \tkzTabLine{d,-, z, +, } + \tkzTabLine{d,+, , +, } + \tkzTabLine{d,-, z, +, } + \tkzTabVar{D+/ , -/ , +/ } + \end{tikzpicture} + + \end{itemize} + \item On a donc une surface minimal pour $x=2.23606797749979$ et $h = 11.18033988749895$. + \end{enumerate} +\end{solution} + +%%% Local Variables: +%%% mode: latex +%%% TeX-master: "master" +%%% End: + +\begin{exercise}[subtitle={Bassin}] + Le tour d'un bassin au niveau du sol présente deux axes de symétrie : l’axe des abscisses et la droite d’équation $x=4$. Il est obtenu par symétrie de la courbe $\mathcal{C}_f$ sur $\intFF{0}{4}$ où $f$ est la fonction définie par + + \[ + f(x) = \left(- x^{2} + 6.9 x - 3.3\right) e^{- x} + 3.3 + \] + On admet que sur $\intFF{0}{4}$ la fonction $f$ est positive. + \begin{enumerate} + \item Sur un repère, tracer l'allure de la courbe $\mathcal{C}_f$, les axes de symétries puis compléter pour dessiner la forme du bassin. + \item Montrer que la fonction $f$ admet comme primitive sur $\R$ la fonction $F$ définie par + \[ + F(x) = 3.3 x + \left( x^{2} - 4.9 x - 1.6\right) e^{- x} + \] + \item Calculer la quantité $\ds \int_0^4 f(x) \; dx$, vous donnerez le résultat sous forme exacte. Interpréter le résultat et reportez cette quantité sur le graphique. + \item On considère que l'échelle de votre graphique est de 1unité pour 15m. Calculer l'aire du bassin. Vous donnerez un résultat arrondi au $m^2$ près. + \end{enumerate} +\end{exercise} + +\begin{solution} + \begin{enumerate} + \item + \begin{tikzpicture}[baseline=(a.north), xscale=1, yscale=0.5] + \tkzInit[xmin=0,xmax=5,xstep=1, + ymin=0,ymax=10,ystep=1] + \tkzGrid + \tkzAxeXY + \tkzFct[domain=0:10,color=red,very thick]% + { (-x**2 + 6.9*x - 3.3)*exp(-x) + 3.3 }; + \end{tikzpicture} + \item Il faut dériver $F(x)$ et vérifier que $F'(x) = f(x)$. + \item $\ds \int_0^4 f(x) \; dx = F(4) - F(0) = 14.8 - \frac{5.2}{e^{4}}$ + \item La quantité calculée à la question précédente se retrouve 4fois pour former le bassin. Il faut ensuite prendre en compte l'échelle, comme 1unité de longueur correspond à 15m, une unité d'air correspond à $15\times15 = 225m^2$. Ainsi l'aire du bassin est égale à + \[ + (14.8 - \frac{5.2}{e^{4}})\times 4 \times 15^2 = 13234.00000 + \] + + \end{enumerate} +\end{solution} + +%%% Local Variables: +%%% mode: latex +%%% TeX-master: "master" +%%% End: + +\begin{exercise}[subtitle={Stylos}] + \emph{Les parties {\rm A} et {\rm B} de cet exercice sont indépendantes.} + + \bigskip + + \begin{minipage}{0.6\linewidth} + \textbf{Partie A} + + \medskip + + Deux ateliers A et B fabriquent des stylos pour une entreprise. + + L'atelier A fabrique 56.00000000000001\,\% des stylos, et parmi ceux-là, 22.0\,\% possèdent un défaut de fabrication. + + De plus, 10.0\,\% des stylos possèdent un défaut de fabrication et sortent de l'atelier B. + + Un stylo est prélevé au hasard dans le stock de l'entreprise. + + On considère les évènements suivants: + + \begin{itemize} + \item A : \og Le stylo a été fabriqué par l'atelier A \fg + \item B : \og Le stylo a été fabriqué par l'atelier B \fg + \item D : \og Le stylo possède un défaut de fabrication \fg + \end{itemize} + \end{minipage} + \begin{minipage}{0.4\linewidth} + \begin{center} + \begin{tikzpicture}[sloped] + \node {.} + child {node {$A$} + child {node {$D$} + edge from parent + node[above] {...} + } + child {node {$\overline{D}$} + edge from parent + node[above] {...} + } + edge from parent + node[above] {...} + } + child[missing] {} + child { node {$B$} + child {node {$D$} + edge from parent + node[above] {...} + } + child {node {$\overline{D}$} + edge from parent + node[above] {...} + } + edge from parent + node[above] {...} + } ; + \end{tikzpicture} + \end{center} + \end{minipage} + + \medskip + + \begin{enumerate} + \item Compléter l'arbre de probabilité ci-contre + \item Interpréter puis donner les probabilités $P(A)$, $P(B)$, $P_A(D)$ et $P(B \cap D)$. + + \item + \begin{enumerate} + \item Calculer la probabilité qu'un stylo provienne de l'atelier A et possède un défaut de fabrication. + \item En déduire que la probabilité qu'un stylo possède un défaut de fabrication est de $0.22$. + \end{enumerate} + \item On prélève un stylo au hasard avec un défaut. Quelle est la probabilité qu'il vienne de l'atelier A? + \end{enumerate} + + \bigskip + + \textbf{Partie B} + \medskip + + Dans cette partie, on suppose que 22.0\,\% des stylos possèdent un défaut de fabrication. + + L'entreprise confectionne des paquets contenant chacun $4$~stylos. + + Le fait qu'un stylo possède ou non un défaut de fabrication est indépendant des autres stylos. + + On appelle $X$ la variable aléatoire donnant pour un paquet le nombre de stylos qui possèdent un défaut de fabrication. + + On admet que la variable aléatoire $X$ suit une loi binomiale. + + \medskip + + \begin{enumerate} + \setcounter{enumi}{4} + \item Avec quelle loi peut-on modéliser $X$. Préciser les paramètres. + \item Calculer et interpréter la probabilité $P(X = 19)$. + \item Le directeur de l'entreprise affirme qu'il y a plus d'une chance sur deux qu'un paquet ne comporte aucun stylo défectueux. A-t-il raison ? + \item Combien de stylos peut-on espérer avoir en moyenne? + \end{enumerate} + \pagebreak +\end{exercise} + +\begin{solution} + \begin{enumerate} + \item + \begin{center} + \begin{tikzpicture}[sloped] + \node {.} + child {node {$A$} + child {node {$D$} + edge from parent + node[above] {0.22} + } + child {node {$\overline{D}$} + edge from parent + node[above] {0.78} + } + edge from parent + node[above] {0.56} + } + child[missing] {} + child { node {$B$} + child {node {$D$} + edge from parent + node[above] {0.23} + } + child {node {$\overline{D}$} + edge from parent + node[above] {0.77} + } + edge from parent + node[above] {0.44} + } ; + \end{tikzpicture} + \end{center} + \item + \begin{itemize} + \item Probabilité que le stylo vienne de l'atelier A + \[ + P(A) = 0.56 + \] + \item Probabilité que le stylo vienne de l'atelier B + \[ + P(B) = 0.44 + \] + \item Probabilité que le stylo ait un défaut sachant qu'il vient de l'atelier A. + \[ + P_A(D) = 0.22 + \] + \item Probabilité que le stylo vienne de l'atelier B et qu'il ait un défaut. + \[ + P(D \cap D) = 0.1 + \] + \end{itemize} + \item + \begin{enumerate} + \item Probabilité qu'un stylo vienne de l'atelier A et qu'il ait un defaut + \[ + P(A\cap D) = P(A) \times P_A(D) = 0.56 \times 0.22 = 0.12 + \] + \item Probabilité que le stylo ai un défaut de fabrication. + \[ + P(D) = P(A\cap D) + P(B\cap D) = 0.12 + 0.1 = 0.22 + \] + \end{enumerate} + \item Probabilité qu'il vienne de l'atelier A sachant qu'il a un defaut + \[ + P_D(A) = \frac{P(A\cap D)}{P(D)} = \frac{0.12}{0.22} = 0.55 + \] + \item $X$ peut être modélisée par une loi binomiale de paramètres $n=19$ et $p=0.22$. + \item (\textit{par de correction automatique disponible pour le résultat final} + \[ + P(X = 19) = \coefBino{19}{19}\times 0.22^{19} \times 0.78^{0} + \] + \item (\textit{par de correction automatique disponible pour le résultat final} + + Il faut calculer la probabilité qu'il y ait 0 stylo avec un defaut. + \[ + P(X = 0) = \coefBino{19}{0}\times 0.22^{0} \times 0.78^{19} + \] + Puis comparer ce nombre à 0,5. + \item Il faut calculer l'espérance + \[ + E[X] = n\times p = 19 \times 0.22 = 4.18 + \] + \end{enumerate} +\end{solution} + +\end{document} + +%%% Local Variables: +%%% mode: latex +%%% TeX-master: "master" +%%% End: diff --git a/Complementaire/DM/2105_DM1/10_2105_DM1.tex b/Complementaire/DM/2105_DM1/10_2105_DM1.tex new file mode 100644 index 0000000..b6e7e23 --- /dev/null +++ b/Complementaire/DM/2105_DM1/10_2105_DM1.tex @@ -0,0 +1,367 @@ +\documentclass[a4paper,10pt]{article} +\usepackage{myXsim} + +% Title Page +\title{DM1 \hfill MOUHOUBI Maïssa} +\tribe{Maths complémentaire} +\date{\hfillÀ render pour le jeudi 27 mai} + +\xsimsetup{ + solution/print = false +} + +\begin{document} +\maketitle + +Les valeurs des exercices sont générés automatiquement. Si une valeur a un nombre adhérant de chiffres après la virgule, vous pouvez l'arrondir à l'entier le plus proche. + +\begin{exercise}[subtitle={Optimisation de matière}] + \begin{minipage}{0.6\textwidth} + On se propose de fabriquer avec le moins de tôle possible une citerne fermée en forme de parallélépipède rectangle dont le volume intérieur doit être de $35m^3$. La longueur est aussi fixée à $5m$ par le cahier des charges. + + On peut donc faire varier uniquement la largeur (notée $x$) et la hauteur (notée $h$) de la cuve. + \end{minipage} + \hfill + \begin{minipage}{0.3\textwidth} + \begin{tikzpicture} + \pgfmathsetmacro{\cubex}{3} + \pgfmathsetmacro{\cubey}{1} + \pgfmathsetmacro{\cubez}{2} + \draw[black,fill=gray] (0,0,0) -- ++(-\cubex,0,0) -- ++(0,-\cubey,0) node [midway, left] {$h$} -- ++(\cubex,0,0) node [midway, below] {$x$} -- cycle; + \draw[black,fill=gray] (0,0,0) -- ++(0,0,-\cubez) -- ++(0,-\cubey,0) -- ++(0,0,\cubez) node [midway, right] {$5m$} -- cycle; + \draw[black,fill=gray] (0,0,0) -- ++(-\cubex,0,0) -- ++(0,0,-\cubez) -- ++(\cubex,0,0) -- cycle; + \end{tikzpicture} + \end{minipage} + + \begin{enumerate} + \item Expliquer pourquoi quand la largeur $x$ change, la hauteur $h$ doit elle aussi changer pour respecter les contraintes. + \item Démontrer que l'on doit avoir $h = \dfrac{7}{x}$. + \item On note $S(x)$ l'aire totale de la citerne (c'est à dire la somme des aires des six faces). Montrer que l'on peut écrire + \[ + S(x) = 10x + 14 + \frac{70}{x} + \] + \item Démontrer que + \[ + S(x) = \frac{10x^2 + 14x + 70}{x} + \] + \item Démontrer que + \[ + S'(x) = \frac{10x^2 - 70}{x^2} + \] + \item En déduire le tableau de variation de $S(x)$ sur $\intOF{0}{10}$. + \item Déterminer les valeurs de $x$ et $h$ correspondant à une utilisation minimal de tôle. + \end{enumerate} +\end{exercise} + +\begin{solution} + \begin{enumerate} + \item Le volume étant fixe si l'on fait varier $x$, $h$ doit aussi varier. + \begin{itemize} + \item Si $x = 2$ alors conserver un volume de $V=35$, $h$ doit être égale à $7 / 2$ + \item Si $x = 3$ alors conserver un volume de $V=35$, $h$ doit être égale à $7 / 3$ + \end{itemize} + \item Pour calculer le volume, on a + \begin{eqnarray*} + V &=& h\times x \times 5 \\ + 35 &=& h\times x \times 5 \\ + x &=& \frac{35}{h\times 5} = \frac{7}{h} + \end{eqnarray*} + \item Pour calculer la surface totale, on ajoute la surface de chaque face. On a donc le calcul suivant + \begin{eqnarray*} + S(x) &=& x\times h \times 2 + x\times5\times2 + h\times 5\times 2\\ + S(x) &=& x\times \frac{7}{x} \times 2 + x\times5\times2 + \frac{7}{x}\times 5\times 2\\ + S(x) &=& 10x + 14 + \frac{70}{x} + \end{eqnarray*} + \item Pour trouver cette nouvelle forme, on met chaque élément sur le même dénominateur + \begin{eqnarray*} + S(x) &=& 10x + 14 + \frac{70}{x}\\ + S(x) &=& \frac{10x\times x}{x} + \frac{14\times x}{x} + \frac{70}{x}\\ + S(x) &=& \frac{10x^2 + 14x + 70}{x} + \end{eqnarray*} + \item On retrouve la formule $\frac{u}{v}$ à dériver + \[ + u(x) = 10x^2 + 14x + 70 \Rightarrow u'(x) = 20x + 14 + \] + \[ + v(x) = x \Rightarrow v'(x) = 1 + \] + Donc au numérateur on obtient + \begin{eqnarray*} + u'(x)\times v(x) - u(x)\times v'(x) &=& (20x + 14)\times x - (10x^2 + 14x + 70)\times 1\\ + &=& 10x^2 - 70 + \end{eqnarray*} + Donc + \[ + S'(x) = \frac{10x^2 - 70}{x^2} + \] + \item Tableau de variations de $S$ + + \begin{itemize} + \item Valeur interdite: $x^2 = 0 \equiv x = 0$ + \item Signe de $10x^2 - 70$: c'est un polynôme du 2e degré + \[ + \Delta = 2800 > 0 + \] + Il y a donc 2 racines + \[ + x_1 = - 2.6457513110645907 \qquad + x_2 = 2.6457513110645907 + \] + Et on sait que $10x^2 - 70$ est du signe de $a$ donc positif en dehors des racines + \item Le dénominateur $x^2$ est toujours positif. + \item Tableau de variations + + \begin{tikzpicture}[baseline=(a.north)] + \tkzTabInit[lgt=3,espcl=3]{$x$/1,$10x^2 - 70$/1, $x^2$/1, $S'$/1, $S$/2}{$0$, $- 2.6457513110645907$, $10$} + \tkzTabLine{d,-, z, +, } + \tkzTabLine{d,+, , +, } + \tkzTabLine{d,-, z, +, } + \tkzTabVar{D+/ , -/ , +/ } + \end{tikzpicture} + + \end{itemize} + \item On a donc une surface minimal pour $x=2.6457513110645907$ et $h = 18.5202591774521349$. + \end{enumerate} +\end{solution} + +%%% Local Variables: +%%% mode: latex +%%% TeX-master: "master" +%%% End: + +\begin{exercise}[subtitle={Bassin}] + Le tour d'un bassin au niveau du sol présente deux axes de symétrie : l’axe des abscisses et la droite d’équation $x=4$. Il est obtenu par symétrie de la courbe $\mathcal{C}_f$ sur $\intFF{0}{4}$ où $f$ est la fonction définie par + + \[ + f(x) = \left(- x^{2} + 1.1 x - 6.1\right) e^{- x} + 6.1 + \] + On admet que sur $\intFF{0}{4}$ la fonction $f$ est positive. + \begin{enumerate} + \item Sur un repère, tracer l'allure de la courbe $\mathcal{C}_f$, les axes de symétries puis compléter pour dessiner la forme du bassin. + \item Montrer que la fonction $f$ admet comme primitive sur $\R$ la fonction $F$ définie par + \[ + F(x) = 6.1 x + \left( x^{2} + 0.9 x + 7.0\right) e^{- x} + \] + \item Calculer la quantité $\ds \int_0^4 f(x) \; dx$, vous donnerez le résultat sous forme exacte. Interpréter le résultat et reportez cette quantité sur le graphique. + \item On considère que l'échelle de votre graphique est de 1unité pour 15m. Calculer l'aire du bassin. Vous donnerez un résultat arrondi au $m^2$ près. + \end{enumerate} +\end{exercise} + +\begin{solution} + \begin{enumerate} + \item + \begin{tikzpicture}[baseline=(a.north), xscale=1, yscale=0.5] + \tkzInit[xmin=0,xmax=5,xstep=1, + ymin=0,ymax=10,ystep=1] + \tkzGrid + \tkzAxeXY + \tkzFct[domain=0:10,color=red,very thick]% + { (-x**2 + 1.1*x - 6.1)*exp(-x) + 6.1 }; + \end{tikzpicture} + \item Il faut dériver $F(x)$ et vérifier que $F'(x) = f(x)$. + \item $\ds \int_0^4 f(x) \; dx = F(4) - F(0) = \frac{26.6}{e^{4}} + 17.4$ + \item La quantité calculée à la question précédente se retrouve 4fois pour former le bassin. Il faut ensuite prendre en compte l'échelle, comme 1unité de longueur correspond à 15m, une unité d'air correspond à $15\times15 = 225m^2$. Ainsi l'aire du bassin est égale à + \[ + (\frac{26.6}{e^{4}} + 17.4)\times 4 \times 15^2 = 16098.00000 + \] + + \end{enumerate} +\end{solution} + +%%% Local Variables: +%%% mode: latex +%%% TeX-master: "master" +%%% End: + +\begin{exercise}[subtitle={Stylos}] + \emph{Les parties {\rm A} et {\rm B} de cet exercice sont indépendantes.} + + \bigskip + + \begin{minipage}{0.6\linewidth} + \textbf{Partie A} + + \medskip + + Deux ateliers A et B fabriquent des stylos pour une entreprise. + + L'atelier A fabrique 23.0\,\% des stylos, et parmi ceux-là, 66.0\,\% possèdent un défaut de fabrication. + + De plus, 68.0\,\% des stylos possèdent un défaut de fabrication et sortent de l'atelier B. + + Un stylo est prélevé au hasard dans le stock de l'entreprise. + + On considère les évènements suivants: + + \begin{itemize} + \item A : \og Le stylo a été fabriqué par l'atelier A \fg + \item B : \og Le stylo a été fabriqué par l'atelier B \fg + \item D : \og Le stylo possède un défaut de fabrication \fg + \end{itemize} + \end{minipage} + \begin{minipage}{0.4\linewidth} + \begin{center} + \begin{tikzpicture}[sloped] + \node {.} + child {node {$A$} + child {node {$D$} + edge from parent + node[above] {...} + } + child {node {$\overline{D}$} + edge from parent + node[above] {...} + } + edge from parent + node[above] {...} + } + child[missing] {} + child { node {$B$} + child {node {$D$} + edge from parent + node[above] {...} + } + child {node {$\overline{D}$} + edge from parent + node[above] {...} + } + edge from parent + node[above] {...} + } ; + \end{tikzpicture} + \end{center} + \end{minipage} + + \medskip + + \begin{enumerate} + \item Compléter l'arbre de probabilité ci-contre + \item Interpréter puis donner les probabilités $P(A)$, $P(B)$, $P_A(D)$ et $P(B \cap D)$. + + \item + \begin{enumerate} + \item Calculer la probabilité qu'un stylo provienne de l'atelier A et possède un défaut de fabrication. + \item En déduire que la probabilité qu'un stylo possède un défaut de fabrication est de $0.83$. + \end{enumerate} + \item On prélève un stylo au hasard avec un défaut. Quelle est la probabilité qu'il vienne de l'atelier A? + \end{enumerate} + + \bigskip + + \textbf{Partie B} + \medskip + + Dans cette partie, on suppose que 83.0\,\% des stylos possèdent un défaut de fabrication. + + L'entreprise confectionne des paquets contenant chacun $4$~stylos. + + Le fait qu'un stylo possède ou non un défaut de fabrication est indépendant des autres stylos. + + On appelle $X$ la variable aléatoire donnant pour un paquet le nombre de stylos qui possèdent un défaut de fabrication. + + On admet que la variable aléatoire $X$ suit une loi binomiale. + + \medskip + + \begin{enumerate} + \setcounter{enumi}{4} + \item Avec quelle loi peut-on modéliser $X$. Préciser les paramètres. + \item Calculer et interpréter la probabilité $P(X = 10)$. + \item Le directeur de l'entreprise affirme qu'il y a plus d'une chance sur deux qu'un paquet ne comporte aucun stylo défectueux. A-t-il raison ? + \item Combien de stylos peut-on espérer avoir en moyenne? + \end{enumerate} + \pagebreak +\end{exercise} + +\begin{solution} + \begin{enumerate} + \item + \begin{center} + \begin{tikzpicture}[sloped] + \node {.} + child {node {$A$} + child {node {$D$} + edge from parent + node[above] {0.66} + } + child {node {$\overline{D}$} + edge from parent + node[above] {0.34} + } + edge from parent + node[above] {0.23} + } + child[missing] {} + child { node {$B$} + child {node {$D$} + edge from parent + node[above] {0.88} + } + child {node {$\overline{D}$} + edge from parent + node[above] {0.12} + } + edge from parent + node[above] {0.77} + } ; + \end{tikzpicture} + \end{center} + \item + \begin{itemize} + \item Probabilité que le stylo vienne de l'atelier A + \[ + P(A) = 0.23 + \] + \item Probabilité que le stylo vienne de l'atelier B + \[ + P(B) = 0.77 + \] + \item Probabilité que le stylo ait un défaut sachant qu'il vient de l'atelier A. + \[ + P_A(D) = 0.66 + \] + \item Probabilité que le stylo vienne de l'atelier B et qu'il ait un défaut. + \[ + P(D \cap D) = 0.68 + \] + \end{itemize} + \item + \begin{enumerate} + \item Probabilité qu'un stylo vienne de l'atelier A et qu'il ait un defaut + \[ + P(A\cap D) = P(A) \times P_A(D) = 0.23 \times 0.66 = 0.15 + \] + \item Probabilité que le stylo ai un défaut de fabrication. + \[ + P(D) = P(A\cap D) + P(B\cap D) = 0.15 + 0.68 = 0.83 + \] + \end{enumerate} + \item Probabilité qu'il vienne de l'atelier A sachant qu'il a un defaut + \[ + P_D(A) = \frac{P(A\cap D)}{P(D)} = \frac{0.15}{0.83} = 0.18 + \] + \item $X$ peut être modélisée par une loi binomiale de paramètres $n=17$ et $p=0.83$. + \item (\textit{par de correction automatique disponible pour le résultat final} + \[ + P(X = 10) = \coefBino{17}{10}\times 0.83^{10} \times 0.17^{7} + \] + \item (\textit{par de correction automatique disponible pour le résultat final} + + Il faut calculer la probabilité qu'il y ait 0 stylo avec un defaut. + \[ + P(X = 0) = \coefBino{17}{0}\times 0.83^{0} \times 0.17^{17} + \] + Puis comparer ce nombre à 0,5. + \item Il faut calculer l'espérance + \[ + E[X] = n\times p = 17 \times 0.83 = 14.11 + \] + \end{enumerate} +\end{solution} + +\end{document} + +%%% Local Variables: +%%% mode: latex +%%% TeX-master: "master" +%%% End: diff --git a/Complementaire/DM/2105_DM1/11_2105_DM1.tex b/Complementaire/DM/2105_DM1/11_2105_DM1.tex new file mode 100644 index 0000000..2d9b608 --- /dev/null +++ b/Complementaire/DM/2105_DM1/11_2105_DM1.tex @@ -0,0 +1,367 @@ +\documentclass[a4paper,10pt]{article} +\usepackage{myXsim} + +% Title Page +\title{DM1 \hfill PERDRIX Camille} +\tribe{Maths complémentaire} +\date{\hfillÀ render pour le jeudi 27 mai} + +\xsimsetup{ + solution/print = false +} + +\begin{document} +\maketitle + +Les valeurs des exercices sont générés automatiquement. Si une valeur a un nombre adhérant de chiffres après la virgule, vous pouvez l'arrondir à l'entier le plus proche. + +\begin{exercise}[subtitle={Optimisation de matière}] + \begin{minipage}{0.6\textwidth} + On se propose de fabriquer avec le moins de tôle possible une citerne fermée en forme de parallélépipède rectangle dont le volume intérieur doit être de $8m^3$. La longueur est aussi fixée à $4m$ par le cahier des charges. + + On peut donc faire varier uniquement la largeur (notée $x$) et la hauteur (notée $h$) de la cuve. + \end{minipage} + \hfill + \begin{minipage}{0.3\textwidth} + \begin{tikzpicture} + \pgfmathsetmacro{\cubex}{3} + \pgfmathsetmacro{\cubey}{1} + \pgfmathsetmacro{\cubez}{2} + \draw[black,fill=gray] (0,0,0) -- ++(-\cubex,0,0) -- ++(0,-\cubey,0) node [midway, left] {$h$} -- ++(\cubex,0,0) node [midway, below] {$x$} -- cycle; + \draw[black,fill=gray] (0,0,0) -- ++(0,0,-\cubez) -- ++(0,-\cubey,0) -- ++(0,0,\cubez) node [midway, right] {$4m$} -- cycle; + \draw[black,fill=gray] (0,0,0) -- ++(-\cubex,0,0) -- ++(0,0,-\cubez) -- ++(\cubex,0,0) -- cycle; + \end{tikzpicture} + \end{minipage} + + \begin{enumerate} + \item Expliquer pourquoi quand la largeur $x$ change, la hauteur $h$ doit elle aussi changer pour respecter les contraintes. + \item Démontrer que l'on doit avoir $h = \dfrac{2}{x}$. + \item On note $S(x)$ l'aire totale de la citerne (c'est à dire la somme des aires des six faces). Montrer que l'on peut écrire + \[ + S(x) = 8x + 4 + \frac{16}{x} + \] + \item Démontrer que + \[ + S(x) = \frac{8x^2 + 4x + 16}{x} + \] + \item Démontrer que + \[ + S'(x) = \frac{8x^2 - 16}{x^2} + \] + \item En déduire le tableau de variation de $S(x)$ sur $\intOF{0}{10}$. + \item Déterminer les valeurs de $x$ et $h$ correspondant à une utilisation minimal de tôle. + \end{enumerate} +\end{exercise} + +\begin{solution} + \begin{enumerate} + \item Le volume étant fixe si l'on fait varier $x$, $h$ doit aussi varier. + \begin{itemize} + \item Si $x = 2$ alors conserver un volume de $V=8$, $h$ doit être égale à $2 / 2$ + \item Si $x = 3$ alors conserver un volume de $V=8$, $h$ doit être égale à $2 / 3$ + \end{itemize} + \item Pour calculer le volume, on a + \begin{eqnarray*} + V &=& h\times x \times 4 \\ + 8 &=& h\times x \times 4 \\ + x &=& \frac{8}{h\times 4} = \frac{2}{h} + \end{eqnarray*} + \item Pour calculer la surface totale, on ajoute la surface de chaque face. On a donc le calcul suivant + \begin{eqnarray*} + S(x) &=& x\times h \times 2 + x\times4\times2 + h\times 4\times 2\\ + S(x) &=& x\times \frac{2}{x} \times 2 + x\times4\times2 + \frac{2}{x}\times 4\times 2\\ + S(x) &=& 8x + 4 + \frac{16}{x} + \end{eqnarray*} + \item Pour trouver cette nouvelle forme, on met chaque élément sur le même dénominateur + \begin{eqnarray*} + S(x) &=& 8x + 4 + \frac{16}{x}\\ + S(x) &=& \frac{8x\times x}{x} + \frac{4\times x}{x} + \frac{16}{x}\\ + S(x) &=& \frac{8x^2 + 4x + 16}{x} + \end{eqnarray*} + \item On retrouve la formule $\frac{u}{v}$ à dériver + \[ + u(x) = 8x^2 + 4x + 16 \Rightarrow u'(x) = 16x + 4 + \] + \[ + v(x) = x \Rightarrow v'(x) = 1 + \] + Donc au numérateur on obtient + \begin{eqnarray*} + u'(x)\times v(x) - u(x)\times v'(x) &=& (16x + 4)\times x - (8x^2 + 4x + 16)\times 1\\ + &=& 8x^2 - 16 + \end{eqnarray*} + Donc + \[ + S'(x) = \frac{8x^2 - 16}{x^2} + \] + \item Tableau de variations de $S$ + + \begin{itemize} + \item Valeur interdite: $x^2 = 0 \equiv x = 0$ + \item Signe de $8x^2 - 16$: c'est un polynôme du 2e degré + \[ + \Delta = 512 > 0 + \] + Il y a donc 2 racines + \[ + x_1 = - 1.4142135623730951 \qquad + x_2 = 1.4142135623730951 + \] + Et on sait que $8x^2 - 16$ est du signe de $a$ donc positif en dehors des racines + \item Le dénominateur $x^2$ est toujours positif. + \item Tableau de variations + + \begin{tikzpicture}[baseline=(a.north)] + \tkzTabInit[lgt=3,espcl=3]{$x$/1,$8x^2 - 16$/1, $x^2$/1, $S'$/1, $S$/2}{$0$, $- 1.4142135623730951$, $10$} + \tkzTabLine{d,-, z, +, } + \tkzTabLine{d,+, , +, } + \tkzTabLine{d,-, z, +, } + \tkzTabVar{D+/ , -/ , +/ } + \end{tikzpicture} + + \end{itemize} + \item On a donc une surface minimal pour $x=1.4142135623730951$ et $h = 2.8284271247461902$. + \end{enumerate} +\end{solution} + +%%% Local Variables: +%%% mode: latex +%%% TeX-master: "master" +%%% End: + +\begin{exercise}[subtitle={Bassin}] + Le tour d'un bassin au niveau du sol présente deux axes de symétrie : l’axe des abscisses et la droite d’équation $x=4$. Il est obtenu par symétrie de la courbe $\mathcal{C}_f$ sur $\intFF{0}{4}$ où $f$ est la fonction définie par + + \[ + f(x) = \left(- x^{2} + 8.3 x - 5.0\right) e^{- x} + 5.0 + \] + On admet que sur $\intFF{0}{4}$ la fonction $f$ est positive. + \begin{enumerate} + \item Sur un repère, tracer l'allure de la courbe $\mathcal{C}_f$, les axes de symétries puis compléter pour dessiner la forme du bassin. + \item Montrer que la fonction $f$ admet comme primitive sur $\R$ la fonction $F$ définie par + \[ + F(x) = 5.0 x + \left( x^{2} - 6.3 x - 1.3\right) e^{- x} + \] + \item Calculer la quantité $\ds \int_0^4 f(x) \; dx$, vous donnerez le résultat sous forme exacte. Interpréter le résultat et reportez cette quantité sur le graphique. + \item On considère que l'échelle de votre graphique est de 1unité pour 15m. Calculer l'aire du bassin. Vous donnerez un résultat arrondi au $m^2$ près. + \end{enumerate} +\end{exercise} + +\begin{solution} + \begin{enumerate} + \item + \begin{tikzpicture}[baseline=(a.north), xscale=1, yscale=0.5] + \tkzInit[xmin=0,xmax=5,xstep=1, + ymin=0,ymax=10,ystep=1] + \tkzGrid + \tkzAxeXY + \tkzFct[domain=0:10,color=red,very thick]% + { (-x**2 + 8.3*x - 5.0)*exp(-x) + 5.0 }; + \end{tikzpicture} + \item Il faut dériver $F(x)$ et vérifier que $F'(x) = f(x)$. + \item $\ds \int_0^4 f(x) \; dx = F(4) - F(0) = 21.3 - \frac{10.5}{e^{4}}$ + \item La quantité calculée à la question précédente se retrouve 4fois pour former le bassin. Il faut ensuite prendre en compte l'échelle, comme 1unité de longueur correspond à 15m, une unité d'air correspond à $15\times15 = 225m^2$. Ainsi l'aire du bassin est égale à + \[ + (21.3 - \frac{10.5}{e^{4}})\times 4 \times 15^2 = 18997.00000 + \] + + \end{enumerate} +\end{solution} + +%%% Local Variables: +%%% mode: latex +%%% TeX-master: "master" +%%% End: + +\begin{exercise}[subtitle={Stylos}] + \emph{Les parties {\rm A} et {\rm B} de cet exercice sont indépendantes.} + + \bigskip + + \begin{minipage}{0.6\linewidth} + \textbf{Partie A} + + \medskip + + Deux ateliers A et B fabriquent des stylos pour une entreprise. + + L'atelier A fabrique 66.0\,\% des stylos, et parmi ceux-là, 35.0\,\% possèdent un défaut de fabrication. + + De plus, 28.000000000000004\,\% des stylos possèdent un défaut de fabrication et sortent de l'atelier B. + + Un stylo est prélevé au hasard dans le stock de l'entreprise. + + On considère les évènements suivants: + + \begin{itemize} + \item A : \og Le stylo a été fabriqué par l'atelier A \fg + \item B : \og Le stylo a été fabriqué par l'atelier B \fg + \item D : \og Le stylo possède un défaut de fabrication \fg + \end{itemize} + \end{minipage} + \begin{minipage}{0.4\linewidth} + \begin{center} + \begin{tikzpicture}[sloped] + \node {.} + child {node {$A$} + child {node {$D$} + edge from parent + node[above] {...} + } + child {node {$\overline{D}$} + edge from parent + node[above] {...} + } + edge from parent + node[above] {...} + } + child[missing] {} + child { node {$B$} + child {node {$D$} + edge from parent + node[above] {...} + } + child {node {$\overline{D}$} + edge from parent + node[above] {...} + } + edge from parent + node[above] {...} + } ; + \end{tikzpicture} + \end{center} + \end{minipage} + + \medskip + + \begin{enumerate} + \item Compléter l'arbre de probabilité ci-contre + \item Interpréter puis donner les probabilités $P(A)$, $P(B)$, $P_A(D)$ et $P(B \cap D)$. + + \item + \begin{enumerate} + \item Calculer la probabilité qu'un stylo provienne de l'atelier A et possède un défaut de fabrication. + \item En déduire que la probabilité qu'un stylo possède un défaut de fabrication est de $0.51$. + \end{enumerate} + \item On prélève un stylo au hasard avec un défaut. Quelle est la probabilité qu'il vienne de l'atelier A? + \end{enumerate} + + \bigskip + + \textbf{Partie B} + \medskip + + Dans cette partie, on suppose que 51.0\,\% des stylos possèdent un défaut de fabrication. + + L'entreprise confectionne des paquets contenant chacun $4$~stylos. + + Le fait qu'un stylo possède ou non un défaut de fabrication est indépendant des autres stylos. + + On appelle $X$ la variable aléatoire donnant pour un paquet le nombre de stylos qui possèdent un défaut de fabrication. + + On admet que la variable aléatoire $X$ suit une loi binomiale. + + \medskip + + \begin{enumerate} + \setcounter{enumi}{4} + \item Avec quelle loi peut-on modéliser $X$. Préciser les paramètres. + \item Calculer et interpréter la probabilité $P(X = 11)$. + \item Le directeur de l'entreprise affirme qu'il y a plus d'une chance sur deux qu'un paquet ne comporte aucun stylo défectueux. A-t-il raison ? + \item Combien de stylos peut-on espérer avoir en moyenne? + \end{enumerate} + \pagebreak +\end{exercise} + +\begin{solution} + \begin{enumerate} + \item + \begin{center} + \begin{tikzpicture}[sloped] + \node {.} + child {node {$A$} + child {node {$D$} + edge from parent + node[above] {0.35} + } + child {node {$\overline{D}$} + edge from parent + node[above] {0.65} + } + edge from parent + node[above] {0.66} + } + child[missing] {} + child { node {$B$} + child {node {$D$} + edge from parent + node[above] {0.81} + } + child {node {$\overline{D}$} + edge from parent + node[above] {0.19} + } + edge from parent + node[above] {0.34} + } ; + \end{tikzpicture} + \end{center} + \item + \begin{itemize} + \item Probabilité que le stylo vienne de l'atelier A + \[ + P(A) = 0.66 + \] + \item Probabilité que le stylo vienne de l'atelier B + \[ + P(B) = 0.34 + \] + \item Probabilité que le stylo ait un défaut sachant qu'il vient de l'atelier A. + \[ + P_A(D) = 0.35 + \] + \item Probabilité que le stylo vienne de l'atelier B et qu'il ait un défaut. + \[ + P(D \cap D) = 0.28 + \] + \end{itemize} + \item + \begin{enumerate} + \item Probabilité qu'un stylo vienne de l'atelier A et qu'il ait un defaut + \[ + P(A\cap D) = P(A) \times P_A(D) = 0.66 \times 0.35 = 0.23 + \] + \item Probabilité que le stylo ai un défaut de fabrication. + \[ + P(D) = P(A\cap D) + P(B\cap D) = 0.23 + 0.28 = 0.51 + \] + \end{enumerate} + \item Probabilité qu'il vienne de l'atelier A sachant qu'il a un defaut + \[ + P_D(A) = \frac{P(A\cap D)}{P(D)} = \frac{0.23}{0.51} = 0.45 + \] + \item $X$ peut être modélisée par une loi binomiale de paramètres $n=19$ et $p=0.51$. + \item (\textit{par de correction automatique disponible pour le résultat final} + \[ + P(X = 11) = \coefBino{19}{11}\times 0.51^{11} \times 0.49^{8} + \] + \item (\textit{par de correction automatique disponible pour le résultat final} + + Il faut calculer la probabilité qu'il y ait 0 stylo avec un defaut. + \[ + P(X = 0) = \coefBino{19}{0}\times 0.51^{0} \times 0.49^{19} + \] + Puis comparer ce nombre à 0,5. + \item Il faut calculer l'espérance + \[ + E[X] = n\times p = 19 \times 0.51 = 9.69 + \] + \end{enumerate} +\end{solution} + +\end{document} + +%%% Local Variables: +%%% mode: latex +%%% TeX-master: "master" +%%% End: diff --git a/Complementaire/DM/2105_DM1/12_2105_DM1.tex b/Complementaire/DM/2105_DM1/12_2105_DM1.tex new file mode 100644 index 0000000..abc8b19 --- /dev/null +++ b/Complementaire/DM/2105_DM1/12_2105_DM1.tex @@ -0,0 +1,367 @@ +\documentclass[a4paper,10pt]{article} +\usepackage{myXsim} + +% Title Page +\title{DM1 \hfill POISON Lorette} +\tribe{Maths complémentaire} +\date{\hfillÀ render pour le jeudi 27 mai} + +\xsimsetup{ + solution/print = false +} + +\begin{document} +\maketitle + +Les valeurs des exercices sont générés automatiquement. Si une valeur a un nombre adhérant de chiffres après la virgule, vous pouvez l'arrondir à l'entier le plus proche. + +\begin{exercise}[subtitle={Optimisation de matière}] + \begin{minipage}{0.6\textwidth} + On se propose de fabriquer avec le moins de tôle possible une citerne fermée en forme de parallélépipède rectangle dont le volume intérieur doit être de $35m^3$. La longueur est aussi fixée à $5m$ par le cahier des charges. + + On peut donc faire varier uniquement la largeur (notée $x$) et la hauteur (notée $h$) de la cuve. + \end{minipage} + \hfill + \begin{minipage}{0.3\textwidth} + \begin{tikzpicture} + \pgfmathsetmacro{\cubex}{3} + \pgfmathsetmacro{\cubey}{1} + \pgfmathsetmacro{\cubez}{2} + \draw[black,fill=gray] (0,0,0) -- ++(-\cubex,0,0) -- ++(0,-\cubey,0) node [midway, left] {$h$} -- ++(\cubex,0,0) node [midway, below] {$x$} -- cycle; + \draw[black,fill=gray] (0,0,0) -- ++(0,0,-\cubez) -- ++(0,-\cubey,0) -- ++(0,0,\cubez) node [midway, right] {$5m$} -- cycle; + \draw[black,fill=gray] (0,0,0) -- ++(-\cubex,0,0) -- ++(0,0,-\cubez) -- ++(\cubex,0,0) -- cycle; + \end{tikzpicture} + \end{minipage} + + \begin{enumerate} + \item Expliquer pourquoi quand la largeur $x$ change, la hauteur $h$ doit elle aussi changer pour respecter les contraintes. + \item Démontrer que l'on doit avoir $h = \dfrac{7}{x}$. + \item On note $S(x)$ l'aire totale de la citerne (c'est à dire la somme des aires des six faces). Montrer que l'on peut écrire + \[ + S(x) = 10x + 14 + \frac{70}{x} + \] + \item Démontrer que + \[ + S(x) = \frac{10x^2 + 14x + 70}{x} + \] + \item Démontrer que + \[ + S'(x) = \frac{10x^2 - 70}{x^2} + \] + \item En déduire le tableau de variation de $S(x)$ sur $\intOF{0}{10}$. + \item Déterminer les valeurs de $x$ et $h$ correspondant à une utilisation minimal de tôle. + \end{enumerate} +\end{exercise} + +\begin{solution} + \begin{enumerate} + \item Le volume étant fixe si l'on fait varier $x$, $h$ doit aussi varier. + \begin{itemize} + \item Si $x = 2$ alors conserver un volume de $V=35$, $h$ doit être égale à $7 / 2$ + \item Si $x = 3$ alors conserver un volume de $V=35$, $h$ doit être égale à $7 / 3$ + \end{itemize} + \item Pour calculer le volume, on a + \begin{eqnarray*} + V &=& h\times x \times 5 \\ + 35 &=& h\times x \times 5 \\ + x &=& \frac{35}{h\times 5} = \frac{7}{h} + \end{eqnarray*} + \item Pour calculer la surface totale, on ajoute la surface de chaque face. On a donc le calcul suivant + \begin{eqnarray*} + S(x) &=& x\times h \times 2 + x\times5\times2 + h\times 5\times 2\\ + S(x) &=& x\times \frac{7}{x} \times 2 + x\times5\times2 + \frac{7}{x}\times 5\times 2\\ + S(x) &=& 10x + 14 + \frac{70}{x} + \end{eqnarray*} + \item Pour trouver cette nouvelle forme, on met chaque élément sur le même dénominateur + \begin{eqnarray*} + S(x) &=& 10x + 14 + \frac{70}{x}\\ + S(x) &=& \frac{10x\times x}{x} + \frac{14\times x}{x} + \frac{70}{x}\\ + S(x) &=& \frac{10x^2 + 14x + 70}{x} + \end{eqnarray*} + \item On retrouve la formule $\frac{u}{v}$ à dériver + \[ + u(x) = 10x^2 + 14x + 70 \Rightarrow u'(x) = 20x + 14 + \] + \[ + v(x) = x \Rightarrow v'(x) = 1 + \] + Donc au numérateur on obtient + \begin{eqnarray*} + u'(x)\times v(x) - u(x)\times v'(x) &=& (20x + 14)\times x - (10x^2 + 14x + 70)\times 1\\ + &=& 10x^2 - 70 + \end{eqnarray*} + Donc + \[ + S'(x) = \frac{10x^2 - 70}{x^2} + \] + \item Tableau de variations de $S$ + + \begin{itemize} + \item Valeur interdite: $x^2 = 0 \equiv x = 0$ + \item Signe de $10x^2 - 70$: c'est un polynôme du 2e degré + \[ + \Delta = 2800 > 0 + \] + Il y a donc 2 racines + \[ + x_1 = - 2.6457513110645907 \qquad + x_2 = 2.6457513110645907 + \] + Et on sait que $10x^2 - 70$ est du signe de $a$ donc positif en dehors des racines + \item Le dénominateur $x^2$ est toujours positif. + \item Tableau de variations + + \begin{tikzpicture}[baseline=(a.north)] + \tkzTabInit[lgt=3,espcl=3]{$x$/1,$10x^2 - 70$/1, $x^2$/1, $S'$/1, $S$/2}{$0$, $- 2.6457513110645907$, $10$} + \tkzTabLine{d,-, z, +, } + \tkzTabLine{d,+, , +, } + \tkzTabLine{d,-, z, +, } + \tkzTabVar{D+/ , -/ , +/ } + \end{tikzpicture} + + \end{itemize} + \item On a donc une surface minimal pour $x=2.6457513110645907$ et $h = 18.5202591774521349$. + \end{enumerate} +\end{solution} + +%%% Local Variables: +%%% mode: latex +%%% TeX-master: "master" +%%% End: + +\begin{exercise}[subtitle={Bassin}] + Le tour d'un bassin au niveau du sol présente deux axes de symétrie : l’axe des abscisses et la droite d’équation $x=4$. Il est obtenu par symétrie de la courbe $\mathcal{C}_f$ sur $\intFF{0}{4}$ où $f$ est la fonction définie par + + \[ + f(x) = \left(- x^{2} + 6.1 x - 3.4\right) e^{- x} + 3.4 + \] + On admet que sur $\intFF{0}{4}$ la fonction $f$ est positive. + \begin{enumerate} + \item Sur un repère, tracer l'allure de la courbe $\mathcal{C}_f$, les axes de symétries puis compléter pour dessiner la forme du bassin. + \item Montrer que la fonction $f$ admet comme primitive sur $\R$ la fonction $F$ définie par + \[ + F(x) = 3.4 x + \left( x^{2} - 4.1 x - 0.7\right) e^{- x} + \] + \item Calculer la quantité $\ds \int_0^4 f(x) \; dx$, vous donnerez le résultat sous forme exacte. Interpréter le résultat et reportez cette quantité sur le graphique. + \item On considère que l'échelle de votre graphique est de 1unité pour 15m. Calculer l'aire du bassin. Vous donnerez un résultat arrondi au $m^2$ près. + \end{enumerate} +\end{exercise} + +\begin{solution} + \begin{enumerate} + \item + \begin{tikzpicture}[baseline=(a.north), xscale=1, yscale=0.5] + \tkzInit[xmin=0,xmax=5,xstep=1, + ymin=0,ymax=10,ystep=1] + \tkzGrid + \tkzAxeXY + \tkzFct[domain=0:10,color=red,very thick]% + { (-x**2 + 6.1*x - 3.4)*exp(-x) + 3.4 }; + \end{tikzpicture} + \item Il faut dériver $F(x)$ et vérifier que $F'(x) = f(x)$. + \item $\ds \int_0^4 f(x) \; dx = F(4) - F(0) = 14.3 - \frac{1.1}{e^{4}}$ + \item La quantité calculée à la question précédente se retrouve 4fois pour former le bassin. Il faut ensuite prendre en compte l'échelle, comme 1unité de longueur correspond à 15m, une unité d'air correspond à $15\times15 = 225m^2$. Ainsi l'aire du bassin est égale à + \[ + (14.3 - \frac{1.1}{e^{4}})\times 4 \times 15^2 = 12852.00000 + \] + + \end{enumerate} +\end{solution} + +%%% Local Variables: +%%% mode: latex +%%% TeX-master: "master" +%%% End: + +\begin{exercise}[subtitle={Stylos}] + \emph{Les parties {\rm A} et {\rm B} de cet exercice sont indépendantes.} + + \bigskip + + \begin{minipage}{0.6\linewidth} + \textbf{Partie A} + + \medskip + + Deux ateliers A et B fabriquent des stylos pour une entreprise. + + L'atelier A fabrique 47.0\,\% des stylos, et parmi ceux-là, 95.0\,\% possèdent un défaut de fabrication. + + De plus, 50.0\,\% des stylos possèdent un défaut de fabrication et sortent de l'atelier B. + + Un stylo est prélevé au hasard dans le stock de l'entreprise. + + On considère les évènements suivants: + + \begin{itemize} + \item A : \og Le stylo a été fabriqué par l'atelier A \fg + \item B : \og Le stylo a été fabriqué par l'atelier B \fg + \item D : \og Le stylo possède un défaut de fabrication \fg + \end{itemize} + \end{minipage} + \begin{minipage}{0.4\linewidth} + \begin{center} + \begin{tikzpicture}[sloped] + \node {.} + child {node {$A$} + child {node {$D$} + edge from parent + node[above] {...} + } + child {node {$\overline{D}$} + edge from parent + node[above] {...} + } + edge from parent + node[above] {...} + } + child[missing] {} + child { node {$B$} + child {node {$D$} + edge from parent + node[above] {...} + } + child {node {$\overline{D}$} + edge from parent + node[above] {...} + } + edge from parent + node[above] {...} + } ; + \end{tikzpicture} + \end{center} + \end{minipage} + + \medskip + + \begin{enumerate} + \item Compléter l'arbre de probabilité ci-contre + \item Interpréter puis donner les probabilités $P(A)$, $P(B)$, $P_A(D)$ et $P(B \cap D)$. + + \item + \begin{enumerate} + \item Calculer la probabilité qu'un stylo provienne de l'atelier A et possède un défaut de fabrication. + \item En déduire que la probabilité qu'un stylo possède un défaut de fabrication est de $0.95$. + \end{enumerate} + \item On prélève un stylo au hasard avec un défaut. Quelle est la probabilité qu'il vienne de l'atelier A? + \end{enumerate} + + \bigskip + + \textbf{Partie B} + \medskip + + Dans cette partie, on suppose que 95.0\,\% des stylos possèdent un défaut de fabrication. + + L'entreprise confectionne des paquets contenant chacun $4$~stylos. + + Le fait qu'un stylo possède ou non un défaut de fabrication est indépendant des autres stylos. + + On appelle $X$ la variable aléatoire donnant pour un paquet le nombre de stylos qui possèdent un défaut de fabrication. + + On admet que la variable aléatoire $X$ suit une loi binomiale. + + \medskip + + \begin{enumerate} + \setcounter{enumi}{4} + \item Avec quelle loi peut-on modéliser $X$. Préciser les paramètres. + \item Calculer et interpréter la probabilité $P(X = 12)$. + \item Le directeur de l'entreprise affirme qu'il y a plus d'une chance sur deux qu'un paquet ne comporte aucun stylo défectueux. A-t-il raison ? + \item Combien de stylos peut-on espérer avoir en moyenne? + \end{enumerate} + \pagebreak +\end{exercise} + +\begin{solution} + \begin{enumerate} + \item + \begin{center} + \begin{tikzpicture}[sloped] + \node {.} + child {node {$A$} + child {node {$D$} + edge from parent + node[above] {0.95} + } + child {node {$\overline{D}$} + edge from parent + node[above] {0.05} + } + edge from parent + node[above] {0.47} + } + child[missing] {} + child { node {$B$} + child {node {$D$} + edge from parent + node[above] {0.95} + } + child {node {$\overline{D}$} + edge from parent + node[above] {0.05} + } + edge from parent + node[above] {0.53} + } ; + \end{tikzpicture} + \end{center} + \item + \begin{itemize} + \item Probabilité que le stylo vienne de l'atelier A + \[ + P(A) = 0.47 + \] + \item Probabilité que le stylo vienne de l'atelier B + \[ + P(B) = 0.53 + \] + \item Probabilité que le stylo ait un défaut sachant qu'il vient de l'atelier A. + \[ + P_A(D) = 0.95 + \] + \item Probabilité que le stylo vienne de l'atelier B et qu'il ait un défaut. + \[ + P(D \cap D) = 0.5 + \] + \end{itemize} + \item + \begin{enumerate} + \item Probabilité qu'un stylo vienne de l'atelier A et qu'il ait un defaut + \[ + P(A\cap D) = P(A) \times P_A(D) = 0.47 \times 0.95 = 0.45 + \] + \item Probabilité que le stylo ai un défaut de fabrication. + \[ + P(D) = P(A\cap D) + P(B\cap D) = 0.45 + 0.5 = 0.95 + \] + \end{enumerate} + \item Probabilité qu'il vienne de l'atelier A sachant qu'il a un defaut + \[ + P_D(A) = \frac{P(A\cap D)}{P(D)} = \frac{0.45}{0.95} = 0.47 + \] + \item $X$ peut être modélisée par une loi binomiale de paramètres $n=15$ et $p=0.95$. + \item (\textit{par de correction automatique disponible pour le résultat final} + \[ + P(X = 12) = \coefBino{15}{12}\times 0.95^{12} \times 0.05^{3} + \] + \item (\textit{par de correction automatique disponible pour le résultat final} + + Il faut calculer la probabilité qu'il y ait 0 stylo avec un defaut. + \[ + P(X = 0) = \coefBino{15}{0}\times 0.95^{0} \times 0.05^{15} + \] + Puis comparer ce nombre à 0,5. + \item Il faut calculer l'espérance + \[ + E[X] = n\times p = 15 \times 0.95 = 14.25 + \] + \end{enumerate} +\end{solution} + +\end{document} + +%%% Local Variables: +%%% mode: latex +%%% TeX-master: "master" +%%% End: diff --git a/Complementaire/DM/2105_DM1/13_2105_DM1.tex b/Complementaire/DM/2105_DM1/13_2105_DM1.tex new file mode 100644 index 0000000..cb6c192 --- /dev/null +++ b/Complementaire/DM/2105_DM1/13_2105_DM1.tex @@ -0,0 +1,367 @@ +\documentclass[a4paper,10pt]{article} +\usepackage{myXsim} + +% Title Page +\title{DM1 \hfill RODRIGUEZ Teddy} +\tribe{Maths complémentaire} +\date{\hfillÀ render pour le jeudi 27 mai} + +\xsimsetup{ + solution/print = false +} + +\begin{document} +\maketitle + +Les valeurs des exercices sont générés automatiquement. Si une valeur a un nombre adhérant de chiffres après la virgule, vous pouvez l'arrondir à l'entier le plus proche. + +\begin{exercise}[subtitle={Optimisation de matière}] + \begin{minipage}{0.6\textwidth} + On se propose de fabriquer avec le moins de tôle possible une citerne fermée en forme de parallélépipède rectangle dont le volume intérieur doit être de $50m^3$. La longueur est aussi fixée à $5m$ par le cahier des charges. + + On peut donc faire varier uniquement la largeur (notée $x$) et la hauteur (notée $h$) de la cuve. + \end{minipage} + \hfill + \begin{minipage}{0.3\textwidth} + \begin{tikzpicture} + \pgfmathsetmacro{\cubex}{3} + \pgfmathsetmacro{\cubey}{1} + \pgfmathsetmacro{\cubez}{2} + \draw[black,fill=gray] (0,0,0) -- ++(-\cubex,0,0) -- ++(0,-\cubey,0) node [midway, left] {$h$} -- ++(\cubex,0,0) node [midway, below] {$x$} -- cycle; + \draw[black,fill=gray] (0,0,0) -- ++(0,0,-\cubez) -- ++(0,-\cubey,0) -- ++(0,0,\cubez) node [midway, right] {$5m$} -- cycle; + \draw[black,fill=gray] (0,0,0) -- ++(-\cubex,0,0) -- ++(0,0,-\cubez) -- ++(\cubex,0,0) -- cycle; + \end{tikzpicture} + \end{minipage} + + \begin{enumerate} + \item Expliquer pourquoi quand la largeur $x$ change, la hauteur $h$ doit elle aussi changer pour respecter les contraintes. + \item Démontrer que l'on doit avoir $h = \dfrac{10}{x}$. + \item On note $S(x)$ l'aire totale de la citerne (c'est à dire la somme des aires des six faces). Montrer que l'on peut écrire + \[ + S(x) = 10x + 20 + \frac{100}{x} + \] + \item Démontrer que + \[ + S(x) = \frac{10x^2 + 20x + 100}{x} + \] + \item Démontrer que + \[ + S'(x) = \frac{10x^2 - 100}{x^2} + \] + \item En déduire le tableau de variation de $S(x)$ sur $\intOF{0}{10}$. + \item Déterminer les valeurs de $x$ et $h$ correspondant à une utilisation minimal de tôle. + \end{enumerate} +\end{exercise} + +\begin{solution} + \begin{enumerate} + \item Le volume étant fixe si l'on fait varier $x$, $h$ doit aussi varier. + \begin{itemize} + \item Si $x = 2$ alors conserver un volume de $V=50$, $h$ doit être égale à $10 / 2$ + \item Si $x = 3$ alors conserver un volume de $V=50$, $h$ doit être égale à $10 / 3$ + \end{itemize} + \item Pour calculer le volume, on a + \begin{eqnarray*} + V &=& h\times x \times 5 \\ + 50 &=& h\times x \times 5 \\ + x &=& \frac{50}{h\times 5} = \frac{10}{h} + \end{eqnarray*} + \item Pour calculer la surface totale, on ajoute la surface de chaque face. On a donc le calcul suivant + \begin{eqnarray*} + S(x) &=& x\times h \times 2 + x\times5\times2 + h\times 5\times 2\\ + S(x) &=& x\times \frac{10}{x} \times 2 + x\times5\times2 + \frac{10}{x}\times 5\times 2\\ + S(x) &=& 10x + 20 + \frac{100}{x} + \end{eqnarray*} + \item Pour trouver cette nouvelle forme, on met chaque élément sur le même dénominateur + \begin{eqnarray*} + S(x) &=& 10x + 20 + \frac{100}{x}\\ + S(x) &=& \frac{10x\times x}{x} + \frac{20\times x}{x} + \frac{100}{x}\\ + S(x) &=& \frac{10x^2 + 20x + 100}{x} + \end{eqnarray*} + \item On retrouve la formule $\frac{u}{v}$ à dériver + \[ + u(x) = 10x^2 + 20x + 100 \Rightarrow u'(x) = 20x + 20 + \] + \[ + v(x) = x \Rightarrow v'(x) = 1 + \] + Donc au numérateur on obtient + \begin{eqnarray*} + u'(x)\times v(x) - u(x)\times v'(x) &=& (20x + 20)\times x - (10x^2 + 20x + 100)\times 1\\ + &=& 10x^2 - 100 + \end{eqnarray*} + Donc + \[ + S'(x) = \frac{10x^2 - 100}{x^2} + \] + \item Tableau de variations de $S$ + + \begin{itemize} + \item Valeur interdite: $x^2 = 0 \equiv x = 0$ + \item Signe de $10x^2 - 100$: c'est un polynôme du 2e degré + \[ + \Delta = 4000 > 0 + \] + Il y a donc 2 racines + \[ + x_1 = - 3.162277660168379 \qquad + x_2 = 3.162277660168379 + \] + Et on sait que $10x^2 - 100$ est du signe de $a$ donc positif en dehors des racines + \item Le dénominateur $x^2$ est toujours positif. + \item Tableau de variations + + \begin{tikzpicture}[baseline=(a.north)] + \tkzTabInit[lgt=3,espcl=3]{$x$/1,$10x^2 - 100$/1, $x^2$/1, $S'$/1, $S$/2}{$0$, $- 3.162277660168379$, $10$} + \tkzTabLine{d,-, z, +, } + \tkzTabLine{d,+, , +, } + \tkzTabLine{d,-, z, +, } + \tkzTabVar{D+/ , -/ , +/ } + \end{tikzpicture} + + \end{itemize} + \item On a donc une surface minimal pour $x=3.162277660168379$ et $h = 31.622776601683790$. + \end{enumerate} +\end{solution} + +%%% Local Variables: +%%% mode: latex +%%% TeX-master: "master" +%%% End: + +\begin{exercise}[subtitle={Bassin}] + Le tour d'un bassin au niveau du sol présente deux axes de symétrie : l’axe des abscisses et la droite d’équation $x=4$. Il est obtenu par symétrie de la courbe $\mathcal{C}_f$ sur $\intFF{0}{4}$ où $f$ est la fonction définie par + + \[ + f(x) = \left(- x^{2} + 1.0 x - 3.6\right) e^{- x} + 3.6 + \] + On admet que sur $\intFF{0}{4}$ la fonction $f$ est positive. + \begin{enumerate} + \item Sur un repère, tracer l'allure de la courbe $\mathcal{C}_f$, les axes de symétries puis compléter pour dessiner la forme du bassin. + \item Montrer que la fonction $f$ admet comme primitive sur $\R$ la fonction $F$ définie par + \[ + F(x) = 3.6 x + \left( x^{2} + x + 4.6\right) e^{- x} + \] + \item Calculer la quantité $\ds \int_0^4 f(x) \; dx$, vous donnerez le résultat sous forme exacte. Interpréter le résultat et reportez cette quantité sur le graphique. + \item On considère que l'échelle de votre graphique est de 1unité pour 15m. Calculer l'aire du bassin. Vous donnerez un résultat arrondi au $m^2$ près. + \end{enumerate} +\end{exercise} + +\begin{solution} + \begin{enumerate} + \item + \begin{tikzpicture}[baseline=(a.north), xscale=1, yscale=0.5] + \tkzInit[xmin=0,xmax=5,xstep=1, + ymin=0,ymax=10,ystep=1] + \tkzGrid + \tkzAxeXY + \tkzFct[domain=0:10,color=red,very thick]% + { (-x**2 + 1.0*x - 3.6)*exp(-x) + 3.6 }; + \end{tikzpicture} + \item Il faut dériver $F(x)$ et vérifier que $F'(x) = f(x)$. + \item $\ds \int_0^4 f(x) \; dx = F(4) - F(0) = \frac{24.6}{e^{4}} + 9.8$ + \item La quantité calculée à la question précédente se retrouve 4fois pour former le bassin. Il faut ensuite prendre en compte l'échelle, comme 1unité de longueur correspond à 15m, une unité d'air correspond à $15\times15 = 225m^2$. Ainsi l'aire du bassin est égale à + \[ + (\frac{24.6}{e^{4}} + 9.8)\times 4 \times 15^2 = 9226.000000 + \] + + \end{enumerate} +\end{solution} + +%%% Local Variables: +%%% mode: latex +%%% TeX-master: "master" +%%% End: + +\begin{exercise}[subtitle={Stylos}] + \emph{Les parties {\rm A} et {\rm B} de cet exercice sont indépendantes.} + + \bigskip + + \begin{minipage}{0.6\linewidth} + \textbf{Partie A} + + \medskip + + Deux ateliers A et B fabriquent des stylos pour une entreprise. + + L'atelier A fabrique 64.0\,\% des stylos, et parmi ceux-là, 4.0\,\% possèdent un défaut de fabrication. + + De plus, 6.0\,\% des stylos possèdent un défaut de fabrication et sortent de l'atelier B. + + Un stylo est prélevé au hasard dans le stock de l'entreprise. + + On considère les évènements suivants: + + \begin{itemize} + \item A : \og Le stylo a été fabriqué par l'atelier A \fg + \item B : \og Le stylo a été fabriqué par l'atelier B \fg + \item D : \og Le stylo possède un défaut de fabrication \fg + \end{itemize} + \end{minipage} + \begin{minipage}{0.4\linewidth} + \begin{center} + \begin{tikzpicture}[sloped] + \node {.} + child {node {$A$} + child {node {$D$} + edge from parent + node[above] {...} + } + child {node {$\overline{D}$} + edge from parent + node[above] {...} + } + edge from parent + node[above] {...} + } + child[missing] {} + child { node {$B$} + child {node {$D$} + edge from parent + node[above] {...} + } + child {node {$\overline{D}$} + edge from parent + node[above] {...} + } + edge from parent + node[above] {...} + } ; + \end{tikzpicture} + \end{center} + \end{minipage} + + \medskip + + \begin{enumerate} + \item Compléter l'arbre de probabilité ci-contre + \item Interpréter puis donner les probabilités $P(A)$, $P(B)$, $P_A(D)$ et $P(B \cap D)$. + + \item + \begin{enumerate} + \item Calculer la probabilité qu'un stylo provienne de l'atelier A et possède un défaut de fabrication. + \item En déduire que la probabilité qu'un stylo possède un défaut de fabrication est de $0.09$. + \end{enumerate} + \item On prélève un stylo au hasard avec un défaut. Quelle est la probabilité qu'il vienne de l'atelier A? + \end{enumerate} + + \bigskip + + \textbf{Partie B} + \medskip + + Dans cette partie, on suppose que 9.0\,\% des stylos possèdent un défaut de fabrication. + + L'entreprise confectionne des paquets contenant chacun $4$~stylos. + + Le fait qu'un stylo possède ou non un défaut de fabrication est indépendant des autres stylos. + + On appelle $X$ la variable aléatoire donnant pour un paquet le nombre de stylos qui possèdent un défaut de fabrication. + + On admet que la variable aléatoire $X$ suit une loi binomiale. + + \medskip + + \begin{enumerate} + \setcounter{enumi}{4} + \item Avec quelle loi peut-on modéliser $X$. Préciser les paramètres. + \item Calculer et interpréter la probabilité $P(X = 9)$. + \item Le directeur de l'entreprise affirme qu'il y a plus d'une chance sur deux qu'un paquet ne comporte aucun stylo défectueux. A-t-il raison ? + \item Combien de stylos peut-on espérer avoir en moyenne? + \end{enumerate} + \pagebreak +\end{exercise} + +\begin{solution} + \begin{enumerate} + \item + \begin{center} + \begin{tikzpicture}[sloped] + \node {.} + child {node {$A$} + child {node {$D$} + edge from parent + node[above] {0.04} + } + child {node {$\overline{D}$} + edge from parent + node[above] {0.96} + } + edge from parent + node[above] {0.64} + } + child[missing] {} + child { node {$B$} + child {node {$D$} + edge from parent + node[above] {0.18} + } + child {node {$\overline{D}$} + edge from parent + node[above] {0.82} + } + edge from parent + node[above] {0.36} + } ; + \end{tikzpicture} + \end{center} + \item + \begin{itemize} + \item Probabilité que le stylo vienne de l'atelier A + \[ + P(A) = 0.64 + \] + \item Probabilité que le stylo vienne de l'atelier B + \[ + P(B) = 0.36 + \] + \item Probabilité que le stylo ait un défaut sachant qu'il vient de l'atelier A. + \[ + P_A(D) = 0.04 + \] + \item Probabilité que le stylo vienne de l'atelier B et qu'il ait un défaut. + \[ + P(D \cap D) = 0.06 + \] + \end{itemize} + \item + \begin{enumerate} + \item Probabilité qu'un stylo vienne de l'atelier A et qu'il ait un defaut + \[ + P(A\cap D) = P(A) \times P_A(D) = 0.64 \times 0.04 = 0.03 + \] + \item Probabilité que le stylo ai un défaut de fabrication. + \[ + P(D) = P(A\cap D) + P(B\cap D) = 0.03 + 0.06 = 0.09 + \] + \end{enumerate} + \item Probabilité qu'il vienne de l'atelier A sachant qu'il a un defaut + \[ + P_D(A) = \frac{P(A\cap D)}{P(D)} = \frac{0.03}{0.09} = 0.33 + \] + \item $X$ peut être modélisée par une loi binomiale de paramètres $n=16$ et $p=0.09$. + \item (\textit{par de correction automatique disponible pour le résultat final} + \[ + P(X = 9) = \coefBino{16}{9}\times 0.09^{9} \times 0.91^{7} + \] + \item (\textit{par de correction automatique disponible pour le résultat final} + + Il faut calculer la probabilité qu'il y ait 0 stylo avec un defaut. + \[ + P(X = 0) = \coefBino{16}{0}\times 0.09^{0} \times 0.91^{16} + \] + Puis comparer ce nombre à 0,5. + \item Il faut calculer l'espérance + \[ + E[X] = n\times p = 16 \times 0.09 = 1.44 + \] + \end{enumerate} +\end{solution} + +\end{document} + +%%% Local Variables: +%%% mode: latex +%%% TeX-master: "master" +%%% End: diff --git a/Complementaire/DM/2105_DM1/14_2105_DM1.tex b/Complementaire/DM/2105_DM1/14_2105_DM1.tex new file mode 100644 index 0000000..25aaf60 --- /dev/null +++ b/Complementaire/DM/2105_DM1/14_2105_DM1.tex @@ -0,0 +1,367 @@ +\documentclass[a4paper,10pt]{article} +\usepackage{myXsim} + +% Title Page +\title{DM1 \hfill SAINT CYR Louis} +\tribe{Maths complémentaire} +\date{\hfillÀ render pour le jeudi 27 mai} + +\xsimsetup{ + solution/print = false +} + +\begin{document} +\maketitle + +Les valeurs des exercices sont générés automatiquement. Si une valeur a un nombre adhérant de chiffres après la virgule, vous pouvez l'arrondir à l'entier le plus proche. + +\begin{exercise}[subtitle={Optimisation de matière}] + \begin{minipage}{0.6\textwidth} + On se propose de fabriquer avec le moins de tôle possible une citerne fermée en forme de parallélépipède rectangle dont le volume intérieur doit être de $45m^3$. La longueur est aussi fixée à $5m$ par le cahier des charges. + + On peut donc faire varier uniquement la largeur (notée $x$) et la hauteur (notée $h$) de la cuve. + \end{minipage} + \hfill + \begin{minipage}{0.3\textwidth} + \begin{tikzpicture} + \pgfmathsetmacro{\cubex}{3} + \pgfmathsetmacro{\cubey}{1} + \pgfmathsetmacro{\cubez}{2} + \draw[black,fill=gray] (0,0,0) -- ++(-\cubex,0,0) -- ++(0,-\cubey,0) node [midway, left] {$h$} -- ++(\cubex,0,0) node [midway, below] {$x$} -- cycle; + \draw[black,fill=gray] (0,0,0) -- ++(0,0,-\cubez) -- ++(0,-\cubey,0) -- ++(0,0,\cubez) node [midway, right] {$5m$} -- cycle; + \draw[black,fill=gray] (0,0,0) -- ++(-\cubex,0,0) -- ++(0,0,-\cubez) -- ++(\cubex,0,0) -- cycle; + \end{tikzpicture} + \end{minipage} + + \begin{enumerate} + \item Expliquer pourquoi quand la largeur $x$ change, la hauteur $h$ doit elle aussi changer pour respecter les contraintes. + \item Démontrer que l'on doit avoir $h = \dfrac{9}{x}$. + \item On note $S(x)$ l'aire totale de la citerne (c'est à dire la somme des aires des six faces). Montrer que l'on peut écrire + \[ + S(x) = 10x + 18 + \frac{90}{x} + \] + \item Démontrer que + \[ + S(x) = \frac{10x^2 + 18x + 90}{x} + \] + \item Démontrer que + \[ + S'(x) = \frac{10x^2 - 90}{x^2} + \] + \item En déduire le tableau de variation de $S(x)$ sur $\intOF{0}{10}$. + \item Déterminer les valeurs de $x$ et $h$ correspondant à une utilisation minimal de tôle. + \end{enumerate} +\end{exercise} + +\begin{solution} + \begin{enumerate} + \item Le volume étant fixe si l'on fait varier $x$, $h$ doit aussi varier. + \begin{itemize} + \item Si $x = 2$ alors conserver un volume de $V=45$, $h$ doit être égale à $9 / 2$ + \item Si $x = 3$ alors conserver un volume de $V=45$, $h$ doit être égale à $9 / 3$ + \end{itemize} + \item Pour calculer le volume, on a + \begin{eqnarray*} + V &=& h\times x \times 5 \\ + 45 &=& h\times x \times 5 \\ + x &=& \frac{45}{h\times 5} = \frac{9}{h} + \end{eqnarray*} + \item Pour calculer la surface totale, on ajoute la surface de chaque face. On a donc le calcul suivant + \begin{eqnarray*} + S(x) &=& x\times h \times 2 + x\times5\times2 + h\times 5\times 2\\ + S(x) &=& x\times \frac{9}{x} \times 2 + x\times5\times2 + \frac{9}{x}\times 5\times 2\\ + S(x) &=& 10x + 18 + \frac{90}{x} + \end{eqnarray*} + \item Pour trouver cette nouvelle forme, on met chaque élément sur le même dénominateur + \begin{eqnarray*} + S(x) &=& 10x + 18 + \frac{90}{x}\\ + S(x) &=& \frac{10x\times x}{x} + \frac{18\times x}{x} + \frac{90}{x}\\ + S(x) &=& \frac{10x^2 + 18x + 90}{x} + \end{eqnarray*} + \item On retrouve la formule $\frac{u}{v}$ à dériver + \[ + u(x) = 10x^2 + 18x + 90 \Rightarrow u'(x) = 20x + 18 + \] + \[ + v(x) = x \Rightarrow v'(x) = 1 + \] + Donc au numérateur on obtient + \begin{eqnarray*} + u'(x)\times v(x) - u(x)\times v'(x) &=& (20x + 18)\times x - (10x^2 + 18x + 90)\times 1\\ + &=& 10x^2 - 90 + \end{eqnarray*} + Donc + \[ + S'(x) = \frac{10x^2 - 90}{x^2} + \] + \item Tableau de variations de $S$ + + \begin{itemize} + \item Valeur interdite: $x^2 = 0 \equiv x = 0$ + \item Signe de $10x^2 - 90$: c'est un polynôme du 2e degré + \[ + \Delta = 3600 > 0 + \] + Il y a donc 2 racines + \[ + x_1 = - 3 \qquad + x_2 = 3 + \] + Et on sait que $10x^2 - 90$ est du signe de $a$ donc positif en dehors des racines + \item Le dénominateur $x^2$ est toujours positif. + \item Tableau de variations + + \begin{tikzpicture}[baseline=(a.north)] + \tkzTabInit[lgt=3,espcl=3]{$x$/1,$10x^2 - 90$/1, $x^2$/1, $S'$/1, $S$/2}{$0$, $- 3$, $10$} + \tkzTabLine{d,-, z, +, } + \tkzTabLine{d,+, , +, } + \tkzTabLine{d,-, z, +, } + \tkzTabVar{D+/ , -/ , +/ } + \end{tikzpicture} + + \end{itemize} + \item On a donc une surface minimal pour $x=3$ et $h = 27$. + \end{enumerate} +\end{solution} + +%%% Local Variables: +%%% mode: latex +%%% TeX-master: "master" +%%% End: + +\begin{exercise}[subtitle={Bassin}] + Le tour d'un bassin au niveau du sol présente deux axes de symétrie : l’axe des abscisses et la droite d’équation $x=4$. Il est obtenu par symétrie de la courbe $\mathcal{C}_f$ sur $\intFF{0}{4}$ où $f$ est la fonction définie par + + \[ + f(x) = \left(- x^{2} + 8.0 x - 0.9\right) e^{- x} + 0.9 + \] + On admet que sur $\intFF{0}{4}$ la fonction $f$ est positive. + \begin{enumerate} + \item Sur un repère, tracer l'allure de la courbe $\mathcal{C}_f$, les axes de symétries puis compléter pour dessiner la forme du bassin. + \item Montrer que la fonction $f$ admet comme primitive sur $\R$ la fonction $F$ définie par + \[ + F(x) = 0.9 x + \left( x^{2} - 6.0 x - 5.1\right) e^{- x} + \] + \item Calculer la quantité $\ds \int_0^4 f(x) \; dx$, vous donnerez le résultat sous forme exacte. Interpréter le résultat et reportez cette quantité sur le graphique. + \item On considère que l'échelle de votre graphique est de 1unité pour 15m. Calculer l'aire du bassin. Vous donnerez un résultat arrondi au $m^2$ près. + \end{enumerate} +\end{exercise} + +\begin{solution} + \begin{enumerate} + \item + \begin{tikzpicture}[baseline=(a.north), xscale=1, yscale=0.5] + \tkzInit[xmin=0,xmax=5,xstep=1, + ymin=0,ymax=10,ystep=1] + \tkzGrid + \tkzAxeXY + \tkzFct[domain=0:10,color=red,very thick]% + { (-x**2 + 8.0*x - 0.9)*exp(-x) + 0.9 }; + \end{tikzpicture} + \item Il faut dériver $F(x)$ et vérifier que $F'(x) = f(x)$. + \item $\ds \int_0^4 f(x) \; dx = F(4) - F(0) = 8.7 - \frac{13.1}{e^{4}}$ + \item La quantité calculée à la question précédente se retrouve 4fois pour former le bassin. Il faut ensuite prendre en compte l'échelle, comme 1unité de longueur correspond à 15m, une unité d'air correspond à $15\times15 = 225m^2$. Ainsi l'aire du bassin est égale à + \[ + (8.7 - \frac{13.1}{e^{4}})\times 4 \times 15^2 = 7614.000000 + \] + + \end{enumerate} +\end{solution} + +%%% Local Variables: +%%% mode: latex +%%% TeX-master: "master" +%%% End: + +\begin{exercise}[subtitle={Stylos}] + \emph{Les parties {\rm A} et {\rm B} de cet exercice sont indépendantes.} + + \bigskip + + \begin{minipage}{0.6\linewidth} + \textbf{Partie A} + + \medskip + + Deux ateliers A et B fabriquent des stylos pour une entreprise. + + L'atelier A fabrique 88.0\,\% des stylos, et parmi ceux-là, 75.0\,\% possèdent un défaut de fabrication. + + De plus, 4.0\,\% des stylos possèdent un défaut de fabrication et sortent de l'atelier B. + + Un stylo est prélevé au hasard dans le stock de l'entreprise. + + On considère les évènements suivants: + + \begin{itemize} + \item A : \og Le stylo a été fabriqué par l'atelier A \fg + \item B : \og Le stylo a été fabriqué par l'atelier B \fg + \item D : \og Le stylo possède un défaut de fabrication \fg + \end{itemize} + \end{minipage} + \begin{minipage}{0.4\linewidth} + \begin{center} + \begin{tikzpicture}[sloped] + \node {.} + child {node {$A$} + child {node {$D$} + edge from parent + node[above] {...} + } + child {node {$\overline{D}$} + edge from parent + node[above] {...} + } + edge from parent + node[above] {...} + } + child[missing] {} + child { node {$B$} + child {node {$D$} + edge from parent + node[above] {...} + } + child {node {$\overline{D}$} + edge from parent + node[above] {...} + } + edge from parent + node[above] {...} + } ; + \end{tikzpicture} + \end{center} + \end{minipage} + + \medskip + + \begin{enumerate} + \item Compléter l'arbre de probabilité ci-contre + \item Interpréter puis donner les probabilités $P(A)$, $P(B)$, $P_A(D)$ et $P(B \cap D)$. + + \item + \begin{enumerate} + \item Calculer la probabilité qu'un stylo provienne de l'atelier A et possède un défaut de fabrication. + \item En déduire que la probabilité qu'un stylo possède un défaut de fabrication est de $0.7$. + \end{enumerate} + \item On prélève un stylo au hasard avec un défaut. Quelle est la probabilité qu'il vienne de l'atelier A? + \end{enumerate} + + \bigskip + + \textbf{Partie B} + \medskip + + Dans cette partie, on suppose que 70.0\,\% des stylos possèdent un défaut de fabrication. + + L'entreprise confectionne des paquets contenant chacun $4$~stylos. + + Le fait qu'un stylo possède ou non un défaut de fabrication est indépendant des autres stylos. + + On appelle $X$ la variable aléatoire donnant pour un paquet le nombre de stylos qui possèdent un défaut de fabrication. + + On admet que la variable aléatoire $X$ suit une loi binomiale. + + \medskip + + \begin{enumerate} + \setcounter{enumi}{4} + \item Avec quelle loi peut-on modéliser $X$. Préciser les paramètres. + \item Calculer et interpréter la probabilité $P(X = 14)$. + \item Le directeur de l'entreprise affirme qu'il y a plus d'une chance sur deux qu'un paquet ne comporte aucun stylo défectueux. A-t-il raison ? + \item Combien de stylos peut-on espérer avoir en moyenne? + \end{enumerate} + \pagebreak +\end{exercise} + +\begin{solution} + \begin{enumerate} + \item + \begin{center} + \begin{tikzpicture}[sloped] + \node {.} + child {node {$A$} + child {node {$D$} + edge from parent + node[above] {0.75} + } + child {node {$\overline{D}$} + edge from parent + node[above] {0.25} + } + edge from parent + node[above] {0.88} + } + child[missing] {} + child { node {$B$} + child {node {$D$} + edge from parent + node[above] {0.34} + } + child {node {$\overline{D}$} + edge from parent + node[above] {0.66} + } + edge from parent + node[above] {0.12} + } ; + \end{tikzpicture} + \end{center} + \item + \begin{itemize} + \item Probabilité que le stylo vienne de l'atelier A + \[ + P(A) = 0.88 + \] + \item Probabilité que le stylo vienne de l'atelier B + \[ + P(B) = 0.12 + \] + \item Probabilité que le stylo ait un défaut sachant qu'il vient de l'atelier A. + \[ + P_A(D) = 0.75 + \] + \item Probabilité que le stylo vienne de l'atelier B et qu'il ait un défaut. + \[ + P(D \cap D) = 0.04 + \] + \end{itemize} + \item + \begin{enumerate} + \item Probabilité qu'un stylo vienne de l'atelier A et qu'il ait un defaut + \[ + P(A\cap D) = P(A) \times P_A(D) = 0.88 \times 0.75 = 0.66 + \] + \item Probabilité que le stylo ai un défaut de fabrication. + \[ + P(D) = P(A\cap D) + P(B\cap D) = 0.66 + 0.04 = 0.7 + \] + \end{enumerate} + \item Probabilité qu'il vienne de l'atelier A sachant qu'il a un defaut + \[ + P_D(A) = \frac{P(A\cap D)}{P(D)} = \frac{0.66}{0.7} = 0.94 + \] + \item $X$ peut être modélisée par une loi binomiale de paramètres $n=15$ et $p=0.7$. + \item (\textit{par de correction automatique disponible pour le résultat final} + \[ + P(X = 14) = \coefBino{15}{14}\times 0.7^{14} \times 0.3^{1} + \] + \item (\textit{par de correction automatique disponible pour le résultat final} + + Il faut calculer la probabilité qu'il y ait 0 stylo avec un defaut. + \[ + P(X = 0) = \coefBino{15}{0}\times 0.7^{0} \times 0.3^{15} + \] + Puis comparer ce nombre à 0,5. + \item Il faut calculer l'espérance + \[ + E[X] = n\times p = 15 \times 0.7 = 10.5 + \] + \end{enumerate} +\end{solution} + +\end{document} + +%%% Local Variables: +%%% mode: latex +%%% TeX-master: "master" +%%% End: diff --git a/Complementaire/DM/2105_DM1/15_2105_DM1.tex b/Complementaire/DM/2105_DM1/15_2105_DM1.tex new file mode 100644 index 0000000..1881db7 --- /dev/null +++ b/Complementaire/DM/2105_DM1/15_2105_DM1.tex @@ -0,0 +1,367 @@ +\documentclass[a4paper,10pt]{article} +\usepackage{myXsim} + +% Title Page +\title{DM1 \hfill SAVIN Lou-Ann} +\tribe{Maths complémentaire} +\date{\hfillÀ render pour le jeudi 27 mai} + +\xsimsetup{ + solution/print = false +} + +\begin{document} +\maketitle + +Les valeurs des exercices sont générés automatiquement. Si une valeur a un nombre adhérant de chiffres après la virgule, vous pouvez l'arrondir à l'entier le plus proche. + +\begin{exercise}[subtitle={Optimisation de matière}] + \begin{minipage}{0.6\textwidth} + On se propose de fabriquer avec le moins de tôle possible une citerne fermée en forme de parallélépipède rectangle dont le volume intérieur doit être de $18m^3$. La longueur est aussi fixée à $3m$ par le cahier des charges. + + On peut donc faire varier uniquement la largeur (notée $x$) et la hauteur (notée $h$) de la cuve. + \end{minipage} + \hfill + \begin{minipage}{0.3\textwidth} + \begin{tikzpicture} + \pgfmathsetmacro{\cubex}{3} + \pgfmathsetmacro{\cubey}{1} + \pgfmathsetmacro{\cubez}{2} + \draw[black,fill=gray] (0,0,0) -- ++(-\cubex,0,0) -- ++(0,-\cubey,0) node [midway, left] {$h$} -- ++(\cubex,0,0) node [midway, below] {$x$} -- cycle; + \draw[black,fill=gray] (0,0,0) -- ++(0,0,-\cubez) -- ++(0,-\cubey,0) -- ++(0,0,\cubez) node [midway, right] {$3m$} -- cycle; + \draw[black,fill=gray] (0,0,0) -- ++(-\cubex,0,0) -- ++(0,0,-\cubez) -- ++(\cubex,0,0) -- cycle; + \end{tikzpicture} + \end{minipage} + + \begin{enumerate} + \item Expliquer pourquoi quand la largeur $x$ change, la hauteur $h$ doit elle aussi changer pour respecter les contraintes. + \item Démontrer que l'on doit avoir $h = \dfrac{6}{x}$. + \item On note $S(x)$ l'aire totale de la citerne (c'est à dire la somme des aires des six faces). Montrer que l'on peut écrire + \[ + S(x) = 6x + 12 + \frac{36}{x} + \] + \item Démontrer que + \[ + S(x) = \frac{6x^2 + 12x + 36}{x} + \] + \item Démontrer que + \[ + S'(x) = \frac{6x^2 - 36}{x^2} + \] + \item En déduire le tableau de variation de $S(x)$ sur $\intOF{0}{10}$. + \item Déterminer les valeurs de $x$ et $h$ correspondant à une utilisation minimal de tôle. + \end{enumerate} +\end{exercise} + +\begin{solution} + \begin{enumerate} + \item Le volume étant fixe si l'on fait varier $x$, $h$ doit aussi varier. + \begin{itemize} + \item Si $x = 2$ alors conserver un volume de $V=18$, $h$ doit être égale à $6 / 2$ + \item Si $x = 3$ alors conserver un volume de $V=18$, $h$ doit être égale à $6 / 3$ + \end{itemize} + \item Pour calculer le volume, on a + \begin{eqnarray*} + V &=& h\times x \times 3 \\ + 18 &=& h\times x \times 3 \\ + x &=& \frac{18}{h\times 3} = \frac{6}{h} + \end{eqnarray*} + \item Pour calculer la surface totale, on ajoute la surface de chaque face. On a donc le calcul suivant + \begin{eqnarray*} + S(x) &=& x\times h \times 2 + x\times3\times2 + h\times 3\times 2\\ + S(x) &=& x\times \frac{6}{x} \times 2 + x\times3\times2 + \frac{6}{x}\times 3\times 2\\ + S(x) &=& 6x + 12 + \frac{36}{x} + \end{eqnarray*} + \item Pour trouver cette nouvelle forme, on met chaque élément sur le même dénominateur + \begin{eqnarray*} + S(x) &=& 6x + 12 + \frac{36}{x}\\ + S(x) &=& \frac{6x\times x}{x} + \frac{12\times x}{x} + \frac{36}{x}\\ + S(x) &=& \frac{6x^2 + 12x + 36}{x} + \end{eqnarray*} + \item On retrouve la formule $\frac{u}{v}$ à dériver + \[ + u(x) = 6x^2 + 12x + 36 \Rightarrow u'(x) = 12x + 12 + \] + \[ + v(x) = x \Rightarrow v'(x) = 1 + \] + Donc au numérateur on obtient + \begin{eqnarray*} + u'(x)\times v(x) - u(x)\times v'(x) &=& (12x + 12)\times x - (6x^2 + 12x + 36)\times 1\\ + &=& 6x^2 - 36 + \end{eqnarray*} + Donc + \[ + S'(x) = \frac{6x^2 - 36}{x^2} + \] + \item Tableau de variations de $S$ + + \begin{itemize} + \item Valeur interdite: $x^2 = 0 \equiv x = 0$ + \item Signe de $6x^2 - 36$: c'est un polynôme du 2e degré + \[ + \Delta = 864 > 0 + \] + Il y a donc 2 racines + \[ + x_1 = - 2.4494897427831783 \qquad + x_2 = 2.4494897427831783 + \] + Et on sait que $6x^2 - 36$ est du signe de $a$ donc positif en dehors des racines + \item Le dénominateur $x^2$ est toujours positif. + \item Tableau de variations + + \begin{tikzpicture}[baseline=(a.north)] + \tkzTabInit[lgt=3,espcl=3]{$x$/1,$6x^2 - 36$/1, $x^2$/1, $S'$/1, $S$/2}{$0$, $- 2.4494897427831783$, $10$} + \tkzTabLine{d,-, z, +, } + \tkzTabLine{d,+, , +, } + \tkzTabLine{d,-, z, +, } + \tkzTabVar{D+/ , -/ , +/ } + \end{tikzpicture} + + \end{itemize} + \item On a donc une surface minimal pour $x=2.4494897427831783$ et $h = 14.6969384566990698$. + \end{enumerate} +\end{solution} + +%%% Local Variables: +%%% mode: latex +%%% TeX-master: "master" +%%% End: + +\begin{exercise}[subtitle={Bassin}] + Le tour d'un bassin au niveau du sol présente deux axes de symétrie : l’axe des abscisses et la droite d’équation $x=4$. Il est obtenu par symétrie de la courbe $\mathcal{C}_f$ sur $\intFF{0}{4}$ où $f$ est la fonction définie par + + \[ + f(x) = \left(- x^{2} + 2.7 x - 6.1\right) e^{- x} + 6.1 + \] + On admet que sur $\intFF{0}{4}$ la fonction $f$ est positive. + \begin{enumerate} + \item Sur un repère, tracer l'allure de la courbe $\mathcal{C}_f$, les axes de symétries puis compléter pour dessiner la forme du bassin. + \item Montrer que la fonction $f$ admet comme primitive sur $\R$ la fonction $F$ définie par + \[ + F(x) = 6.1 x + \left( x^{2} - 0.7 x + 5.4\right) e^{- x} + \] + \item Calculer la quantité $\ds \int_0^4 f(x) \; dx$, vous donnerez le résultat sous forme exacte. Interpréter le résultat et reportez cette quantité sur le graphique. + \item On considère que l'échelle de votre graphique est de 1unité pour 15m. Calculer l'aire du bassin. Vous donnerez un résultat arrondi au $m^2$ près. + \end{enumerate} +\end{exercise} + +\begin{solution} + \begin{enumerate} + \item + \begin{tikzpicture}[baseline=(a.north), xscale=1, yscale=0.5] + \tkzInit[xmin=0,xmax=5,xstep=1, + ymin=0,ymax=10,ystep=1] + \tkzGrid + \tkzAxeXY + \tkzFct[domain=0:10,color=red,very thick]% + { (-x**2 + 2.7*x - 6.1)*exp(-x) + 6.1 }; + \end{tikzpicture} + \item Il faut dériver $F(x)$ et vérifier que $F'(x) = f(x)$. + \item $\ds \int_0^4 f(x) \; dx = F(4) - F(0) = \frac{18.6}{e^{4}} + 19.0$ + \item La quantité calculée à la question précédente se retrouve 4fois pour former le bassin. Il faut ensuite prendre en compte l'échelle, comme 1unité de longueur correspond à 15m, une unité d'air correspond à $15\times15 = 225m^2$. Ainsi l'aire du bassin est égale à + \[ + (\frac{18.6}{e^{4}} + 19.0)\times 4 \times 15^2 = 17407.00000 + \] + + \end{enumerate} +\end{solution} + +%%% Local Variables: +%%% mode: latex +%%% TeX-master: "master" +%%% End: + +\begin{exercise}[subtitle={Stylos}] + \emph{Les parties {\rm A} et {\rm B} de cet exercice sont indépendantes.} + + \bigskip + + \begin{minipage}{0.6\linewidth} + \textbf{Partie A} + + \medskip + + Deux ateliers A et B fabriquent des stylos pour une entreprise. + + L'atelier A fabrique 47.0\,\% des stylos, et parmi ceux-là, 45.0\,\% possèdent un défaut de fabrication. + + De plus, 3.0\,\% des stylos possèdent un défaut de fabrication et sortent de l'atelier B. + + Un stylo est prélevé au hasard dans le stock de l'entreprise. + + On considère les évènements suivants: + + \begin{itemize} + \item A : \og Le stylo a été fabriqué par l'atelier A \fg + \item B : \og Le stylo a été fabriqué par l'atelier B \fg + \item D : \og Le stylo possède un défaut de fabrication \fg + \end{itemize} + \end{minipage} + \begin{minipage}{0.4\linewidth} + \begin{center} + \begin{tikzpicture}[sloped] + \node {.} + child {node {$A$} + child {node {$D$} + edge from parent + node[above] {...} + } + child {node {$\overline{D}$} + edge from parent + node[above] {...} + } + edge from parent + node[above] {...} + } + child[missing] {} + child { node {$B$} + child {node {$D$} + edge from parent + node[above] {...} + } + child {node {$\overline{D}$} + edge from parent + node[above] {...} + } + edge from parent + node[above] {...} + } ; + \end{tikzpicture} + \end{center} + \end{minipage} + + \medskip + + \begin{enumerate} + \item Compléter l'arbre de probabilité ci-contre + \item Interpréter puis donner les probabilités $P(A)$, $P(B)$, $P_A(D)$ et $P(B \cap D)$. + + \item + \begin{enumerate} + \item Calculer la probabilité qu'un stylo provienne de l'atelier A et possède un défaut de fabrication. + \item En déduire que la probabilité qu'un stylo possède un défaut de fabrication est de $0.24$. + \end{enumerate} + \item On prélève un stylo au hasard avec un défaut. Quelle est la probabilité qu'il vienne de l'atelier A? + \end{enumerate} + + \bigskip + + \textbf{Partie B} + \medskip + + Dans cette partie, on suppose que 24.0\,\% des stylos possèdent un défaut de fabrication. + + L'entreprise confectionne des paquets contenant chacun $4$~stylos. + + Le fait qu'un stylo possède ou non un défaut de fabrication est indépendant des autres stylos. + + On appelle $X$ la variable aléatoire donnant pour un paquet le nombre de stylos qui possèdent un défaut de fabrication. + + On admet que la variable aléatoire $X$ suit une loi binomiale. + + \medskip + + \begin{enumerate} + \setcounter{enumi}{4} + \item Avec quelle loi peut-on modéliser $X$. Préciser les paramètres. + \item Calculer et interpréter la probabilité $P(X = 9)$. + \item Le directeur de l'entreprise affirme qu'il y a plus d'une chance sur deux qu'un paquet ne comporte aucun stylo défectueux. A-t-il raison ? + \item Combien de stylos peut-on espérer avoir en moyenne? + \end{enumerate} + \pagebreak +\end{exercise} + +\begin{solution} + \begin{enumerate} + \item + \begin{center} + \begin{tikzpicture}[sloped] + \node {.} + child {node {$A$} + child {node {$D$} + edge from parent + node[above] {0.45} + } + child {node {$\overline{D}$} + edge from parent + node[above] {0.55} + } + edge from parent + node[above] {0.47} + } + child[missing] {} + child { node {$B$} + child {node {$D$} + edge from parent + node[above] {0.05} + } + child {node {$\overline{D}$} + edge from parent + node[above] {0.95} + } + edge from parent + node[above] {0.53} + } ; + \end{tikzpicture} + \end{center} + \item + \begin{itemize} + \item Probabilité que le stylo vienne de l'atelier A + \[ + P(A) = 0.47 + \] + \item Probabilité que le stylo vienne de l'atelier B + \[ + P(B) = 0.53 + \] + \item Probabilité que le stylo ait un défaut sachant qu'il vient de l'atelier A. + \[ + P_A(D) = 0.45 + \] + \item Probabilité que le stylo vienne de l'atelier B et qu'il ait un défaut. + \[ + P(D \cap D) = 0.03 + \] + \end{itemize} + \item + \begin{enumerate} + \item Probabilité qu'un stylo vienne de l'atelier A et qu'il ait un defaut + \[ + P(A\cap D) = P(A) \times P_A(D) = 0.47 \times 0.45 = 0.21 + \] + \item Probabilité que le stylo ai un défaut de fabrication. + \[ + P(D) = P(A\cap D) + P(B\cap D) = 0.21 + 0.03 = 0.24 + \] + \end{enumerate} + \item Probabilité qu'il vienne de l'atelier A sachant qu'il a un defaut + \[ + P_D(A) = \frac{P(A\cap D)}{P(D)} = \frac{0.21}{0.24} = 0.88 + \] + \item $X$ peut être modélisée par une loi binomiale de paramètres $n=19$ et $p=0.24$. + \item (\textit{par de correction automatique disponible pour le résultat final} + \[ + P(X = 9) = \coefBino{19}{9}\times 0.24^{9} \times 0.76^{10} + \] + \item (\textit{par de correction automatique disponible pour le résultat final} + + Il faut calculer la probabilité qu'il y ait 0 stylo avec un defaut. + \[ + P(X = 0) = \coefBino{19}{0}\times 0.24^{0} \times 0.76^{19} + \] + Puis comparer ce nombre à 0,5. + \item Il faut calculer l'espérance + \[ + E[X] = n\times p = 19 \times 0.24 = 4.56 + \] + \end{enumerate} +\end{solution} + +\end{document} + +%%% Local Variables: +%%% mode: latex +%%% TeX-master: "master" +%%% End: diff --git a/Complementaire/DM/2105_DM1/16_2105_DM1.tex b/Complementaire/DM/2105_DM1/16_2105_DM1.tex new file mode 100644 index 0000000..0b951c8 --- /dev/null +++ b/Complementaire/DM/2105_DM1/16_2105_DM1.tex @@ -0,0 +1,367 @@ +\documentclass[a4paper,10pt]{article} +\usepackage{myXsim} + +% Title Page +\title{DM1 \hfill SILVA LOPES Katleen} +\tribe{Maths complémentaire} +\date{\hfillÀ render pour le jeudi 27 mai} + +\xsimsetup{ + solution/print = false +} + +\begin{document} +\maketitle + +Les valeurs des exercices sont générés automatiquement. Si une valeur a un nombre adhérant de chiffres après la virgule, vous pouvez l'arrondir à l'entier le plus proche. + +\begin{exercise}[subtitle={Optimisation de matière}] + \begin{minipage}{0.6\textwidth} + On se propose de fabriquer avec le moins de tôle possible une citerne fermée en forme de parallélépipède rectangle dont le volume intérieur doit être de $27m^3$. La longueur est aussi fixée à $3m$ par le cahier des charges. + + On peut donc faire varier uniquement la largeur (notée $x$) et la hauteur (notée $h$) de la cuve. + \end{minipage} + \hfill + \begin{minipage}{0.3\textwidth} + \begin{tikzpicture} + \pgfmathsetmacro{\cubex}{3} + \pgfmathsetmacro{\cubey}{1} + \pgfmathsetmacro{\cubez}{2} + \draw[black,fill=gray] (0,0,0) -- ++(-\cubex,0,0) -- ++(0,-\cubey,0) node [midway, left] {$h$} -- ++(\cubex,0,0) node [midway, below] {$x$} -- cycle; + \draw[black,fill=gray] (0,0,0) -- ++(0,0,-\cubez) -- ++(0,-\cubey,0) -- ++(0,0,\cubez) node [midway, right] {$3m$} -- cycle; + \draw[black,fill=gray] (0,0,0) -- ++(-\cubex,0,0) -- ++(0,0,-\cubez) -- ++(\cubex,0,0) -- cycle; + \end{tikzpicture} + \end{minipage} + + \begin{enumerate} + \item Expliquer pourquoi quand la largeur $x$ change, la hauteur $h$ doit elle aussi changer pour respecter les contraintes. + \item Démontrer que l'on doit avoir $h = \dfrac{9}{x}$. + \item On note $S(x)$ l'aire totale de la citerne (c'est à dire la somme des aires des six faces). Montrer que l'on peut écrire + \[ + S(x) = 6x + 18 + \frac{54}{x} + \] + \item Démontrer que + \[ + S(x) = \frac{6x^2 + 18x + 54}{x} + \] + \item Démontrer que + \[ + S'(x) = \frac{6x^2 - 54}{x^2} + \] + \item En déduire le tableau de variation de $S(x)$ sur $\intOF{0}{10}$. + \item Déterminer les valeurs de $x$ et $h$ correspondant à une utilisation minimal de tôle. + \end{enumerate} +\end{exercise} + +\begin{solution} + \begin{enumerate} + \item Le volume étant fixe si l'on fait varier $x$, $h$ doit aussi varier. + \begin{itemize} + \item Si $x = 2$ alors conserver un volume de $V=27$, $h$ doit être égale à $9 / 2$ + \item Si $x = 3$ alors conserver un volume de $V=27$, $h$ doit être égale à $9 / 3$ + \end{itemize} + \item Pour calculer le volume, on a + \begin{eqnarray*} + V &=& h\times x \times 3 \\ + 27 &=& h\times x \times 3 \\ + x &=& \frac{27}{h\times 3} = \frac{9}{h} + \end{eqnarray*} + \item Pour calculer la surface totale, on ajoute la surface de chaque face. On a donc le calcul suivant + \begin{eqnarray*} + S(x) &=& x\times h \times 2 + x\times3\times2 + h\times 3\times 2\\ + S(x) &=& x\times \frac{9}{x} \times 2 + x\times3\times2 + \frac{9}{x}\times 3\times 2\\ + S(x) &=& 6x + 18 + \frac{54}{x} + \end{eqnarray*} + \item Pour trouver cette nouvelle forme, on met chaque élément sur le même dénominateur + \begin{eqnarray*} + S(x) &=& 6x + 18 + \frac{54}{x}\\ + S(x) &=& \frac{6x\times x}{x} + \frac{18\times x}{x} + \frac{54}{x}\\ + S(x) &=& \frac{6x^2 + 18x + 54}{x} + \end{eqnarray*} + \item On retrouve la formule $\frac{u}{v}$ à dériver + \[ + u(x) = 6x^2 + 18x + 54 \Rightarrow u'(x) = 12x + 18 + \] + \[ + v(x) = x \Rightarrow v'(x) = 1 + \] + Donc au numérateur on obtient + \begin{eqnarray*} + u'(x)\times v(x) - u(x)\times v'(x) &=& (12x + 18)\times x - (6x^2 + 18x + 54)\times 1\\ + &=& 6x^2 - 54 + \end{eqnarray*} + Donc + \[ + S'(x) = \frac{6x^2 - 54}{x^2} + \] + \item Tableau de variations de $S$ + + \begin{itemize} + \item Valeur interdite: $x^2 = 0 \equiv x = 0$ + \item Signe de $6x^2 - 54$: c'est un polynôme du 2e degré + \[ + \Delta = 1296 > 0 + \] + Il y a donc 2 racines + \[ + x_1 = - 3 \qquad + x_2 = 3 + \] + Et on sait que $6x^2 - 54$ est du signe de $a$ donc positif en dehors des racines + \item Le dénominateur $x^2$ est toujours positif. + \item Tableau de variations + + \begin{tikzpicture}[baseline=(a.north)] + \tkzTabInit[lgt=3,espcl=3]{$x$/1,$6x^2 - 54$/1, $x^2$/1, $S'$/1, $S$/2}{$0$, $- 3$, $10$} + \tkzTabLine{d,-, z, +, } + \tkzTabLine{d,+, , +, } + \tkzTabLine{d,-, z, +, } + \tkzTabVar{D+/ , -/ , +/ } + \end{tikzpicture} + + \end{itemize} + \item On a donc une surface minimal pour $x=3$ et $h = 27$. + \end{enumerate} +\end{solution} + +%%% Local Variables: +%%% mode: latex +%%% TeX-master: "master" +%%% End: + +\begin{exercise}[subtitle={Bassin}] + Le tour d'un bassin au niveau du sol présente deux axes de symétrie : l’axe des abscisses et la droite d’équation $x=4$. Il est obtenu par symétrie de la courbe $\mathcal{C}_f$ sur $\intFF{0}{4}$ où $f$ est la fonction définie par + + \[ + f(x) = \left(- x^{2} + 3.0 x - 6.6\right) e^{- x} + 6.6 + \] + On admet que sur $\intFF{0}{4}$ la fonction $f$ est positive. + \begin{enumerate} + \item Sur un repère, tracer l'allure de la courbe $\mathcal{C}_f$, les axes de symétries puis compléter pour dessiner la forme du bassin. + \item Montrer que la fonction $f$ admet comme primitive sur $\R$ la fonction $F$ définie par + \[ + F(x) = 6.6 x + \left( x^{2} - x + 5.6\right) e^{- x} + \] + \item Calculer la quantité $\ds \int_0^4 f(x) \; dx$, vous donnerez le résultat sous forme exacte. Interpréter le résultat et reportez cette quantité sur le graphique. + \item On considère que l'échelle de votre graphique est de 1unité pour 15m. Calculer l'aire du bassin. Vous donnerez un résultat arrondi au $m^2$ près. + \end{enumerate} +\end{exercise} + +\begin{solution} + \begin{enumerate} + \item + \begin{tikzpicture}[baseline=(a.north), xscale=1, yscale=0.5] + \tkzInit[xmin=0,xmax=5,xstep=1, + ymin=0,ymax=10,ystep=1] + \tkzGrid + \tkzAxeXY + \tkzFct[domain=0:10,color=red,very thick]% + { (-x**2 + 3.0*x - 6.6)*exp(-x) + 6.6 }; + \end{tikzpicture} + \item Il faut dériver $F(x)$ et vérifier que $F'(x) = f(x)$. + \item $\ds \int_0^4 f(x) \; dx = F(4) - F(0) = \frac{17.6}{e^{4}} + 20.8$ + \item La quantité calculée à la question précédente se retrouve 4fois pour former le bassin. Il faut ensuite prendre en compte l'échelle, comme 1unité de longueur correspond à 15m, une unité d'air correspond à $15\times15 = 225m^2$. Ainsi l'aire du bassin est égale à + \[ + (\frac{17.6}{e^{4}} + 20.8)\times 4 \times 15^2 = 19010.00000 + \] + + \end{enumerate} +\end{solution} + +%%% Local Variables: +%%% mode: latex +%%% TeX-master: "master" +%%% End: + +\begin{exercise}[subtitle={Stylos}] + \emph{Les parties {\rm A} et {\rm B} de cet exercice sont indépendantes.} + + \bigskip + + \begin{minipage}{0.6\linewidth} + \textbf{Partie A} + + \medskip + + Deux ateliers A et B fabriquent des stylos pour une entreprise. + + L'atelier A fabrique 43.0\,\% des stylos, et parmi ceux-là, 31.0\,\% possèdent un défaut de fabrication. + + De plus, 19.0\,\% des stylos possèdent un défaut de fabrication et sortent de l'atelier B. + + Un stylo est prélevé au hasard dans le stock de l'entreprise. + + On considère les évènements suivants: + + \begin{itemize} + \item A : \og Le stylo a été fabriqué par l'atelier A \fg + \item B : \og Le stylo a été fabriqué par l'atelier B \fg + \item D : \og Le stylo possède un défaut de fabrication \fg + \end{itemize} + \end{minipage} + \begin{minipage}{0.4\linewidth} + \begin{center} + \begin{tikzpicture}[sloped] + \node {.} + child {node {$A$} + child {node {$D$} + edge from parent + node[above] {...} + } + child {node {$\overline{D}$} + edge from parent + node[above] {...} + } + edge from parent + node[above] {...} + } + child[missing] {} + child { node {$B$} + child {node {$D$} + edge from parent + node[above] {...} + } + child {node {$\overline{D}$} + edge from parent + node[above] {...} + } + edge from parent + node[above] {...} + } ; + \end{tikzpicture} + \end{center} + \end{minipage} + + \medskip + + \begin{enumerate} + \item Compléter l'arbre de probabilité ci-contre + \item Interpréter puis donner les probabilités $P(A)$, $P(B)$, $P_A(D)$ et $P(B \cap D)$. + + \item + \begin{enumerate} + \item Calculer la probabilité qu'un stylo provienne de l'atelier A et possède un défaut de fabrication. + \item En déduire que la probabilité qu'un stylo possède un défaut de fabrication est de $0.32$. + \end{enumerate} + \item On prélève un stylo au hasard avec un défaut. Quelle est la probabilité qu'il vienne de l'atelier A? + \end{enumerate} + + \bigskip + + \textbf{Partie B} + \medskip + + Dans cette partie, on suppose que 32.0\,\% des stylos possèdent un défaut de fabrication. + + L'entreprise confectionne des paquets contenant chacun $4$~stylos. + + Le fait qu'un stylo possède ou non un défaut de fabrication est indépendant des autres stylos. + + On appelle $X$ la variable aléatoire donnant pour un paquet le nombre de stylos qui possèdent un défaut de fabrication. + + On admet que la variable aléatoire $X$ suit une loi binomiale. + + \medskip + + \begin{enumerate} + \setcounter{enumi}{4} + \item Avec quelle loi peut-on modéliser $X$. Préciser les paramètres. + \item Calculer et interpréter la probabilité $P(X = 7)$. + \item Le directeur de l'entreprise affirme qu'il y a plus d'une chance sur deux qu'un paquet ne comporte aucun stylo défectueux. A-t-il raison ? + \item Combien de stylos peut-on espérer avoir en moyenne? + \end{enumerate} + \pagebreak +\end{exercise} + +\begin{solution} + \begin{enumerate} + \item + \begin{center} + \begin{tikzpicture}[sloped] + \node {.} + child {node {$A$} + child {node {$D$} + edge from parent + node[above] {0.31} + } + child {node {$\overline{D}$} + edge from parent + node[above] {0.69} + } + edge from parent + node[above] {0.43} + } + child[missing] {} + child { node {$B$} + child {node {$D$} + edge from parent + node[above] {0.34} + } + child {node {$\overline{D}$} + edge from parent + node[above] {0.66} + } + edge from parent + node[above] {0.57} + } ; + \end{tikzpicture} + \end{center} + \item + \begin{itemize} + \item Probabilité que le stylo vienne de l'atelier A + \[ + P(A) = 0.43 + \] + \item Probabilité que le stylo vienne de l'atelier B + \[ + P(B) = 0.57 + \] + \item Probabilité que le stylo ait un défaut sachant qu'il vient de l'atelier A. + \[ + P_A(D) = 0.31 + \] + \item Probabilité que le stylo vienne de l'atelier B et qu'il ait un défaut. + \[ + P(D \cap D) = 0.19 + \] + \end{itemize} + \item + \begin{enumerate} + \item Probabilité qu'un stylo vienne de l'atelier A et qu'il ait un defaut + \[ + P(A\cap D) = P(A) \times P_A(D) = 0.43 \times 0.31 = 0.13 + \] + \item Probabilité que le stylo ai un défaut de fabrication. + \[ + P(D) = P(A\cap D) + P(B\cap D) = 0.13 + 0.19 = 0.32 + \] + \end{enumerate} + \item Probabilité qu'il vienne de l'atelier A sachant qu'il a un defaut + \[ + P_D(A) = \frac{P(A\cap D)}{P(D)} = \frac{0.13}{0.32} = 0.41 + \] + \item $X$ peut être modélisée par une loi binomiale de paramètres $n=10$ et $p=0.32$. + \item (\textit{par de correction automatique disponible pour le résultat final} + \[ + P(X = 7) = \coefBino{10}{7}\times 0.32^{7} \times 0.68^{3} + \] + \item (\textit{par de correction automatique disponible pour le résultat final} + + Il faut calculer la probabilité qu'il y ait 0 stylo avec un defaut. + \[ + P(X = 0) = \coefBino{10}{0}\times 0.32^{0} \times 0.68^{10} + \] + Puis comparer ce nombre à 0,5. + \item Il faut calculer l'espérance + \[ + E[X] = n\times p = 10 \times 0.32 = 3.2 + \] + \end{enumerate} +\end{solution} + +\end{document} + +%%% Local Variables: +%%% mode: latex +%%% TeX-master: "master" +%%% End: diff --git a/Complementaire/DM/2105_DM1/17_2105_DM1.tex b/Complementaire/DM/2105_DM1/17_2105_DM1.tex new file mode 100644 index 0000000..7c3bac4 --- /dev/null +++ b/Complementaire/DM/2105_DM1/17_2105_DM1.tex @@ -0,0 +1,367 @@ +\documentclass[a4paper,10pt]{article} +\usepackage{myXsim} + +% Title Page +\title{DM1 \hfill VANDROUX Guillemette} +\tribe{Maths complémentaire} +\date{\hfillÀ render pour le jeudi 27 mai} + +\xsimsetup{ + solution/print = false +} + +\begin{document} +\maketitle + +Les valeurs des exercices sont générés automatiquement. Si une valeur a un nombre adhérant de chiffres après la virgule, vous pouvez l'arrondir à l'entier le plus proche. + +\begin{exercise}[subtitle={Optimisation de matière}] + \begin{minipage}{0.6\textwidth} + On se propose de fabriquer avec le moins de tôle possible une citerne fermée en forme de parallélépipède rectangle dont le volume intérieur doit être de $30m^3$. La longueur est aussi fixée à $3m$ par le cahier des charges. + + On peut donc faire varier uniquement la largeur (notée $x$) et la hauteur (notée $h$) de la cuve. + \end{minipage} + \hfill + \begin{minipage}{0.3\textwidth} + \begin{tikzpicture} + \pgfmathsetmacro{\cubex}{3} + \pgfmathsetmacro{\cubey}{1} + \pgfmathsetmacro{\cubez}{2} + \draw[black,fill=gray] (0,0,0) -- ++(-\cubex,0,0) -- ++(0,-\cubey,0) node [midway, left] {$h$} -- ++(\cubex,0,0) node [midway, below] {$x$} -- cycle; + \draw[black,fill=gray] (0,0,0) -- ++(0,0,-\cubez) -- ++(0,-\cubey,0) -- ++(0,0,\cubez) node [midway, right] {$3m$} -- cycle; + \draw[black,fill=gray] (0,0,0) -- ++(-\cubex,0,0) -- ++(0,0,-\cubez) -- ++(\cubex,0,0) -- cycle; + \end{tikzpicture} + \end{minipage} + + \begin{enumerate} + \item Expliquer pourquoi quand la largeur $x$ change, la hauteur $h$ doit elle aussi changer pour respecter les contraintes. + \item Démontrer que l'on doit avoir $h = \dfrac{10}{x}$. + \item On note $S(x)$ l'aire totale de la citerne (c'est à dire la somme des aires des six faces). Montrer que l'on peut écrire + \[ + S(x) = 6x + 20 + \frac{60}{x} + \] + \item Démontrer que + \[ + S(x) = \frac{6x^2 + 20x + 60}{x} + \] + \item Démontrer que + \[ + S'(x) = \frac{6x^2 - 60}{x^2} + \] + \item En déduire le tableau de variation de $S(x)$ sur $\intOF{0}{10}$. + \item Déterminer les valeurs de $x$ et $h$ correspondant à une utilisation minimal de tôle. + \end{enumerate} +\end{exercise} + +\begin{solution} + \begin{enumerate} + \item Le volume étant fixe si l'on fait varier $x$, $h$ doit aussi varier. + \begin{itemize} + \item Si $x = 2$ alors conserver un volume de $V=30$, $h$ doit être égale à $10 / 2$ + \item Si $x = 3$ alors conserver un volume de $V=30$, $h$ doit être égale à $10 / 3$ + \end{itemize} + \item Pour calculer le volume, on a + \begin{eqnarray*} + V &=& h\times x \times 3 \\ + 30 &=& h\times x \times 3 \\ + x &=& \frac{30}{h\times 3} = \frac{10}{h} + \end{eqnarray*} + \item Pour calculer la surface totale, on ajoute la surface de chaque face. On a donc le calcul suivant + \begin{eqnarray*} + S(x) &=& x\times h \times 2 + x\times3\times2 + h\times 3\times 2\\ + S(x) &=& x\times \frac{10}{x} \times 2 + x\times3\times2 + \frac{10}{x}\times 3\times 2\\ + S(x) &=& 6x + 20 + \frac{60}{x} + \end{eqnarray*} + \item Pour trouver cette nouvelle forme, on met chaque élément sur le même dénominateur + \begin{eqnarray*} + S(x) &=& 6x + 20 + \frac{60}{x}\\ + S(x) &=& \frac{6x\times x}{x} + \frac{20\times x}{x} + \frac{60}{x}\\ + S(x) &=& \frac{6x^2 + 20x + 60}{x} + \end{eqnarray*} + \item On retrouve la formule $\frac{u}{v}$ à dériver + \[ + u(x) = 6x^2 + 20x + 60 \Rightarrow u'(x) = 12x + 20 + \] + \[ + v(x) = x \Rightarrow v'(x) = 1 + \] + Donc au numérateur on obtient + \begin{eqnarray*} + u'(x)\times v(x) - u(x)\times v'(x) &=& (12x + 20)\times x - (6x^2 + 20x + 60)\times 1\\ + &=& 6x^2 - 60 + \end{eqnarray*} + Donc + \[ + S'(x) = \frac{6x^2 - 60}{x^2} + \] + \item Tableau de variations de $S$ + + \begin{itemize} + \item Valeur interdite: $x^2 = 0 \equiv x = 0$ + \item Signe de $6x^2 - 60$: c'est un polynôme du 2e degré + \[ + \Delta = 1440 > 0 + \] + Il y a donc 2 racines + \[ + x_1 = - 3.1622776601683795 \qquad + x_2 = 3.1622776601683795 + \] + Et on sait que $6x^2 - 60$ est du signe de $a$ donc positif en dehors des racines + \item Le dénominateur $x^2$ est toujours positif. + \item Tableau de variations + + \begin{tikzpicture}[baseline=(a.north)] + \tkzTabInit[lgt=3,espcl=3]{$x$/1,$6x^2 - 60$/1, $x^2$/1, $S'$/1, $S$/2}{$0$, $- 3.1622776601683795$, $10$} + \tkzTabLine{d,-, z, +, } + \tkzTabLine{d,+, , +, } + \tkzTabLine{d,-, z, +, } + \tkzTabVar{D+/ , -/ , +/ } + \end{tikzpicture} + + \end{itemize} + \item On a donc une surface minimal pour $x=3.1622776601683795$ et $h = 31.6227766016837950$. + \end{enumerate} +\end{solution} + +%%% Local Variables: +%%% mode: latex +%%% TeX-master: "master" +%%% End: + +\begin{exercise}[subtitle={Bassin}] + Le tour d'un bassin au niveau du sol présente deux axes de symétrie : l’axe des abscisses et la droite d’équation $x=4$. Il est obtenu par symétrie de la courbe $\mathcal{C}_f$ sur $\intFF{0}{4}$ où $f$ est la fonction définie par + + \[ + f(x) = \left(- x^{2} + 5.0 x - 1.8\right) e^{- x} + 1.8 + \] + On admet que sur $\intFF{0}{4}$ la fonction $f$ est positive. + \begin{enumerate} + \item Sur un repère, tracer l'allure de la courbe $\mathcal{C}_f$, les axes de symétries puis compléter pour dessiner la forme du bassin. + \item Montrer que la fonction $f$ admet comme primitive sur $\R$ la fonction $F$ définie par + \[ + F(x) = 1.8 x + \left( x^{2} - 3.0 x - 1.2\right) e^{- x} + \] + \item Calculer la quantité $\ds \int_0^4 f(x) \; dx$, vous donnerez le résultat sous forme exacte. Interpréter le résultat et reportez cette quantité sur le graphique. + \item On considère que l'échelle de votre graphique est de 1unité pour 15m. Calculer l'aire du bassin. Vous donnerez un résultat arrondi au $m^2$ près. + \end{enumerate} +\end{exercise} + +\begin{solution} + \begin{enumerate} + \item + \begin{tikzpicture}[baseline=(a.north), xscale=1, yscale=0.5] + \tkzInit[xmin=0,xmax=5,xstep=1, + ymin=0,ymax=10,ystep=1] + \tkzGrid + \tkzAxeXY + \tkzFct[domain=0:10,color=red,very thick]% + { (-x**2 + 5.0*x - 1.8)*exp(-x) + 1.8 }; + \end{tikzpicture} + \item Il faut dériver $F(x)$ et vérifier que $F'(x) = f(x)$. + \item $\ds \int_0^4 f(x) \; dx = F(4) - F(0) = \frac{2.8}{e^{4}} + 8.4$ + \item La quantité calculée à la question précédente se retrouve 4fois pour former le bassin. Il faut ensuite prendre en compte l'échelle, comme 1unité de longueur correspond à 15m, une unité d'air correspond à $15\times15 = 225m^2$. Ainsi l'aire du bassin est égale à + \[ + (\frac{2.8}{e^{4}} + 8.4)\times 4 \times 15^2 = 7606.000000 + \] + + \end{enumerate} +\end{solution} + +%%% Local Variables: +%%% mode: latex +%%% TeX-master: "master" +%%% End: + +\begin{exercise}[subtitle={Stylos}] + \emph{Les parties {\rm A} et {\rm B} de cet exercice sont indépendantes.} + + \bigskip + + \begin{minipage}{0.6\linewidth} + \textbf{Partie A} + + \medskip + + Deux ateliers A et B fabriquent des stylos pour une entreprise. + + L'atelier A fabrique 55.00000000000001\,\% des stylos, et parmi ceux-là, 46.0\,\% possèdent un défaut de fabrication. + + De plus, 20.0\,\% des stylos possèdent un défaut de fabrication et sortent de l'atelier B. + + Un stylo est prélevé au hasard dans le stock de l'entreprise. + + On considère les évènements suivants: + + \begin{itemize} + \item A : \og Le stylo a été fabriqué par l'atelier A \fg + \item B : \og Le stylo a été fabriqué par l'atelier B \fg + \item D : \og Le stylo possède un défaut de fabrication \fg + \end{itemize} + \end{minipage} + \begin{minipage}{0.4\linewidth} + \begin{center} + \begin{tikzpicture}[sloped] + \node {.} + child {node {$A$} + child {node {$D$} + edge from parent + node[above] {...} + } + child {node {$\overline{D}$} + edge from parent + node[above] {...} + } + edge from parent + node[above] {...} + } + child[missing] {} + child { node {$B$} + child {node {$D$} + edge from parent + node[above] {...} + } + child {node {$\overline{D}$} + edge from parent + node[above] {...} + } + edge from parent + node[above] {...} + } ; + \end{tikzpicture} + \end{center} + \end{minipage} + + \medskip + + \begin{enumerate} + \item Compléter l'arbre de probabilité ci-contre + \item Interpréter puis donner les probabilités $P(A)$, $P(B)$, $P_A(D)$ et $P(B \cap D)$. + + \item + \begin{enumerate} + \item Calculer la probabilité qu'un stylo provienne de l'atelier A et possède un défaut de fabrication. + \item En déduire que la probabilité qu'un stylo possède un défaut de fabrication est de $0.45$. + \end{enumerate} + \item On prélève un stylo au hasard avec un défaut. Quelle est la probabilité qu'il vienne de l'atelier A? + \end{enumerate} + + \bigskip + + \textbf{Partie B} + \medskip + + Dans cette partie, on suppose que 45.0\,\% des stylos possèdent un défaut de fabrication. + + L'entreprise confectionne des paquets contenant chacun $4$~stylos. + + Le fait qu'un stylo possède ou non un défaut de fabrication est indépendant des autres stylos. + + On appelle $X$ la variable aléatoire donnant pour un paquet le nombre de stylos qui possèdent un défaut de fabrication. + + On admet que la variable aléatoire $X$ suit une loi binomiale. + + \medskip + + \begin{enumerate} + \setcounter{enumi}{4} + \item Avec quelle loi peut-on modéliser $X$. Préciser les paramètres. + \item Calculer et interpréter la probabilité $P(X = 13)$. + \item Le directeur de l'entreprise affirme qu'il y a plus d'une chance sur deux qu'un paquet ne comporte aucun stylo défectueux. A-t-il raison ? + \item Combien de stylos peut-on espérer avoir en moyenne? + \end{enumerate} + \pagebreak +\end{exercise} + +\begin{solution} + \begin{enumerate} + \item + \begin{center} + \begin{tikzpicture}[sloped] + \node {.} + child {node {$A$} + child {node {$D$} + edge from parent + node[above] {0.46} + } + child {node {$\overline{D}$} + edge from parent + node[above] {0.54} + } + edge from parent + node[above] {0.55} + } + child[missing] {} + child { node {$B$} + child {node {$D$} + edge from parent + node[above] {0.44} + } + child {node {$\overline{D}$} + edge from parent + node[above] {0.56} + } + edge from parent + node[above] {0.45} + } ; + \end{tikzpicture} + \end{center} + \item + \begin{itemize} + \item Probabilité que le stylo vienne de l'atelier A + \[ + P(A) = 0.55 + \] + \item Probabilité que le stylo vienne de l'atelier B + \[ + P(B) = 0.45 + \] + \item Probabilité que le stylo ait un défaut sachant qu'il vient de l'atelier A. + \[ + P_A(D) = 0.46 + \] + \item Probabilité que le stylo vienne de l'atelier B et qu'il ait un défaut. + \[ + P(D \cap D) = 0.2 + \] + \end{itemize} + \item + \begin{enumerate} + \item Probabilité qu'un stylo vienne de l'atelier A et qu'il ait un defaut + \[ + P(A\cap D) = P(A) \times P_A(D) = 0.55 \times 0.46 = 0.25 + \] + \item Probabilité que le stylo ai un défaut de fabrication. + \[ + P(D) = P(A\cap D) + P(B\cap D) = 0.25 + 0.2 = 0.45 + \] + \end{enumerate} + \item Probabilité qu'il vienne de l'atelier A sachant qu'il a un defaut + \[ + P_D(A) = \frac{P(A\cap D)}{P(D)} = \frac{0.25}{0.45} = 0.56 + \] + \item $X$ peut être modélisée par une loi binomiale de paramètres $n=15$ et $p=0.45$. + \item (\textit{par de correction automatique disponible pour le résultat final} + \[ + P(X = 13) = \coefBino{15}{13}\times 0.45^{13} \times 0.55^{2} + \] + \item (\textit{par de correction automatique disponible pour le résultat final} + + Il faut calculer la probabilité qu'il y ait 0 stylo avec un defaut. + \[ + P(X = 0) = \coefBino{15}{0}\times 0.45^{0} \times 0.55^{15} + \] + Puis comparer ce nombre à 0,5. + \item Il faut calculer l'espérance + \[ + E[X] = n\times p = 15 \times 0.45 = 6.75 + \] + \end{enumerate} +\end{solution} + +\end{document} + +%%% Local Variables: +%%% mode: latex +%%% TeX-master: "master" +%%% End: diff --git a/Complementaire/DM/2105_DM1/corr_01_2105_DM1.tex b/Complementaire/DM/2105_DM1/corr_01_2105_DM1.tex new file mode 100644 index 0000000..4fa9832 --- /dev/null +++ b/Complementaire/DM/2105_DM1/corr_01_2105_DM1.tex @@ -0,0 +1,367 @@ +\documentclass[a4paper,10pt]{article} +\usepackage{myXsim} + +% Title Page +\title{DM1 \hfill BALLARD Antoine} +\tribe{Maths complémentaire} +\date{\hfillÀ render pour le jeudi 27 mai} + +\xsimsetup{ + solution/print = true +} + +\begin{document} +\maketitle + +Les valeurs des exercices sont générés automatiquement. Si une valeur a un nombre adhérant de chiffres après la virgule, vous pouvez l'arrondir à l'entier le plus proche. + +\begin{exercise}[subtitle={Optimisation de matière}] + \begin{minipage}{0.6\textwidth} + On se propose de fabriquer avec le moins de tôle possible une citerne fermée en forme de parallélépipède rectangle dont le volume intérieur doit être de $8m^3$. La longueur est aussi fixée à $4m$ par le cahier des charges. + + On peut donc faire varier uniquement la largeur (notée $x$) et la hauteur (notée $h$) de la cuve. + \end{minipage} + \hfill + \begin{minipage}{0.3\textwidth} + \begin{tikzpicture} + \pgfmathsetmacro{\cubex}{3} + \pgfmathsetmacro{\cubey}{1} + \pgfmathsetmacro{\cubez}{2} + \draw[black,fill=gray] (0,0,0) -- ++(-\cubex,0,0) -- ++(0,-\cubey,0) node [midway, left] {$h$} -- ++(\cubex,0,0) node [midway, below] {$x$} -- cycle; + \draw[black,fill=gray] (0,0,0) -- ++(0,0,-\cubez) -- ++(0,-\cubey,0) -- ++(0,0,\cubez) node [midway, right] {$4m$} -- cycle; + \draw[black,fill=gray] (0,0,0) -- ++(-\cubex,0,0) -- ++(0,0,-\cubez) -- ++(\cubex,0,0) -- cycle; + \end{tikzpicture} + \end{minipage} + + \begin{enumerate} + \item Expliquer pourquoi quand la largeur $x$ change, la hauteur $h$ doit elle aussi changer pour respecter les contraintes. + \item Démontrer que l'on doit avoir $h = \dfrac{2}{x}$. + \item On note $S(x)$ l'aire totale de la citerne (c'est à dire la somme des aires des six faces). Montrer que l'on peut écrire + \[ + S(x) = 8x + 4 + \frac{16}{x} + \] + \item Démontrer que + \[ + S(x) = \frac{8x^2 + 4x + 16}{x} + \] + \item Démontrer que + \[ + S'(x) = \frac{8x^2 - 16}{x^2} + \] + \item En déduire le tableau de variation de $S(x)$ sur $\intOF{0}{10}$. + \item Déterminer les valeurs de $x$ et $h$ correspondant à une utilisation minimal de tôle. + \end{enumerate} +\end{exercise} + +\begin{solution} + \begin{enumerate} + \item Le volume étant fixe si l'on fait varier $x$, $h$ doit aussi varier. + \begin{itemize} + \item Si $x = 2$ alors conserver un volume de $V=8$, $h$ doit être égale à $2 / 2$ + \item Si $x = 3$ alors conserver un volume de $V=8$, $h$ doit être égale à $2 / 3$ + \end{itemize} + \item Pour calculer le volume, on a + \begin{eqnarray*} + V &=& h\times x \times 4 \\ + 8 &=& h\times x \times 4 \\ + x &=& \frac{8}{h\times 4} = \frac{2}{h} + \end{eqnarray*} + \item Pour calculer la surface totale, on ajoute la surface de chaque face. On a donc le calcul suivant + \begin{eqnarray*} + S(x) &=& x\times h \times 2 + x\times4\times2 + h\times 4\times 2\\ + S(x) &=& x\times \frac{2}{x} \times 2 + x\times4\times2 + \frac{2}{x}\times 4\times 2\\ + S(x) &=& 8x + 4 + \frac{16}{x} + \end{eqnarray*} + \item Pour trouver cette nouvelle forme, on met chaque élément sur le même dénominateur + \begin{eqnarray*} + S(x) &=& 8x + 4 + \frac{16}{x}\\ + S(x) &=& \frac{8x\times x}{x} + \frac{4\times x}{x} + \frac{16}{x}\\ + S(x) &=& \frac{8x^2 + 4x + 16}{x} + \end{eqnarray*} + \item On retrouve la formule $\frac{u}{v}$ à dériver + \[ + u(x) = 8x^2 + 4x + 16 \Rightarrow u'(x) = 16x + 4 + \] + \[ + v(x) = x \Rightarrow v'(x) = 1 + \] + Donc au numérateur on obtient + \begin{eqnarray*} + u'(x)\times v(x) - u(x)\times v'(x) &=& (16x + 4)\times x - (8x^2 + 4x + 16)\times 1\\ + &=& 8x^2 - 16 + \end{eqnarray*} + Donc + \[ + S'(x) = \frac{8x^2 - 16}{x^2} + \] + \item Tableau de variations de $S$ + + \begin{itemize} + \item Valeur interdite: $x^2 = 0 \equiv x = 0$ + \item Signe de $8x^2 - 16$: c'est un polynôme du 2e degré + \[ + \Delta = 512 > 0 + \] + Il y a donc 2 racines + \[ + x_1 = - 1.4142135623730951 \qquad + x_2 = 1.4142135623730951 + \] + Et on sait que $8x^2 - 16$ est du signe de $a$ donc positif en dehors des racines + \item Le dénominateur $x^2$ est toujours positif. + \item Tableau de variations + + \begin{tikzpicture}[baseline=(a.north)] + \tkzTabInit[lgt=3,espcl=3]{$x$/1,$8x^2 - 16$/1, $x^2$/1, $S'$/1, $S$/2}{$0$, $- 1.4142135623730951$, $10$} + \tkzTabLine{d,-, z, +, } + \tkzTabLine{d,+, , +, } + \tkzTabLine{d,-, z, +, } + \tkzTabVar{D+/ , -/ , +/ } + \end{tikzpicture} + + \end{itemize} + \item On a donc une surface minimal pour $x=1.4142135623730951$ et $h = 2.8284271247461902$. + \end{enumerate} +\end{solution} + +%%% Local Variables: +%%% mode: latex +%%% TeX-master: "master" +%%% End: + +\begin{exercise}[subtitle={Bassin}] + Le tour d'un bassin au niveau du sol présente deux axes de symétrie : l’axe des abscisses et la droite d’équation $x=4$. Il est obtenu par symétrie de la courbe $\mathcal{C}_f$ sur $\intFF{0}{4}$ où $f$ est la fonction définie par + + \[ + f(x) = \left(- x^{2} + 0.3 x - 7.3\right) e^{- x} + 7.3 + \] + On admet que sur $\intFF{0}{4}$ la fonction $f$ est positive. + \begin{enumerate} + \item Sur un repère, tracer l'allure de la courbe $\mathcal{C}_f$, les axes de symétries puis compléter pour dessiner la forme du bassin. + \item Montrer que la fonction $f$ admet comme primitive sur $\R$ la fonction $F$ définie par + \[ + F(x) = 7.3 x + \left( x^{2} + 1.7 x + 9.0\right) e^{- x} + \] + \item Calculer la quantité $\ds \int_0^4 f(x) \; dx$, vous donnerez le résultat sous forme exacte. Interpréter le résultat et reportez cette quantité sur le graphique. + \item On considère que l'échelle de votre graphique est de 1unité pour 15m. Calculer l'aire du bassin. Vous donnerez un résultat arrondi au $m^2$ près. + \end{enumerate} +\end{exercise} + +\begin{solution} + \begin{enumerate} + \item + \begin{tikzpicture}[baseline=(a.north), xscale=1, yscale=0.5] + \tkzInit[xmin=0,xmax=5,xstep=1, + ymin=0,ymax=10,ystep=1] + \tkzGrid + \tkzAxeXY + \tkzFct[domain=0:10,color=red,very thick]% + { (-x**2 + 0.3*x - 7.3)*exp(-x) + 7.3 }; + \end{tikzpicture} + \item Il faut dériver $F(x)$ et vérifier que $F'(x) = f(x)$. + \item $\ds \int_0^4 f(x) \; dx = F(4) - F(0) = \frac{31.8}{e^{4}} + 20.2$ + \item La quantité calculée à la question précédente se retrouve 4fois pour former le bassin. Il faut ensuite prendre en compte l'échelle, comme 1unité de longueur correspond à 15m, une unité d'air correspond à $15\times15 = 225m^2$. Ainsi l'aire du bassin est égale à + \[ + (\frac{31.8}{e^{4}} + 20.2)\times 4 \times 15^2 = 18704.00000 + \] + + \end{enumerate} +\end{solution} + +%%% Local Variables: +%%% mode: latex +%%% TeX-master: "master" +%%% End: + +\begin{exercise}[subtitle={Stylos}] + \emph{Les parties {\rm A} et {\rm B} de cet exercice sont indépendantes.} + + \bigskip + + \begin{minipage}{0.6\linewidth} + \textbf{Partie A} + + \medskip + + Deux ateliers A et B fabriquent des stylos pour une entreprise. + + L'atelier A fabrique 86.0\,\% des stylos, et parmi ceux-là, 34.0\,\% possèdent un défaut de fabrication. + + De plus, 8.0\,\% des stylos possèdent un défaut de fabrication et sortent de l'atelier B. + + Un stylo est prélevé au hasard dans le stock de l'entreprise. + + On considère les évènements suivants: + + \begin{itemize} + \item A : \og Le stylo a été fabriqué par l'atelier A \fg + \item B : \og Le stylo a été fabriqué par l'atelier B \fg + \item D : \og Le stylo possède un défaut de fabrication \fg + \end{itemize} + \end{minipage} + \begin{minipage}{0.4\linewidth} + \begin{center} + \begin{tikzpicture}[sloped] + \node {.} + child {node {$A$} + child {node {$D$} + edge from parent + node[above] {...} + } + child {node {$\overline{D}$} + edge from parent + node[above] {...} + } + edge from parent + node[above] {...} + } + child[missing] {} + child { node {$B$} + child {node {$D$} + edge from parent + node[above] {...} + } + child {node {$\overline{D}$} + edge from parent + node[above] {...} + } + edge from parent + node[above] {...} + } ; + \end{tikzpicture} + \end{center} + \end{minipage} + + \medskip + + \begin{enumerate} + \item Compléter l'arbre de probabilité ci-contre + \item Interpréter puis donner les probabilités $P(A)$, $P(B)$, $P_A(D)$ et $P(B \cap D)$. + + \item + \begin{enumerate} + \item Calculer la probabilité qu'un stylo provienne de l'atelier A et possède un défaut de fabrication. + \item En déduire que la probabilité qu'un stylo possède un défaut de fabrication est de $0.37$. + \end{enumerate} + \item On prélève un stylo au hasard avec un défaut. Quelle est la probabilité qu'il vienne de l'atelier A? + \end{enumerate} + + \bigskip + + \textbf{Partie B} + \medskip + + Dans cette partie, on suppose que 37.0\,\% des stylos possèdent un défaut de fabrication. + + L'entreprise confectionne des paquets contenant chacun $4$~stylos. + + Le fait qu'un stylo possède ou non un défaut de fabrication est indépendant des autres stylos. + + On appelle $X$ la variable aléatoire donnant pour un paquet le nombre de stylos qui possèdent un défaut de fabrication. + + On admet que la variable aléatoire $X$ suit une loi binomiale. + + \medskip + + \begin{enumerate} + \setcounter{enumi}{4} + \item Avec quelle loi peut-on modéliser $X$. Préciser les paramètres. + \item Calculer et interpréter la probabilité $P(X = 10)$. + \item Le directeur de l'entreprise affirme qu'il y a plus d'une chance sur deux qu'un paquet ne comporte aucun stylo défectueux. A-t-il raison ? + \item Combien de stylos peut-on espérer avoir en moyenne? + \end{enumerate} + \pagebreak +\end{exercise} + +\begin{solution} + \begin{enumerate} + \item + \begin{center} + \begin{tikzpicture}[sloped] + \node {.} + child {node {$A$} + child {node {$D$} + edge from parent + node[above] {0.34} + } + child {node {$\overline{D}$} + edge from parent + node[above] {0.66} + } + edge from parent + node[above] {0.86} + } + child[missing] {} + child { node {$B$} + child {node {$D$} + edge from parent + node[above] {0.58} + } + child {node {$\overline{D}$} + edge from parent + node[above] {0.42} + } + edge from parent + node[above] {0.14} + } ; + \end{tikzpicture} + \end{center} + \item + \begin{itemize} + \item Probabilité que le stylo vienne de l'atelier A + \[ + P(A) = 0.86 + \] + \item Probabilité que le stylo vienne de l'atelier B + \[ + P(B) = 0.14 + \] + \item Probabilité que le stylo ait un défaut sachant qu'il vient de l'atelier A. + \[ + P_A(D) = 0.34 + \] + \item Probabilité que le stylo vienne de l'atelier B et qu'il ait un défaut. + \[ + P(D \cap D) = 0.08 + \] + \end{itemize} + \item + \begin{enumerate} + \item Probabilité qu'un stylo vienne de l'atelier A et qu'il ait un defaut + \[ + P(A\cap D) = P(A) \times P_A(D) = 0.86 \times 0.34 = 0.29 + \] + \item Probabilité que le stylo ai un défaut de fabrication. + \[ + P(D) = P(A\cap D) + P(B\cap D) = 0.29 + 0.08 = 0.37 + \] + \end{enumerate} + \item Probabilité qu'il vienne de l'atelier A sachant qu'il a un defaut + \[ + P_D(A) = \frac{P(A\cap D)}{P(D)} = \frac{0.29}{0.37} = 0.78 + \] + \item $X$ peut être modélisée par une loi binomiale de paramètres $n=20$ et $p=0.37$. + \item (\textit{par de correction automatique disponible pour le résultat final} + \[ + P(X = 10) = \coefBino{20}{10}\times 0.37^{10} \times 0.63^{10} + \] + \item (\textit{par de correction automatique disponible pour le résultat final} + + Il faut calculer la probabilité qu'il y ait 0 stylo avec un defaut. + \[ + P(X = 0) = \coefBino{20}{0}\times 0.37^{0} \times 0.63^{20} + \] + Puis comparer ce nombre à 0,5. + \item Il faut calculer l'espérance + \[ + E[X] = n\times p = 20 \times 0.37 = 7.4 + \] + \end{enumerate} +\end{solution} + +\end{document} + +%%% Local Variables: +%%% mode: latex +%%% TeX-master: "master" +%%% End: diff --git a/Complementaire/DM/2105_DM1/corr_02_2105_DM1.tex b/Complementaire/DM/2105_DM1/corr_02_2105_DM1.tex new file mode 100644 index 0000000..cb874c4 --- /dev/null +++ b/Complementaire/DM/2105_DM1/corr_02_2105_DM1.tex @@ -0,0 +1,367 @@ +\documentclass[a4paper,10pt]{article} +\usepackage{myXsim} + +% Title Page +\title{DM1 \hfill BALUKHATYY Alexandre} +\tribe{Maths complémentaire} +\date{\hfillÀ render pour le jeudi 27 mai} + +\xsimsetup{ + solution/print = true +} + +\begin{document} +\maketitle + +Les valeurs des exercices sont générés automatiquement. Si une valeur a un nombre adhérant de chiffres après la virgule, vous pouvez l'arrondir à l'entier le plus proche. + +\begin{exercise}[subtitle={Optimisation de matière}] + \begin{minipage}{0.6\textwidth} + On se propose de fabriquer avec le moins de tôle possible une citerne fermée en forme de parallélépipède rectangle dont le volume intérieur doit être de $15m^3$. La longueur est aussi fixée à $3m$ par le cahier des charges. + + On peut donc faire varier uniquement la largeur (notée $x$) et la hauteur (notée $h$) de la cuve. + \end{minipage} + \hfill + \begin{minipage}{0.3\textwidth} + \begin{tikzpicture} + \pgfmathsetmacro{\cubex}{3} + \pgfmathsetmacro{\cubey}{1} + \pgfmathsetmacro{\cubez}{2} + \draw[black,fill=gray] (0,0,0) -- ++(-\cubex,0,0) -- ++(0,-\cubey,0) node [midway, left] {$h$} -- ++(\cubex,0,0) node [midway, below] {$x$} -- cycle; + \draw[black,fill=gray] (0,0,0) -- ++(0,0,-\cubez) -- ++(0,-\cubey,0) -- ++(0,0,\cubez) node [midway, right] {$3m$} -- cycle; + \draw[black,fill=gray] (0,0,0) -- ++(-\cubex,0,0) -- ++(0,0,-\cubez) -- ++(\cubex,0,0) -- cycle; + \end{tikzpicture} + \end{minipage} + + \begin{enumerate} + \item Expliquer pourquoi quand la largeur $x$ change, la hauteur $h$ doit elle aussi changer pour respecter les contraintes. + \item Démontrer que l'on doit avoir $h = \dfrac{5}{x}$. + \item On note $S(x)$ l'aire totale de la citerne (c'est à dire la somme des aires des six faces). Montrer que l'on peut écrire + \[ + S(x) = 6x + 10 + \frac{30}{x} + \] + \item Démontrer que + \[ + S(x) = \frac{6x^2 + 10x + 30}{x} + \] + \item Démontrer que + \[ + S'(x) = \frac{6x^2 - 30}{x^2} + \] + \item En déduire le tableau de variation de $S(x)$ sur $\intOF{0}{10}$. + \item Déterminer les valeurs de $x$ et $h$ correspondant à une utilisation minimal de tôle. + \end{enumerate} +\end{exercise} + +\begin{solution} + \begin{enumerate} + \item Le volume étant fixe si l'on fait varier $x$, $h$ doit aussi varier. + \begin{itemize} + \item Si $x = 2$ alors conserver un volume de $V=15$, $h$ doit être égale à $5 / 2$ + \item Si $x = 3$ alors conserver un volume de $V=15$, $h$ doit être égale à $5 / 3$ + \end{itemize} + \item Pour calculer le volume, on a + \begin{eqnarray*} + V &=& h\times x \times 3 \\ + 15 &=& h\times x \times 3 \\ + x &=& \frac{15}{h\times 3} = \frac{5}{h} + \end{eqnarray*} + \item Pour calculer la surface totale, on ajoute la surface de chaque face. On a donc le calcul suivant + \begin{eqnarray*} + S(x) &=& x\times h \times 2 + x\times3\times2 + h\times 3\times 2\\ + S(x) &=& x\times \frac{5}{x} \times 2 + x\times3\times2 + \frac{5}{x}\times 3\times 2\\ + S(x) &=& 6x + 10 + \frac{30}{x} + \end{eqnarray*} + \item Pour trouver cette nouvelle forme, on met chaque élément sur le même dénominateur + \begin{eqnarray*} + S(x) &=& 6x + 10 + \frac{30}{x}\\ + S(x) &=& \frac{6x\times x}{x} + \frac{10\times x}{x} + \frac{30}{x}\\ + S(x) &=& \frac{6x^2 + 10x + 30}{x} + \end{eqnarray*} + \item On retrouve la formule $\frac{u}{v}$ à dériver + \[ + u(x) = 6x^2 + 10x + 30 \Rightarrow u'(x) = 12x + 10 + \] + \[ + v(x) = x \Rightarrow v'(x) = 1 + \] + Donc au numérateur on obtient + \begin{eqnarray*} + u'(x)\times v(x) - u(x)\times v'(x) &=& (12x + 10)\times x - (6x^2 + 10x + 30)\times 1\\ + &=& 6x^2 - 30 + \end{eqnarray*} + Donc + \[ + S'(x) = \frac{6x^2 - 30}{x^2} + \] + \item Tableau de variations de $S$ + + \begin{itemize} + \item Valeur interdite: $x^2 = 0 \equiv x = 0$ + \item Signe de $6x^2 - 30$: c'est un polynôme du 2e degré + \[ + \Delta = 720 > 0 + \] + Il y a donc 2 racines + \[ + x_1 = - 2.23606797749979 \qquad + x_2 = 2.23606797749979 + \] + Et on sait que $6x^2 - 30$ est du signe de $a$ donc positif en dehors des racines + \item Le dénominateur $x^2$ est toujours positif. + \item Tableau de variations + + \begin{tikzpicture}[baseline=(a.north)] + \tkzTabInit[lgt=3,espcl=3]{$x$/1,$6x^2 - 30$/1, $x^2$/1, $S'$/1, $S$/2}{$0$, $- 2.23606797749979$, $10$} + \tkzTabLine{d,-, z, +, } + \tkzTabLine{d,+, , +, } + \tkzTabLine{d,-, z, +, } + \tkzTabVar{D+/ , -/ , +/ } + \end{tikzpicture} + + \end{itemize} + \item On a donc une surface minimal pour $x=2.23606797749979$ et $h = 11.18033988749895$. + \end{enumerate} +\end{solution} + +%%% Local Variables: +%%% mode: latex +%%% TeX-master: "master" +%%% End: + +\begin{exercise}[subtitle={Bassin}] + Le tour d'un bassin au niveau du sol présente deux axes de symétrie : l’axe des abscisses et la droite d’équation $x=4$. Il est obtenu par symétrie de la courbe $\mathcal{C}_f$ sur $\intFF{0}{4}$ où $f$ est la fonction définie par + + \[ + f(x) = \left(- x^{2} + 8.4 x - 0.6\right) e^{- x} + 0.6 + \] + On admet que sur $\intFF{0}{4}$ la fonction $f$ est positive. + \begin{enumerate} + \item Sur un repère, tracer l'allure de la courbe $\mathcal{C}_f$, les axes de symétries puis compléter pour dessiner la forme du bassin. + \item Montrer que la fonction $f$ admet comme primitive sur $\R$ la fonction $F$ définie par + \[ + F(x) = 0.6 x + \left( x^{2} - 6.4 x - 5.8\right) e^{- x} + \] + \item Calculer la quantité $\ds \int_0^4 f(x) \; dx$, vous donnerez le résultat sous forme exacte. Interpréter le résultat et reportez cette quantité sur le graphique. + \item On considère que l'échelle de votre graphique est de 1unité pour 15m. Calculer l'aire du bassin. Vous donnerez un résultat arrondi au $m^2$ près. + \end{enumerate} +\end{exercise} + +\begin{solution} + \begin{enumerate} + \item + \begin{tikzpicture}[baseline=(a.north), xscale=1, yscale=0.5] + \tkzInit[xmin=0,xmax=5,xstep=1, + ymin=0,ymax=10,ystep=1] + \tkzGrid + \tkzAxeXY + \tkzFct[domain=0:10,color=red,very thick]% + { (-x**2 + 8.4*x - 0.6)*exp(-x) + 0.6 }; + \end{tikzpicture} + \item Il faut dériver $F(x)$ et vérifier que $F'(x) = f(x)$. + \item $\ds \int_0^4 f(x) \; dx = F(4) - F(0) = 8.2 - \frac{15.4}{e^{4}}$ + \item La quantité calculée à la question précédente se retrouve 4fois pour former le bassin. Il faut ensuite prendre en compte l'échelle, comme 1unité de longueur correspond à 15m, une unité d'air correspond à $15\times15 = 225m^2$. Ainsi l'aire du bassin est égale à + \[ + (8.2 - \frac{15.4}{e^{4}})\times 4 \times 15^2 = 7126.000000 + \] + + \end{enumerate} +\end{solution} + +%%% Local Variables: +%%% mode: latex +%%% TeX-master: "master" +%%% End: + +\begin{exercise}[subtitle={Stylos}] + \emph{Les parties {\rm A} et {\rm B} de cet exercice sont indépendantes.} + + \bigskip + + \begin{minipage}{0.6\linewidth} + \textbf{Partie A} + + \medskip + + Deux ateliers A et B fabriquent des stylos pour une entreprise. + + L'atelier A fabrique 25.0\,\% des stylos, et parmi ceux-là, 84.0\,\% possèdent un défaut de fabrication. + + De plus, 57.99999999999999\,\% des stylos possèdent un défaut de fabrication et sortent de l'atelier B. + + Un stylo est prélevé au hasard dans le stock de l'entreprise. + + On considère les évènements suivants: + + \begin{itemize} + \item A : \og Le stylo a été fabriqué par l'atelier A \fg + \item B : \og Le stylo a été fabriqué par l'atelier B \fg + \item D : \og Le stylo possède un défaut de fabrication \fg + \end{itemize} + \end{minipage} + \begin{minipage}{0.4\linewidth} + \begin{center} + \begin{tikzpicture}[sloped] + \node {.} + child {node {$A$} + child {node {$D$} + edge from parent + node[above] {...} + } + child {node {$\overline{D}$} + edge from parent + node[above] {...} + } + edge from parent + node[above] {...} + } + child[missing] {} + child { node {$B$} + child {node {$D$} + edge from parent + node[above] {...} + } + child {node {$\overline{D}$} + edge from parent + node[above] {...} + } + edge from parent + node[above] {...} + } ; + \end{tikzpicture} + \end{center} + \end{minipage} + + \medskip + + \begin{enumerate} + \item Compléter l'arbre de probabilité ci-contre + \item Interpréter puis donner les probabilités $P(A)$, $P(B)$, $P_A(D)$ et $P(B \cap D)$. + + \item + \begin{enumerate} + \item Calculer la probabilité qu'un stylo provienne de l'atelier A et possède un défaut de fabrication. + \item En déduire que la probabilité qu'un stylo possède un défaut de fabrication est de $0.79$. + \end{enumerate} + \item On prélève un stylo au hasard avec un défaut. Quelle est la probabilité qu'il vienne de l'atelier A? + \end{enumerate} + + \bigskip + + \textbf{Partie B} + \medskip + + Dans cette partie, on suppose que 79.0\,\% des stylos possèdent un défaut de fabrication. + + L'entreprise confectionne des paquets contenant chacun $4$~stylos. + + Le fait qu'un stylo possède ou non un défaut de fabrication est indépendant des autres stylos. + + On appelle $X$ la variable aléatoire donnant pour un paquet le nombre de stylos qui possèdent un défaut de fabrication. + + On admet que la variable aléatoire $X$ suit une loi binomiale. + + \medskip + + \begin{enumerate} + \setcounter{enumi}{4} + \item Avec quelle loi peut-on modéliser $X$. Préciser les paramètres. + \item Calculer et interpréter la probabilité $P(X = 11)$. + \item Le directeur de l'entreprise affirme qu'il y a plus d'une chance sur deux qu'un paquet ne comporte aucun stylo défectueux. A-t-il raison ? + \item Combien de stylos peut-on espérer avoir en moyenne? + \end{enumerate} + \pagebreak +\end{exercise} + +\begin{solution} + \begin{enumerate} + \item + \begin{center} + \begin{tikzpicture}[sloped] + \node {.} + child {node {$A$} + child {node {$D$} + edge from parent + node[above] {0.84} + } + child {node {$\overline{D}$} + edge from parent + node[above] {0.16} + } + edge from parent + node[above] {0.25} + } + child[missing] {} + child { node {$B$} + child {node {$D$} + edge from parent + node[above] {0.78} + } + child {node {$\overline{D}$} + edge from parent + node[above] {0.22} + } + edge from parent + node[above] {0.75} + } ; + \end{tikzpicture} + \end{center} + \item + \begin{itemize} + \item Probabilité que le stylo vienne de l'atelier A + \[ + P(A) = 0.25 + \] + \item Probabilité que le stylo vienne de l'atelier B + \[ + P(B) = 0.75 + \] + \item Probabilité que le stylo ait un défaut sachant qu'il vient de l'atelier A. + \[ + P_A(D) = 0.84 + \] + \item Probabilité que le stylo vienne de l'atelier B et qu'il ait un défaut. + \[ + P(D \cap D) = 0.58 + \] + \end{itemize} + \item + \begin{enumerate} + \item Probabilité qu'un stylo vienne de l'atelier A et qu'il ait un defaut + \[ + P(A\cap D) = P(A) \times P_A(D) = 0.25 \times 0.84 = 0.21 + \] + \item Probabilité que le stylo ai un défaut de fabrication. + \[ + P(D) = P(A\cap D) + P(B\cap D) = 0.21 + 0.58 = 0.79 + \] + \end{enumerate} + \item Probabilité qu'il vienne de l'atelier A sachant qu'il a un defaut + \[ + P_D(A) = \frac{P(A\cap D)}{P(D)} = \frac{0.21}{0.79} = 0.27 + \] + \item $X$ peut être modélisée par une loi binomiale de paramètres $n=12$ et $p=0.79$. + \item (\textit{par de correction automatique disponible pour le résultat final} + \[ + P(X = 11) = \coefBino{12}{11}\times 0.79^{11} \times 0.21^{1} + \] + \item (\textit{par de correction automatique disponible pour le résultat final} + + Il faut calculer la probabilité qu'il y ait 0 stylo avec un defaut. + \[ + P(X = 0) = \coefBino{12}{0}\times 0.79^{0} \times 0.21^{12} + \] + Puis comparer ce nombre à 0,5. + \item Il faut calculer l'espérance + \[ + E[X] = n\times p = 12 \times 0.79 = 9.48 + \] + \end{enumerate} +\end{solution} + +\end{document} + +%%% Local Variables: +%%% mode: latex +%%% TeX-master: "master" +%%% End: diff --git a/Complementaire/DM/2105_DM1/corr_03_2105_DM1.tex b/Complementaire/DM/2105_DM1/corr_03_2105_DM1.tex new file mode 100644 index 0000000..a084592 --- /dev/null +++ b/Complementaire/DM/2105_DM1/corr_03_2105_DM1.tex @@ -0,0 +1,367 @@ +\documentclass[a4paper,10pt]{article} +\usepackage{myXsim} + +% Title Page +\title{DM1 \hfill CALES Mathis} +\tribe{Maths complémentaire} +\date{\hfillÀ render pour le jeudi 27 mai} + +\xsimsetup{ + solution/print = true +} + +\begin{document} +\maketitle + +Les valeurs des exercices sont générés automatiquement. Si une valeur a un nombre adhérant de chiffres après la virgule, vous pouvez l'arrondir à l'entier le plus proche. + +\begin{exercise}[subtitle={Optimisation de matière}] + \begin{minipage}{0.6\textwidth} + On se propose de fabriquer avec le moins de tôle possible une citerne fermée en forme de parallélépipède rectangle dont le volume intérieur doit être de $6m^3$. La longueur est aussi fixée à $2m$ par le cahier des charges. + + On peut donc faire varier uniquement la largeur (notée $x$) et la hauteur (notée $h$) de la cuve. + \end{minipage} + \hfill + \begin{minipage}{0.3\textwidth} + \begin{tikzpicture} + \pgfmathsetmacro{\cubex}{3} + \pgfmathsetmacro{\cubey}{1} + \pgfmathsetmacro{\cubez}{2} + \draw[black,fill=gray] (0,0,0) -- ++(-\cubex,0,0) -- ++(0,-\cubey,0) node [midway, left] {$h$} -- ++(\cubex,0,0) node [midway, below] {$x$} -- cycle; + \draw[black,fill=gray] (0,0,0) -- ++(0,0,-\cubez) -- ++(0,-\cubey,0) -- ++(0,0,\cubez) node [midway, right] {$2m$} -- cycle; + \draw[black,fill=gray] (0,0,0) -- ++(-\cubex,0,0) -- ++(0,0,-\cubez) -- ++(\cubex,0,0) -- cycle; + \end{tikzpicture} + \end{minipage} + + \begin{enumerate} + \item Expliquer pourquoi quand la largeur $x$ change, la hauteur $h$ doit elle aussi changer pour respecter les contraintes. + \item Démontrer que l'on doit avoir $h = \dfrac{3}{x}$. + \item On note $S(x)$ l'aire totale de la citerne (c'est à dire la somme des aires des six faces). Montrer que l'on peut écrire + \[ + S(x) = 4x + 6 + \frac{12}{x} + \] + \item Démontrer que + \[ + S(x) = \frac{4x^2 + 6x + 12}{x} + \] + \item Démontrer que + \[ + S'(x) = \frac{4x^2 - 12}{x^2} + \] + \item En déduire le tableau de variation de $S(x)$ sur $\intOF{0}{10}$. + \item Déterminer les valeurs de $x$ et $h$ correspondant à une utilisation minimal de tôle. + \end{enumerate} +\end{exercise} + +\begin{solution} + \begin{enumerate} + \item Le volume étant fixe si l'on fait varier $x$, $h$ doit aussi varier. + \begin{itemize} + \item Si $x = 2$ alors conserver un volume de $V=6$, $h$ doit être égale à $3 / 2$ + \item Si $x = 3$ alors conserver un volume de $V=6$, $h$ doit être égale à $3 / 3$ + \end{itemize} + \item Pour calculer le volume, on a + \begin{eqnarray*} + V &=& h\times x \times 2 \\ + 6 &=& h\times x \times 2 \\ + x &=& \frac{6}{h\times 2} = \frac{3}{h} + \end{eqnarray*} + \item Pour calculer la surface totale, on ajoute la surface de chaque face. On a donc le calcul suivant + \begin{eqnarray*} + S(x) &=& x\times h \times 2 + x\times2\times2 + h\times 2\times 2\\ + S(x) &=& x\times \frac{3}{x} \times 2 + x\times2\times2 + \frac{3}{x}\times 2\times 2\\ + S(x) &=& 4x + 6 + \frac{12}{x} + \end{eqnarray*} + \item Pour trouver cette nouvelle forme, on met chaque élément sur le même dénominateur + \begin{eqnarray*} + S(x) &=& 4x + 6 + \frac{12}{x}\\ + S(x) &=& \frac{4x\times x}{x} + \frac{6\times x}{x} + \frac{12}{x}\\ + S(x) &=& \frac{4x^2 + 6x + 12}{x} + \end{eqnarray*} + \item On retrouve la formule $\frac{u}{v}$ à dériver + \[ + u(x) = 4x^2 + 6x + 12 \Rightarrow u'(x) = 8x + 6 + \] + \[ + v(x) = x \Rightarrow v'(x) = 1 + \] + Donc au numérateur on obtient + \begin{eqnarray*} + u'(x)\times v(x) - u(x)\times v'(x) &=& (8x + 6)\times x - (4x^2 + 6x + 12)\times 1\\ + &=& 4x^2 - 12 + \end{eqnarray*} + Donc + \[ + S'(x) = \frac{4x^2 - 12}{x^2} + \] + \item Tableau de variations de $S$ + + \begin{itemize} + \item Valeur interdite: $x^2 = 0 \equiv x = 0$ + \item Signe de $4x^2 - 12$: c'est un polynôme du 2e degré + \[ + \Delta = 192 > 0 + \] + Il y a donc 2 racines + \[ + x_1 = - 1.7320508075688772 \qquad + x_2 = 1.7320508075688772 + \] + Et on sait que $4x^2 - 12$ est du signe de $a$ donc positif en dehors des racines + \item Le dénominateur $x^2$ est toujours positif. + \item Tableau de variations + + \begin{tikzpicture}[baseline=(a.north)] + \tkzTabInit[lgt=3,espcl=3]{$x$/1,$4x^2 - 12$/1, $x^2$/1, $S'$/1, $S$/2}{$0$, $- 1.7320508075688772$, $10$} + \tkzTabLine{d,-, z, +, } + \tkzTabLine{d,+, , +, } + \tkzTabLine{d,-, z, +, } + \tkzTabVar{D+/ , -/ , +/ } + \end{tikzpicture} + + \end{itemize} + \item On a donc une surface minimal pour $x=1.7320508075688772$ et $h = 5.1961524227066316$. + \end{enumerate} +\end{solution} + +%%% Local Variables: +%%% mode: latex +%%% TeX-master: "master" +%%% End: + +\begin{exercise}[subtitle={Bassin}] + Le tour d'un bassin au niveau du sol présente deux axes de symétrie : l’axe des abscisses et la droite d’équation $x=4$. Il est obtenu par symétrie de la courbe $\mathcal{C}_f$ sur $\intFF{0}{4}$ où $f$ est la fonction définie par + + \[ + f(x) = \left(- x^{2} + 2.8 x - 8.7\right) e^{- x} + 8.7 + \] + On admet que sur $\intFF{0}{4}$ la fonction $f$ est positive. + \begin{enumerate} + \item Sur un repère, tracer l'allure de la courbe $\mathcal{C}_f$, les axes de symétries puis compléter pour dessiner la forme du bassin. + \item Montrer que la fonction $f$ admet comme primitive sur $\R$ la fonction $F$ définie par + \[ + F(x) = 8.7 x + \left( x^{2} - 0.8 x + 7.9\right) e^{- x} + \] + \item Calculer la quantité $\ds \int_0^4 f(x) \; dx$, vous donnerez le résultat sous forme exacte. Interpréter le résultat et reportez cette quantité sur le graphique. + \item On considère que l'échelle de votre graphique est de 1unité pour 15m. Calculer l'aire du bassin. Vous donnerez un résultat arrondi au $m^2$ près. + \end{enumerate} +\end{exercise} + +\begin{solution} + \begin{enumerate} + \item + \begin{tikzpicture}[baseline=(a.north), xscale=1, yscale=0.5] + \tkzInit[xmin=0,xmax=5,xstep=1, + ymin=0,ymax=10,ystep=1] + \tkzGrid + \tkzAxeXY + \tkzFct[domain=0:10,color=red,very thick]% + { (-x**2 + 2.8*x - 8.7)*exp(-x) + 8.7 }; + \end{tikzpicture} + \item Il faut dériver $F(x)$ et vérifier que $F'(x) = f(x)$. + \item $\ds \int_0^4 f(x) \; dx = F(4) - F(0) = \frac{20.7}{e^{4}} + 26.9$ + \item La quantité calculée à la question précédente se retrouve 4fois pour former le bassin. Il faut ensuite prendre en compte l'échelle, comme 1unité de longueur correspond à 15m, une unité d'air correspond à $15\times15 = 225m^2$. Ainsi l'aire du bassin est égale à + \[ + (\frac{20.7}{e^{4}} + 26.9)\times 4 \times 15^2 = 24551.00000 + \] + + \end{enumerate} +\end{solution} + +%%% Local Variables: +%%% mode: latex +%%% TeX-master: "master" +%%% End: + +\begin{exercise}[subtitle={Stylos}] + \emph{Les parties {\rm A} et {\rm B} de cet exercice sont indépendantes.} + + \bigskip + + \begin{minipage}{0.6\linewidth} + \textbf{Partie A} + + \medskip + + Deux ateliers A et B fabriquent des stylos pour une entreprise. + + L'atelier A fabrique 48.0\,\% des stylos, et parmi ceux-là, 99.0\,\% possèdent un défaut de fabrication. + + De plus, 10.0\,\% des stylos possèdent un défaut de fabrication et sortent de l'atelier B. + + Un stylo est prélevé au hasard dans le stock de l'entreprise. + + On considère les évènements suivants: + + \begin{itemize} + \item A : \og Le stylo a été fabriqué par l'atelier A \fg + \item B : \og Le stylo a été fabriqué par l'atelier B \fg + \item D : \og Le stylo possède un défaut de fabrication \fg + \end{itemize} + \end{minipage} + \begin{minipage}{0.4\linewidth} + \begin{center} + \begin{tikzpicture}[sloped] + \node {.} + child {node {$A$} + child {node {$D$} + edge from parent + node[above] {...} + } + child {node {$\overline{D}$} + edge from parent + node[above] {...} + } + edge from parent + node[above] {...} + } + child[missing] {} + child { node {$B$} + child {node {$D$} + edge from parent + node[above] {...} + } + child {node {$\overline{D}$} + edge from parent + node[above] {...} + } + edge from parent + node[above] {...} + } ; + \end{tikzpicture} + \end{center} + \end{minipage} + + \medskip + + \begin{enumerate} + \item Compléter l'arbre de probabilité ci-contre + \item Interpréter puis donner les probabilités $P(A)$, $P(B)$, $P_A(D)$ et $P(B \cap D)$. + + \item + \begin{enumerate} + \item Calculer la probabilité qu'un stylo provienne de l'atelier A et possède un défaut de fabrication. + \item En déduire que la probabilité qu'un stylo possède un défaut de fabrication est de $0.58$. + \end{enumerate} + \item On prélève un stylo au hasard avec un défaut. Quelle est la probabilité qu'il vienne de l'atelier A? + \end{enumerate} + + \bigskip + + \textbf{Partie B} + \medskip + + Dans cette partie, on suppose que 57.99999999999999\,\% des stylos possèdent un défaut de fabrication. + + L'entreprise confectionne des paquets contenant chacun $4$~stylos. + + Le fait qu'un stylo possède ou non un défaut de fabrication est indépendant des autres stylos. + + On appelle $X$ la variable aléatoire donnant pour un paquet le nombre de stylos qui possèdent un défaut de fabrication. + + On admet que la variable aléatoire $X$ suit une loi binomiale. + + \medskip + + \begin{enumerate} + \setcounter{enumi}{4} + \item Avec quelle loi peut-on modéliser $X$. Préciser les paramètres. + \item Calculer et interpréter la probabilité $P(X = 10)$. + \item Le directeur de l'entreprise affirme qu'il y a plus d'une chance sur deux qu'un paquet ne comporte aucun stylo défectueux. A-t-il raison ? + \item Combien de stylos peut-on espérer avoir en moyenne? + \end{enumerate} + \pagebreak +\end{exercise} + +\begin{solution} + \begin{enumerate} + \item + \begin{center} + \begin{tikzpicture}[sloped] + \node {.} + child {node {$A$} + child {node {$D$} + edge from parent + node[above] {0.99} + } + child {node {$\overline{D}$} + edge from parent + node[above] {0.01} + } + edge from parent + node[above] {0.48} + } + child[missing] {} + child { node {$B$} + child {node {$D$} + edge from parent + node[above] {0.2} + } + child {node {$\overline{D}$} + edge from parent + node[above] {0.8} + } + edge from parent + node[above] {0.52} + } ; + \end{tikzpicture} + \end{center} + \item + \begin{itemize} + \item Probabilité que le stylo vienne de l'atelier A + \[ + P(A) = 0.48 + \] + \item Probabilité que le stylo vienne de l'atelier B + \[ + P(B) = 0.52 + \] + \item Probabilité que le stylo ait un défaut sachant qu'il vient de l'atelier A. + \[ + P_A(D) = 0.99 + \] + \item Probabilité que le stylo vienne de l'atelier B et qu'il ait un défaut. + \[ + P(D \cap D) = 0.1 + \] + \end{itemize} + \item + \begin{enumerate} + \item Probabilité qu'un stylo vienne de l'atelier A et qu'il ait un defaut + \[ + P(A\cap D) = P(A) \times P_A(D) = 0.48 \times 0.99 = 0.48 + \] + \item Probabilité que le stylo ai un défaut de fabrication. + \[ + P(D) = P(A\cap D) + P(B\cap D) = 0.48 + 0.1 = 0.58 + \] + \end{enumerate} + \item Probabilité qu'il vienne de l'atelier A sachant qu'il a un defaut + \[ + P_D(A) = \frac{P(A\cap D)}{P(D)} = \frac{0.48}{0.58} = 0.83 + \] + \item $X$ peut être modélisée par une loi binomiale de paramètres $n=17$ et $p=0.58$. + \item (\textit{par de correction automatique disponible pour le résultat final} + \[ + P(X = 10) = \coefBino{17}{10}\times 0.58^{10} \times 0.42^{7} + \] + \item (\textit{par de correction automatique disponible pour le résultat final} + + Il faut calculer la probabilité qu'il y ait 0 stylo avec un defaut. + \[ + P(X = 0) = \coefBino{17}{0}\times 0.58^{0} \times 0.42^{17} + \] + Puis comparer ce nombre à 0,5. + \item Il faut calculer l'espérance + \[ + E[X] = n\times p = 17 \times 0.58 = 9.86 + \] + \end{enumerate} +\end{solution} + +\end{document} + +%%% Local Variables: +%%% mode: latex +%%% TeX-master: "master" +%%% End: diff --git a/Complementaire/DM/2105_DM1/corr_04_2105_DM1.tex b/Complementaire/DM/2105_DM1/corr_04_2105_DM1.tex new file mode 100644 index 0000000..1362fb9 --- /dev/null +++ b/Complementaire/DM/2105_DM1/corr_04_2105_DM1.tex @@ -0,0 +1,367 @@ +\documentclass[a4paper,10pt]{article} +\usepackage{myXsim} + +% Title Page +\title{DM1 \hfill CHAKIR Iman} +\tribe{Maths complémentaire} +\date{\hfillÀ render pour le jeudi 27 mai} + +\xsimsetup{ + solution/print = true +} + +\begin{document} +\maketitle + +Les valeurs des exercices sont générés automatiquement. Si une valeur a un nombre adhérant de chiffres après la virgule, vous pouvez l'arrondir à l'entier le plus proche. + +\begin{exercise}[subtitle={Optimisation de matière}] + \begin{minipage}{0.6\textwidth} + On se propose de fabriquer avec le moins de tôle possible une citerne fermée en forme de parallélépipède rectangle dont le volume intérieur doit être de $20m^3$. La longueur est aussi fixée à $4m$ par le cahier des charges. + + On peut donc faire varier uniquement la largeur (notée $x$) et la hauteur (notée $h$) de la cuve. + \end{minipage} + \hfill + \begin{minipage}{0.3\textwidth} + \begin{tikzpicture} + \pgfmathsetmacro{\cubex}{3} + \pgfmathsetmacro{\cubey}{1} + \pgfmathsetmacro{\cubez}{2} + \draw[black,fill=gray] (0,0,0) -- ++(-\cubex,0,0) -- ++(0,-\cubey,0) node [midway, left] {$h$} -- ++(\cubex,0,0) node [midway, below] {$x$} -- cycle; + \draw[black,fill=gray] (0,0,0) -- ++(0,0,-\cubez) -- ++(0,-\cubey,0) -- ++(0,0,\cubez) node [midway, right] {$4m$} -- cycle; + \draw[black,fill=gray] (0,0,0) -- ++(-\cubex,0,0) -- ++(0,0,-\cubez) -- ++(\cubex,0,0) -- cycle; + \end{tikzpicture} + \end{minipage} + + \begin{enumerate} + \item Expliquer pourquoi quand la largeur $x$ change, la hauteur $h$ doit elle aussi changer pour respecter les contraintes. + \item Démontrer que l'on doit avoir $h = \dfrac{5}{x}$. + \item On note $S(x)$ l'aire totale de la citerne (c'est à dire la somme des aires des six faces). Montrer que l'on peut écrire + \[ + S(x) = 8x + 10 + \frac{40}{x} + \] + \item Démontrer que + \[ + S(x) = \frac{8x^2 + 10x + 40}{x} + \] + \item Démontrer que + \[ + S'(x) = \frac{8x^2 - 40}{x^2} + \] + \item En déduire le tableau de variation de $S(x)$ sur $\intOF{0}{10}$. + \item Déterminer les valeurs de $x$ et $h$ correspondant à une utilisation minimal de tôle. + \end{enumerate} +\end{exercise} + +\begin{solution} + \begin{enumerate} + \item Le volume étant fixe si l'on fait varier $x$, $h$ doit aussi varier. + \begin{itemize} + \item Si $x = 2$ alors conserver un volume de $V=20$, $h$ doit être égale à $5 / 2$ + \item Si $x = 3$ alors conserver un volume de $V=20$, $h$ doit être égale à $5 / 3$ + \end{itemize} + \item Pour calculer le volume, on a + \begin{eqnarray*} + V &=& h\times x \times 4 \\ + 20 &=& h\times x \times 4 \\ + x &=& \frac{20}{h\times 4} = \frac{5}{h} + \end{eqnarray*} + \item Pour calculer la surface totale, on ajoute la surface de chaque face. On a donc le calcul suivant + \begin{eqnarray*} + S(x) &=& x\times h \times 2 + x\times4\times2 + h\times 4\times 2\\ + S(x) &=& x\times \frac{5}{x} \times 2 + x\times4\times2 + \frac{5}{x}\times 4\times 2\\ + S(x) &=& 8x + 10 + \frac{40}{x} + \end{eqnarray*} + \item Pour trouver cette nouvelle forme, on met chaque élément sur le même dénominateur + \begin{eqnarray*} + S(x) &=& 8x + 10 + \frac{40}{x}\\ + S(x) &=& \frac{8x\times x}{x} + \frac{10\times x}{x} + \frac{40}{x}\\ + S(x) &=& \frac{8x^2 + 10x + 40}{x} + \end{eqnarray*} + \item On retrouve la formule $\frac{u}{v}$ à dériver + \[ + u(x) = 8x^2 + 10x + 40 \Rightarrow u'(x) = 16x + 10 + \] + \[ + v(x) = x \Rightarrow v'(x) = 1 + \] + Donc au numérateur on obtient + \begin{eqnarray*} + u'(x)\times v(x) - u(x)\times v'(x) &=& (16x + 10)\times x - (8x^2 + 10x + 40)\times 1\\ + &=& 8x^2 - 40 + \end{eqnarray*} + Donc + \[ + S'(x) = \frac{8x^2 - 40}{x^2} + \] + \item Tableau de variations de $S$ + + \begin{itemize} + \item Valeur interdite: $x^2 = 0 \equiv x = 0$ + \item Signe de $8x^2 - 40$: c'est un polynôme du 2e degré + \[ + \Delta = 1280 > 0 + \] + Il y a donc 2 racines + \[ + x_1 = - 2.23606797749979 \qquad + x_2 = 2.23606797749979 + \] + Et on sait que $8x^2 - 40$ est du signe de $a$ donc positif en dehors des racines + \item Le dénominateur $x^2$ est toujours positif. + \item Tableau de variations + + \begin{tikzpicture}[baseline=(a.north)] + \tkzTabInit[lgt=3,espcl=3]{$x$/1,$8x^2 - 40$/1, $x^2$/1, $S'$/1, $S$/2}{$0$, $- 2.23606797749979$, $10$} + \tkzTabLine{d,-, z, +, } + \tkzTabLine{d,+, , +, } + \tkzTabLine{d,-, z, +, } + \tkzTabVar{D+/ , -/ , +/ } + \end{tikzpicture} + + \end{itemize} + \item On a donc une surface minimal pour $x=2.23606797749979$ et $h = 11.18033988749895$. + \end{enumerate} +\end{solution} + +%%% Local Variables: +%%% mode: latex +%%% TeX-master: "master" +%%% End: + +\begin{exercise}[subtitle={Bassin}] + Le tour d'un bassin au niveau du sol présente deux axes de symétrie : l’axe des abscisses et la droite d’équation $x=4$. Il est obtenu par symétrie de la courbe $\mathcal{C}_f$ sur $\intFF{0}{4}$ où $f$ est la fonction définie par + + \[ + f(x) = \left(- x^{2} + 9.0 x - 9.0\right) e^{- x} + 9.0 + \] + On admet que sur $\intFF{0}{4}$ la fonction $f$ est positive. + \begin{enumerate} + \item Sur un repère, tracer l'allure de la courbe $\mathcal{C}_f$, les axes de symétries puis compléter pour dessiner la forme du bassin. + \item Montrer que la fonction $f$ admet comme primitive sur $\R$ la fonction $F$ définie par + \[ + F(x) = 9.0 x + \left( x^{2} - 7.0 x + 2.0\right) e^{- x} + \] + \item Calculer la quantité $\ds \int_0^4 f(x) \; dx$, vous donnerez le résultat sous forme exacte. Interpréter le résultat et reportez cette quantité sur le graphique. + \item On considère que l'échelle de votre graphique est de 1unité pour 15m. Calculer l'aire du bassin. Vous donnerez un résultat arrondi au $m^2$ près. + \end{enumerate} +\end{exercise} + +\begin{solution} + \begin{enumerate} + \item + \begin{tikzpicture}[baseline=(a.north), xscale=1, yscale=0.5] + \tkzInit[xmin=0,xmax=5,xstep=1, + ymin=0,ymax=10,ystep=1] + \tkzGrid + \tkzAxeXY + \tkzFct[domain=0:10,color=red,very thick]% + { (-x**2 + 9.0*x - 9.0)*exp(-x) + 9.0 }; + \end{tikzpicture} + \item Il faut dériver $F(x)$ et vérifier que $F'(x) = f(x)$. + \item $\ds \int_0^4 f(x) \; dx = F(4) - F(0) = 34.0 - \frac{10.0}{e^{4}}$ + \item La quantité calculée à la question précédente se retrouve 4fois pour former le bassin. Il faut ensuite prendre en compte l'échelle, comme 1unité de longueur correspond à 15m, une unité d'air correspond à $15\times15 = 225m^2$. Ainsi l'aire du bassin est égale à + \[ + (34.0 - \frac{10.0}{e^{4}})\times 4 \times 15^2 = 30435.00000 + \] + + \end{enumerate} +\end{solution} + +%%% Local Variables: +%%% mode: latex +%%% TeX-master: "master" +%%% End: + +\begin{exercise}[subtitle={Stylos}] + \emph{Les parties {\rm A} et {\rm B} de cet exercice sont indépendantes.} + + \bigskip + + \begin{minipage}{0.6\linewidth} + \textbf{Partie A} + + \medskip + + Deux ateliers A et B fabriquent des stylos pour une entreprise. + + L'atelier A fabrique 27.0\,\% des stylos, et parmi ceux-là, 39.0\,\% possèdent un défaut de fabrication. + + De plus, 4.0\,\% des stylos possèdent un défaut de fabrication et sortent de l'atelier B. + + Un stylo est prélevé au hasard dans le stock de l'entreprise. + + On considère les évènements suivants: + + \begin{itemize} + \item A : \og Le stylo a été fabriqué par l'atelier A \fg + \item B : \og Le stylo a été fabriqué par l'atelier B \fg + \item D : \og Le stylo possède un défaut de fabrication \fg + \end{itemize} + \end{minipage} + \begin{minipage}{0.4\linewidth} + \begin{center} + \begin{tikzpicture}[sloped] + \node {.} + child {node {$A$} + child {node {$D$} + edge from parent + node[above] {...} + } + child {node {$\overline{D}$} + edge from parent + node[above] {...} + } + edge from parent + node[above] {...} + } + child[missing] {} + child { node {$B$} + child {node {$D$} + edge from parent + node[above] {...} + } + child {node {$\overline{D}$} + edge from parent + node[above] {...} + } + edge from parent + node[above] {...} + } ; + \end{tikzpicture} + \end{center} + \end{minipage} + + \medskip + + \begin{enumerate} + \item Compléter l'arbre de probabilité ci-contre + \item Interpréter puis donner les probabilités $P(A)$, $P(B)$, $P_A(D)$ et $P(B \cap D)$. + + \item + \begin{enumerate} + \item Calculer la probabilité qu'un stylo provienne de l'atelier A et possède un défaut de fabrication. + \item En déduire que la probabilité qu'un stylo possède un défaut de fabrication est de $0.15$. + \end{enumerate} + \item On prélève un stylo au hasard avec un défaut. Quelle est la probabilité qu'il vienne de l'atelier A? + \end{enumerate} + + \bigskip + + \textbf{Partie B} + \medskip + + Dans cette partie, on suppose que 15.0\,\% des stylos possèdent un défaut de fabrication. + + L'entreprise confectionne des paquets contenant chacun $4$~stylos. + + Le fait qu'un stylo possède ou non un défaut de fabrication est indépendant des autres stylos. + + On appelle $X$ la variable aléatoire donnant pour un paquet le nombre de stylos qui possèdent un défaut de fabrication. + + On admet que la variable aléatoire $X$ suit une loi binomiale. + + \medskip + + \begin{enumerate} + \setcounter{enumi}{4} + \item Avec quelle loi peut-on modéliser $X$. Préciser les paramètres. + \item Calculer et interpréter la probabilité $P(X = 9)$. + \item Le directeur de l'entreprise affirme qu'il y a plus d'une chance sur deux qu'un paquet ne comporte aucun stylo défectueux. A-t-il raison ? + \item Combien de stylos peut-on espérer avoir en moyenne? + \end{enumerate} + \pagebreak +\end{exercise} + +\begin{solution} + \begin{enumerate} + \item + \begin{center} + \begin{tikzpicture}[sloped] + \node {.} + child {node {$A$} + child {node {$D$} + edge from parent + node[above] {0.39} + } + child {node {$\overline{D}$} + edge from parent + node[above] {0.61} + } + edge from parent + node[above] {0.27} + } + child[missing] {} + child { node {$B$} + child {node {$D$} + edge from parent + node[above] {0.05} + } + child {node {$\overline{D}$} + edge from parent + node[above] {0.95} + } + edge from parent + node[above] {0.73} + } ; + \end{tikzpicture} + \end{center} + \item + \begin{itemize} + \item Probabilité que le stylo vienne de l'atelier A + \[ + P(A) = 0.27 + \] + \item Probabilité que le stylo vienne de l'atelier B + \[ + P(B) = 0.73 + \] + \item Probabilité que le stylo ait un défaut sachant qu'il vient de l'atelier A. + \[ + P_A(D) = 0.39 + \] + \item Probabilité que le stylo vienne de l'atelier B et qu'il ait un défaut. + \[ + P(D \cap D) = 0.04 + \] + \end{itemize} + \item + \begin{enumerate} + \item Probabilité qu'un stylo vienne de l'atelier A et qu'il ait un defaut + \[ + P(A\cap D) = P(A) \times P_A(D) = 0.27 \times 0.39 = 0.11 + \] + \item Probabilité que le stylo ai un défaut de fabrication. + \[ + P(D) = P(A\cap D) + P(B\cap D) = 0.11 + 0.04 = 0.15 + \] + \end{enumerate} + \item Probabilité qu'il vienne de l'atelier A sachant qu'il a un defaut + \[ + P_D(A) = \frac{P(A\cap D)}{P(D)} = \frac{0.11}{0.15} = 0.73 + \] + \item $X$ peut être modélisée par une loi binomiale de paramètres $n=17$ et $p=0.15$. + \item (\textit{par de correction automatique disponible pour le résultat final} + \[ + P(X = 9) = \coefBino{17}{9}\times 0.15^{9} \times 0.85^{8} + \] + \item (\textit{par de correction automatique disponible pour le résultat final} + + Il faut calculer la probabilité qu'il y ait 0 stylo avec un defaut. + \[ + P(X = 0) = \coefBino{17}{0}\times 0.15^{0} \times 0.85^{17} + \] + Puis comparer ce nombre à 0,5. + \item Il faut calculer l'espérance + \[ + E[X] = n\times p = 17 \times 0.15 = 2.55 + \] + \end{enumerate} +\end{solution} + +\end{document} + +%%% Local Variables: +%%% mode: latex +%%% TeX-master: "master" +%%% End: diff --git a/Complementaire/DM/2105_DM1/corr_05_2105_DM1.tex b/Complementaire/DM/2105_DM1/corr_05_2105_DM1.tex new file mode 100644 index 0000000..4f4859e --- /dev/null +++ b/Complementaire/DM/2105_DM1/corr_05_2105_DM1.tex @@ -0,0 +1,367 @@ +\documentclass[a4paper,10pt]{article} +\usepackage{myXsim} + +% Title Page +\title{DM1 \hfill GERMAIN Margot} +\tribe{Maths complémentaire} +\date{\hfillÀ render pour le jeudi 27 mai} + +\xsimsetup{ + solution/print = true +} + +\begin{document} +\maketitle + +Les valeurs des exercices sont générés automatiquement. Si une valeur a un nombre adhérant de chiffres après la virgule, vous pouvez l'arrondir à l'entier le plus proche. + +\begin{exercise}[subtitle={Optimisation de matière}] + \begin{minipage}{0.6\textwidth} + On se propose de fabriquer avec le moins de tôle possible une citerne fermée en forme de parallélépipède rectangle dont le volume intérieur doit être de $35m^3$. La longueur est aussi fixée à $5m$ par le cahier des charges. + + On peut donc faire varier uniquement la largeur (notée $x$) et la hauteur (notée $h$) de la cuve. + \end{minipage} + \hfill + \begin{minipage}{0.3\textwidth} + \begin{tikzpicture} + \pgfmathsetmacro{\cubex}{3} + \pgfmathsetmacro{\cubey}{1} + \pgfmathsetmacro{\cubez}{2} + \draw[black,fill=gray] (0,0,0) -- ++(-\cubex,0,0) -- ++(0,-\cubey,0) node [midway, left] {$h$} -- ++(\cubex,0,0) node [midway, below] {$x$} -- cycle; + \draw[black,fill=gray] (0,0,0) -- ++(0,0,-\cubez) -- ++(0,-\cubey,0) -- ++(0,0,\cubez) node [midway, right] {$5m$} -- cycle; + \draw[black,fill=gray] (0,0,0) -- ++(-\cubex,0,0) -- ++(0,0,-\cubez) -- ++(\cubex,0,0) -- cycle; + \end{tikzpicture} + \end{minipage} + + \begin{enumerate} + \item Expliquer pourquoi quand la largeur $x$ change, la hauteur $h$ doit elle aussi changer pour respecter les contraintes. + \item Démontrer que l'on doit avoir $h = \dfrac{7}{x}$. + \item On note $S(x)$ l'aire totale de la citerne (c'est à dire la somme des aires des six faces). Montrer que l'on peut écrire + \[ + S(x) = 10x + 14 + \frac{70}{x} + \] + \item Démontrer que + \[ + S(x) = \frac{10x^2 + 14x + 70}{x} + \] + \item Démontrer que + \[ + S'(x) = \frac{10x^2 - 70}{x^2} + \] + \item En déduire le tableau de variation de $S(x)$ sur $\intOF{0}{10}$. + \item Déterminer les valeurs de $x$ et $h$ correspondant à une utilisation minimal de tôle. + \end{enumerate} +\end{exercise} + +\begin{solution} + \begin{enumerate} + \item Le volume étant fixe si l'on fait varier $x$, $h$ doit aussi varier. + \begin{itemize} + \item Si $x = 2$ alors conserver un volume de $V=35$, $h$ doit être égale à $7 / 2$ + \item Si $x = 3$ alors conserver un volume de $V=35$, $h$ doit être égale à $7 / 3$ + \end{itemize} + \item Pour calculer le volume, on a + \begin{eqnarray*} + V &=& h\times x \times 5 \\ + 35 &=& h\times x \times 5 \\ + x &=& \frac{35}{h\times 5} = \frac{7}{h} + \end{eqnarray*} + \item Pour calculer la surface totale, on ajoute la surface de chaque face. On a donc le calcul suivant + \begin{eqnarray*} + S(x) &=& x\times h \times 2 + x\times5\times2 + h\times 5\times 2\\ + S(x) &=& x\times \frac{7}{x} \times 2 + x\times5\times2 + \frac{7}{x}\times 5\times 2\\ + S(x) &=& 10x + 14 + \frac{70}{x} + \end{eqnarray*} + \item Pour trouver cette nouvelle forme, on met chaque élément sur le même dénominateur + \begin{eqnarray*} + S(x) &=& 10x + 14 + \frac{70}{x}\\ + S(x) &=& \frac{10x\times x}{x} + \frac{14\times x}{x} + \frac{70}{x}\\ + S(x) &=& \frac{10x^2 + 14x + 70}{x} + \end{eqnarray*} + \item On retrouve la formule $\frac{u}{v}$ à dériver + \[ + u(x) = 10x^2 + 14x + 70 \Rightarrow u'(x) = 20x + 14 + \] + \[ + v(x) = x \Rightarrow v'(x) = 1 + \] + Donc au numérateur on obtient + \begin{eqnarray*} + u'(x)\times v(x) - u(x)\times v'(x) &=& (20x + 14)\times x - (10x^2 + 14x + 70)\times 1\\ + &=& 10x^2 - 70 + \end{eqnarray*} + Donc + \[ + S'(x) = \frac{10x^2 - 70}{x^2} + \] + \item Tableau de variations de $S$ + + \begin{itemize} + \item Valeur interdite: $x^2 = 0 \equiv x = 0$ + \item Signe de $10x^2 - 70$: c'est un polynôme du 2e degré + \[ + \Delta = 2800 > 0 + \] + Il y a donc 2 racines + \[ + x_1 = - 2.6457513110645907 \qquad + x_2 = 2.6457513110645907 + \] + Et on sait que $10x^2 - 70$ est du signe de $a$ donc positif en dehors des racines + \item Le dénominateur $x^2$ est toujours positif. + \item Tableau de variations + + \begin{tikzpicture}[baseline=(a.north)] + \tkzTabInit[lgt=3,espcl=3]{$x$/1,$10x^2 - 70$/1, $x^2$/1, $S'$/1, $S$/2}{$0$, $- 2.6457513110645907$, $10$} + \tkzTabLine{d,-, z, +, } + \tkzTabLine{d,+, , +, } + \tkzTabLine{d,-, z, +, } + \tkzTabVar{D+/ , -/ , +/ } + \end{tikzpicture} + + \end{itemize} + \item On a donc une surface minimal pour $x=2.6457513110645907$ et $h = 18.5202591774521349$. + \end{enumerate} +\end{solution} + +%%% Local Variables: +%%% mode: latex +%%% TeX-master: "master" +%%% End: + +\begin{exercise}[subtitle={Bassin}] + Le tour d'un bassin au niveau du sol présente deux axes de symétrie : l’axe des abscisses et la droite d’équation $x=4$. Il est obtenu par symétrie de la courbe $\mathcal{C}_f$ sur $\intFF{0}{4}$ où $f$ est la fonction définie par + + \[ + f(x) = \left(- x^{2} + 8.0 x - 1.1\right) e^{- x} + 1.1 + \] + On admet que sur $\intFF{0}{4}$ la fonction $f$ est positive. + \begin{enumerate} + \item Sur un repère, tracer l'allure de la courbe $\mathcal{C}_f$, les axes de symétries puis compléter pour dessiner la forme du bassin. + \item Montrer que la fonction $f$ admet comme primitive sur $\R$ la fonction $F$ définie par + \[ + F(x) = 1.1 x + \left( x^{2} - 6.0 x - 4.9\right) e^{- x} + \] + \item Calculer la quantité $\ds \int_0^4 f(x) \; dx$, vous donnerez le résultat sous forme exacte. Interpréter le résultat et reportez cette quantité sur le graphique. + \item On considère que l'échelle de votre graphique est de 1unité pour 15m. Calculer l'aire du bassin. Vous donnerez un résultat arrondi au $m^2$ près. + \end{enumerate} +\end{exercise} + +\begin{solution} + \begin{enumerate} + \item + \begin{tikzpicture}[baseline=(a.north), xscale=1, yscale=0.5] + \tkzInit[xmin=0,xmax=5,xstep=1, + ymin=0,ymax=10,ystep=1] + \tkzGrid + \tkzAxeXY + \tkzFct[domain=0:10,color=red,very thick]% + { (-x**2 + 8.0*x - 1.1)*exp(-x) + 1.1 }; + \end{tikzpicture} + \item Il faut dériver $F(x)$ et vérifier que $F'(x) = f(x)$. + \item $\ds \int_0^4 f(x) \; dx = F(4) - F(0) = 9.3 - \frac{12.9}{e^{4}}$ + \item La quantité calculée à la question précédente se retrouve 4fois pour former le bassin. Il faut ensuite prendre en compte l'échelle, comme 1unité de longueur correspond à 15m, une unité d'air correspond à $15\times15 = 225m^2$. Ainsi l'aire du bassin est égale à + \[ + (9.3 - \frac{12.9}{e^{4}})\times 4 \times 15^2 = 8157.000000 + \] + + \end{enumerate} +\end{solution} + +%%% Local Variables: +%%% mode: latex +%%% TeX-master: "master" +%%% End: + +\begin{exercise}[subtitle={Stylos}] + \emph{Les parties {\rm A} et {\rm B} de cet exercice sont indépendantes.} + + \bigskip + + \begin{minipage}{0.6\linewidth} + \textbf{Partie A} + + \medskip + + Deux ateliers A et B fabriquent des stylos pour une entreprise. + + L'atelier A fabrique 81.0\,\% des stylos, et parmi ceux-là, 57.99999999999999\,\% possèdent un défaut de fabrication. + + De plus, 2.0\,\% des stylos possèdent un défaut de fabrication et sortent de l'atelier B. + + Un stylo est prélevé au hasard dans le stock de l'entreprise. + + On considère les évènements suivants: + + \begin{itemize} + \item A : \og Le stylo a été fabriqué par l'atelier A \fg + \item B : \og Le stylo a été fabriqué par l'atelier B \fg + \item D : \og Le stylo possède un défaut de fabrication \fg + \end{itemize} + \end{minipage} + \begin{minipage}{0.4\linewidth} + \begin{center} + \begin{tikzpicture}[sloped] + \node {.} + child {node {$A$} + child {node {$D$} + edge from parent + node[above] {...} + } + child {node {$\overline{D}$} + edge from parent + node[above] {...} + } + edge from parent + node[above] {...} + } + child[missing] {} + child { node {$B$} + child {node {$D$} + edge from parent + node[above] {...} + } + child {node {$\overline{D}$} + edge from parent + node[above] {...} + } + edge from parent + node[above] {...} + } ; + \end{tikzpicture} + \end{center} + \end{minipage} + + \medskip + + \begin{enumerate} + \item Compléter l'arbre de probabilité ci-contre + \item Interpréter puis donner les probabilités $P(A)$, $P(B)$, $P_A(D)$ et $P(B \cap D)$. + + \item + \begin{enumerate} + \item Calculer la probabilité qu'un stylo provienne de l'atelier A et possède un défaut de fabrication. + \item En déduire que la probabilité qu'un stylo possède un défaut de fabrication est de $0.49$. + \end{enumerate} + \item On prélève un stylo au hasard avec un défaut. Quelle est la probabilité qu'il vienne de l'atelier A? + \end{enumerate} + + \bigskip + + \textbf{Partie B} + \medskip + + Dans cette partie, on suppose que 49.0\,\% des stylos possèdent un défaut de fabrication. + + L'entreprise confectionne des paquets contenant chacun $4$~stylos. + + Le fait qu'un stylo possède ou non un défaut de fabrication est indépendant des autres stylos. + + On appelle $X$ la variable aléatoire donnant pour un paquet le nombre de stylos qui possèdent un défaut de fabrication. + + On admet que la variable aléatoire $X$ suit une loi binomiale. + + \medskip + + \begin{enumerate} + \setcounter{enumi}{4} + \item Avec quelle loi peut-on modéliser $X$. Préciser les paramètres. + \item Calculer et interpréter la probabilité $P(X = 14)$. + \item Le directeur de l'entreprise affirme qu'il y a plus d'une chance sur deux qu'un paquet ne comporte aucun stylo défectueux. A-t-il raison ? + \item Combien de stylos peut-on espérer avoir en moyenne? + \end{enumerate} + \pagebreak +\end{exercise} + +\begin{solution} + \begin{enumerate} + \item + \begin{center} + \begin{tikzpicture}[sloped] + \node {.} + child {node {$A$} + child {node {$D$} + edge from parent + node[above] {0.58} + } + child {node {$\overline{D}$} + edge from parent + node[above] {0.42} + } + edge from parent + node[above] {0.81} + } + child[missing] {} + child { node {$B$} + child {node {$D$} + edge from parent + node[above] {0.13} + } + child {node {$\overline{D}$} + edge from parent + node[above] {0.87} + } + edge from parent + node[above] {0.19} + } ; + \end{tikzpicture} + \end{center} + \item + \begin{itemize} + \item Probabilité que le stylo vienne de l'atelier A + \[ + P(A) = 0.81 + \] + \item Probabilité que le stylo vienne de l'atelier B + \[ + P(B) = 0.19 + \] + \item Probabilité que le stylo ait un défaut sachant qu'il vient de l'atelier A. + \[ + P_A(D) = 0.58 + \] + \item Probabilité que le stylo vienne de l'atelier B et qu'il ait un défaut. + \[ + P(D \cap D) = 0.02 + \] + \end{itemize} + \item + \begin{enumerate} + \item Probabilité qu'un stylo vienne de l'atelier A et qu'il ait un defaut + \[ + P(A\cap D) = P(A) \times P_A(D) = 0.81 \times 0.58 = 0.47 + \] + \item Probabilité que le stylo ai un défaut de fabrication. + \[ + P(D) = P(A\cap D) + P(B\cap D) = 0.47 + 0.02 = 0.49 + \] + \end{enumerate} + \item Probabilité qu'il vienne de l'atelier A sachant qu'il a un defaut + \[ + P_D(A) = \frac{P(A\cap D)}{P(D)} = \frac{0.47}{0.49} = 0.96 + \] + \item $X$ peut être modélisée par une loi binomiale de paramètres $n=20$ et $p=0.49$. + \item (\textit{par de correction automatique disponible pour le résultat final} + \[ + P(X = 14) = \coefBino{20}{14}\times 0.49^{14} \times 0.51^{6} + \] + \item (\textit{par de correction automatique disponible pour le résultat final} + + Il faut calculer la probabilité qu'il y ait 0 stylo avec un defaut. + \[ + P(X = 0) = \coefBino{20}{0}\times 0.49^{0} \times 0.51^{20} + \] + Puis comparer ce nombre à 0,5. + \item Il faut calculer l'espérance + \[ + E[X] = n\times p = 20 \times 0.49 = 9.8 + \] + \end{enumerate} +\end{solution} + +\end{document} + +%%% Local Variables: +%%% mode: latex +%%% TeX-master: "master" +%%% End: diff --git a/Complementaire/DM/2105_DM1/corr_06_2105_DM1.tex b/Complementaire/DM/2105_DM1/corr_06_2105_DM1.tex new file mode 100644 index 0000000..db0426e --- /dev/null +++ b/Complementaire/DM/2105_DM1/corr_06_2105_DM1.tex @@ -0,0 +1,367 @@ +\documentclass[a4paper,10pt]{article} +\usepackage{myXsim} + +% Title Page +\title{DM1 \hfill HOKELEKLI Damla} +\tribe{Maths complémentaire} +\date{\hfillÀ render pour le jeudi 27 mai} + +\xsimsetup{ + solution/print = true +} + +\begin{document} +\maketitle + +Les valeurs des exercices sont générés automatiquement. Si une valeur a un nombre adhérant de chiffres après la virgule, vous pouvez l'arrondir à l'entier le plus proche. + +\begin{exercise}[subtitle={Optimisation de matière}] + \begin{minipage}{0.6\textwidth} + On se propose de fabriquer avec le moins de tôle possible une citerne fermée en forme de parallélépipède rectangle dont le volume intérieur doit être de $10m^3$. La longueur est aussi fixée à $2m$ par le cahier des charges. + + On peut donc faire varier uniquement la largeur (notée $x$) et la hauteur (notée $h$) de la cuve. + \end{minipage} + \hfill + \begin{minipage}{0.3\textwidth} + \begin{tikzpicture} + \pgfmathsetmacro{\cubex}{3} + \pgfmathsetmacro{\cubey}{1} + \pgfmathsetmacro{\cubez}{2} + \draw[black,fill=gray] (0,0,0) -- ++(-\cubex,0,0) -- ++(0,-\cubey,0) node [midway, left] {$h$} -- ++(\cubex,0,0) node [midway, below] {$x$} -- cycle; + \draw[black,fill=gray] (0,0,0) -- ++(0,0,-\cubez) -- ++(0,-\cubey,0) -- ++(0,0,\cubez) node [midway, right] {$2m$} -- cycle; + \draw[black,fill=gray] (0,0,0) -- ++(-\cubex,0,0) -- ++(0,0,-\cubez) -- ++(\cubex,0,0) -- cycle; + \end{tikzpicture} + \end{minipage} + + \begin{enumerate} + \item Expliquer pourquoi quand la largeur $x$ change, la hauteur $h$ doit elle aussi changer pour respecter les contraintes. + \item Démontrer que l'on doit avoir $h = \dfrac{5}{x}$. + \item On note $S(x)$ l'aire totale de la citerne (c'est à dire la somme des aires des six faces). Montrer que l'on peut écrire + \[ + S(x) = 4x + 10 + \frac{20}{x} + \] + \item Démontrer que + \[ + S(x) = \frac{4x^2 + 10x + 20}{x} + \] + \item Démontrer que + \[ + S'(x) = \frac{4x^2 - 20}{x^2} + \] + \item En déduire le tableau de variation de $S(x)$ sur $\intOF{0}{10}$. + \item Déterminer les valeurs de $x$ et $h$ correspondant à une utilisation minimal de tôle. + \end{enumerate} +\end{exercise} + +\begin{solution} + \begin{enumerate} + \item Le volume étant fixe si l'on fait varier $x$, $h$ doit aussi varier. + \begin{itemize} + \item Si $x = 2$ alors conserver un volume de $V=10$, $h$ doit être égale à $5 / 2$ + \item Si $x = 3$ alors conserver un volume de $V=10$, $h$ doit être égale à $5 / 3$ + \end{itemize} + \item Pour calculer le volume, on a + \begin{eqnarray*} + V &=& h\times x \times 2 \\ + 10 &=& h\times x \times 2 \\ + x &=& \frac{10}{h\times 2} = \frac{5}{h} + \end{eqnarray*} + \item Pour calculer la surface totale, on ajoute la surface de chaque face. On a donc le calcul suivant + \begin{eqnarray*} + S(x) &=& x\times h \times 2 + x\times2\times2 + h\times 2\times 2\\ + S(x) &=& x\times \frac{5}{x} \times 2 + x\times2\times2 + \frac{5}{x}\times 2\times 2\\ + S(x) &=& 4x + 10 + \frac{20}{x} + \end{eqnarray*} + \item Pour trouver cette nouvelle forme, on met chaque élément sur le même dénominateur + \begin{eqnarray*} + S(x) &=& 4x + 10 + \frac{20}{x}\\ + S(x) &=& \frac{4x\times x}{x} + \frac{10\times x}{x} + \frac{20}{x}\\ + S(x) &=& \frac{4x^2 + 10x + 20}{x} + \end{eqnarray*} + \item On retrouve la formule $\frac{u}{v}$ à dériver + \[ + u(x) = 4x^2 + 10x + 20 \Rightarrow u'(x) = 8x + 10 + \] + \[ + v(x) = x \Rightarrow v'(x) = 1 + \] + Donc au numérateur on obtient + \begin{eqnarray*} + u'(x)\times v(x) - u(x)\times v'(x) &=& (8x + 10)\times x - (4x^2 + 10x + 20)\times 1\\ + &=& 4x^2 - 20 + \end{eqnarray*} + Donc + \[ + S'(x) = \frac{4x^2 - 20}{x^2} + \] + \item Tableau de variations de $S$ + + \begin{itemize} + \item Valeur interdite: $x^2 = 0 \equiv x = 0$ + \item Signe de $4x^2 - 20$: c'est un polynôme du 2e degré + \[ + \Delta = 320 > 0 + \] + Il y a donc 2 racines + \[ + x_1 = - 2.23606797749979 \qquad + x_2 = 2.23606797749979 + \] + Et on sait que $4x^2 - 20$ est du signe de $a$ donc positif en dehors des racines + \item Le dénominateur $x^2$ est toujours positif. + \item Tableau de variations + + \begin{tikzpicture}[baseline=(a.north)] + \tkzTabInit[lgt=3,espcl=3]{$x$/1,$4x^2 - 20$/1, $x^2$/1, $S'$/1, $S$/2}{$0$, $- 2.23606797749979$, $10$} + \tkzTabLine{d,-, z, +, } + \tkzTabLine{d,+, , +, } + \tkzTabLine{d,-, z, +, } + \tkzTabVar{D+/ , -/ , +/ } + \end{tikzpicture} + + \end{itemize} + \item On a donc une surface minimal pour $x=2.23606797749979$ et $h = 11.18033988749895$. + \end{enumerate} +\end{solution} + +%%% Local Variables: +%%% mode: latex +%%% TeX-master: "master" +%%% End: + +\begin{exercise}[subtitle={Bassin}] + Le tour d'un bassin au niveau du sol présente deux axes de symétrie : l’axe des abscisses et la droite d’équation $x=4$. Il est obtenu par symétrie de la courbe $\mathcal{C}_f$ sur $\intFF{0}{4}$ où $f$ est la fonction définie par + + \[ + f(x) = \left(- x^{2} + 5.6 x - 1.6\right) e^{- x} + 1.6 + \] + On admet que sur $\intFF{0}{4}$ la fonction $f$ est positive. + \begin{enumerate} + \item Sur un repère, tracer l'allure de la courbe $\mathcal{C}_f$, les axes de symétries puis compléter pour dessiner la forme du bassin. + \item Montrer que la fonction $f$ admet comme primitive sur $\R$ la fonction $F$ définie par + \[ + F(x) = 1.6 x + \left( x^{2} - 3.6 x - 2.0\right) e^{- x} + \] + \item Calculer la quantité $\ds \int_0^4 f(x) \; dx$, vous donnerez le résultat sous forme exacte. Interpréter le résultat et reportez cette quantité sur le graphique. + \item On considère que l'échelle de votre graphique est de 1unité pour 15m. Calculer l'aire du bassin. Vous donnerez un résultat arrondi au $m^2$ près. + \end{enumerate} +\end{exercise} + +\begin{solution} + \begin{enumerate} + \item + \begin{tikzpicture}[baseline=(a.north), xscale=1, yscale=0.5] + \tkzInit[xmin=0,xmax=5,xstep=1, + ymin=0,ymax=10,ystep=1] + \tkzGrid + \tkzAxeXY + \tkzFct[domain=0:10,color=red,very thick]% + { (-x**2 + 5.6*x - 1.6)*exp(-x) + 1.6 }; + \end{tikzpicture} + \item Il faut dériver $F(x)$ et vérifier que $F'(x) = f(x)$. + \item $\ds \int_0^4 f(x) \; dx = F(4) - F(0) = 8.4 - \frac{0.399999999999999}{e^{4}}$ + \item La quantité calculée à la question précédente se retrouve 4fois pour former le bassin. Il faut ensuite prendre en compte l'échelle, comme 1unité de longueur correspond à 15m, une unité d'air correspond à $15\times15 = 225m^2$. Ainsi l'aire du bassin est égale à + \[ + (8.4 - \frac{0.399999999999999}{e^{4}})\times 4 \times 15^2 = 7553.000000 + \] + + \end{enumerate} +\end{solution} + +%%% Local Variables: +%%% mode: latex +%%% TeX-master: "master" +%%% End: + +\begin{exercise}[subtitle={Stylos}] + \emph{Les parties {\rm A} et {\rm B} de cet exercice sont indépendantes.} + + \bigskip + + \begin{minipage}{0.6\linewidth} + \textbf{Partie A} + + \medskip + + Deux ateliers A et B fabriquent des stylos pour une entreprise. + + L'atelier A fabrique 47.0\,\% des stylos, et parmi ceux-là, 25.0\,\% possèdent un défaut de fabrication. + + De plus, 23.0\,\% des stylos possèdent un défaut de fabrication et sortent de l'atelier B. + + Un stylo est prélevé au hasard dans le stock de l'entreprise. + + On considère les évènements suivants: + + \begin{itemize} + \item A : \og Le stylo a été fabriqué par l'atelier A \fg + \item B : \og Le stylo a été fabriqué par l'atelier B \fg + \item D : \og Le stylo possède un défaut de fabrication \fg + \end{itemize} + \end{minipage} + \begin{minipage}{0.4\linewidth} + \begin{center} + \begin{tikzpicture}[sloped] + \node {.} + child {node {$A$} + child {node {$D$} + edge from parent + node[above] {...} + } + child {node {$\overline{D}$} + edge from parent + node[above] {...} + } + edge from parent + node[above] {...} + } + child[missing] {} + child { node {$B$} + child {node {$D$} + edge from parent + node[above] {...} + } + child {node {$\overline{D}$} + edge from parent + node[above] {...} + } + edge from parent + node[above] {...} + } ; + \end{tikzpicture} + \end{center} + \end{minipage} + + \medskip + + \begin{enumerate} + \item Compléter l'arbre de probabilité ci-contre + \item Interpréter puis donner les probabilités $P(A)$, $P(B)$, $P_A(D)$ et $P(B \cap D)$. + + \item + \begin{enumerate} + \item Calculer la probabilité qu'un stylo provienne de l'atelier A et possède un défaut de fabrication. + \item En déduire que la probabilité qu'un stylo possède un défaut de fabrication est de $0.35$. + \end{enumerate} + \item On prélève un stylo au hasard avec un défaut. Quelle est la probabilité qu'il vienne de l'atelier A? + \end{enumerate} + + \bigskip + + \textbf{Partie B} + \medskip + + Dans cette partie, on suppose que 35.0\,\% des stylos possèdent un défaut de fabrication. + + L'entreprise confectionne des paquets contenant chacun $4$~stylos. + + Le fait qu'un stylo possède ou non un défaut de fabrication est indépendant des autres stylos. + + On appelle $X$ la variable aléatoire donnant pour un paquet le nombre de stylos qui possèdent un défaut de fabrication. + + On admet que la variable aléatoire $X$ suit une loi binomiale. + + \medskip + + \begin{enumerate} + \setcounter{enumi}{4} + \item Avec quelle loi peut-on modéliser $X$. Préciser les paramètres. + \item Calculer et interpréter la probabilité $P(X = 17)$. + \item Le directeur de l'entreprise affirme qu'il y a plus d'une chance sur deux qu'un paquet ne comporte aucun stylo défectueux. A-t-il raison ? + \item Combien de stylos peut-on espérer avoir en moyenne? + \end{enumerate} + \pagebreak +\end{exercise} + +\begin{solution} + \begin{enumerate} + \item + \begin{center} + \begin{tikzpicture}[sloped] + \node {.} + child {node {$A$} + child {node {$D$} + edge from parent + node[above] {0.25} + } + child {node {$\overline{D}$} + edge from parent + node[above] {0.75} + } + edge from parent + node[above] {0.47} + } + child[missing] {} + child { node {$B$} + child {node {$D$} + edge from parent + node[above] {0.44} + } + child {node {$\overline{D}$} + edge from parent + node[above] {0.56} + } + edge from parent + node[above] {0.53} + } ; + \end{tikzpicture} + \end{center} + \item + \begin{itemize} + \item Probabilité que le stylo vienne de l'atelier A + \[ + P(A) = 0.47 + \] + \item Probabilité que le stylo vienne de l'atelier B + \[ + P(B) = 0.53 + \] + \item Probabilité que le stylo ait un défaut sachant qu'il vient de l'atelier A. + \[ + P_A(D) = 0.25 + \] + \item Probabilité que le stylo vienne de l'atelier B et qu'il ait un défaut. + \[ + P(D \cap D) = 0.23 + \] + \end{itemize} + \item + \begin{enumerate} + \item Probabilité qu'un stylo vienne de l'atelier A et qu'il ait un defaut + \[ + P(A\cap D) = P(A) \times P_A(D) = 0.47 \times 0.25 = 0.12 + \] + \item Probabilité que le stylo ai un défaut de fabrication. + \[ + P(D) = P(A\cap D) + P(B\cap D) = 0.12 + 0.23 = 0.35 + \] + \end{enumerate} + \item Probabilité qu'il vienne de l'atelier A sachant qu'il a un defaut + \[ + P_D(A) = \frac{P(A\cap D)}{P(D)} = \frac{0.12}{0.35} = 0.34 + \] + \item $X$ peut être modélisée par une loi binomiale de paramètres $n=19$ et $p=0.35$. + \item (\textit{par de correction automatique disponible pour le résultat final} + \[ + P(X = 17) = \coefBino{19}{17}\times 0.35^{17} \times 0.65^{2} + \] + \item (\textit{par de correction automatique disponible pour le résultat final} + + Il faut calculer la probabilité qu'il y ait 0 stylo avec un defaut. + \[ + P(X = 0) = \coefBino{19}{0}\times 0.35^{0} \times 0.65^{19} + \] + Puis comparer ce nombre à 0,5. + \item Il faut calculer l'espérance + \[ + E[X] = n\times p = 19 \times 0.35 = 6.65 + \] + \end{enumerate} +\end{solution} + +\end{document} + +%%% Local Variables: +%%% mode: latex +%%% TeX-master: "master" +%%% End: diff --git a/Complementaire/DM/2105_DM1/corr_07_2105_DM1.tex b/Complementaire/DM/2105_DM1/corr_07_2105_DM1.tex new file mode 100644 index 0000000..fa24c39 --- /dev/null +++ b/Complementaire/DM/2105_DM1/corr_07_2105_DM1.tex @@ -0,0 +1,367 @@ +\documentclass[a4paper,10pt]{article} +\usepackage{myXsim} + +% Title Page +\title{DM1 \hfill KICHENASSAMY Kévin} +\tribe{Maths complémentaire} +\date{\hfillÀ render pour le jeudi 27 mai} + +\xsimsetup{ + solution/print = true +} + +\begin{document} +\maketitle + +Les valeurs des exercices sont générés automatiquement. Si une valeur a un nombre adhérant de chiffres après la virgule, vous pouvez l'arrondir à l'entier le plus proche. + +\begin{exercise}[subtitle={Optimisation de matière}] + \begin{minipage}{0.6\textwidth} + On se propose de fabriquer avec le moins de tôle possible une citerne fermée en forme de parallélépipède rectangle dont le volume intérieur doit être de $16m^3$. La longueur est aussi fixée à $4m$ par le cahier des charges. + + On peut donc faire varier uniquement la largeur (notée $x$) et la hauteur (notée $h$) de la cuve. + \end{minipage} + \hfill + \begin{minipage}{0.3\textwidth} + \begin{tikzpicture} + \pgfmathsetmacro{\cubex}{3} + \pgfmathsetmacro{\cubey}{1} + \pgfmathsetmacro{\cubez}{2} + \draw[black,fill=gray] (0,0,0) -- ++(-\cubex,0,0) -- ++(0,-\cubey,0) node [midway, left] {$h$} -- ++(\cubex,0,0) node [midway, below] {$x$} -- cycle; + \draw[black,fill=gray] (0,0,0) -- ++(0,0,-\cubez) -- ++(0,-\cubey,0) -- ++(0,0,\cubez) node [midway, right] {$4m$} -- cycle; + \draw[black,fill=gray] (0,0,0) -- ++(-\cubex,0,0) -- ++(0,0,-\cubez) -- ++(\cubex,0,0) -- cycle; + \end{tikzpicture} + \end{minipage} + + \begin{enumerate} + \item Expliquer pourquoi quand la largeur $x$ change, la hauteur $h$ doit elle aussi changer pour respecter les contraintes. + \item Démontrer que l'on doit avoir $h = \dfrac{4}{x}$. + \item On note $S(x)$ l'aire totale de la citerne (c'est à dire la somme des aires des six faces). Montrer que l'on peut écrire + \[ + S(x) = 8x + 8 + \frac{32}{x} + \] + \item Démontrer que + \[ + S(x) = \frac{8x^2 + 8x + 32}{x} + \] + \item Démontrer que + \[ + S'(x) = \frac{8x^2 - 32}{x^2} + \] + \item En déduire le tableau de variation de $S(x)$ sur $\intOF{0}{10}$. + \item Déterminer les valeurs de $x$ et $h$ correspondant à une utilisation minimal de tôle. + \end{enumerate} +\end{exercise} + +\begin{solution} + \begin{enumerate} + \item Le volume étant fixe si l'on fait varier $x$, $h$ doit aussi varier. + \begin{itemize} + \item Si $x = 2$ alors conserver un volume de $V=16$, $h$ doit être égale à $4 / 2$ + \item Si $x = 3$ alors conserver un volume de $V=16$, $h$ doit être égale à $4 / 3$ + \end{itemize} + \item Pour calculer le volume, on a + \begin{eqnarray*} + V &=& h\times x \times 4 \\ + 16 &=& h\times x \times 4 \\ + x &=& \frac{16}{h\times 4} = \frac{4}{h} + \end{eqnarray*} + \item Pour calculer la surface totale, on ajoute la surface de chaque face. On a donc le calcul suivant + \begin{eqnarray*} + S(x) &=& x\times h \times 2 + x\times4\times2 + h\times 4\times 2\\ + S(x) &=& x\times \frac{4}{x} \times 2 + x\times4\times2 + \frac{4}{x}\times 4\times 2\\ + S(x) &=& 8x + 8 + \frac{32}{x} + \end{eqnarray*} + \item Pour trouver cette nouvelle forme, on met chaque élément sur le même dénominateur + \begin{eqnarray*} + S(x) &=& 8x + 8 + \frac{32}{x}\\ + S(x) &=& \frac{8x\times x}{x} + \frac{8\times x}{x} + \frac{32}{x}\\ + S(x) &=& \frac{8x^2 + 8x + 32}{x} + \end{eqnarray*} + \item On retrouve la formule $\frac{u}{v}$ à dériver + \[ + u(x) = 8x^2 + 8x + 32 \Rightarrow u'(x) = 16x + 8 + \] + \[ + v(x) = x \Rightarrow v'(x) = 1 + \] + Donc au numérateur on obtient + \begin{eqnarray*} + u'(x)\times v(x) - u(x)\times v'(x) &=& (16x + 8)\times x - (8x^2 + 8x + 32)\times 1\\ + &=& 8x^2 - 32 + \end{eqnarray*} + Donc + \[ + S'(x) = \frac{8x^2 - 32}{x^2} + \] + \item Tableau de variations de $S$ + + \begin{itemize} + \item Valeur interdite: $x^2 = 0 \equiv x = 0$ + \item Signe de $8x^2 - 32$: c'est un polynôme du 2e degré + \[ + \Delta = 1024 > 0 + \] + Il y a donc 2 racines + \[ + x_1 = - 2 \qquad + x_2 = 2 + \] + Et on sait que $8x^2 - 32$ est du signe de $a$ donc positif en dehors des racines + \item Le dénominateur $x^2$ est toujours positif. + \item Tableau de variations + + \begin{tikzpicture}[baseline=(a.north)] + \tkzTabInit[lgt=3,espcl=3]{$x$/1,$8x^2 - 32$/1, $x^2$/1, $S'$/1, $S$/2}{$0$, $- 2$, $10$} + \tkzTabLine{d,-, z, +, } + \tkzTabLine{d,+, , +, } + \tkzTabLine{d,-, z, +, } + \tkzTabVar{D+/ , -/ , +/ } + \end{tikzpicture} + + \end{itemize} + \item On a donc une surface minimal pour $x=2$ et $h = 8$. + \end{enumerate} +\end{solution} + +%%% Local Variables: +%%% mode: latex +%%% TeX-master: "master" +%%% End: + +\begin{exercise}[subtitle={Bassin}] + Le tour d'un bassin au niveau du sol présente deux axes de symétrie : l’axe des abscisses et la droite d’équation $x=4$. Il est obtenu par symétrie de la courbe $\mathcal{C}_f$ sur $\intFF{0}{4}$ où $f$ est la fonction définie par + + \[ + f(x) = \left(- x^{2} + 0.4 x - 6.0\right) e^{- x} + 6.0 + \] + On admet que sur $\intFF{0}{4}$ la fonction $f$ est positive. + \begin{enumerate} + \item Sur un repère, tracer l'allure de la courbe $\mathcal{C}_f$, les axes de symétries puis compléter pour dessiner la forme du bassin. + \item Montrer que la fonction $f$ admet comme primitive sur $\R$ la fonction $F$ définie par + \[ + F(x) = 6.0 x + \left( x^{2} + 1.6 x + 7.6\right) e^{- x} + \] + \item Calculer la quantité $\ds \int_0^4 f(x) \; dx$, vous donnerez le résultat sous forme exacte. Interpréter le résultat et reportez cette quantité sur le graphique. + \item On considère que l'échelle de votre graphique est de 1unité pour 15m. Calculer l'aire du bassin. Vous donnerez un résultat arrondi au $m^2$ près. + \end{enumerate} +\end{exercise} + +\begin{solution} + \begin{enumerate} + \item + \begin{tikzpicture}[baseline=(a.north), xscale=1, yscale=0.5] + \tkzInit[xmin=0,xmax=5,xstep=1, + ymin=0,ymax=10,ystep=1] + \tkzGrid + \tkzAxeXY + \tkzFct[domain=0:10,color=red,very thick]% + { (-x**2 + 0.4*x - 6.0)*exp(-x) + 6.0 }; + \end{tikzpicture} + \item Il faut dériver $F(x)$ et vérifier que $F'(x) = f(x)$. + \item $\ds \int_0^4 f(x) \; dx = F(4) - F(0) = \frac{30.0}{e^{4}} + 16.4$ + \item La quantité calculée à la question précédente se retrouve 4fois pour former le bassin. Il faut ensuite prendre en compte l'échelle, comme 1unité de longueur correspond à 15m, une unité d'air correspond à $15\times15 = 225m^2$. Ainsi l'aire du bassin est égale à + \[ + (\frac{30.0}{e^{4}} + 16.4)\times 4 \times 15^2 = 15255.00000 + \] + + \end{enumerate} +\end{solution} + +%%% Local Variables: +%%% mode: latex +%%% TeX-master: "master" +%%% End: + +\begin{exercise}[subtitle={Stylos}] + \emph{Les parties {\rm A} et {\rm B} de cet exercice sont indépendantes.} + + \bigskip + + \begin{minipage}{0.6\linewidth} + \textbf{Partie A} + + \medskip + + Deux ateliers A et B fabriquent des stylos pour une entreprise. + + L'atelier A fabrique 84.0\,\% des stylos, et parmi ceux-là, 85.0\,\% possèdent un défaut de fabrication. + + De plus, 4.0\,\% des stylos possèdent un défaut de fabrication et sortent de l'atelier B. + + Un stylo est prélevé au hasard dans le stock de l'entreprise. + + On considère les évènements suivants: + + \begin{itemize} + \item A : \og Le stylo a été fabriqué par l'atelier A \fg + \item B : \og Le stylo a été fabriqué par l'atelier B \fg + \item D : \og Le stylo possède un défaut de fabrication \fg + \end{itemize} + \end{minipage} + \begin{minipage}{0.4\linewidth} + \begin{center} + \begin{tikzpicture}[sloped] + \node {.} + child {node {$A$} + child {node {$D$} + edge from parent + node[above] {...} + } + child {node {$\overline{D}$} + edge from parent + node[above] {...} + } + edge from parent + node[above] {...} + } + child[missing] {} + child { node {$B$} + child {node {$D$} + edge from parent + node[above] {...} + } + child {node {$\overline{D}$} + edge from parent + node[above] {...} + } + edge from parent + node[above] {...} + } ; + \end{tikzpicture} + \end{center} + \end{minipage} + + \medskip + + \begin{enumerate} + \item Compléter l'arbre de probabilité ci-contre + \item Interpréter puis donner les probabilités $P(A)$, $P(B)$, $P_A(D)$ et $P(B \cap D)$. + + \item + \begin{enumerate} + \item Calculer la probabilité qu'un stylo provienne de l'atelier A et possède un défaut de fabrication. + \item En déduire que la probabilité qu'un stylo possède un défaut de fabrication est de $0.75$. + \end{enumerate} + \item On prélève un stylo au hasard avec un défaut. Quelle est la probabilité qu'il vienne de l'atelier A? + \end{enumerate} + + \bigskip + + \textbf{Partie B} + \medskip + + Dans cette partie, on suppose que 75.0\,\% des stylos possèdent un défaut de fabrication. + + L'entreprise confectionne des paquets contenant chacun $4$~stylos. + + Le fait qu'un stylo possède ou non un défaut de fabrication est indépendant des autres stylos. + + On appelle $X$ la variable aléatoire donnant pour un paquet le nombre de stylos qui possèdent un défaut de fabrication. + + On admet que la variable aléatoire $X$ suit une loi binomiale. + + \medskip + + \begin{enumerate} + \setcounter{enumi}{4} + \item Avec quelle loi peut-on modéliser $X$. Préciser les paramètres. + \item Calculer et interpréter la probabilité $P(X = 14)$. + \item Le directeur de l'entreprise affirme qu'il y a plus d'une chance sur deux qu'un paquet ne comporte aucun stylo défectueux. A-t-il raison ? + \item Combien de stylos peut-on espérer avoir en moyenne? + \end{enumerate} + \pagebreak +\end{exercise} + +\begin{solution} + \begin{enumerate} + \item + \begin{center} + \begin{tikzpicture}[sloped] + \node {.} + child {node {$A$} + child {node {$D$} + edge from parent + node[above] {0.85} + } + child {node {$\overline{D}$} + edge from parent + node[above] {0.15} + } + edge from parent + node[above] {0.84} + } + child[missing] {} + child { node {$B$} + child {node {$D$} + edge from parent + node[above] {0.26} + } + child {node {$\overline{D}$} + edge from parent + node[above] {0.74} + } + edge from parent + node[above] {0.16} + } ; + \end{tikzpicture} + \end{center} + \item + \begin{itemize} + \item Probabilité que le stylo vienne de l'atelier A + \[ + P(A) = 0.84 + \] + \item Probabilité que le stylo vienne de l'atelier B + \[ + P(B) = 0.16 + \] + \item Probabilité que le stylo ait un défaut sachant qu'il vient de l'atelier A. + \[ + P_A(D) = 0.85 + \] + \item Probabilité que le stylo vienne de l'atelier B et qu'il ait un défaut. + \[ + P(D \cap D) = 0.04 + \] + \end{itemize} + \item + \begin{enumerate} + \item Probabilité qu'un stylo vienne de l'atelier A et qu'il ait un defaut + \[ + P(A\cap D) = P(A) \times P_A(D) = 0.84 \times 0.85 = 0.71 + \] + \item Probabilité que le stylo ai un défaut de fabrication. + \[ + P(D) = P(A\cap D) + P(B\cap D) = 0.71 + 0.04 = 0.75 + \] + \end{enumerate} + \item Probabilité qu'il vienne de l'atelier A sachant qu'il a un defaut + \[ + P_D(A) = \frac{P(A\cap D)}{P(D)} = \frac{0.71}{0.75} = 0.95 + \] + \item $X$ peut être modélisée par une loi binomiale de paramètres $n=17$ et $p=0.75$. + \item (\textit{par de correction automatique disponible pour le résultat final} + \[ + P(X = 14) = \coefBino{17}{14}\times 0.75^{14} \times 0.25^{3} + \] + \item (\textit{par de correction automatique disponible pour le résultat final} + + Il faut calculer la probabilité qu'il y ait 0 stylo avec un defaut. + \[ + P(X = 0) = \coefBino{17}{0}\times 0.75^{0} \times 0.25^{17} + \] + Puis comparer ce nombre à 0,5. + \item Il faut calculer l'espérance + \[ + E[X] = n\times p = 17 \times 0.75 = 12.75 + \] + \end{enumerate} +\end{solution} + +\end{document} + +%%% Local Variables: +%%% mode: latex +%%% TeX-master: "master" +%%% End: diff --git a/Complementaire/DM/2105_DM1/corr_08_2105_DM1.tex b/Complementaire/DM/2105_DM1/corr_08_2105_DM1.tex new file mode 100644 index 0000000..6d3231c --- /dev/null +++ b/Complementaire/DM/2105_DM1/corr_08_2105_DM1.tex @@ -0,0 +1,367 @@ +\documentclass[a4paper,10pt]{article} +\usepackage{myXsim} + +% Title Page +\title{DM1 \hfill MATHIEU Allan} +\tribe{Maths complémentaire} +\date{\hfillÀ render pour le jeudi 27 mai} + +\xsimsetup{ + solution/print = true +} + +\begin{document} +\maketitle + +Les valeurs des exercices sont générés automatiquement. Si une valeur a un nombre adhérant de chiffres après la virgule, vous pouvez l'arrondir à l'entier le plus proche. + +\begin{exercise}[subtitle={Optimisation de matière}] + \begin{minipage}{0.6\textwidth} + On se propose de fabriquer avec le moins de tôle possible une citerne fermée en forme de parallélépipède rectangle dont le volume intérieur doit être de $40m^3$. La longueur est aussi fixée à $4m$ par le cahier des charges. + + On peut donc faire varier uniquement la largeur (notée $x$) et la hauteur (notée $h$) de la cuve. + \end{minipage} + \hfill + \begin{minipage}{0.3\textwidth} + \begin{tikzpicture} + \pgfmathsetmacro{\cubex}{3} + \pgfmathsetmacro{\cubey}{1} + \pgfmathsetmacro{\cubez}{2} + \draw[black,fill=gray] (0,0,0) -- ++(-\cubex,0,0) -- ++(0,-\cubey,0) node [midway, left] {$h$} -- ++(\cubex,0,0) node [midway, below] {$x$} -- cycle; + \draw[black,fill=gray] (0,0,0) -- ++(0,0,-\cubez) -- ++(0,-\cubey,0) -- ++(0,0,\cubez) node [midway, right] {$4m$} -- cycle; + \draw[black,fill=gray] (0,0,0) -- ++(-\cubex,0,0) -- ++(0,0,-\cubez) -- ++(\cubex,0,0) -- cycle; + \end{tikzpicture} + \end{minipage} + + \begin{enumerate} + \item Expliquer pourquoi quand la largeur $x$ change, la hauteur $h$ doit elle aussi changer pour respecter les contraintes. + \item Démontrer que l'on doit avoir $h = \dfrac{10}{x}$. + \item On note $S(x)$ l'aire totale de la citerne (c'est à dire la somme des aires des six faces). Montrer que l'on peut écrire + \[ + S(x) = 8x + 20 + \frac{80}{x} + \] + \item Démontrer que + \[ + S(x) = \frac{8x^2 + 20x + 80}{x} + \] + \item Démontrer que + \[ + S'(x) = \frac{8x^2 - 80}{x^2} + \] + \item En déduire le tableau de variation de $S(x)$ sur $\intOF{0}{10}$. + \item Déterminer les valeurs de $x$ et $h$ correspondant à une utilisation minimal de tôle. + \end{enumerate} +\end{exercise} + +\begin{solution} + \begin{enumerate} + \item Le volume étant fixe si l'on fait varier $x$, $h$ doit aussi varier. + \begin{itemize} + \item Si $x = 2$ alors conserver un volume de $V=40$, $h$ doit être égale à $10 / 2$ + \item Si $x = 3$ alors conserver un volume de $V=40$, $h$ doit être égale à $10 / 3$ + \end{itemize} + \item Pour calculer le volume, on a + \begin{eqnarray*} + V &=& h\times x \times 4 \\ + 40 &=& h\times x \times 4 \\ + x &=& \frac{40}{h\times 4} = \frac{10}{h} + \end{eqnarray*} + \item Pour calculer la surface totale, on ajoute la surface de chaque face. On a donc le calcul suivant + \begin{eqnarray*} + S(x) &=& x\times h \times 2 + x\times4\times2 + h\times 4\times 2\\ + S(x) &=& x\times \frac{10}{x} \times 2 + x\times4\times2 + \frac{10}{x}\times 4\times 2\\ + S(x) &=& 8x + 20 + \frac{80}{x} + \end{eqnarray*} + \item Pour trouver cette nouvelle forme, on met chaque élément sur le même dénominateur + \begin{eqnarray*} + S(x) &=& 8x + 20 + \frac{80}{x}\\ + S(x) &=& \frac{8x\times x}{x} + \frac{20\times x}{x} + \frac{80}{x}\\ + S(x) &=& \frac{8x^2 + 20x + 80}{x} + \end{eqnarray*} + \item On retrouve la formule $\frac{u}{v}$ à dériver + \[ + u(x) = 8x^2 + 20x + 80 \Rightarrow u'(x) = 16x + 20 + \] + \[ + v(x) = x \Rightarrow v'(x) = 1 + \] + Donc au numérateur on obtient + \begin{eqnarray*} + u'(x)\times v(x) - u(x)\times v'(x) &=& (16x + 20)\times x - (8x^2 + 20x + 80)\times 1\\ + &=& 8x^2 - 80 + \end{eqnarray*} + Donc + \[ + S'(x) = \frac{8x^2 - 80}{x^2} + \] + \item Tableau de variations de $S$ + + \begin{itemize} + \item Valeur interdite: $x^2 = 0 \equiv x = 0$ + \item Signe de $8x^2 - 80$: c'est un polynôme du 2e degré + \[ + \Delta = 2560 > 0 + \] + Il y a donc 2 racines + \[ + x_1 = - 3.1622776601683795 \qquad + x_2 = 3.1622776601683795 + \] + Et on sait que $8x^2 - 80$ est du signe de $a$ donc positif en dehors des racines + \item Le dénominateur $x^2$ est toujours positif. + \item Tableau de variations + + \begin{tikzpicture}[baseline=(a.north)] + \tkzTabInit[lgt=3,espcl=3]{$x$/1,$8x^2 - 80$/1, $x^2$/1, $S'$/1, $S$/2}{$0$, $- 3.1622776601683795$, $10$} + \tkzTabLine{d,-, z, +, } + \tkzTabLine{d,+, , +, } + \tkzTabLine{d,-, z, +, } + \tkzTabVar{D+/ , -/ , +/ } + \end{tikzpicture} + + \end{itemize} + \item On a donc une surface minimal pour $x=3.1622776601683795$ et $h = 31.6227766016837950$. + \end{enumerate} +\end{solution} + +%%% Local Variables: +%%% mode: latex +%%% TeX-master: "master" +%%% End: + +\begin{exercise}[subtitle={Bassin}] + Le tour d'un bassin au niveau du sol présente deux axes de symétrie : l’axe des abscisses et la droite d’équation $x=4$. Il est obtenu par symétrie de la courbe $\mathcal{C}_f$ sur $\intFF{0}{4}$ où $f$ est la fonction définie par + + \[ + f(x) = \left(- x^{2} + 6.1 x - 9.3\right) e^{- x} + 9.3 + \] + On admet que sur $\intFF{0}{4}$ la fonction $f$ est positive. + \begin{enumerate} + \item Sur un repère, tracer l'allure de la courbe $\mathcal{C}_f$, les axes de symétries puis compléter pour dessiner la forme du bassin. + \item Montrer que la fonction $f$ admet comme primitive sur $\R$ la fonction $F$ définie par + \[ + F(x) = 9.3 x + \left( x^{2} - 4.1 x + 5.2\right) e^{- x} + \] + \item Calculer la quantité $\ds \int_0^4 f(x) \; dx$, vous donnerez le résultat sous forme exacte. Interpréter le résultat et reportez cette quantité sur le graphique. + \item On considère que l'échelle de votre graphique est de 1unité pour 15m. Calculer l'aire du bassin. Vous donnerez un résultat arrondi au $m^2$ près. + \end{enumerate} +\end{exercise} + +\begin{solution} + \begin{enumerate} + \item + \begin{tikzpicture}[baseline=(a.north), xscale=1, yscale=0.5] + \tkzInit[xmin=0,xmax=5,xstep=1, + ymin=0,ymax=10,ystep=1] + \tkzGrid + \tkzAxeXY + \tkzFct[domain=0:10,color=red,very thick]% + { (-x**2 + 6.1*x - 9.3)*exp(-x) + 9.3 }; + \end{tikzpicture} + \item Il faut dériver $F(x)$ et vérifier que $F'(x) = f(x)$. + \item $\ds \int_0^4 f(x) \; dx = F(4) - F(0) = \frac{4.8}{e^{4}} + 32.0$ + \item La quantité calculée à la question précédente se retrouve 4fois pour former le bassin. Il faut ensuite prendre en compte l'échelle, comme 1unité de longueur correspond à 15m, une unité d'air correspond à $15\times15 = 225m^2$. Ainsi l'aire du bassin est égale à + \[ + (\frac{4.8}{e^{4}} + 32.0)\times 4 \times 15^2 = 28879.00000 + \] + + \end{enumerate} +\end{solution} + +%%% Local Variables: +%%% mode: latex +%%% TeX-master: "master" +%%% End: + +\begin{exercise}[subtitle={Stylos}] + \emph{Les parties {\rm A} et {\rm B} de cet exercice sont indépendantes.} + + \bigskip + + \begin{minipage}{0.6\linewidth} + \textbf{Partie A} + + \medskip + + Deux ateliers A et B fabriquent des stylos pour une entreprise. + + L'atelier A fabrique 92.0\,\% des stylos, et parmi ceux-là, 47.0\,\% possèdent un défaut de fabrication. + + De plus, 1.0\,\% des stylos possèdent un défaut de fabrication et sortent de l'atelier B. + + Un stylo est prélevé au hasard dans le stock de l'entreprise. + + On considère les évènements suivants: + + \begin{itemize} + \item A : \og Le stylo a été fabriqué par l'atelier A \fg + \item B : \og Le stylo a été fabriqué par l'atelier B \fg + \item D : \og Le stylo possède un défaut de fabrication \fg + \end{itemize} + \end{minipage} + \begin{minipage}{0.4\linewidth} + \begin{center} + \begin{tikzpicture}[sloped] + \node {.} + child {node {$A$} + child {node {$D$} + edge from parent + node[above] {...} + } + child {node {$\overline{D}$} + edge from parent + node[above] {...} + } + edge from parent + node[above] {...} + } + child[missing] {} + child { node {$B$} + child {node {$D$} + edge from parent + node[above] {...} + } + child {node {$\overline{D}$} + edge from parent + node[above] {...} + } + edge from parent + node[above] {...} + } ; + \end{tikzpicture} + \end{center} + \end{minipage} + + \medskip + + \begin{enumerate} + \item Compléter l'arbre de probabilité ci-contre + \item Interpréter puis donner les probabilités $P(A)$, $P(B)$, $P_A(D)$ et $P(B \cap D)$. + + \item + \begin{enumerate} + \item Calculer la probabilité qu'un stylo provienne de l'atelier A et possède un défaut de fabrication. + \item En déduire que la probabilité qu'un stylo possède un défaut de fabrication est de $0.44$. + \end{enumerate} + \item On prélève un stylo au hasard avec un défaut. Quelle est la probabilité qu'il vienne de l'atelier A? + \end{enumerate} + + \bigskip + + \textbf{Partie B} + \medskip + + Dans cette partie, on suppose que 44.0\,\% des stylos possèdent un défaut de fabrication. + + L'entreprise confectionne des paquets contenant chacun $4$~stylos. + + Le fait qu'un stylo possède ou non un défaut de fabrication est indépendant des autres stylos. + + On appelle $X$ la variable aléatoire donnant pour un paquet le nombre de stylos qui possèdent un défaut de fabrication. + + On admet que la variable aléatoire $X$ suit une loi binomiale. + + \medskip + + \begin{enumerate} + \setcounter{enumi}{4} + \item Avec quelle loi peut-on modéliser $X$. Préciser les paramètres. + \item Calculer et interpréter la probabilité $P(X = 15)$. + \item Le directeur de l'entreprise affirme qu'il y a plus d'une chance sur deux qu'un paquet ne comporte aucun stylo défectueux. A-t-il raison ? + \item Combien de stylos peut-on espérer avoir en moyenne? + \end{enumerate} + \pagebreak +\end{exercise} + +\begin{solution} + \begin{enumerate} + \item + \begin{center} + \begin{tikzpicture}[sloped] + \node {.} + child {node {$A$} + child {node {$D$} + edge from parent + node[above] {0.47} + } + child {node {$\overline{D}$} + edge from parent + node[above] {0.53} + } + edge from parent + node[above] {0.92} + } + child[missing] {} + child { node {$B$} + child {node {$D$} + edge from parent + node[above] {0.17} + } + child {node {$\overline{D}$} + edge from parent + node[above] {0.83} + } + edge from parent + node[above] {0.08} + } ; + \end{tikzpicture} + \end{center} + \item + \begin{itemize} + \item Probabilité que le stylo vienne de l'atelier A + \[ + P(A) = 0.92 + \] + \item Probabilité que le stylo vienne de l'atelier B + \[ + P(B) = 0.08 + \] + \item Probabilité que le stylo ait un défaut sachant qu'il vient de l'atelier A. + \[ + P_A(D) = 0.47 + \] + \item Probabilité que le stylo vienne de l'atelier B et qu'il ait un défaut. + \[ + P(D \cap D) = 0.01 + \] + \end{itemize} + \item + \begin{enumerate} + \item Probabilité qu'un stylo vienne de l'atelier A et qu'il ait un defaut + \[ + P(A\cap D) = P(A) \times P_A(D) = 0.92 \times 0.47 = 0.43 + \] + \item Probabilité que le stylo ai un défaut de fabrication. + \[ + P(D) = P(A\cap D) + P(B\cap D) = 0.43 + 0.01 = 0.44 + \] + \end{enumerate} + \item Probabilité qu'il vienne de l'atelier A sachant qu'il a un defaut + \[ + P_D(A) = \frac{P(A\cap D)}{P(D)} = \frac{0.43}{0.44} = 0.98 + \] + \item $X$ peut être modélisée par une loi binomiale de paramètres $n=17$ et $p=0.44$. + \item (\textit{par de correction automatique disponible pour le résultat final} + \[ + P(X = 15) = \coefBino{17}{15}\times 0.44^{15} \times 0.56^{2} + \] + \item (\textit{par de correction automatique disponible pour le résultat final} + + Il faut calculer la probabilité qu'il y ait 0 stylo avec un defaut. + \[ + P(X = 0) = \coefBino{17}{0}\times 0.44^{0} \times 0.56^{17} + \] + Puis comparer ce nombre à 0,5. + \item Il faut calculer l'espérance + \[ + E[X] = n\times p = 17 \times 0.44 = 7.48 + \] + \end{enumerate} +\end{solution} + +\end{document} + +%%% Local Variables: +%%% mode: latex +%%% TeX-master: "master" +%%% End: diff --git a/Complementaire/DM/2105_DM1/corr_09_2105_DM1.tex b/Complementaire/DM/2105_DM1/corr_09_2105_DM1.tex new file mode 100644 index 0000000..f2a9090 --- /dev/null +++ b/Complementaire/DM/2105_DM1/corr_09_2105_DM1.tex @@ -0,0 +1,367 @@ +\documentclass[a4paper,10pt]{article} +\usepackage{myXsim} + +% Title Page +\title{DM1 \hfill MOLINIER Annelise} +\tribe{Maths complémentaire} +\date{\hfillÀ render pour le jeudi 27 mai} + +\xsimsetup{ + solution/print = true +} + +\begin{document} +\maketitle + +Les valeurs des exercices sont générés automatiquement. Si une valeur a un nombre adhérant de chiffres après la virgule, vous pouvez l'arrondir à l'entier le plus proche. + +\begin{exercise}[subtitle={Optimisation de matière}] + \begin{minipage}{0.6\textwidth} + On se propose de fabriquer avec le moins de tôle possible une citerne fermée en forme de parallélépipède rectangle dont le volume intérieur doit être de $25m^3$. La longueur est aussi fixée à $5m$ par le cahier des charges. + + On peut donc faire varier uniquement la largeur (notée $x$) et la hauteur (notée $h$) de la cuve. + \end{minipage} + \hfill + \begin{minipage}{0.3\textwidth} + \begin{tikzpicture} + \pgfmathsetmacro{\cubex}{3} + \pgfmathsetmacro{\cubey}{1} + \pgfmathsetmacro{\cubez}{2} + \draw[black,fill=gray] (0,0,0) -- ++(-\cubex,0,0) -- ++(0,-\cubey,0) node [midway, left] {$h$} -- ++(\cubex,0,0) node [midway, below] {$x$} -- cycle; + \draw[black,fill=gray] (0,0,0) -- ++(0,0,-\cubez) -- ++(0,-\cubey,0) -- ++(0,0,\cubez) node [midway, right] {$5m$} -- cycle; + \draw[black,fill=gray] (0,0,0) -- ++(-\cubex,0,0) -- ++(0,0,-\cubez) -- ++(\cubex,0,0) -- cycle; + \end{tikzpicture} + \end{minipage} + + \begin{enumerate} + \item Expliquer pourquoi quand la largeur $x$ change, la hauteur $h$ doit elle aussi changer pour respecter les contraintes. + \item Démontrer que l'on doit avoir $h = \dfrac{5}{x}$. + \item On note $S(x)$ l'aire totale de la citerne (c'est à dire la somme des aires des six faces). Montrer que l'on peut écrire + \[ + S(x) = 10x + 10 + \frac{50}{x} + \] + \item Démontrer que + \[ + S(x) = \frac{10x^2 + 10x + 50}{x} + \] + \item Démontrer que + \[ + S'(x) = \frac{10x^2 - 50}{x^2} + \] + \item En déduire le tableau de variation de $S(x)$ sur $\intOF{0}{10}$. + \item Déterminer les valeurs de $x$ et $h$ correspondant à une utilisation minimal de tôle. + \end{enumerate} +\end{exercise} + +\begin{solution} + \begin{enumerate} + \item Le volume étant fixe si l'on fait varier $x$, $h$ doit aussi varier. + \begin{itemize} + \item Si $x = 2$ alors conserver un volume de $V=25$, $h$ doit être égale à $5 / 2$ + \item Si $x = 3$ alors conserver un volume de $V=25$, $h$ doit être égale à $5 / 3$ + \end{itemize} + \item Pour calculer le volume, on a + \begin{eqnarray*} + V &=& h\times x \times 5 \\ + 25 &=& h\times x \times 5 \\ + x &=& \frac{25}{h\times 5} = \frac{5}{h} + \end{eqnarray*} + \item Pour calculer la surface totale, on ajoute la surface de chaque face. On a donc le calcul suivant + \begin{eqnarray*} + S(x) &=& x\times h \times 2 + x\times5\times2 + h\times 5\times 2\\ + S(x) &=& x\times \frac{5}{x} \times 2 + x\times5\times2 + \frac{5}{x}\times 5\times 2\\ + S(x) &=& 10x + 10 + \frac{50}{x} + \end{eqnarray*} + \item Pour trouver cette nouvelle forme, on met chaque élément sur le même dénominateur + \begin{eqnarray*} + S(x) &=& 10x + 10 + \frac{50}{x}\\ + S(x) &=& \frac{10x\times x}{x} + \frac{10\times x}{x} + \frac{50}{x}\\ + S(x) &=& \frac{10x^2 + 10x + 50}{x} + \end{eqnarray*} + \item On retrouve la formule $\frac{u}{v}$ à dériver + \[ + u(x) = 10x^2 + 10x + 50 \Rightarrow u'(x) = 20x + 10 + \] + \[ + v(x) = x \Rightarrow v'(x) = 1 + \] + Donc au numérateur on obtient + \begin{eqnarray*} + u'(x)\times v(x) - u(x)\times v'(x) &=& (20x + 10)\times x - (10x^2 + 10x + 50)\times 1\\ + &=& 10x^2 - 50 + \end{eqnarray*} + Donc + \[ + S'(x) = \frac{10x^2 - 50}{x^2} + \] + \item Tableau de variations de $S$ + + \begin{itemize} + \item Valeur interdite: $x^2 = 0 \equiv x = 0$ + \item Signe de $10x^2 - 50$: c'est un polynôme du 2e degré + \[ + \Delta = 2000 > 0 + \] + Il y a donc 2 racines + \[ + x_1 = - 2.23606797749979 \qquad + x_2 = 2.23606797749979 + \] + Et on sait que $10x^2 - 50$ est du signe de $a$ donc positif en dehors des racines + \item Le dénominateur $x^2$ est toujours positif. + \item Tableau de variations + + \begin{tikzpicture}[baseline=(a.north)] + \tkzTabInit[lgt=3,espcl=3]{$x$/1,$10x^2 - 50$/1, $x^2$/1, $S'$/1, $S$/2}{$0$, $- 2.23606797749979$, $10$} + \tkzTabLine{d,-, z, +, } + \tkzTabLine{d,+, , +, } + \tkzTabLine{d,-, z, +, } + \tkzTabVar{D+/ , -/ , +/ } + \end{tikzpicture} + + \end{itemize} + \item On a donc une surface minimal pour $x=2.23606797749979$ et $h = 11.18033988749895$. + \end{enumerate} +\end{solution} + +%%% Local Variables: +%%% mode: latex +%%% TeX-master: "master" +%%% End: + +\begin{exercise}[subtitle={Bassin}] + Le tour d'un bassin au niveau du sol présente deux axes de symétrie : l’axe des abscisses et la droite d’équation $x=4$. Il est obtenu par symétrie de la courbe $\mathcal{C}_f$ sur $\intFF{0}{4}$ où $f$ est la fonction définie par + + \[ + f(x) = \left(- x^{2} + 6.9 x - 3.3\right) e^{- x} + 3.3 + \] + On admet que sur $\intFF{0}{4}$ la fonction $f$ est positive. + \begin{enumerate} + \item Sur un repère, tracer l'allure de la courbe $\mathcal{C}_f$, les axes de symétries puis compléter pour dessiner la forme du bassin. + \item Montrer que la fonction $f$ admet comme primitive sur $\R$ la fonction $F$ définie par + \[ + F(x) = 3.3 x + \left( x^{2} - 4.9 x - 1.6\right) e^{- x} + \] + \item Calculer la quantité $\ds \int_0^4 f(x) \; dx$, vous donnerez le résultat sous forme exacte. Interpréter le résultat et reportez cette quantité sur le graphique. + \item On considère que l'échelle de votre graphique est de 1unité pour 15m. Calculer l'aire du bassin. Vous donnerez un résultat arrondi au $m^2$ près. + \end{enumerate} +\end{exercise} + +\begin{solution} + \begin{enumerate} + \item + \begin{tikzpicture}[baseline=(a.north), xscale=1, yscale=0.5] + \tkzInit[xmin=0,xmax=5,xstep=1, + ymin=0,ymax=10,ystep=1] + \tkzGrid + \tkzAxeXY + \tkzFct[domain=0:10,color=red,very thick]% + { (-x**2 + 6.9*x - 3.3)*exp(-x) + 3.3 }; + \end{tikzpicture} + \item Il faut dériver $F(x)$ et vérifier que $F'(x) = f(x)$. + \item $\ds \int_0^4 f(x) \; dx = F(4) - F(0) = 14.8 - \frac{5.2}{e^{4}}$ + \item La quantité calculée à la question précédente se retrouve 4fois pour former le bassin. Il faut ensuite prendre en compte l'échelle, comme 1unité de longueur correspond à 15m, une unité d'air correspond à $15\times15 = 225m^2$. Ainsi l'aire du bassin est égale à + \[ + (14.8 - \frac{5.2}{e^{4}})\times 4 \times 15^2 = 13234.00000 + \] + + \end{enumerate} +\end{solution} + +%%% Local Variables: +%%% mode: latex +%%% TeX-master: "master" +%%% End: + +\begin{exercise}[subtitle={Stylos}] + \emph{Les parties {\rm A} et {\rm B} de cet exercice sont indépendantes.} + + \bigskip + + \begin{minipage}{0.6\linewidth} + \textbf{Partie A} + + \medskip + + Deux ateliers A et B fabriquent des stylos pour une entreprise. + + L'atelier A fabrique 56.00000000000001\,\% des stylos, et parmi ceux-là, 22.0\,\% possèdent un défaut de fabrication. + + De plus, 10.0\,\% des stylos possèdent un défaut de fabrication et sortent de l'atelier B. + + Un stylo est prélevé au hasard dans le stock de l'entreprise. + + On considère les évènements suivants: + + \begin{itemize} + \item A : \og Le stylo a été fabriqué par l'atelier A \fg + \item B : \og Le stylo a été fabriqué par l'atelier B \fg + \item D : \og Le stylo possède un défaut de fabrication \fg + \end{itemize} + \end{minipage} + \begin{minipage}{0.4\linewidth} + \begin{center} + \begin{tikzpicture}[sloped] + \node {.} + child {node {$A$} + child {node {$D$} + edge from parent + node[above] {...} + } + child {node {$\overline{D}$} + edge from parent + node[above] {...} + } + edge from parent + node[above] {...} + } + child[missing] {} + child { node {$B$} + child {node {$D$} + edge from parent + node[above] {...} + } + child {node {$\overline{D}$} + edge from parent + node[above] {...} + } + edge from parent + node[above] {...} + } ; + \end{tikzpicture} + \end{center} + \end{minipage} + + \medskip + + \begin{enumerate} + \item Compléter l'arbre de probabilité ci-contre + \item Interpréter puis donner les probabilités $P(A)$, $P(B)$, $P_A(D)$ et $P(B \cap D)$. + + \item + \begin{enumerate} + \item Calculer la probabilité qu'un stylo provienne de l'atelier A et possède un défaut de fabrication. + \item En déduire que la probabilité qu'un stylo possède un défaut de fabrication est de $0.22$. + \end{enumerate} + \item On prélève un stylo au hasard avec un défaut. Quelle est la probabilité qu'il vienne de l'atelier A? + \end{enumerate} + + \bigskip + + \textbf{Partie B} + \medskip + + Dans cette partie, on suppose que 22.0\,\% des stylos possèdent un défaut de fabrication. + + L'entreprise confectionne des paquets contenant chacun $4$~stylos. + + Le fait qu'un stylo possède ou non un défaut de fabrication est indépendant des autres stylos. + + On appelle $X$ la variable aléatoire donnant pour un paquet le nombre de stylos qui possèdent un défaut de fabrication. + + On admet que la variable aléatoire $X$ suit une loi binomiale. + + \medskip + + \begin{enumerate} + \setcounter{enumi}{4} + \item Avec quelle loi peut-on modéliser $X$. Préciser les paramètres. + \item Calculer et interpréter la probabilité $P(X = 19)$. + \item Le directeur de l'entreprise affirme qu'il y a plus d'une chance sur deux qu'un paquet ne comporte aucun stylo défectueux. A-t-il raison ? + \item Combien de stylos peut-on espérer avoir en moyenne? + \end{enumerate} + \pagebreak +\end{exercise} + +\begin{solution} + \begin{enumerate} + \item + \begin{center} + \begin{tikzpicture}[sloped] + \node {.} + child {node {$A$} + child {node {$D$} + edge from parent + node[above] {0.22} + } + child {node {$\overline{D}$} + edge from parent + node[above] {0.78} + } + edge from parent + node[above] {0.56} + } + child[missing] {} + child { node {$B$} + child {node {$D$} + edge from parent + node[above] {0.23} + } + child {node {$\overline{D}$} + edge from parent + node[above] {0.77} + } + edge from parent + node[above] {0.44} + } ; + \end{tikzpicture} + \end{center} + \item + \begin{itemize} + \item Probabilité que le stylo vienne de l'atelier A + \[ + P(A) = 0.56 + \] + \item Probabilité que le stylo vienne de l'atelier B + \[ + P(B) = 0.44 + \] + \item Probabilité que le stylo ait un défaut sachant qu'il vient de l'atelier A. + \[ + P_A(D) = 0.22 + \] + \item Probabilité que le stylo vienne de l'atelier B et qu'il ait un défaut. + \[ + P(D \cap D) = 0.1 + \] + \end{itemize} + \item + \begin{enumerate} + \item Probabilité qu'un stylo vienne de l'atelier A et qu'il ait un defaut + \[ + P(A\cap D) = P(A) \times P_A(D) = 0.56 \times 0.22 = 0.12 + \] + \item Probabilité que le stylo ai un défaut de fabrication. + \[ + P(D) = P(A\cap D) + P(B\cap D) = 0.12 + 0.1 = 0.22 + \] + \end{enumerate} + \item Probabilité qu'il vienne de l'atelier A sachant qu'il a un defaut + \[ + P_D(A) = \frac{P(A\cap D)}{P(D)} = \frac{0.12}{0.22} = 0.55 + \] + \item $X$ peut être modélisée par une loi binomiale de paramètres $n=19$ et $p=0.22$. + \item (\textit{par de correction automatique disponible pour le résultat final} + \[ + P(X = 19) = \coefBino{19}{19}\times 0.22^{19} \times 0.78^{0} + \] + \item (\textit{par de correction automatique disponible pour le résultat final} + + Il faut calculer la probabilité qu'il y ait 0 stylo avec un defaut. + \[ + P(X = 0) = \coefBino{19}{0}\times 0.22^{0} \times 0.78^{19} + \] + Puis comparer ce nombre à 0,5. + \item Il faut calculer l'espérance + \[ + E[X] = n\times p = 19 \times 0.22 = 4.18 + \] + \end{enumerate} +\end{solution} + +\end{document} + +%%% Local Variables: +%%% mode: latex +%%% TeX-master: "master" +%%% End: diff --git a/Complementaire/DM/2105_DM1/corr_10_2105_DM1.tex b/Complementaire/DM/2105_DM1/corr_10_2105_DM1.tex new file mode 100644 index 0000000..4cead8f --- /dev/null +++ b/Complementaire/DM/2105_DM1/corr_10_2105_DM1.tex @@ -0,0 +1,367 @@ +\documentclass[a4paper,10pt]{article} +\usepackage{myXsim} + +% Title Page +\title{DM1 \hfill MOUHOUBI Maïssa} +\tribe{Maths complémentaire} +\date{\hfillÀ render pour le jeudi 27 mai} + +\xsimsetup{ + solution/print = true +} + +\begin{document} +\maketitle + +Les valeurs des exercices sont générés automatiquement. Si une valeur a un nombre adhérant de chiffres après la virgule, vous pouvez l'arrondir à l'entier le plus proche. + +\begin{exercise}[subtitle={Optimisation de matière}] + \begin{minipage}{0.6\textwidth} + On se propose de fabriquer avec le moins de tôle possible une citerne fermée en forme de parallélépipède rectangle dont le volume intérieur doit être de $35m^3$. La longueur est aussi fixée à $5m$ par le cahier des charges. + + On peut donc faire varier uniquement la largeur (notée $x$) et la hauteur (notée $h$) de la cuve. + \end{minipage} + \hfill + \begin{minipage}{0.3\textwidth} + \begin{tikzpicture} + \pgfmathsetmacro{\cubex}{3} + \pgfmathsetmacro{\cubey}{1} + \pgfmathsetmacro{\cubez}{2} + \draw[black,fill=gray] (0,0,0) -- ++(-\cubex,0,0) -- ++(0,-\cubey,0) node [midway, left] {$h$} -- ++(\cubex,0,0) node [midway, below] {$x$} -- cycle; + \draw[black,fill=gray] (0,0,0) -- ++(0,0,-\cubez) -- ++(0,-\cubey,0) -- ++(0,0,\cubez) node [midway, right] {$5m$} -- cycle; + \draw[black,fill=gray] (0,0,0) -- ++(-\cubex,0,0) -- ++(0,0,-\cubez) -- ++(\cubex,0,0) -- cycle; + \end{tikzpicture} + \end{minipage} + + \begin{enumerate} + \item Expliquer pourquoi quand la largeur $x$ change, la hauteur $h$ doit elle aussi changer pour respecter les contraintes. + \item Démontrer que l'on doit avoir $h = \dfrac{7}{x}$. + \item On note $S(x)$ l'aire totale de la citerne (c'est à dire la somme des aires des six faces). Montrer que l'on peut écrire + \[ + S(x) = 10x + 14 + \frac{70}{x} + \] + \item Démontrer que + \[ + S(x) = \frac{10x^2 + 14x + 70}{x} + \] + \item Démontrer que + \[ + S'(x) = \frac{10x^2 - 70}{x^2} + \] + \item En déduire le tableau de variation de $S(x)$ sur $\intOF{0}{10}$. + \item Déterminer les valeurs de $x$ et $h$ correspondant à une utilisation minimal de tôle. + \end{enumerate} +\end{exercise} + +\begin{solution} + \begin{enumerate} + \item Le volume étant fixe si l'on fait varier $x$, $h$ doit aussi varier. + \begin{itemize} + \item Si $x = 2$ alors conserver un volume de $V=35$, $h$ doit être égale à $7 / 2$ + \item Si $x = 3$ alors conserver un volume de $V=35$, $h$ doit être égale à $7 / 3$ + \end{itemize} + \item Pour calculer le volume, on a + \begin{eqnarray*} + V &=& h\times x \times 5 \\ + 35 &=& h\times x \times 5 \\ + x &=& \frac{35}{h\times 5} = \frac{7}{h} + \end{eqnarray*} + \item Pour calculer la surface totale, on ajoute la surface de chaque face. On a donc le calcul suivant + \begin{eqnarray*} + S(x) &=& x\times h \times 2 + x\times5\times2 + h\times 5\times 2\\ + S(x) &=& x\times \frac{7}{x} \times 2 + x\times5\times2 + \frac{7}{x}\times 5\times 2\\ + S(x) &=& 10x + 14 + \frac{70}{x} + \end{eqnarray*} + \item Pour trouver cette nouvelle forme, on met chaque élément sur le même dénominateur + \begin{eqnarray*} + S(x) &=& 10x + 14 + \frac{70}{x}\\ + S(x) &=& \frac{10x\times x}{x} + \frac{14\times x}{x} + \frac{70}{x}\\ + S(x) &=& \frac{10x^2 + 14x + 70}{x} + \end{eqnarray*} + \item On retrouve la formule $\frac{u}{v}$ à dériver + \[ + u(x) = 10x^2 + 14x + 70 \Rightarrow u'(x) = 20x + 14 + \] + \[ + v(x) = x \Rightarrow v'(x) = 1 + \] + Donc au numérateur on obtient + \begin{eqnarray*} + u'(x)\times v(x) - u(x)\times v'(x) &=& (20x + 14)\times x - (10x^2 + 14x + 70)\times 1\\ + &=& 10x^2 - 70 + \end{eqnarray*} + Donc + \[ + S'(x) = \frac{10x^2 - 70}{x^2} + \] + \item Tableau de variations de $S$ + + \begin{itemize} + \item Valeur interdite: $x^2 = 0 \equiv x = 0$ + \item Signe de $10x^2 - 70$: c'est un polynôme du 2e degré + \[ + \Delta = 2800 > 0 + \] + Il y a donc 2 racines + \[ + x_1 = - 2.6457513110645907 \qquad + x_2 = 2.6457513110645907 + \] + Et on sait que $10x^2 - 70$ est du signe de $a$ donc positif en dehors des racines + \item Le dénominateur $x^2$ est toujours positif. + \item Tableau de variations + + \begin{tikzpicture}[baseline=(a.north)] + \tkzTabInit[lgt=3,espcl=3]{$x$/1,$10x^2 - 70$/1, $x^2$/1, $S'$/1, $S$/2}{$0$, $- 2.6457513110645907$, $10$} + \tkzTabLine{d,-, z, +, } + \tkzTabLine{d,+, , +, } + \tkzTabLine{d,-, z, +, } + \tkzTabVar{D+/ , -/ , +/ } + \end{tikzpicture} + + \end{itemize} + \item On a donc une surface minimal pour $x=2.6457513110645907$ et $h = 18.5202591774521349$. + \end{enumerate} +\end{solution} + +%%% Local Variables: +%%% mode: latex +%%% TeX-master: "master" +%%% End: + +\begin{exercise}[subtitle={Bassin}] + Le tour d'un bassin au niveau du sol présente deux axes de symétrie : l’axe des abscisses et la droite d’équation $x=4$. Il est obtenu par symétrie de la courbe $\mathcal{C}_f$ sur $\intFF{0}{4}$ où $f$ est la fonction définie par + + \[ + f(x) = \left(- x^{2} + 1.1 x - 6.1\right) e^{- x} + 6.1 + \] + On admet que sur $\intFF{0}{4}$ la fonction $f$ est positive. + \begin{enumerate} + \item Sur un repère, tracer l'allure de la courbe $\mathcal{C}_f$, les axes de symétries puis compléter pour dessiner la forme du bassin. + \item Montrer que la fonction $f$ admet comme primitive sur $\R$ la fonction $F$ définie par + \[ + F(x) = 6.1 x + \left( x^{2} + 0.9 x + 7.0\right) e^{- x} + \] + \item Calculer la quantité $\ds \int_0^4 f(x) \; dx$, vous donnerez le résultat sous forme exacte. Interpréter le résultat et reportez cette quantité sur le graphique. + \item On considère que l'échelle de votre graphique est de 1unité pour 15m. Calculer l'aire du bassin. Vous donnerez un résultat arrondi au $m^2$ près. + \end{enumerate} +\end{exercise} + +\begin{solution} + \begin{enumerate} + \item + \begin{tikzpicture}[baseline=(a.north), xscale=1, yscale=0.5] + \tkzInit[xmin=0,xmax=5,xstep=1, + ymin=0,ymax=10,ystep=1] + \tkzGrid + \tkzAxeXY + \tkzFct[domain=0:10,color=red,very thick]% + { (-x**2 + 1.1*x - 6.1)*exp(-x) + 6.1 }; + \end{tikzpicture} + \item Il faut dériver $F(x)$ et vérifier que $F'(x) = f(x)$. + \item $\ds \int_0^4 f(x) \; dx = F(4) - F(0) = \frac{26.6}{e^{4}} + 17.4$ + \item La quantité calculée à la question précédente se retrouve 4fois pour former le bassin. Il faut ensuite prendre en compte l'échelle, comme 1unité de longueur correspond à 15m, une unité d'air correspond à $15\times15 = 225m^2$. Ainsi l'aire du bassin est égale à + \[ + (\frac{26.6}{e^{4}} + 17.4)\times 4 \times 15^2 = 16098.00000 + \] + + \end{enumerate} +\end{solution} + +%%% Local Variables: +%%% mode: latex +%%% TeX-master: "master" +%%% End: + +\begin{exercise}[subtitle={Stylos}] + \emph{Les parties {\rm A} et {\rm B} de cet exercice sont indépendantes.} + + \bigskip + + \begin{minipage}{0.6\linewidth} + \textbf{Partie A} + + \medskip + + Deux ateliers A et B fabriquent des stylos pour une entreprise. + + L'atelier A fabrique 23.0\,\% des stylos, et parmi ceux-là, 66.0\,\% possèdent un défaut de fabrication. + + De plus, 68.0\,\% des stylos possèdent un défaut de fabrication et sortent de l'atelier B. + + Un stylo est prélevé au hasard dans le stock de l'entreprise. + + On considère les évènements suivants: + + \begin{itemize} + \item A : \og Le stylo a été fabriqué par l'atelier A \fg + \item B : \og Le stylo a été fabriqué par l'atelier B \fg + \item D : \og Le stylo possède un défaut de fabrication \fg + \end{itemize} + \end{minipage} + \begin{minipage}{0.4\linewidth} + \begin{center} + \begin{tikzpicture}[sloped] + \node {.} + child {node {$A$} + child {node {$D$} + edge from parent + node[above] {...} + } + child {node {$\overline{D}$} + edge from parent + node[above] {...} + } + edge from parent + node[above] {...} + } + child[missing] {} + child { node {$B$} + child {node {$D$} + edge from parent + node[above] {...} + } + child {node {$\overline{D}$} + edge from parent + node[above] {...} + } + edge from parent + node[above] {...} + } ; + \end{tikzpicture} + \end{center} + \end{minipage} + + \medskip + + \begin{enumerate} + \item Compléter l'arbre de probabilité ci-contre + \item Interpréter puis donner les probabilités $P(A)$, $P(B)$, $P_A(D)$ et $P(B \cap D)$. + + \item + \begin{enumerate} + \item Calculer la probabilité qu'un stylo provienne de l'atelier A et possède un défaut de fabrication. + \item En déduire que la probabilité qu'un stylo possède un défaut de fabrication est de $0.83$. + \end{enumerate} + \item On prélève un stylo au hasard avec un défaut. Quelle est la probabilité qu'il vienne de l'atelier A? + \end{enumerate} + + \bigskip + + \textbf{Partie B} + \medskip + + Dans cette partie, on suppose que 83.0\,\% des stylos possèdent un défaut de fabrication. + + L'entreprise confectionne des paquets contenant chacun $4$~stylos. + + Le fait qu'un stylo possède ou non un défaut de fabrication est indépendant des autres stylos. + + On appelle $X$ la variable aléatoire donnant pour un paquet le nombre de stylos qui possèdent un défaut de fabrication. + + On admet que la variable aléatoire $X$ suit une loi binomiale. + + \medskip + + \begin{enumerate} + \setcounter{enumi}{4} + \item Avec quelle loi peut-on modéliser $X$. Préciser les paramètres. + \item Calculer et interpréter la probabilité $P(X = 10)$. + \item Le directeur de l'entreprise affirme qu'il y a plus d'une chance sur deux qu'un paquet ne comporte aucun stylo défectueux. A-t-il raison ? + \item Combien de stylos peut-on espérer avoir en moyenne? + \end{enumerate} + \pagebreak +\end{exercise} + +\begin{solution} + \begin{enumerate} + \item + \begin{center} + \begin{tikzpicture}[sloped] + \node {.} + child {node {$A$} + child {node {$D$} + edge from parent + node[above] {0.66} + } + child {node {$\overline{D}$} + edge from parent + node[above] {0.34} + } + edge from parent + node[above] {0.23} + } + child[missing] {} + child { node {$B$} + child {node {$D$} + edge from parent + node[above] {0.88} + } + child {node {$\overline{D}$} + edge from parent + node[above] {0.12} + } + edge from parent + node[above] {0.77} + } ; + \end{tikzpicture} + \end{center} + \item + \begin{itemize} + \item Probabilité que le stylo vienne de l'atelier A + \[ + P(A) = 0.23 + \] + \item Probabilité que le stylo vienne de l'atelier B + \[ + P(B) = 0.77 + \] + \item Probabilité que le stylo ait un défaut sachant qu'il vient de l'atelier A. + \[ + P_A(D) = 0.66 + \] + \item Probabilité que le stylo vienne de l'atelier B et qu'il ait un défaut. + \[ + P(D \cap D) = 0.68 + \] + \end{itemize} + \item + \begin{enumerate} + \item Probabilité qu'un stylo vienne de l'atelier A et qu'il ait un defaut + \[ + P(A\cap D) = P(A) \times P_A(D) = 0.23 \times 0.66 = 0.15 + \] + \item Probabilité que le stylo ai un défaut de fabrication. + \[ + P(D) = P(A\cap D) + P(B\cap D) = 0.15 + 0.68 = 0.83 + \] + \end{enumerate} + \item Probabilité qu'il vienne de l'atelier A sachant qu'il a un defaut + \[ + P_D(A) = \frac{P(A\cap D)}{P(D)} = \frac{0.15}{0.83} = 0.18 + \] + \item $X$ peut être modélisée par une loi binomiale de paramètres $n=17$ et $p=0.83$. + \item (\textit{par de correction automatique disponible pour le résultat final} + \[ + P(X = 10) = \coefBino{17}{10}\times 0.83^{10} \times 0.17^{7} + \] + \item (\textit{par de correction automatique disponible pour le résultat final} + + Il faut calculer la probabilité qu'il y ait 0 stylo avec un defaut. + \[ + P(X = 0) = \coefBino{17}{0}\times 0.83^{0} \times 0.17^{17} + \] + Puis comparer ce nombre à 0,5. + \item Il faut calculer l'espérance + \[ + E[X] = n\times p = 17 \times 0.83 = 14.11 + \] + \end{enumerate} +\end{solution} + +\end{document} + +%%% Local Variables: +%%% mode: latex +%%% TeX-master: "master" +%%% End: diff --git a/Complementaire/DM/2105_DM1/corr_11_2105_DM1.tex b/Complementaire/DM/2105_DM1/corr_11_2105_DM1.tex new file mode 100644 index 0000000..76ad41c --- /dev/null +++ b/Complementaire/DM/2105_DM1/corr_11_2105_DM1.tex @@ -0,0 +1,367 @@ +\documentclass[a4paper,10pt]{article} +\usepackage{myXsim} + +% Title Page +\title{DM1 \hfill PERDRIX Camille} +\tribe{Maths complémentaire} +\date{\hfillÀ render pour le jeudi 27 mai} + +\xsimsetup{ + solution/print = true +} + +\begin{document} +\maketitle + +Les valeurs des exercices sont générés automatiquement. Si une valeur a un nombre adhérant de chiffres après la virgule, vous pouvez l'arrondir à l'entier le plus proche. + +\begin{exercise}[subtitle={Optimisation de matière}] + \begin{minipage}{0.6\textwidth} + On se propose de fabriquer avec le moins de tôle possible une citerne fermée en forme de parallélépipède rectangle dont le volume intérieur doit être de $8m^3$. La longueur est aussi fixée à $4m$ par le cahier des charges. + + On peut donc faire varier uniquement la largeur (notée $x$) et la hauteur (notée $h$) de la cuve. + \end{minipage} + \hfill + \begin{minipage}{0.3\textwidth} + \begin{tikzpicture} + \pgfmathsetmacro{\cubex}{3} + \pgfmathsetmacro{\cubey}{1} + \pgfmathsetmacro{\cubez}{2} + \draw[black,fill=gray] (0,0,0) -- ++(-\cubex,0,0) -- ++(0,-\cubey,0) node [midway, left] {$h$} -- ++(\cubex,0,0) node [midway, below] {$x$} -- cycle; + \draw[black,fill=gray] (0,0,0) -- ++(0,0,-\cubez) -- ++(0,-\cubey,0) -- ++(0,0,\cubez) node [midway, right] {$4m$} -- cycle; + \draw[black,fill=gray] (0,0,0) -- ++(-\cubex,0,0) -- ++(0,0,-\cubez) -- ++(\cubex,0,0) -- cycle; + \end{tikzpicture} + \end{minipage} + + \begin{enumerate} + \item Expliquer pourquoi quand la largeur $x$ change, la hauteur $h$ doit elle aussi changer pour respecter les contraintes. + \item Démontrer que l'on doit avoir $h = \dfrac{2}{x}$. + \item On note $S(x)$ l'aire totale de la citerne (c'est à dire la somme des aires des six faces). Montrer que l'on peut écrire + \[ + S(x) = 8x + 4 + \frac{16}{x} + \] + \item Démontrer que + \[ + S(x) = \frac{8x^2 + 4x + 16}{x} + \] + \item Démontrer que + \[ + S'(x) = \frac{8x^2 - 16}{x^2} + \] + \item En déduire le tableau de variation de $S(x)$ sur $\intOF{0}{10}$. + \item Déterminer les valeurs de $x$ et $h$ correspondant à une utilisation minimal de tôle. + \end{enumerate} +\end{exercise} + +\begin{solution} + \begin{enumerate} + \item Le volume étant fixe si l'on fait varier $x$, $h$ doit aussi varier. + \begin{itemize} + \item Si $x = 2$ alors conserver un volume de $V=8$, $h$ doit être égale à $2 / 2$ + \item Si $x = 3$ alors conserver un volume de $V=8$, $h$ doit être égale à $2 / 3$ + \end{itemize} + \item Pour calculer le volume, on a + \begin{eqnarray*} + V &=& h\times x \times 4 \\ + 8 &=& h\times x \times 4 \\ + x &=& \frac{8}{h\times 4} = \frac{2}{h} + \end{eqnarray*} + \item Pour calculer la surface totale, on ajoute la surface de chaque face. On a donc le calcul suivant + \begin{eqnarray*} + S(x) &=& x\times h \times 2 + x\times4\times2 + h\times 4\times 2\\ + S(x) &=& x\times \frac{2}{x} \times 2 + x\times4\times2 + \frac{2}{x}\times 4\times 2\\ + S(x) &=& 8x + 4 + \frac{16}{x} + \end{eqnarray*} + \item Pour trouver cette nouvelle forme, on met chaque élément sur le même dénominateur + \begin{eqnarray*} + S(x) &=& 8x + 4 + \frac{16}{x}\\ + S(x) &=& \frac{8x\times x}{x} + \frac{4\times x}{x} + \frac{16}{x}\\ + S(x) &=& \frac{8x^2 + 4x + 16}{x} + \end{eqnarray*} + \item On retrouve la formule $\frac{u}{v}$ à dériver + \[ + u(x) = 8x^2 + 4x + 16 \Rightarrow u'(x) = 16x + 4 + \] + \[ + v(x) = x \Rightarrow v'(x) = 1 + \] + Donc au numérateur on obtient + \begin{eqnarray*} + u'(x)\times v(x) - u(x)\times v'(x) &=& (16x + 4)\times x - (8x^2 + 4x + 16)\times 1\\ + &=& 8x^2 - 16 + \end{eqnarray*} + Donc + \[ + S'(x) = \frac{8x^2 - 16}{x^2} + \] + \item Tableau de variations de $S$ + + \begin{itemize} + \item Valeur interdite: $x^2 = 0 \equiv x = 0$ + \item Signe de $8x^2 - 16$: c'est un polynôme du 2e degré + \[ + \Delta = 512 > 0 + \] + Il y a donc 2 racines + \[ + x_1 = - 1.4142135623730951 \qquad + x_2 = 1.4142135623730951 + \] + Et on sait que $8x^2 - 16$ est du signe de $a$ donc positif en dehors des racines + \item Le dénominateur $x^2$ est toujours positif. + \item Tableau de variations + + \begin{tikzpicture}[baseline=(a.north)] + \tkzTabInit[lgt=3,espcl=3]{$x$/1,$8x^2 - 16$/1, $x^2$/1, $S'$/1, $S$/2}{$0$, $- 1.4142135623730951$, $10$} + \tkzTabLine{d,-, z, +, } + \tkzTabLine{d,+, , +, } + \tkzTabLine{d,-, z, +, } + \tkzTabVar{D+/ , -/ , +/ } + \end{tikzpicture} + + \end{itemize} + \item On a donc une surface minimal pour $x=1.4142135623730951$ et $h = 2.8284271247461902$. + \end{enumerate} +\end{solution} + +%%% Local Variables: +%%% mode: latex +%%% TeX-master: "master" +%%% End: + +\begin{exercise}[subtitle={Bassin}] + Le tour d'un bassin au niveau du sol présente deux axes de symétrie : l’axe des abscisses et la droite d’équation $x=4$. Il est obtenu par symétrie de la courbe $\mathcal{C}_f$ sur $\intFF{0}{4}$ où $f$ est la fonction définie par + + \[ + f(x) = \left(- x^{2} + 8.3 x - 5.0\right) e^{- x} + 5.0 + \] + On admet que sur $\intFF{0}{4}$ la fonction $f$ est positive. + \begin{enumerate} + \item Sur un repère, tracer l'allure de la courbe $\mathcal{C}_f$, les axes de symétries puis compléter pour dessiner la forme du bassin. + \item Montrer que la fonction $f$ admet comme primitive sur $\R$ la fonction $F$ définie par + \[ + F(x) = 5.0 x + \left( x^{2} - 6.3 x - 1.3\right) e^{- x} + \] + \item Calculer la quantité $\ds \int_0^4 f(x) \; dx$, vous donnerez le résultat sous forme exacte. Interpréter le résultat et reportez cette quantité sur le graphique. + \item On considère que l'échelle de votre graphique est de 1unité pour 15m. Calculer l'aire du bassin. Vous donnerez un résultat arrondi au $m^2$ près. + \end{enumerate} +\end{exercise} + +\begin{solution} + \begin{enumerate} + \item + \begin{tikzpicture}[baseline=(a.north), xscale=1, yscale=0.5] + \tkzInit[xmin=0,xmax=5,xstep=1, + ymin=0,ymax=10,ystep=1] + \tkzGrid + \tkzAxeXY + \tkzFct[domain=0:10,color=red,very thick]% + { (-x**2 + 8.3*x - 5.0)*exp(-x) + 5.0 }; + \end{tikzpicture} + \item Il faut dériver $F(x)$ et vérifier que $F'(x) = f(x)$. + \item $\ds \int_0^4 f(x) \; dx = F(4) - F(0) = 21.3 - \frac{10.5}{e^{4}}$ + \item La quantité calculée à la question précédente se retrouve 4fois pour former le bassin. Il faut ensuite prendre en compte l'échelle, comme 1unité de longueur correspond à 15m, une unité d'air correspond à $15\times15 = 225m^2$. Ainsi l'aire du bassin est égale à + \[ + (21.3 - \frac{10.5}{e^{4}})\times 4 \times 15^2 = 18997.00000 + \] + + \end{enumerate} +\end{solution} + +%%% Local Variables: +%%% mode: latex +%%% TeX-master: "master" +%%% End: + +\begin{exercise}[subtitle={Stylos}] + \emph{Les parties {\rm A} et {\rm B} de cet exercice sont indépendantes.} + + \bigskip + + \begin{minipage}{0.6\linewidth} + \textbf{Partie A} + + \medskip + + Deux ateliers A et B fabriquent des stylos pour une entreprise. + + L'atelier A fabrique 66.0\,\% des stylos, et parmi ceux-là, 35.0\,\% possèdent un défaut de fabrication. + + De plus, 28.000000000000004\,\% des stylos possèdent un défaut de fabrication et sortent de l'atelier B. + + Un stylo est prélevé au hasard dans le stock de l'entreprise. + + On considère les évènements suivants: + + \begin{itemize} + \item A : \og Le stylo a été fabriqué par l'atelier A \fg + \item B : \og Le stylo a été fabriqué par l'atelier B \fg + \item D : \og Le stylo possède un défaut de fabrication \fg + \end{itemize} + \end{minipage} + \begin{minipage}{0.4\linewidth} + \begin{center} + \begin{tikzpicture}[sloped] + \node {.} + child {node {$A$} + child {node {$D$} + edge from parent + node[above] {...} + } + child {node {$\overline{D}$} + edge from parent + node[above] {...} + } + edge from parent + node[above] {...} + } + child[missing] {} + child { node {$B$} + child {node {$D$} + edge from parent + node[above] {...} + } + child {node {$\overline{D}$} + edge from parent + node[above] {...} + } + edge from parent + node[above] {...} + } ; + \end{tikzpicture} + \end{center} + \end{minipage} + + \medskip + + \begin{enumerate} + \item Compléter l'arbre de probabilité ci-contre + \item Interpréter puis donner les probabilités $P(A)$, $P(B)$, $P_A(D)$ et $P(B \cap D)$. + + \item + \begin{enumerate} + \item Calculer la probabilité qu'un stylo provienne de l'atelier A et possède un défaut de fabrication. + \item En déduire que la probabilité qu'un stylo possède un défaut de fabrication est de $0.51$. + \end{enumerate} + \item On prélève un stylo au hasard avec un défaut. Quelle est la probabilité qu'il vienne de l'atelier A? + \end{enumerate} + + \bigskip + + \textbf{Partie B} + \medskip + + Dans cette partie, on suppose que 51.0\,\% des stylos possèdent un défaut de fabrication. + + L'entreprise confectionne des paquets contenant chacun $4$~stylos. + + Le fait qu'un stylo possède ou non un défaut de fabrication est indépendant des autres stylos. + + On appelle $X$ la variable aléatoire donnant pour un paquet le nombre de stylos qui possèdent un défaut de fabrication. + + On admet que la variable aléatoire $X$ suit une loi binomiale. + + \medskip + + \begin{enumerate} + \setcounter{enumi}{4} + \item Avec quelle loi peut-on modéliser $X$. Préciser les paramètres. + \item Calculer et interpréter la probabilité $P(X = 11)$. + \item Le directeur de l'entreprise affirme qu'il y a plus d'une chance sur deux qu'un paquet ne comporte aucun stylo défectueux. A-t-il raison ? + \item Combien de stylos peut-on espérer avoir en moyenne? + \end{enumerate} + \pagebreak +\end{exercise} + +\begin{solution} + \begin{enumerate} + \item + \begin{center} + \begin{tikzpicture}[sloped] + \node {.} + child {node {$A$} + child {node {$D$} + edge from parent + node[above] {0.35} + } + child {node {$\overline{D}$} + edge from parent + node[above] {0.65} + } + edge from parent + node[above] {0.66} + } + child[missing] {} + child { node {$B$} + child {node {$D$} + edge from parent + node[above] {0.81} + } + child {node {$\overline{D}$} + edge from parent + node[above] {0.19} + } + edge from parent + node[above] {0.34} + } ; + \end{tikzpicture} + \end{center} + \item + \begin{itemize} + \item Probabilité que le stylo vienne de l'atelier A + \[ + P(A) = 0.66 + \] + \item Probabilité que le stylo vienne de l'atelier B + \[ + P(B) = 0.34 + \] + \item Probabilité que le stylo ait un défaut sachant qu'il vient de l'atelier A. + \[ + P_A(D) = 0.35 + \] + \item Probabilité que le stylo vienne de l'atelier B et qu'il ait un défaut. + \[ + P(D \cap D) = 0.28 + \] + \end{itemize} + \item + \begin{enumerate} + \item Probabilité qu'un stylo vienne de l'atelier A et qu'il ait un defaut + \[ + P(A\cap D) = P(A) \times P_A(D) = 0.66 \times 0.35 = 0.23 + \] + \item Probabilité que le stylo ai un défaut de fabrication. + \[ + P(D) = P(A\cap D) + P(B\cap D) = 0.23 + 0.28 = 0.51 + \] + \end{enumerate} + \item Probabilité qu'il vienne de l'atelier A sachant qu'il a un defaut + \[ + P_D(A) = \frac{P(A\cap D)}{P(D)} = \frac{0.23}{0.51} = 0.45 + \] + \item $X$ peut être modélisée par une loi binomiale de paramètres $n=19$ et $p=0.51$. + \item (\textit{par de correction automatique disponible pour le résultat final} + \[ + P(X = 11) = \coefBino{19}{11}\times 0.51^{11} \times 0.49^{8} + \] + \item (\textit{par de correction automatique disponible pour le résultat final} + + Il faut calculer la probabilité qu'il y ait 0 stylo avec un defaut. + \[ + P(X = 0) = \coefBino{19}{0}\times 0.51^{0} \times 0.49^{19} + \] + Puis comparer ce nombre à 0,5. + \item Il faut calculer l'espérance + \[ + E[X] = n\times p = 19 \times 0.51 = 9.69 + \] + \end{enumerate} +\end{solution} + +\end{document} + +%%% Local Variables: +%%% mode: latex +%%% TeX-master: "master" +%%% End: diff --git a/Complementaire/DM/2105_DM1/corr_12_2105_DM1.tex b/Complementaire/DM/2105_DM1/corr_12_2105_DM1.tex new file mode 100644 index 0000000..fe30e8c --- /dev/null +++ b/Complementaire/DM/2105_DM1/corr_12_2105_DM1.tex @@ -0,0 +1,367 @@ +\documentclass[a4paper,10pt]{article} +\usepackage{myXsim} + +% Title Page +\title{DM1 \hfill POISON Lorette} +\tribe{Maths complémentaire} +\date{\hfillÀ render pour le jeudi 27 mai} + +\xsimsetup{ + solution/print = true +} + +\begin{document} +\maketitle + +Les valeurs des exercices sont générés automatiquement. Si une valeur a un nombre adhérant de chiffres après la virgule, vous pouvez l'arrondir à l'entier le plus proche. + +\begin{exercise}[subtitle={Optimisation de matière}] + \begin{minipage}{0.6\textwidth} + On se propose de fabriquer avec le moins de tôle possible une citerne fermée en forme de parallélépipède rectangle dont le volume intérieur doit être de $35m^3$. La longueur est aussi fixée à $5m$ par le cahier des charges. + + On peut donc faire varier uniquement la largeur (notée $x$) et la hauteur (notée $h$) de la cuve. + \end{minipage} + \hfill + \begin{minipage}{0.3\textwidth} + \begin{tikzpicture} + \pgfmathsetmacro{\cubex}{3} + \pgfmathsetmacro{\cubey}{1} + \pgfmathsetmacro{\cubez}{2} + \draw[black,fill=gray] (0,0,0) -- ++(-\cubex,0,0) -- ++(0,-\cubey,0) node [midway, left] {$h$} -- ++(\cubex,0,0) node [midway, below] {$x$} -- cycle; + \draw[black,fill=gray] (0,0,0) -- ++(0,0,-\cubez) -- ++(0,-\cubey,0) -- ++(0,0,\cubez) node [midway, right] {$5m$} -- cycle; + \draw[black,fill=gray] (0,0,0) -- ++(-\cubex,0,0) -- ++(0,0,-\cubez) -- ++(\cubex,0,0) -- cycle; + \end{tikzpicture} + \end{minipage} + + \begin{enumerate} + \item Expliquer pourquoi quand la largeur $x$ change, la hauteur $h$ doit elle aussi changer pour respecter les contraintes. + \item Démontrer que l'on doit avoir $h = \dfrac{7}{x}$. + \item On note $S(x)$ l'aire totale de la citerne (c'est à dire la somme des aires des six faces). Montrer que l'on peut écrire + \[ + S(x) = 10x + 14 + \frac{70}{x} + \] + \item Démontrer que + \[ + S(x) = \frac{10x^2 + 14x + 70}{x} + \] + \item Démontrer que + \[ + S'(x) = \frac{10x^2 - 70}{x^2} + \] + \item En déduire le tableau de variation de $S(x)$ sur $\intOF{0}{10}$. + \item Déterminer les valeurs de $x$ et $h$ correspondant à une utilisation minimal de tôle. + \end{enumerate} +\end{exercise} + +\begin{solution} + \begin{enumerate} + \item Le volume étant fixe si l'on fait varier $x$, $h$ doit aussi varier. + \begin{itemize} + \item Si $x = 2$ alors conserver un volume de $V=35$, $h$ doit être égale à $7 / 2$ + \item Si $x = 3$ alors conserver un volume de $V=35$, $h$ doit être égale à $7 / 3$ + \end{itemize} + \item Pour calculer le volume, on a + \begin{eqnarray*} + V &=& h\times x \times 5 \\ + 35 &=& h\times x \times 5 \\ + x &=& \frac{35}{h\times 5} = \frac{7}{h} + \end{eqnarray*} + \item Pour calculer la surface totale, on ajoute la surface de chaque face. On a donc le calcul suivant + \begin{eqnarray*} + S(x) &=& x\times h \times 2 + x\times5\times2 + h\times 5\times 2\\ + S(x) &=& x\times \frac{7}{x} \times 2 + x\times5\times2 + \frac{7}{x}\times 5\times 2\\ + S(x) &=& 10x + 14 + \frac{70}{x} + \end{eqnarray*} + \item Pour trouver cette nouvelle forme, on met chaque élément sur le même dénominateur + \begin{eqnarray*} + S(x) &=& 10x + 14 + \frac{70}{x}\\ + S(x) &=& \frac{10x\times x}{x} + \frac{14\times x}{x} + \frac{70}{x}\\ + S(x) &=& \frac{10x^2 + 14x + 70}{x} + \end{eqnarray*} + \item On retrouve la formule $\frac{u}{v}$ à dériver + \[ + u(x) = 10x^2 + 14x + 70 \Rightarrow u'(x) = 20x + 14 + \] + \[ + v(x) = x \Rightarrow v'(x) = 1 + \] + Donc au numérateur on obtient + \begin{eqnarray*} + u'(x)\times v(x) - u(x)\times v'(x) &=& (20x + 14)\times x - (10x^2 + 14x + 70)\times 1\\ + &=& 10x^2 - 70 + \end{eqnarray*} + Donc + \[ + S'(x) = \frac{10x^2 - 70}{x^2} + \] + \item Tableau de variations de $S$ + + \begin{itemize} + \item Valeur interdite: $x^2 = 0 \equiv x = 0$ + \item Signe de $10x^2 - 70$: c'est un polynôme du 2e degré + \[ + \Delta = 2800 > 0 + \] + Il y a donc 2 racines + \[ + x_1 = - 2.6457513110645907 \qquad + x_2 = 2.6457513110645907 + \] + Et on sait que $10x^2 - 70$ est du signe de $a$ donc positif en dehors des racines + \item Le dénominateur $x^2$ est toujours positif. + \item Tableau de variations + + \begin{tikzpicture}[baseline=(a.north)] + \tkzTabInit[lgt=3,espcl=3]{$x$/1,$10x^2 - 70$/1, $x^2$/1, $S'$/1, $S$/2}{$0$, $- 2.6457513110645907$, $10$} + \tkzTabLine{d,-, z, +, } + \tkzTabLine{d,+, , +, } + \tkzTabLine{d,-, z, +, } + \tkzTabVar{D+/ , -/ , +/ } + \end{tikzpicture} + + \end{itemize} + \item On a donc une surface minimal pour $x=2.6457513110645907$ et $h = 18.5202591774521349$. + \end{enumerate} +\end{solution} + +%%% Local Variables: +%%% mode: latex +%%% TeX-master: "master" +%%% End: + +\begin{exercise}[subtitle={Bassin}] + Le tour d'un bassin au niveau du sol présente deux axes de symétrie : l’axe des abscisses et la droite d’équation $x=4$. Il est obtenu par symétrie de la courbe $\mathcal{C}_f$ sur $\intFF{0}{4}$ où $f$ est la fonction définie par + + \[ + f(x) = \left(- x^{2} + 6.1 x - 3.4\right) e^{- x} + 3.4 + \] + On admet que sur $\intFF{0}{4}$ la fonction $f$ est positive. + \begin{enumerate} + \item Sur un repère, tracer l'allure de la courbe $\mathcal{C}_f$, les axes de symétries puis compléter pour dessiner la forme du bassin. + \item Montrer que la fonction $f$ admet comme primitive sur $\R$ la fonction $F$ définie par + \[ + F(x) = 3.4 x + \left( x^{2} - 4.1 x - 0.7\right) e^{- x} + \] + \item Calculer la quantité $\ds \int_0^4 f(x) \; dx$, vous donnerez le résultat sous forme exacte. Interpréter le résultat et reportez cette quantité sur le graphique. + \item On considère que l'échelle de votre graphique est de 1unité pour 15m. Calculer l'aire du bassin. Vous donnerez un résultat arrondi au $m^2$ près. + \end{enumerate} +\end{exercise} + +\begin{solution} + \begin{enumerate} + \item + \begin{tikzpicture}[baseline=(a.north), xscale=1, yscale=0.5] + \tkzInit[xmin=0,xmax=5,xstep=1, + ymin=0,ymax=10,ystep=1] + \tkzGrid + \tkzAxeXY + \tkzFct[domain=0:10,color=red,very thick]% + { (-x**2 + 6.1*x - 3.4)*exp(-x) + 3.4 }; + \end{tikzpicture} + \item Il faut dériver $F(x)$ et vérifier que $F'(x) = f(x)$. + \item $\ds \int_0^4 f(x) \; dx = F(4) - F(0) = 14.3 - \frac{1.1}{e^{4}}$ + \item La quantité calculée à la question précédente se retrouve 4fois pour former le bassin. Il faut ensuite prendre en compte l'échelle, comme 1unité de longueur correspond à 15m, une unité d'air correspond à $15\times15 = 225m^2$. Ainsi l'aire du bassin est égale à + \[ + (14.3 - \frac{1.1}{e^{4}})\times 4 \times 15^2 = 12852.00000 + \] + + \end{enumerate} +\end{solution} + +%%% Local Variables: +%%% mode: latex +%%% TeX-master: "master" +%%% End: + +\begin{exercise}[subtitle={Stylos}] + \emph{Les parties {\rm A} et {\rm B} de cet exercice sont indépendantes.} + + \bigskip + + \begin{minipage}{0.6\linewidth} + \textbf{Partie A} + + \medskip + + Deux ateliers A et B fabriquent des stylos pour une entreprise. + + L'atelier A fabrique 47.0\,\% des stylos, et parmi ceux-là, 95.0\,\% possèdent un défaut de fabrication. + + De plus, 50.0\,\% des stylos possèdent un défaut de fabrication et sortent de l'atelier B. + + Un stylo est prélevé au hasard dans le stock de l'entreprise. + + On considère les évènements suivants: + + \begin{itemize} + \item A : \og Le stylo a été fabriqué par l'atelier A \fg + \item B : \og Le stylo a été fabriqué par l'atelier B \fg + \item D : \og Le stylo possède un défaut de fabrication \fg + \end{itemize} + \end{minipage} + \begin{minipage}{0.4\linewidth} + \begin{center} + \begin{tikzpicture}[sloped] + \node {.} + child {node {$A$} + child {node {$D$} + edge from parent + node[above] {...} + } + child {node {$\overline{D}$} + edge from parent + node[above] {...} + } + edge from parent + node[above] {...} + } + child[missing] {} + child { node {$B$} + child {node {$D$} + edge from parent + node[above] {...} + } + child {node {$\overline{D}$} + edge from parent + node[above] {...} + } + edge from parent + node[above] {...} + } ; + \end{tikzpicture} + \end{center} + \end{minipage} + + \medskip + + \begin{enumerate} + \item Compléter l'arbre de probabilité ci-contre + \item Interpréter puis donner les probabilités $P(A)$, $P(B)$, $P_A(D)$ et $P(B \cap D)$. + + \item + \begin{enumerate} + \item Calculer la probabilité qu'un stylo provienne de l'atelier A et possède un défaut de fabrication. + \item En déduire que la probabilité qu'un stylo possède un défaut de fabrication est de $0.95$. + \end{enumerate} + \item On prélève un stylo au hasard avec un défaut. Quelle est la probabilité qu'il vienne de l'atelier A? + \end{enumerate} + + \bigskip + + \textbf{Partie B} + \medskip + + Dans cette partie, on suppose que 95.0\,\% des stylos possèdent un défaut de fabrication. + + L'entreprise confectionne des paquets contenant chacun $4$~stylos. + + Le fait qu'un stylo possède ou non un défaut de fabrication est indépendant des autres stylos. + + On appelle $X$ la variable aléatoire donnant pour un paquet le nombre de stylos qui possèdent un défaut de fabrication. + + On admet que la variable aléatoire $X$ suit une loi binomiale. + + \medskip + + \begin{enumerate} + \setcounter{enumi}{4} + \item Avec quelle loi peut-on modéliser $X$. Préciser les paramètres. + \item Calculer et interpréter la probabilité $P(X = 12)$. + \item Le directeur de l'entreprise affirme qu'il y a plus d'une chance sur deux qu'un paquet ne comporte aucun stylo défectueux. A-t-il raison ? + \item Combien de stylos peut-on espérer avoir en moyenne? + \end{enumerate} + \pagebreak +\end{exercise} + +\begin{solution} + \begin{enumerate} + \item + \begin{center} + \begin{tikzpicture}[sloped] + \node {.} + child {node {$A$} + child {node {$D$} + edge from parent + node[above] {0.95} + } + child {node {$\overline{D}$} + edge from parent + node[above] {0.05} + } + edge from parent + node[above] {0.47} + } + child[missing] {} + child { node {$B$} + child {node {$D$} + edge from parent + node[above] {0.95} + } + child {node {$\overline{D}$} + edge from parent + node[above] {0.05} + } + edge from parent + node[above] {0.53} + } ; + \end{tikzpicture} + \end{center} + \item + \begin{itemize} + \item Probabilité que le stylo vienne de l'atelier A + \[ + P(A) = 0.47 + \] + \item Probabilité que le stylo vienne de l'atelier B + \[ + P(B) = 0.53 + \] + \item Probabilité que le stylo ait un défaut sachant qu'il vient de l'atelier A. + \[ + P_A(D) = 0.95 + \] + \item Probabilité que le stylo vienne de l'atelier B et qu'il ait un défaut. + \[ + P(D \cap D) = 0.5 + \] + \end{itemize} + \item + \begin{enumerate} + \item Probabilité qu'un stylo vienne de l'atelier A et qu'il ait un defaut + \[ + P(A\cap D) = P(A) \times P_A(D) = 0.47 \times 0.95 = 0.45 + \] + \item Probabilité que le stylo ai un défaut de fabrication. + \[ + P(D) = P(A\cap D) + P(B\cap D) = 0.45 + 0.5 = 0.95 + \] + \end{enumerate} + \item Probabilité qu'il vienne de l'atelier A sachant qu'il a un defaut + \[ + P_D(A) = \frac{P(A\cap D)}{P(D)} = \frac{0.45}{0.95} = 0.47 + \] + \item $X$ peut être modélisée par une loi binomiale de paramètres $n=15$ et $p=0.95$. + \item (\textit{par de correction automatique disponible pour le résultat final} + \[ + P(X = 12) = \coefBino{15}{12}\times 0.95^{12} \times 0.05^{3} + \] + \item (\textit{par de correction automatique disponible pour le résultat final} + + Il faut calculer la probabilité qu'il y ait 0 stylo avec un defaut. + \[ + P(X = 0) = \coefBino{15}{0}\times 0.95^{0} \times 0.05^{15} + \] + Puis comparer ce nombre à 0,5. + \item Il faut calculer l'espérance + \[ + E[X] = n\times p = 15 \times 0.95 = 14.25 + \] + \end{enumerate} +\end{solution} + +\end{document} + +%%% Local Variables: +%%% mode: latex +%%% TeX-master: "master" +%%% End: diff --git a/Complementaire/DM/2105_DM1/corr_13_2105_DM1.tex b/Complementaire/DM/2105_DM1/corr_13_2105_DM1.tex new file mode 100644 index 0000000..93a14b9 --- /dev/null +++ b/Complementaire/DM/2105_DM1/corr_13_2105_DM1.tex @@ -0,0 +1,367 @@ +\documentclass[a4paper,10pt]{article} +\usepackage{myXsim} + +% Title Page +\title{DM1 \hfill RODRIGUEZ Teddy} +\tribe{Maths complémentaire} +\date{\hfillÀ render pour le jeudi 27 mai} + +\xsimsetup{ + solution/print = true +} + +\begin{document} +\maketitle + +Les valeurs des exercices sont générés automatiquement. Si une valeur a un nombre adhérant de chiffres après la virgule, vous pouvez l'arrondir à l'entier le plus proche. + +\begin{exercise}[subtitle={Optimisation de matière}] + \begin{minipage}{0.6\textwidth} + On se propose de fabriquer avec le moins de tôle possible une citerne fermée en forme de parallélépipède rectangle dont le volume intérieur doit être de $50m^3$. La longueur est aussi fixée à $5m$ par le cahier des charges. + + On peut donc faire varier uniquement la largeur (notée $x$) et la hauteur (notée $h$) de la cuve. + \end{minipage} + \hfill + \begin{minipage}{0.3\textwidth} + \begin{tikzpicture} + \pgfmathsetmacro{\cubex}{3} + \pgfmathsetmacro{\cubey}{1} + \pgfmathsetmacro{\cubez}{2} + \draw[black,fill=gray] (0,0,0) -- ++(-\cubex,0,0) -- ++(0,-\cubey,0) node [midway, left] {$h$} -- ++(\cubex,0,0) node [midway, below] {$x$} -- cycle; + \draw[black,fill=gray] (0,0,0) -- ++(0,0,-\cubez) -- ++(0,-\cubey,0) -- ++(0,0,\cubez) node [midway, right] {$5m$} -- cycle; + \draw[black,fill=gray] (0,0,0) -- ++(-\cubex,0,0) -- ++(0,0,-\cubez) -- ++(\cubex,0,0) -- cycle; + \end{tikzpicture} + \end{minipage} + + \begin{enumerate} + \item Expliquer pourquoi quand la largeur $x$ change, la hauteur $h$ doit elle aussi changer pour respecter les contraintes. + \item Démontrer que l'on doit avoir $h = \dfrac{10}{x}$. + \item On note $S(x)$ l'aire totale de la citerne (c'est à dire la somme des aires des six faces). Montrer que l'on peut écrire + \[ + S(x) = 10x + 20 + \frac{100}{x} + \] + \item Démontrer que + \[ + S(x) = \frac{10x^2 + 20x + 100}{x} + \] + \item Démontrer que + \[ + S'(x) = \frac{10x^2 - 100}{x^2} + \] + \item En déduire le tableau de variation de $S(x)$ sur $\intOF{0}{10}$. + \item Déterminer les valeurs de $x$ et $h$ correspondant à une utilisation minimal de tôle. + \end{enumerate} +\end{exercise} + +\begin{solution} + \begin{enumerate} + \item Le volume étant fixe si l'on fait varier $x$, $h$ doit aussi varier. + \begin{itemize} + \item Si $x = 2$ alors conserver un volume de $V=50$, $h$ doit être égale à $10 / 2$ + \item Si $x = 3$ alors conserver un volume de $V=50$, $h$ doit être égale à $10 / 3$ + \end{itemize} + \item Pour calculer le volume, on a + \begin{eqnarray*} + V &=& h\times x \times 5 \\ + 50 &=& h\times x \times 5 \\ + x &=& \frac{50}{h\times 5} = \frac{10}{h} + \end{eqnarray*} + \item Pour calculer la surface totale, on ajoute la surface de chaque face. On a donc le calcul suivant + \begin{eqnarray*} + S(x) &=& x\times h \times 2 + x\times5\times2 + h\times 5\times 2\\ + S(x) &=& x\times \frac{10}{x} \times 2 + x\times5\times2 + \frac{10}{x}\times 5\times 2\\ + S(x) &=& 10x + 20 + \frac{100}{x} + \end{eqnarray*} + \item Pour trouver cette nouvelle forme, on met chaque élément sur le même dénominateur + \begin{eqnarray*} + S(x) &=& 10x + 20 + \frac{100}{x}\\ + S(x) &=& \frac{10x\times x}{x} + \frac{20\times x}{x} + \frac{100}{x}\\ + S(x) &=& \frac{10x^2 + 20x + 100}{x} + \end{eqnarray*} + \item On retrouve la formule $\frac{u}{v}$ à dériver + \[ + u(x) = 10x^2 + 20x + 100 \Rightarrow u'(x) = 20x + 20 + \] + \[ + v(x) = x \Rightarrow v'(x) = 1 + \] + Donc au numérateur on obtient + \begin{eqnarray*} + u'(x)\times v(x) - u(x)\times v'(x) &=& (20x + 20)\times x - (10x^2 + 20x + 100)\times 1\\ + &=& 10x^2 - 100 + \end{eqnarray*} + Donc + \[ + S'(x) = \frac{10x^2 - 100}{x^2} + \] + \item Tableau de variations de $S$ + + \begin{itemize} + \item Valeur interdite: $x^2 = 0 \equiv x = 0$ + \item Signe de $10x^2 - 100$: c'est un polynôme du 2e degré + \[ + \Delta = 4000 > 0 + \] + Il y a donc 2 racines + \[ + x_1 = - 3.162277660168379 \qquad + x_2 = 3.162277660168379 + \] + Et on sait que $10x^2 - 100$ est du signe de $a$ donc positif en dehors des racines + \item Le dénominateur $x^2$ est toujours positif. + \item Tableau de variations + + \begin{tikzpicture}[baseline=(a.north)] + \tkzTabInit[lgt=3,espcl=3]{$x$/1,$10x^2 - 100$/1, $x^2$/1, $S'$/1, $S$/2}{$0$, $- 3.162277660168379$, $10$} + \tkzTabLine{d,-, z, +, } + \tkzTabLine{d,+, , +, } + \tkzTabLine{d,-, z, +, } + \tkzTabVar{D+/ , -/ , +/ } + \end{tikzpicture} + + \end{itemize} + \item On a donc une surface minimal pour $x=3.162277660168379$ et $h = 31.622776601683790$. + \end{enumerate} +\end{solution} + +%%% Local Variables: +%%% mode: latex +%%% TeX-master: "master" +%%% End: + +\begin{exercise}[subtitle={Bassin}] + Le tour d'un bassin au niveau du sol présente deux axes de symétrie : l’axe des abscisses et la droite d’équation $x=4$. Il est obtenu par symétrie de la courbe $\mathcal{C}_f$ sur $\intFF{0}{4}$ où $f$ est la fonction définie par + + \[ + f(x) = \left(- x^{2} + 1.0 x - 3.6\right) e^{- x} + 3.6 + \] + On admet que sur $\intFF{0}{4}$ la fonction $f$ est positive. + \begin{enumerate} + \item Sur un repère, tracer l'allure de la courbe $\mathcal{C}_f$, les axes de symétries puis compléter pour dessiner la forme du bassin. + \item Montrer que la fonction $f$ admet comme primitive sur $\R$ la fonction $F$ définie par + \[ + F(x) = 3.6 x + \left( x^{2} + x + 4.6\right) e^{- x} + \] + \item Calculer la quantité $\ds \int_0^4 f(x) \; dx$, vous donnerez le résultat sous forme exacte. Interpréter le résultat et reportez cette quantité sur le graphique. + \item On considère que l'échelle de votre graphique est de 1unité pour 15m. Calculer l'aire du bassin. Vous donnerez un résultat arrondi au $m^2$ près. + \end{enumerate} +\end{exercise} + +\begin{solution} + \begin{enumerate} + \item + \begin{tikzpicture}[baseline=(a.north), xscale=1, yscale=0.5] + \tkzInit[xmin=0,xmax=5,xstep=1, + ymin=0,ymax=10,ystep=1] + \tkzGrid + \tkzAxeXY + \tkzFct[domain=0:10,color=red,very thick]% + { (-x**2 + 1.0*x - 3.6)*exp(-x) + 3.6 }; + \end{tikzpicture} + \item Il faut dériver $F(x)$ et vérifier que $F'(x) = f(x)$. + \item $\ds \int_0^4 f(x) \; dx = F(4) - F(0) = \frac{24.6}{e^{4}} + 9.8$ + \item La quantité calculée à la question précédente se retrouve 4fois pour former le bassin. Il faut ensuite prendre en compte l'échelle, comme 1unité de longueur correspond à 15m, une unité d'air correspond à $15\times15 = 225m^2$. Ainsi l'aire du bassin est égale à + \[ + (\frac{24.6}{e^{4}} + 9.8)\times 4 \times 15^2 = 9226.000000 + \] + + \end{enumerate} +\end{solution} + +%%% Local Variables: +%%% mode: latex +%%% TeX-master: "master" +%%% End: + +\begin{exercise}[subtitle={Stylos}] + \emph{Les parties {\rm A} et {\rm B} de cet exercice sont indépendantes.} + + \bigskip + + \begin{minipage}{0.6\linewidth} + \textbf{Partie A} + + \medskip + + Deux ateliers A et B fabriquent des stylos pour une entreprise. + + L'atelier A fabrique 64.0\,\% des stylos, et parmi ceux-là, 4.0\,\% possèdent un défaut de fabrication. + + De plus, 6.0\,\% des stylos possèdent un défaut de fabrication et sortent de l'atelier B. + + Un stylo est prélevé au hasard dans le stock de l'entreprise. + + On considère les évènements suivants: + + \begin{itemize} + \item A : \og Le stylo a été fabriqué par l'atelier A \fg + \item B : \og Le stylo a été fabriqué par l'atelier B \fg + \item D : \og Le stylo possède un défaut de fabrication \fg + \end{itemize} + \end{minipage} + \begin{minipage}{0.4\linewidth} + \begin{center} + \begin{tikzpicture}[sloped] + \node {.} + child {node {$A$} + child {node {$D$} + edge from parent + node[above] {...} + } + child {node {$\overline{D}$} + edge from parent + node[above] {...} + } + edge from parent + node[above] {...} + } + child[missing] {} + child { node {$B$} + child {node {$D$} + edge from parent + node[above] {...} + } + child {node {$\overline{D}$} + edge from parent + node[above] {...} + } + edge from parent + node[above] {...} + } ; + \end{tikzpicture} + \end{center} + \end{minipage} + + \medskip + + \begin{enumerate} + \item Compléter l'arbre de probabilité ci-contre + \item Interpréter puis donner les probabilités $P(A)$, $P(B)$, $P_A(D)$ et $P(B \cap D)$. + + \item + \begin{enumerate} + \item Calculer la probabilité qu'un stylo provienne de l'atelier A et possède un défaut de fabrication. + \item En déduire que la probabilité qu'un stylo possède un défaut de fabrication est de $0.09$. + \end{enumerate} + \item On prélève un stylo au hasard avec un défaut. Quelle est la probabilité qu'il vienne de l'atelier A? + \end{enumerate} + + \bigskip + + \textbf{Partie B} + \medskip + + Dans cette partie, on suppose que 9.0\,\% des stylos possèdent un défaut de fabrication. + + L'entreprise confectionne des paquets contenant chacun $4$~stylos. + + Le fait qu'un stylo possède ou non un défaut de fabrication est indépendant des autres stylos. + + On appelle $X$ la variable aléatoire donnant pour un paquet le nombre de stylos qui possèdent un défaut de fabrication. + + On admet que la variable aléatoire $X$ suit une loi binomiale. + + \medskip + + \begin{enumerate} + \setcounter{enumi}{4} + \item Avec quelle loi peut-on modéliser $X$. Préciser les paramètres. + \item Calculer et interpréter la probabilité $P(X = 9)$. + \item Le directeur de l'entreprise affirme qu'il y a plus d'une chance sur deux qu'un paquet ne comporte aucun stylo défectueux. A-t-il raison ? + \item Combien de stylos peut-on espérer avoir en moyenne? + \end{enumerate} + \pagebreak +\end{exercise} + +\begin{solution} + \begin{enumerate} + \item + \begin{center} + \begin{tikzpicture}[sloped] + \node {.} + child {node {$A$} + child {node {$D$} + edge from parent + node[above] {0.04} + } + child {node {$\overline{D}$} + edge from parent + node[above] {0.96} + } + edge from parent + node[above] {0.64} + } + child[missing] {} + child { node {$B$} + child {node {$D$} + edge from parent + node[above] {0.18} + } + child {node {$\overline{D}$} + edge from parent + node[above] {0.82} + } + edge from parent + node[above] {0.36} + } ; + \end{tikzpicture} + \end{center} + \item + \begin{itemize} + \item Probabilité que le stylo vienne de l'atelier A + \[ + P(A) = 0.64 + \] + \item Probabilité que le stylo vienne de l'atelier B + \[ + P(B) = 0.36 + \] + \item Probabilité que le stylo ait un défaut sachant qu'il vient de l'atelier A. + \[ + P_A(D) = 0.04 + \] + \item Probabilité que le stylo vienne de l'atelier B et qu'il ait un défaut. + \[ + P(D \cap D) = 0.06 + \] + \end{itemize} + \item + \begin{enumerate} + \item Probabilité qu'un stylo vienne de l'atelier A et qu'il ait un defaut + \[ + P(A\cap D) = P(A) \times P_A(D) = 0.64 \times 0.04 = 0.03 + \] + \item Probabilité que le stylo ai un défaut de fabrication. + \[ + P(D) = P(A\cap D) + P(B\cap D) = 0.03 + 0.06 = 0.09 + \] + \end{enumerate} + \item Probabilité qu'il vienne de l'atelier A sachant qu'il a un defaut + \[ + P_D(A) = \frac{P(A\cap D)}{P(D)} = \frac{0.03}{0.09} = 0.33 + \] + \item $X$ peut être modélisée par une loi binomiale de paramètres $n=16$ et $p=0.09$. + \item (\textit{par de correction automatique disponible pour le résultat final} + \[ + P(X = 9) = \coefBino{16}{9}\times 0.09^{9} \times 0.91^{7} + \] + \item (\textit{par de correction automatique disponible pour le résultat final} + + Il faut calculer la probabilité qu'il y ait 0 stylo avec un defaut. + \[ + P(X = 0) = \coefBino{16}{0}\times 0.09^{0} \times 0.91^{16} + \] + Puis comparer ce nombre à 0,5. + \item Il faut calculer l'espérance + \[ + E[X] = n\times p = 16 \times 0.09 = 1.44 + \] + \end{enumerate} +\end{solution} + +\end{document} + +%%% Local Variables: +%%% mode: latex +%%% TeX-master: "master" +%%% End: diff --git a/Complementaire/DM/2105_DM1/corr_14_2105_DM1.tex b/Complementaire/DM/2105_DM1/corr_14_2105_DM1.tex new file mode 100644 index 0000000..d9f7526 --- /dev/null +++ b/Complementaire/DM/2105_DM1/corr_14_2105_DM1.tex @@ -0,0 +1,367 @@ +\documentclass[a4paper,10pt]{article} +\usepackage{myXsim} + +% Title Page +\title{DM1 \hfill SAINT CYR Louis} +\tribe{Maths complémentaire} +\date{\hfillÀ render pour le jeudi 27 mai} + +\xsimsetup{ + solution/print = true +} + +\begin{document} +\maketitle + +Les valeurs des exercices sont générés automatiquement. Si une valeur a un nombre adhérant de chiffres après la virgule, vous pouvez l'arrondir à l'entier le plus proche. + +\begin{exercise}[subtitle={Optimisation de matière}] + \begin{minipage}{0.6\textwidth} + On se propose de fabriquer avec le moins de tôle possible une citerne fermée en forme de parallélépipède rectangle dont le volume intérieur doit être de $45m^3$. La longueur est aussi fixée à $5m$ par le cahier des charges. + + On peut donc faire varier uniquement la largeur (notée $x$) et la hauteur (notée $h$) de la cuve. + \end{minipage} + \hfill + \begin{minipage}{0.3\textwidth} + \begin{tikzpicture} + \pgfmathsetmacro{\cubex}{3} + \pgfmathsetmacro{\cubey}{1} + \pgfmathsetmacro{\cubez}{2} + \draw[black,fill=gray] (0,0,0) -- ++(-\cubex,0,0) -- ++(0,-\cubey,0) node [midway, left] {$h$} -- ++(\cubex,0,0) node [midway, below] {$x$} -- cycle; + \draw[black,fill=gray] (0,0,0) -- ++(0,0,-\cubez) -- ++(0,-\cubey,0) -- ++(0,0,\cubez) node [midway, right] {$5m$} -- cycle; + \draw[black,fill=gray] (0,0,0) -- ++(-\cubex,0,0) -- ++(0,0,-\cubez) -- ++(\cubex,0,0) -- cycle; + \end{tikzpicture} + \end{minipage} + + \begin{enumerate} + \item Expliquer pourquoi quand la largeur $x$ change, la hauteur $h$ doit elle aussi changer pour respecter les contraintes. + \item Démontrer que l'on doit avoir $h = \dfrac{9}{x}$. + \item On note $S(x)$ l'aire totale de la citerne (c'est à dire la somme des aires des six faces). Montrer que l'on peut écrire + \[ + S(x) = 10x + 18 + \frac{90}{x} + \] + \item Démontrer que + \[ + S(x) = \frac{10x^2 + 18x + 90}{x} + \] + \item Démontrer que + \[ + S'(x) = \frac{10x^2 - 90}{x^2} + \] + \item En déduire le tableau de variation de $S(x)$ sur $\intOF{0}{10}$. + \item Déterminer les valeurs de $x$ et $h$ correspondant à une utilisation minimal de tôle. + \end{enumerate} +\end{exercise} + +\begin{solution} + \begin{enumerate} + \item Le volume étant fixe si l'on fait varier $x$, $h$ doit aussi varier. + \begin{itemize} + \item Si $x = 2$ alors conserver un volume de $V=45$, $h$ doit être égale à $9 / 2$ + \item Si $x = 3$ alors conserver un volume de $V=45$, $h$ doit être égale à $9 / 3$ + \end{itemize} + \item Pour calculer le volume, on a + \begin{eqnarray*} + V &=& h\times x \times 5 \\ + 45 &=& h\times x \times 5 \\ + x &=& \frac{45}{h\times 5} = \frac{9}{h} + \end{eqnarray*} + \item Pour calculer la surface totale, on ajoute la surface de chaque face. On a donc le calcul suivant + \begin{eqnarray*} + S(x) &=& x\times h \times 2 + x\times5\times2 + h\times 5\times 2\\ + S(x) &=& x\times \frac{9}{x} \times 2 + x\times5\times2 + \frac{9}{x}\times 5\times 2\\ + S(x) &=& 10x + 18 + \frac{90}{x} + \end{eqnarray*} + \item Pour trouver cette nouvelle forme, on met chaque élément sur le même dénominateur + \begin{eqnarray*} + S(x) &=& 10x + 18 + \frac{90}{x}\\ + S(x) &=& \frac{10x\times x}{x} + \frac{18\times x}{x} + \frac{90}{x}\\ + S(x) &=& \frac{10x^2 + 18x + 90}{x} + \end{eqnarray*} + \item On retrouve la formule $\frac{u}{v}$ à dériver + \[ + u(x) = 10x^2 + 18x + 90 \Rightarrow u'(x) = 20x + 18 + \] + \[ + v(x) = x \Rightarrow v'(x) = 1 + \] + Donc au numérateur on obtient + \begin{eqnarray*} + u'(x)\times v(x) - u(x)\times v'(x) &=& (20x + 18)\times x - (10x^2 + 18x + 90)\times 1\\ + &=& 10x^2 - 90 + \end{eqnarray*} + Donc + \[ + S'(x) = \frac{10x^2 - 90}{x^2} + \] + \item Tableau de variations de $S$ + + \begin{itemize} + \item Valeur interdite: $x^2 = 0 \equiv x = 0$ + \item Signe de $10x^2 - 90$: c'est un polynôme du 2e degré + \[ + \Delta = 3600 > 0 + \] + Il y a donc 2 racines + \[ + x_1 = - 3 \qquad + x_2 = 3 + \] + Et on sait que $10x^2 - 90$ est du signe de $a$ donc positif en dehors des racines + \item Le dénominateur $x^2$ est toujours positif. + \item Tableau de variations + + \begin{tikzpicture}[baseline=(a.north)] + \tkzTabInit[lgt=3,espcl=3]{$x$/1,$10x^2 - 90$/1, $x^2$/1, $S'$/1, $S$/2}{$0$, $- 3$, $10$} + \tkzTabLine{d,-, z, +, } + \tkzTabLine{d,+, , +, } + \tkzTabLine{d,-, z, +, } + \tkzTabVar{D+/ , -/ , +/ } + \end{tikzpicture} + + \end{itemize} + \item On a donc une surface minimal pour $x=3$ et $h = 27$. + \end{enumerate} +\end{solution} + +%%% Local Variables: +%%% mode: latex +%%% TeX-master: "master" +%%% End: + +\begin{exercise}[subtitle={Bassin}] + Le tour d'un bassin au niveau du sol présente deux axes de symétrie : l’axe des abscisses et la droite d’équation $x=4$. Il est obtenu par symétrie de la courbe $\mathcal{C}_f$ sur $\intFF{0}{4}$ où $f$ est la fonction définie par + + \[ + f(x) = \left(- x^{2} + 8.0 x - 0.9\right) e^{- x} + 0.9 + \] + On admet que sur $\intFF{0}{4}$ la fonction $f$ est positive. + \begin{enumerate} + \item Sur un repère, tracer l'allure de la courbe $\mathcal{C}_f$, les axes de symétries puis compléter pour dessiner la forme du bassin. + \item Montrer que la fonction $f$ admet comme primitive sur $\R$ la fonction $F$ définie par + \[ + F(x) = 0.9 x + \left( x^{2} - 6.0 x - 5.1\right) e^{- x} + \] + \item Calculer la quantité $\ds \int_0^4 f(x) \; dx$, vous donnerez le résultat sous forme exacte. Interpréter le résultat et reportez cette quantité sur le graphique. + \item On considère que l'échelle de votre graphique est de 1unité pour 15m. Calculer l'aire du bassin. Vous donnerez un résultat arrondi au $m^2$ près. + \end{enumerate} +\end{exercise} + +\begin{solution} + \begin{enumerate} + \item + \begin{tikzpicture}[baseline=(a.north), xscale=1, yscale=0.5] + \tkzInit[xmin=0,xmax=5,xstep=1, + ymin=0,ymax=10,ystep=1] + \tkzGrid + \tkzAxeXY + \tkzFct[domain=0:10,color=red,very thick]% + { (-x**2 + 8.0*x - 0.9)*exp(-x) + 0.9 }; + \end{tikzpicture} + \item Il faut dériver $F(x)$ et vérifier que $F'(x) = f(x)$. + \item $\ds \int_0^4 f(x) \; dx = F(4) - F(0) = 8.7 - \frac{13.1}{e^{4}}$ + \item La quantité calculée à la question précédente se retrouve 4fois pour former le bassin. Il faut ensuite prendre en compte l'échelle, comme 1unité de longueur correspond à 15m, une unité d'air correspond à $15\times15 = 225m^2$. Ainsi l'aire du bassin est égale à + \[ + (8.7 - \frac{13.1}{e^{4}})\times 4 \times 15^2 = 7614.000000 + \] + + \end{enumerate} +\end{solution} + +%%% Local Variables: +%%% mode: latex +%%% TeX-master: "master" +%%% End: + +\begin{exercise}[subtitle={Stylos}] + \emph{Les parties {\rm A} et {\rm B} de cet exercice sont indépendantes.} + + \bigskip + + \begin{minipage}{0.6\linewidth} + \textbf{Partie A} + + \medskip + + Deux ateliers A et B fabriquent des stylos pour une entreprise. + + L'atelier A fabrique 88.0\,\% des stylos, et parmi ceux-là, 75.0\,\% possèdent un défaut de fabrication. + + De plus, 4.0\,\% des stylos possèdent un défaut de fabrication et sortent de l'atelier B. + + Un stylo est prélevé au hasard dans le stock de l'entreprise. + + On considère les évènements suivants: + + \begin{itemize} + \item A : \og Le stylo a été fabriqué par l'atelier A \fg + \item B : \og Le stylo a été fabriqué par l'atelier B \fg + \item D : \og Le stylo possède un défaut de fabrication \fg + \end{itemize} + \end{minipage} + \begin{minipage}{0.4\linewidth} + \begin{center} + \begin{tikzpicture}[sloped] + \node {.} + child {node {$A$} + child {node {$D$} + edge from parent + node[above] {...} + } + child {node {$\overline{D}$} + edge from parent + node[above] {...} + } + edge from parent + node[above] {...} + } + child[missing] {} + child { node {$B$} + child {node {$D$} + edge from parent + node[above] {...} + } + child {node {$\overline{D}$} + edge from parent + node[above] {...} + } + edge from parent + node[above] {...} + } ; + \end{tikzpicture} + \end{center} + \end{minipage} + + \medskip + + \begin{enumerate} + \item Compléter l'arbre de probabilité ci-contre + \item Interpréter puis donner les probabilités $P(A)$, $P(B)$, $P_A(D)$ et $P(B \cap D)$. + + \item + \begin{enumerate} + \item Calculer la probabilité qu'un stylo provienne de l'atelier A et possède un défaut de fabrication. + \item En déduire que la probabilité qu'un stylo possède un défaut de fabrication est de $0.7$. + \end{enumerate} + \item On prélève un stylo au hasard avec un défaut. Quelle est la probabilité qu'il vienne de l'atelier A? + \end{enumerate} + + \bigskip + + \textbf{Partie B} + \medskip + + Dans cette partie, on suppose que 70.0\,\% des stylos possèdent un défaut de fabrication. + + L'entreprise confectionne des paquets contenant chacun $4$~stylos. + + Le fait qu'un stylo possède ou non un défaut de fabrication est indépendant des autres stylos. + + On appelle $X$ la variable aléatoire donnant pour un paquet le nombre de stylos qui possèdent un défaut de fabrication. + + On admet que la variable aléatoire $X$ suit une loi binomiale. + + \medskip + + \begin{enumerate} + \setcounter{enumi}{4} + \item Avec quelle loi peut-on modéliser $X$. Préciser les paramètres. + \item Calculer et interpréter la probabilité $P(X = 14)$. + \item Le directeur de l'entreprise affirme qu'il y a plus d'une chance sur deux qu'un paquet ne comporte aucun stylo défectueux. A-t-il raison ? + \item Combien de stylos peut-on espérer avoir en moyenne? + \end{enumerate} + \pagebreak +\end{exercise} + +\begin{solution} + \begin{enumerate} + \item + \begin{center} + \begin{tikzpicture}[sloped] + \node {.} + child {node {$A$} + child {node {$D$} + edge from parent + node[above] {0.75} + } + child {node {$\overline{D}$} + edge from parent + node[above] {0.25} + } + edge from parent + node[above] {0.88} + } + child[missing] {} + child { node {$B$} + child {node {$D$} + edge from parent + node[above] {0.34} + } + child {node {$\overline{D}$} + edge from parent + node[above] {0.66} + } + edge from parent + node[above] {0.12} + } ; + \end{tikzpicture} + \end{center} + \item + \begin{itemize} + \item Probabilité que le stylo vienne de l'atelier A + \[ + P(A) = 0.88 + \] + \item Probabilité que le stylo vienne de l'atelier B + \[ + P(B) = 0.12 + \] + \item Probabilité que le stylo ait un défaut sachant qu'il vient de l'atelier A. + \[ + P_A(D) = 0.75 + \] + \item Probabilité que le stylo vienne de l'atelier B et qu'il ait un défaut. + \[ + P(D \cap D) = 0.04 + \] + \end{itemize} + \item + \begin{enumerate} + \item Probabilité qu'un stylo vienne de l'atelier A et qu'il ait un defaut + \[ + P(A\cap D) = P(A) \times P_A(D) = 0.88 \times 0.75 = 0.66 + \] + \item Probabilité que le stylo ai un défaut de fabrication. + \[ + P(D) = P(A\cap D) + P(B\cap D) = 0.66 + 0.04 = 0.7 + \] + \end{enumerate} + \item Probabilité qu'il vienne de l'atelier A sachant qu'il a un defaut + \[ + P_D(A) = \frac{P(A\cap D)}{P(D)} = \frac{0.66}{0.7} = 0.94 + \] + \item $X$ peut être modélisée par une loi binomiale de paramètres $n=15$ et $p=0.7$. + \item (\textit{par de correction automatique disponible pour le résultat final} + \[ + P(X = 14) = \coefBino{15}{14}\times 0.7^{14} \times 0.3^{1} + \] + \item (\textit{par de correction automatique disponible pour le résultat final} + + Il faut calculer la probabilité qu'il y ait 0 stylo avec un defaut. + \[ + P(X = 0) = \coefBino{15}{0}\times 0.7^{0} \times 0.3^{15} + \] + Puis comparer ce nombre à 0,5. + \item Il faut calculer l'espérance + \[ + E[X] = n\times p = 15 \times 0.7 = 10.5 + \] + \end{enumerate} +\end{solution} + +\end{document} + +%%% Local Variables: +%%% mode: latex +%%% TeX-master: "master" +%%% End: diff --git a/Complementaire/DM/2105_DM1/corr_15_2105_DM1.tex b/Complementaire/DM/2105_DM1/corr_15_2105_DM1.tex new file mode 100644 index 0000000..527fb0f --- /dev/null +++ b/Complementaire/DM/2105_DM1/corr_15_2105_DM1.tex @@ -0,0 +1,367 @@ +\documentclass[a4paper,10pt]{article} +\usepackage{myXsim} + +% Title Page +\title{DM1 \hfill SAVIN Lou-Ann} +\tribe{Maths complémentaire} +\date{\hfillÀ render pour le jeudi 27 mai} + +\xsimsetup{ + solution/print = true +} + +\begin{document} +\maketitle + +Les valeurs des exercices sont générés automatiquement. Si une valeur a un nombre adhérant de chiffres après la virgule, vous pouvez l'arrondir à l'entier le plus proche. + +\begin{exercise}[subtitle={Optimisation de matière}] + \begin{minipage}{0.6\textwidth} + On se propose de fabriquer avec le moins de tôle possible une citerne fermée en forme de parallélépipède rectangle dont le volume intérieur doit être de $18m^3$. La longueur est aussi fixée à $3m$ par le cahier des charges. + + On peut donc faire varier uniquement la largeur (notée $x$) et la hauteur (notée $h$) de la cuve. + \end{minipage} + \hfill + \begin{minipage}{0.3\textwidth} + \begin{tikzpicture} + \pgfmathsetmacro{\cubex}{3} + \pgfmathsetmacro{\cubey}{1} + \pgfmathsetmacro{\cubez}{2} + \draw[black,fill=gray] (0,0,0) -- ++(-\cubex,0,0) -- ++(0,-\cubey,0) node [midway, left] {$h$} -- ++(\cubex,0,0) node [midway, below] {$x$} -- cycle; + \draw[black,fill=gray] (0,0,0) -- ++(0,0,-\cubez) -- ++(0,-\cubey,0) -- ++(0,0,\cubez) node [midway, right] {$3m$} -- cycle; + \draw[black,fill=gray] (0,0,0) -- ++(-\cubex,0,0) -- ++(0,0,-\cubez) -- ++(\cubex,0,0) -- cycle; + \end{tikzpicture} + \end{minipage} + + \begin{enumerate} + \item Expliquer pourquoi quand la largeur $x$ change, la hauteur $h$ doit elle aussi changer pour respecter les contraintes. + \item Démontrer que l'on doit avoir $h = \dfrac{6}{x}$. + \item On note $S(x)$ l'aire totale de la citerne (c'est à dire la somme des aires des six faces). Montrer que l'on peut écrire + \[ + S(x) = 6x + 12 + \frac{36}{x} + \] + \item Démontrer que + \[ + S(x) = \frac{6x^2 + 12x + 36}{x} + \] + \item Démontrer que + \[ + S'(x) = \frac{6x^2 - 36}{x^2} + \] + \item En déduire le tableau de variation de $S(x)$ sur $\intOF{0}{10}$. + \item Déterminer les valeurs de $x$ et $h$ correspondant à une utilisation minimal de tôle. + \end{enumerate} +\end{exercise} + +\begin{solution} + \begin{enumerate} + \item Le volume étant fixe si l'on fait varier $x$, $h$ doit aussi varier. + \begin{itemize} + \item Si $x = 2$ alors conserver un volume de $V=18$, $h$ doit être égale à $6 / 2$ + \item Si $x = 3$ alors conserver un volume de $V=18$, $h$ doit être égale à $6 / 3$ + \end{itemize} + \item Pour calculer le volume, on a + \begin{eqnarray*} + V &=& h\times x \times 3 \\ + 18 &=& h\times x \times 3 \\ + x &=& \frac{18}{h\times 3} = \frac{6}{h} + \end{eqnarray*} + \item Pour calculer la surface totale, on ajoute la surface de chaque face. On a donc le calcul suivant + \begin{eqnarray*} + S(x) &=& x\times h \times 2 + x\times3\times2 + h\times 3\times 2\\ + S(x) &=& x\times \frac{6}{x} \times 2 + x\times3\times2 + \frac{6}{x}\times 3\times 2\\ + S(x) &=& 6x + 12 + \frac{36}{x} + \end{eqnarray*} + \item Pour trouver cette nouvelle forme, on met chaque élément sur le même dénominateur + \begin{eqnarray*} + S(x) &=& 6x + 12 + \frac{36}{x}\\ + S(x) &=& \frac{6x\times x}{x} + \frac{12\times x}{x} + \frac{36}{x}\\ + S(x) &=& \frac{6x^2 + 12x + 36}{x} + \end{eqnarray*} + \item On retrouve la formule $\frac{u}{v}$ à dériver + \[ + u(x) = 6x^2 + 12x + 36 \Rightarrow u'(x) = 12x + 12 + \] + \[ + v(x) = x \Rightarrow v'(x) = 1 + \] + Donc au numérateur on obtient + \begin{eqnarray*} + u'(x)\times v(x) - u(x)\times v'(x) &=& (12x + 12)\times x - (6x^2 + 12x + 36)\times 1\\ + &=& 6x^2 - 36 + \end{eqnarray*} + Donc + \[ + S'(x) = \frac{6x^2 - 36}{x^2} + \] + \item Tableau de variations de $S$ + + \begin{itemize} + \item Valeur interdite: $x^2 = 0 \equiv x = 0$ + \item Signe de $6x^2 - 36$: c'est un polynôme du 2e degré + \[ + \Delta = 864 > 0 + \] + Il y a donc 2 racines + \[ + x_1 = - 2.4494897427831783 \qquad + x_2 = 2.4494897427831783 + \] + Et on sait que $6x^2 - 36$ est du signe de $a$ donc positif en dehors des racines + \item Le dénominateur $x^2$ est toujours positif. + \item Tableau de variations + + \begin{tikzpicture}[baseline=(a.north)] + \tkzTabInit[lgt=3,espcl=3]{$x$/1,$6x^2 - 36$/1, $x^2$/1, $S'$/1, $S$/2}{$0$, $- 2.4494897427831783$, $10$} + \tkzTabLine{d,-, z, +, } + \tkzTabLine{d,+, , +, } + \tkzTabLine{d,-, z, +, } + \tkzTabVar{D+/ , -/ , +/ } + \end{tikzpicture} + + \end{itemize} + \item On a donc une surface minimal pour $x=2.4494897427831783$ et $h = 14.6969384566990698$. + \end{enumerate} +\end{solution} + +%%% Local Variables: +%%% mode: latex +%%% TeX-master: "master" +%%% End: + +\begin{exercise}[subtitle={Bassin}] + Le tour d'un bassin au niveau du sol présente deux axes de symétrie : l’axe des abscisses et la droite d’équation $x=4$. Il est obtenu par symétrie de la courbe $\mathcal{C}_f$ sur $\intFF{0}{4}$ où $f$ est la fonction définie par + + \[ + f(x) = \left(- x^{2} + 2.7 x - 6.1\right) e^{- x} + 6.1 + \] + On admet que sur $\intFF{0}{4}$ la fonction $f$ est positive. + \begin{enumerate} + \item Sur un repère, tracer l'allure de la courbe $\mathcal{C}_f$, les axes de symétries puis compléter pour dessiner la forme du bassin. + \item Montrer que la fonction $f$ admet comme primitive sur $\R$ la fonction $F$ définie par + \[ + F(x) = 6.1 x + \left( x^{2} - 0.7 x + 5.4\right) e^{- x} + \] + \item Calculer la quantité $\ds \int_0^4 f(x) \; dx$, vous donnerez le résultat sous forme exacte. Interpréter le résultat et reportez cette quantité sur le graphique. + \item On considère que l'échelle de votre graphique est de 1unité pour 15m. Calculer l'aire du bassin. Vous donnerez un résultat arrondi au $m^2$ près. + \end{enumerate} +\end{exercise} + +\begin{solution} + \begin{enumerate} + \item + \begin{tikzpicture}[baseline=(a.north), xscale=1, yscale=0.5] + \tkzInit[xmin=0,xmax=5,xstep=1, + ymin=0,ymax=10,ystep=1] + \tkzGrid + \tkzAxeXY + \tkzFct[domain=0:10,color=red,very thick]% + { (-x**2 + 2.7*x - 6.1)*exp(-x) + 6.1 }; + \end{tikzpicture} + \item Il faut dériver $F(x)$ et vérifier que $F'(x) = f(x)$. + \item $\ds \int_0^4 f(x) \; dx = F(4) - F(0) = \frac{18.6}{e^{4}} + 19.0$ + \item La quantité calculée à la question précédente se retrouve 4fois pour former le bassin. Il faut ensuite prendre en compte l'échelle, comme 1unité de longueur correspond à 15m, une unité d'air correspond à $15\times15 = 225m^2$. Ainsi l'aire du bassin est égale à + \[ + (\frac{18.6}{e^{4}} + 19.0)\times 4 \times 15^2 = 17407.00000 + \] + + \end{enumerate} +\end{solution} + +%%% Local Variables: +%%% mode: latex +%%% TeX-master: "master" +%%% End: + +\begin{exercise}[subtitle={Stylos}] + \emph{Les parties {\rm A} et {\rm B} de cet exercice sont indépendantes.} + + \bigskip + + \begin{minipage}{0.6\linewidth} + \textbf{Partie A} + + \medskip + + Deux ateliers A et B fabriquent des stylos pour une entreprise. + + L'atelier A fabrique 47.0\,\% des stylos, et parmi ceux-là, 45.0\,\% possèdent un défaut de fabrication. + + De plus, 3.0\,\% des stylos possèdent un défaut de fabrication et sortent de l'atelier B. + + Un stylo est prélevé au hasard dans le stock de l'entreprise. + + On considère les évènements suivants: + + \begin{itemize} + \item A : \og Le stylo a été fabriqué par l'atelier A \fg + \item B : \og Le stylo a été fabriqué par l'atelier B \fg + \item D : \og Le stylo possède un défaut de fabrication \fg + \end{itemize} + \end{minipage} + \begin{minipage}{0.4\linewidth} + \begin{center} + \begin{tikzpicture}[sloped] + \node {.} + child {node {$A$} + child {node {$D$} + edge from parent + node[above] {...} + } + child {node {$\overline{D}$} + edge from parent + node[above] {...} + } + edge from parent + node[above] {...} + } + child[missing] {} + child { node {$B$} + child {node {$D$} + edge from parent + node[above] {...} + } + child {node {$\overline{D}$} + edge from parent + node[above] {...} + } + edge from parent + node[above] {...} + } ; + \end{tikzpicture} + \end{center} + \end{minipage} + + \medskip + + \begin{enumerate} + \item Compléter l'arbre de probabilité ci-contre + \item Interpréter puis donner les probabilités $P(A)$, $P(B)$, $P_A(D)$ et $P(B \cap D)$. + + \item + \begin{enumerate} + \item Calculer la probabilité qu'un stylo provienne de l'atelier A et possède un défaut de fabrication. + \item En déduire que la probabilité qu'un stylo possède un défaut de fabrication est de $0.24$. + \end{enumerate} + \item On prélève un stylo au hasard avec un défaut. Quelle est la probabilité qu'il vienne de l'atelier A? + \end{enumerate} + + \bigskip + + \textbf{Partie B} + \medskip + + Dans cette partie, on suppose que 24.0\,\% des stylos possèdent un défaut de fabrication. + + L'entreprise confectionne des paquets contenant chacun $4$~stylos. + + Le fait qu'un stylo possède ou non un défaut de fabrication est indépendant des autres stylos. + + On appelle $X$ la variable aléatoire donnant pour un paquet le nombre de stylos qui possèdent un défaut de fabrication. + + On admet que la variable aléatoire $X$ suit une loi binomiale. + + \medskip + + \begin{enumerate} + \setcounter{enumi}{4} + \item Avec quelle loi peut-on modéliser $X$. Préciser les paramètres. + \item Calculer et interpréter la probabilité $P(X = 9)$. + \item Le directeur de l'entreprise affirme qu'il y a plus d'une chance sur deux qu'un paquet ne comporte aucun stylo défectueux. A-t-il raison ? + \item Combien de stylos peut-on espérer avoir en moyenne? + \end{enumerate} + \pagebreak +\end{exercise} + +\begin{solution} + \begin{enumerate} + \item + \begin{center} + \begin{tikzpicture}[sloped] + \node {.} + child {node {$A$} + child {node {$D$} + edge from parent + node[above] {0.45} + } + child {node {$\overline{D}$} + edge from parent + node[above] {0.55} + } + edge from parent + node[above] {0.47} + } + child[missing] {} + child { node {$B$} + child {node {$D$} + edge from parent + node[above] {0.05} + } + child {node {$\overline{D}$} + edge from parent + node[above] {0.95} + } + edge from parent + node[above] {0.53} + } ; + \end{tikzpicture} + \end{center} + \item + \begin{itemize} + \item Probabilité que le stylo vienne de l'atelier A + \[ + P(A) = 0.47 + \] + \item Probabilité que le stylo vienne de l'atelier B + \[ + P(B) = 0.53 + \] + \item Probabilité que le stylo ait un défaut sachant qu'il vient de l'atelier A. + \[ + P_A(D) = 0.45 + \] + \item Probabilité que le stylo vienne de l'atelier B et qu'il ait un défaut. + \[ + P(D \cap D) = 0.03 + \] + \end{itemize} + \item + \begin{enumerate} + \item Probabilité qu'un stylo vienne de l'atelier A et qu'il ait un defaut + \[ + P(A\cap D) = P(A) \times P_A(D) = 0.47 \times 0.45 = 0.21 + \] + \item Probabilité que le stylo ai un défaut de fabrication. + \[ + P(D) = P(A\cap D) + P(B\cap D) = 0.21 + 0.03 = 0.24 + \] + \end{enumerate} + \item Probabilité qu'il vienne de l'atelier A sachant qu'il a un defaut + \[ + P_D(A) = \frac{P(A\cap D)}{P(D)} = \frac{0.21}{0.24} = 0.88 + \] + \item $X$ peut être modélisée par une loi binomiale de paramètres $n=19$ et $p=0.24$. + \item (\textit{par de correction automatique disponible pour le résultat final} + \[ + P(X = 9) = \coefBino{19}{9}\times 0.24^{9} \times 0.76^{10} + \] + \item (\textit{par de correction automatique disponible pour le résultat final} + + Il faut calculer la probabilité qu'il y ait 0 stylo avec un defaut. + \[ + P(X = 0) = \coefBino{19}{0}\times 0.24^{0} \times 0.76^{19} + \] + Puis comparer ce nombre à 0,5. + \item Il faut calculer l'espérance + \[ + E[X] = n\times p = 19 \times 0.24 = 4.56 + \] + \end{enumerate} +\end{solution} + +\end{document} + +%%% Local Variables: +%%% mode: latex +%%% TeX-master: "master" +%%% End: diff --git a/Complementaire/DM/2105_DM1/corr_16_2105_DM1.tex b/Complementaire/DM/2105_DM1/corr_16_2105_DM1.tex new file mode 100644 index 0000000..f59beea --- /dev/null +++ b/Complementaire/DM/2105_DM1/corr_16_2105_DM1.tex @@ -0,0 +1,367 @@ +\documentclass[a4paper,10pt]{article} +\usepackage{myXsim} + +% Title Page +\title{DM1 \hfill SILVA LOPES Katleen} +\tribe{Maths complémentaire} +\date{\hfillÀ render pour le jeudi 27 mai} + +\xsimsetup{ + solution/print = true +} + +\begin{document} +\maketitle + +Les valeurs des exercices sont générés automatiquement. Si une valeur a un nombre adhérant de chiffres après la virgule, vous pouvez l'arrondir à l'entier le plus proche. + +\begin{exercise}[subtitle={Optimisation de matière}] + \begin{minipage}{0.6\textwidth} + On se propose de fabriquer avec le moins de tôle possible une citerne fermée en forme de parallélépipède rectangle dont le volume intérieur doit être de $27m^3$. La longueur est aussi fixée à $3m$ par le cahier des charges. + + On peut donc faire varier uniquement la largeur (notée $x$) et la hauteur (notée $h$) de la cuve. + \end{minipage} + \hfill + \begin{minipage}{0.3\textwidth} + \begin{tikzpicture} + \pgfmathsetmacro{\cubex}{3} + \pgfmathsetmacro{\cubey}{1} + \pgfmathsetmacro{\cubez}{2} + \draw[black,fill=gray] (0,0,0) -- ++(-\cubex,0,0) -- ++(0,-\cubey,0) node [midway, left] {$h$} -- ++(\cubex,0,0) node [midway, below] {$x$} -- cycle; + \draw[black,fill=gray] (0,0,0) -- ++(0,0,-\cubez) -- ++(0,-\cubey,0) -- ++(0,0,\cubez) node [midway, right] {$3m$} -- cycle; + \draw[black,fill=gray] (0,0,0) -- ++(-\cubex,0,0) -- ++(0,0,-\cubez) -- ++(\cubex,0,0) -- cycle; + \end{tikzpicture} + \end{minipage} + + \begin{enumerate} + \item Expliquer pourquoi quand la largeur $x$ change, la hauteur $h$ doit elle aussi changer pour respecter les contraintes. + \item Démontrer que l'on doit avoir $h = \dfrac{9}{x}$. + \item On note $S(x)$ l'aire totale de la citerne (c'est à dire la somme des aires des six faces). Montrer que l'on peut écrire + \[ + S(x) = 6x + 18 + \frac{54}{x} + \] + \item Démontrer que + \[ + S(x) = \frac{6x^2 + 18x + 54}{x} + \] + \item Démontrer que + \[ + S'(x) = \frac{6x^2 - 54}{x^2} + \] + \item En déduire le tableau de variation de $S(x)$ sur $\intOF{0}{10}$. + \item Déterminer les valeurs de $x$ et $h$ correspondant à une utilisation minimal de tôle. + \end{enumerate} +\end{exercise} + +\begin{solution} + \begin{enumerate} + \item Le volume étant fixe si l'on fait varier $x$, $h$ doit aussi varier. + \begin{itemize} + \item Si $x = 2$ alors conserver un volume de $V=27$, $h$ doit être égale à $9 / 2$ + \item Si $x = 3$ alors conserver un volume de $V=27$, $h$ doit être égale à $9 / 3$ + \end{itemize} + \item Pour calculer le volume, on a + \begin{eqnarray*} + V &=& h\times x \times 3 \\ + 27 &=& h\times x \times 3 \\ + x &=& \frac{27}{h\times 3} = \frac{9}{h} + \end{eqnarray*} + \item Pour calculer la surface totale, on ajoute la surface de chaque face. On a donc le calcul suivant + \begin{eqnarray*} + S(x) &=& x\times h \times 2 + x\times3\times2 + h\times 3\times 2\\ + S(x) &=& x\times \frac{9}{x} \times 2 + x\times3\times2 + \frac{9}{x}\times 3\times 2\\ + S(x) &=& 6x + 18 + \frac{54}{x} + \end{eqnarray*} + \item Pour trouver cette nouvelle forme, on met chaque élément sur le même dénominateur + \begin{eqnarray*} + S(x) &=& 6x + 18 + \frac{54}{x}\\ + S(x) &=& \frac{6x\times x}{x} + \frac{18\times x}{x} + \frac{54}{x}\\ + S(x) &=& \frac{6x^2 + 18x + 54}{x} + \end{eqnarray*} + \item On retrouve la formule $\frac{u}{v}$ à dériver + \[ + u(x) = 6x^2 + 18x + 54 \Rightarrow u'(x) = 12x + 18 + \] + \[ + v(x) = x \Rightarrow v'(x) = 1 + \] + Donc au numérateur on obtient + \begin{eqnarray*} + u'(x)\times v(x) - u(x)\times v'(x) &=& (12x + 18)\times x - (6x^2 + 18x + 54)\times 1\\ + &=& 6x^2 - 54 + \end{eqnarray*} + Donc + \[ + S'(x) = \frac{6x^2 - 54}{x^2} + \] + \item Tableau de variations de $S$ + + \begin{itemize} + \item Valeur interdite: $x^2 = 0 \equiv x = 0$ + \item Signe de $6x^2 - 54$: c'est un polynôme du 2e degré + \[ + \Delta = 1296 > 0 + \] + Il y a donc 2 racines + \[ + x_1 = - 3 \qquad + x_2 = 3 + \] + Et on sait que $6x^2 - 54$ est du signe de $a$ donc positif en dehors des racines + \item Le dénominateur $x^2$ est toujours positif. + \item Tableau de variations + + \begin{tikzpicture}[baseline=(a.north)] + \tkzTabInit[lgt=3,espcl=3]{$x$/1,$6x^2 - 54$/1, $x^2$/1, $S'$/1, $S$/2}{$0$, $- 3$, $10$} + \tkzTabLine{d,-, z, +, } + \tkzTabLine{d,+, , +, } + \tkzTabLine{d,-, z, +, } + \tkzTabVar{D+/ , -/ , +/ } + \end{tikzpicture} + + \end{itemize} + \item On a donc une surface minimal pour $x=3$ et $h = 27$. + \end{enumerate} +\end{solution} + +%%% Local Variables: +%%% mode: latex +%%% TeX-master: "master" +%%% End: + +\begin{exercise}[subtitle={Bassin}] + Le tour d'un bassin au niveau du sol présente deux axes de symétrie : l’axe des abscisses et la droite d’équation $x=4$. Il est obtenu par symétrie de la courbe $\mathcal{C}_f$ sur $\intFF{0}{4}$ où $f$ est la fonction définie par + + \[ + f(x) = \left(- x^{2} + 3.0 x - 6.6\right) e^{- x} + 6.6 + \] + On admet que sur $\intFF{0}{4}$ la fonction $f$ est positive. + \begin{enumerate} + \item Sur un repère, tracer l'allure de la courbe $\mathcal{C}_f$, les axes de symétries puis compléter pour dessiner la forme du bassin. + \item Montrer que la fonction $f$ admet comme primitive sur $\R$ la fonction $F$ définie par + \[ + F(x) = 6.6 x + \left( x^{2} - x + 5.6\right) e^{- x} + \] + \item Calculer la quantité $\ds \int_0^4 f(x) \; dx$, vous donnerez le résultat sous forme exacte. Interpréter le résultat et reportez cette quantité sur le graphique. + \item On considère que l'échelle de votre graphique est de 1unité pour 15m. Calculer l'aire du bassin. Vous donnerez un résultat arrondi au $m^2$ près. + \end{enumerate} +\end{exercise} + +\begin{solution} + \begin{enumerate} + \item + \begin{tikzpicture}[baseline=(a.north), xscale=1, yscale=0.5] + \tkzInit[xmin=0,xmax=5,xstep=1, + ymin=0,ymax=10,ystep=1] + \tkzGrid + \tkzAxeXY + \tkzFct[domain=0:10,color=red,very thick]% + { (-x**2 + 3.0*x - 6.6)*exp(-x) + 6.6 }; + \end{tikzpicture} + \item Il faut dériver $F(x)$ et vérifier que $F'(x) = f(x)$. + \item $\ds \int_0^4 f(x) \; dx = F(4) - F(0) = \frac{17.6}{e^{4}} + 20.8$ + \item La quantité calculée à la question précédente se retrouve 4fois pour former le bassin. Il faut ensuite prendre en compte l'échelle, comme 1unité de longueur correspond à 15m, une unité d'air correspond à $15\times15 = 225m^2$. Ainsi l'aire du bassin est égale à + \[ + (\frac{17.6}{e^{4}} + 20.8)\times 4 \times 15^2 = 19010.00000 + \] + + \end{enumerate} +\end{solution} + +%%% Local Variables: +%%% mode: latex +%%% TeX-master: "master" +%%% End: + +\begin{exercise}[subtitle={Stylos}] + \emph{Les parties {\rm A} et {\rm B} de cet exercice sont indépendantes.} + + \bigskip + + \begin{minipage}{0.6\linewidth} + \textbf{Partie A} + + \medskip + + Deux ateliers A et B fabriquent des stylos pour une entreprise. + + L'atelier A fabrique 43.0\,\% des stylos, et parmi ceux-là, 31.0\,\% possèdent un défaut de fabrication. + + De plus, 19.0\,\% des stylos possèdent un défaut de fabrication et sortent de l'atelier B. + + Un stylo est prélevé au hasard dans le stock de l'entreprise. + + On considère les évènements suivants: + + \begin{itemize} + \item A : \og Le stylo a été fabriqué par l'atelier A \fg + \item B : \og Le stylo a été fabriqué par l'atelier B \fg + \item D : \og Le stylo possède un défaut de fabrication \fg + \end{itemize} + \end{minipage} + \begin{minipage}{0.4\linewidth} + \begin{center} + \begin{tikzpicture}[sloped] + \node {.} + child {node {$A$} + child {node {$D$} + edge from parent + node[above] {...} + } + child {node {$\overline{D}$} + edge from parent + node[above] {...} + } + edge from parent + node[above] {...} + } + child[missing] {} + child { node {$B$} + child {node {$D$} + edge from parent + node[above] {...} + } + child {node {$\overline{D}$} + edge from parent + node[above] {...} + } + edge from parent + node[above] {...} + } ; + \end{tikzpicture} + \end{center} + \end{minipage} + + \medskip + + \begin{enumerate} + \item Compléter l'arbre de probabilité ci-contre + \item Interpréter puis donner les probabilités $P(A)$, $P(B)$, $P_A(D)$ et $P(B \cap D)$. + + \item + \begin{enumerate} + \item Calculer la probabilité qu'un stylo provienne de l'atelier A et possède un défaut de fabrication. + \item En déduire que la probabilité qu'un stylo possède un défaut de fabrication est de $0.32$. + \end{enumerate} + \item On prélève un stylo au hasard avec un défaut. Quelle est la probabilité qu'il vienne de l'atelier A? + \end{enumerate} + + \bigskip + + \textbf{Partie B} + \medskip + + Dans cette partie, on suppose que 32.0\,\% des stylos possèdent un défaut de fabrication. + + L'entreprise confectionne des paquets contenant chacun $4$~stylos. + + Le fait qu'un stylo possède ou non un défaut de fabrication est indépendant des autres stylos. + + On appelle $X$ la variable aléatoire donnant pour un paquet le nombre de stylos qui possèdent un défaut de fabrication. + + On admet que la variable aléatoire $X$ suit une loi binomiale. + + \medskip + + \begin{enumerate} + \setcounter{enumi}{4} + \item Avec quelle loi peut-on modéliser $X$. Préciser les paramètres. + \item Calculer et interpréter la probabilité $P(X = 7)$. + \item Le directeur de l'entreprise affirme qu'il y a plus d'une chance sur deux qu'un paquet ne comporte aucun stylo défectueux. A-t-il raison ? + \item Combien de stylos peut-on espérer avoir en moyenne? + \end{enumerate} + \pagebreak +\end{exercise} + +\begin{solution} + \begin{enumerate} + \item + \begin{center} + \begin{tikzpicture}[sloped] + \node {.} + child {node {$A$} + child {node {$D$} + edge from parent + node[above] {0.31} + } + child {node {$\overline{D}$} + edge from parent + node[above] {0.69} + } + edge from parent + node[above] {0.43} + } + child[missing] {} + child { node {$B$} + child {node {$D$} + edge from parent + node[above] {0.34} + } + child {node {$\overline{D}$} + edge from parent + node[above] {0.66} + } + edge from parent + node[above] {0.57} + } ; + \end{tikzpicture} + \end{center} + \item + \begin{itemize} + \item Probabilité que le stylo vienne de l'atelier A + \[ + P(A) = 0.43 + \] + \item Probabilité que le stylo vienne de l'atelier B + \[ + P(B) = 0.57 + \] + \item Probabilité que le stylo ait un défaut sachant qu'il vient de l'atelier A. + \[ + P_A(D) = 0.31 + \] + \item Probabilité que le stylo vienne de l'atelier B et qu'il ait un défaut. + \[ + P(D \cap D) = 0.19 + \] + \end{itemize} + \item + \begin{enumerate} + \item Probabilité qu'un stylo vienne de l'atelier A et qu'il ait un defaut + \[ + P(A\cap D) = P(A) \times P_A(D) = 0.43 \times 0.31 = 0.13 + \] + \item Probabilité que le stylo ai un défaut de fabrication. + \[ + P(D) = P(A\cap D) + P(B\cap D) = 0.13 + 0.19 = 0.32 + \] + \end{enumerate} + \item Probabilité qu'il vienne de l'atelier A sachant qu'il a un defaut + \[ + P_D(A) = \frac{P(A\cap D)}{P(D)} = \frac{0.13}{0.32} = 0.41 + \] + \item $X$ peut être modélisée par une loi binomiale de paramètres $n=10$ et $p=0.32$. + \item (\textit{par de correction automatique disponible pour le résultat final} + \[ + P(X = 7) = \coefBino{10}{7}\times 0.32^{7} \times 0.68^{3} + \] + \item (\textit{par de correction automatique disponible pour le résultat final} + + Il faut calculer la probabilité qu'il y ait 0 stylo avec un defaut. + \[ + P(X = 0) = \coefBino{10}{0}\times 0.32^{0} \times 0.68^{10} + \] + Puis comparer ce nombre à 0,5. + \item Il faut calculer l'espérance + \[ + E[X] = n\times p = 10 \times 0.32 = 3.2 + \] + \end{enumerate} +\end{solution} + +\end{document} + +%%% Local Variables: +%%% mode: latex +%%% TeX-master: "master" +%%% End: diff --git a/Complementaire/DM/2105_DM1/corr_17_2105_DM1.tex b/Complementaire/DM/2105_DM1/corr_17_2105_DM1.tex new file mode 100644 index 0000000..30569d8 --- /dev/null +++ b/Complementaire/DM/2105_DM1/corr_17_2105_DM1.tex @@ -0,0 +1,367 @@ +\documentclass[a4paper,10pt]{article} +\usepackage{myXsim} + +% Title Page +\title{DM1 \hfill VANDROUX Guillemette} +\tribe{Maths complémentaire} +\date{\hfillÀ render pour le jeudi 27 mai} + +\xsimsetup{ + solution/print = true +} + +\begin{document} +\maketitle + +Les valeurs des exercices sont générés automatiquement. Si une valeur a un nombre adhérant de chiffres après la virgule, vous pouvez l'arrondir à l'entier le plus proche. + +\begin{exercise}[subtitle={Optimisation de matière}] + \begin{minipage}{0.6\textwidth} + On se propose de fabriquer avec le moins de tôle possible une citerne fermée en forme de parallélépipède rectangle dont le volume intérieur doit être de $30m^3$. La longueur est aussi fixée à $3m$ par le cahier des charges. + + On peut donc faire varier uniquement la largeur (notée $x$) et la hauteur (notée $h$) de la cuve. + \end{minipage} + \hfill + \begin{minipage}{0.3\textwidth} + \begin{tikzpicture} + \pgfmathsetmacro{\cubex}{3} + \pgfmathsetmacro{\cubey}{1} + \pgfmathsetmacro{\cubez}{2} + \draw[black,fill=gray] (0,0,0) -- ++(-\cubex,0,0) -- ++(0,-\cubey,0) node [midway, left] {$h$} -- ++(\cubex,0,0) node [midway, below] {$x$} -- cycle; + \draw[black,fill=gray] (0,0,0) -- ++(0,0,-\cubez) -- ++(0,-\cubey,0) -- ++(0,0,\cubez) node [midway, right] {$3m$} -- cycle; + \draw[black,fill=gray] (0,0,0) -- ++(-\cubex,0,0) -- ++(0,0,-\cubez) -- ++(\cubex,0,0) -- cycle; + \end{tikzpicture} + \end{minipage} + + \begin{enumerate} + \item Expliquer pourquoi quand la largeur $x$ change, la hauteur $h$ doit elle aussi changer pour respecter les contraintes. + \item Démontrer que l'on doit avoir $h = \dfrac{10}{x}$. + \item On note $S(x)$ l'aire totale de la citerne (c'est à dire la somme des aires des six faces). Montrer que l'on peut écrire + \[ + S(x) = 6x + 20 + \frac{60}{x} + \] + \item Démontrer que + \[ + S(x) = \frac{6x^2 + 20x + 60}{x} + \] + \item Démontrer que + \[ + S'(x) = \frac{6x^2 - 60}{x^2} + \] + \item En déduire le tableau de variation de $S(x)$ sur $\intOF{0}{10}$. + \item Déterminer les valeurs de $x$ et $h$ correspondant à une utilisation minimal de tôle. + \end{enumerate} +\end{exercise} + +\begin{solution} + \begin{enumerate} + \item Le volume étant fixe si l'on fait varier $x$, $h$ doit aussi varier. + \begin{itemize} + \item Si $x = 2$ alors conserver un volume de $V=30$, $h$ doit être égale à $10 / 2$ + \item Si $x = 3$ alors conserver un volume de $V=30$, $h$ doit être égale à $10 / 3$ + \end{itemize} + \item Pour calculer le volume, on a + \begin{eqnarray*} + V &=& h\times x \times 3 \\ + 30 &=& h\times x \times 3 \\ + x &=& \frac{30}{h\times 3} = \frac{10}{h} + \end{eqnarray*} + \item Pour calculer la surface totale, on ajoute la surface de chaque face. On a donc le calcul suivant + \begin{eqnarray*} + S(x) &=& x\times h \times 2 + x\times3\times2 + h\times 3\times 2\\ + S(x) &=& x\times \frac{10}{x} \times 2 + x\times3\times2 + \frac{10}{x}\times 3\times 2\\ + S(x) &=& 6x + 20 + \frac{60}{x} + \end{eqnarray*} + \item Pour trouver cette nouvelle forme, on met chaque élément sur le même dénominateur + \begin{eqnarray*} + S(x) &=& 6x + 20 + \frac{60}{x}\\ + S(x) &=& \frac{6x\times x}{x} + \frac{20\times x}{x} + \frac{60}{x}\\ + S(x) &=& \frac{6x^2 + 20x + 60}{x} + \end{eqnarray*} + \item On retrouve la formule $\frac{u}{v}$ à dériver + \[ + u(x) = 6x^2 + 20x + 60 \Rightarrow u'(x) = 12x + 20 + \] + \[ + v(x) = x \Rightarrow v'(x) = 1 + \] + Donc au numérateur on obtient + \begin{eqnarray*} + u'(x)\times v(x) - u(x)\times v'(x) &=& (12x + 20)\times x - (6x^2 + 20x + 60)\times 1\\ + &=& 6x^2 - 60 + \end{eqnarray*} + Donc + \[ + S'(x) = \frac{6x^2 - 60}{x^2} + \] + \item Tableau de variations de $S$ + + \begin{itemize} + \item Valeur interdite: $x^2 = 0 \equiv x = 0$ + \item Signe de $6x^2 - 60$: c'est un polynôme du 2e degré + \[ + \Delta = 1440 > 0 + \] + Il y a donc 2 racines + \[ + x_1 = - 3.1622776601683795 \qquad + x_2 = 3.1622776601683795 + \] + Et on sait que $6x^2 - 60$ est du signe de $a$ donc positif en dehors des racines + \item Le dénominateur $x^2$ est toujours positif. + \item Tableau de variations + + \begin{tikzpicture}[baseline=(a.north)] + \tkzTabInit[lgt=3,espcl=3]{$x$/1,$6x^2 - 60$/1, $x^2$/1, $S'$/1, $S$/2}{$0$, $- 3.1622776601683795$, $10$} + \tkzTabLine{d,-, z, +, } + \tkzTabLine{d,+, , +, } + \tkzTabLine{d,-, z, +, } + \tkzTabVar{D+/ , -/ , +/ } + \end{tikzpicture} + + \end{itemize} + \item On a donc une surface minimal pour $x=3.1622776601683795$ et $h = 31.6227766016837950$. + \end{enumerate} +\end{solution} + +%%% Local Variables: +%%% mode: latex +%%% TeX-master: "master" +%%% End: + +\begin{exercise}[subtitle={Bassin}] + Le tour d'un bassin au niveau du sol présente deux axes de symétrie : l’axe des abscisses et la droite d’équation $x=4$. Il est obtenu par symétrie de la courbe $\mathcal{C}_f$ sur $\intFF{0}{4}$ où $f$ est la fonction définie par + + \[ + f(x) = \left(- x^{2} + 5.0 x - 1.8\right) e^{- x} + 1.8 + \] + On admet que sur $\intFF{0}{4}$ la fonction $f$ est positive. + \begin{enumerate} + \item Sur un repère, tracer l'allure de la courbe $\mathcal{C}_f$, les axes de symétries puis compléter pour dessiner la forme du bassin. + \item Montrer que la fonction $f$ admet comme primitive sur $\R$ la fonction $F$ définie par + \[ + F(x) = 1.8 x + \left( x^{2} - 3.0 x - 1.2\right) e^{- x} + \] + \item Calculer la quantité $\ds \int_0^4 f(x) \; dx$, vous donnerez le résultat sous forme exacte. Interpréter le résultat et reportez cette quantité sur le graphique. + \item On considère que l'échelle de votre graphique est de 1unité pour 15m. Calculer l'aire du bassin. Vous donnerez un résultat arrondi au $m^2$ près. + \end{enumerate} +\end{exercise} + +\begin{solution} + \begin{enumerate} + \item + \begin{tikzpicture}[baseline=(a.north), xscale=1, yscale=0.5] + \tkzInit[xmin=0,xmax=5,xstep=1, + ymin=0,ymax=10,ystep=1] + \tkzGrid + \tkzAxeXY + \tkzFct[domain=0:10,color=red,very thick]% + { (-x**2 + 5.0*x - 1.8)*exp(-x) + 1.8 }; + \end{tikzpicture} + \item Il faut dériver $F(x)$ et vérifier que $F'(x) = f(x)$. + \item $\ds \int_0^4 f(x) \; dx = F(4) - F(0) = \frac{2.8}{e^{4}} + 8.4$ + \item La quantité calculée à la question précédente se retrouve 4fois pour former le bassin. Il faut ensuite prendre en compte l'échelle, comme 1unité de longueur correspond à 15m, une unité d'air correspond à $15\times15 = 225m^2$. Ainsi l'aire du bassin est égale à + \[ + (\frac{2.8}{e^{4}} + 8.4)\times 4 \times 15^2 = 7606.000000 + \] + + \end{enumerate} +\end{solution} + +%%% Local Variables: +%%% mode: latex +%%% TeX-master: "master" +%%% End: + +\begin{exercise}[subtitle={Stylos}] + \emph{Les parties {\rm A} et {\rm B} de cet exercice sont indépendantes.} + + \bigskip + + \begin{minipage}{0.6\linewidth} + \textbf{Partie A} + + \medskip + + Deux ateliers A et B fabriquent des stylos pour une entreprise. + + L'atelier A fabrique 55.00000000000001\,\% des stylos, et parmi ceux-là, 46.0\,\% possèdent un défaut de fabrication. + + De plus, 20.0\,\% des stylos possèdent un défaut de fabrication et sortent de l'atelier B. + + Un stylo est prélevé au hasard dans le stock de l'entreprise. + + On considère les évènements suivants: + + \begin{itemize} + \item A : \og Le stylo a été fabriqué par l'atelier A \fg + \item B : \og Le stylo a été fabriqué par l'atelier B \fg + \item D : \og Le stylo possède un défaut de fabrication \fg + \end{itemize} + \end{minipage} + \begin{minipage}{0.4\linewidth} + \begin{center} + \begin{tikzpicture}[sloped] + \node {.} + child {node {$A$} + child {node {$D$} + edge from parent + node[above] {...} + } + child {node {$\overline{D}$} + edge from parent + node[above] {...} + } + edge from parent + node[above] {...} + } + child[missing] {} + child { node {$B$} + child {node {$D$} + edge from parent + node[above] {...} + } + child {node {$\overline{D}$} + edge from parent + node[above] {...} + } + edge from parent + node[above] {...} + } ; + \end{tikzpicture} + \end{center} + \end{minipage} + + \medskip + + \begin{enumerate} + \item Compléter l'arbre de probabilité ci-contre + \item Interpréter puis donner les probabilités $P(A)$, $P(B)$, $P_A(D)$ et $P(B \cap D)$. + + \item + \begin{enumerate} + \item Calculer la probabilité qu'un stylo provienne de l'atelier A et possède un défaut de fabrication. + \item En déduire que la probabilité qu'un stylo possède un défaut de fabrication est de $0.45$. + \end{enumerate} + \item On prélève un stylo au hasard avec un défaut. Quelle est la probabilité qu'il vienne de l'atelier A? + \end{enumerate} + + \bigskip + + \textbf{Partie B} + \medskip + + Dans cette partie, on suppose que 45.0\,\% des stylos possèdent un défaut de fabrication. + + L'entreprise confectionne des paquets contenant chacun $4$~stylos. + + Le fait qu'un stylo possède ou non un défaut de fabrication est indépendant des autres stylos. + + On appelle $X$ la variable aléatoire donnant pour un paquet le nombre de stylos qui possèdent un défaut de fabrication. + + On admet que la variable aléatoire $X$ suit une loi binomiale. + + \medskip + + \begin{enumerate} + \setcounter{enumi}{4} + \item Avec quelle loi peut-on modéliser $X$. Préciser les paramètres. + \item Calculer et interpréter la probabilité $P(X = 13)$. + \item Le directeur de l'entreprise affirme qu'il y a plus d'une chance sur deux qu'un paquet ne comporte aucun stylo défectueux. A-t-il raison ? + \item Combien de stylos peut-on espérer avoir en moyenne? + \end{enumerate} + \pagebreak +\end{exercise} + +\begin{solution} + \begin{enumerate} + \item + \begin{center} + \begin{tikzpicture}[sloped] + \node {.} + child {node {$A$} + child {node {$D$} + edge from parent + node[above] {0.46} + } + child {node {$\overline{D}$} + edge from parent + node[above] {0.54} + } + edge from parent + node[above] {0.55} + } + child[missing] {} + child { node {$B$} + child {node {$D$} + edge from parent + node[above] {0.44} + } + child {node {$\overline{D}$} + edge from parent + node[above] {0.56} + } + edge from parent + node[above] {0.45} + } ; + \end{tikzpicture} + \end{center} + \item + \begin{itemize} + \item Probabilité que le stylo vienne de l'atelier A + \[ + P(A) = 0.55 + \] + \item Probabilité que le stylo vienne de l'atelier B + \[ + P(B) = 0.45 + \] + \item Probabilité que le stylo ait un défaut sachant qu'il vient de l'atelier A. + \[ + P_A(D) = 0.46 + \] + \item Probabilité que le stylo vienne de l'atelier B et qu'il ait un défaut. + \[ + P(D \cap D) = 0.2 + \] + \end{itemize} + \item + \begin{enumerate} + \item Probabilité qu'un stylo vienne de l'atelier A et qu'il ait un defaut + \[ + P(A\cap D) = P(A) \times P_A(D) = 0.55 \times 0.46 = 0.25 + \] + \item Probabilité que le stylo ai un défaut de fabrication. + \[ + P(D) = P(A\cap D) + P(B\cap D) = 0.25 + 0.2 = 0.45 + \] + \end{enumerate} + \item Probabilité qu'il vienne de l'atelier A sachant qu'il a un defaut + \[ + P_D(A) = \frac{P(A\cap D)}{P(D)} = \frac{0.25}{0.45} = 0.56 + \] + \item $X$ peut être modélisée par une loi binomiale de paramètres $n=15$ et $p=0.45$. + \item (\textit{par de correction automatique disponible pour le résultat final} + \[ + P(X = 13) = \coefBino{15}{13}\times 0.45^{13} \times 0.55^{2} + \] + \item (\textit{par de correction automatique disponible pour le résultat final} + + Il faut calculer la probabilité qu'il y ait 0 stylo avec un defaut. + \[ + P(X = 0) = \coefBino{15}{0}\times 0.45^{0} \times 0.55^{15} + \] + Puis comparer ce nombre à 0,5. + \item Il faut calculer l'espérance + \[ + E[X] = n\times p = 15 \times 0.45 = 6.75 + \] + \end{enumerate} +\end{solution} + +\end{document} + +%%% Local Variables: +%%% mode: latex +%%% TeX-master: "master" +%%% End: diff --git a/Complementaire/DM/2105_DM1/corr_all_2105_DM1.pdf b/Complementaire/DM/2105_DM1/corr_all_2105_DM1.pdf new file mode 100644 index 0000000..1309797 Binary files /dev/null and b/Complementaire/DM/2105_DM1/corr_all_2105_DM1.pdf differ diff --git a/Complementaire/DM/2105_DM1/tpl_2105_DM1.tex b/Complementaire/DM/2105_DM1/tpl_2105_DM1.tex index 41a8da8..908ecb9 100644 --- a/Complementaire/DM/2105_DM1/tpl_2105_DM1.tex +++ b/Complementaire/DM/2105_DM1/tpl_2105_DM1.tex @@ -13,8 +13,11 @@ \begin{document} \maketitle +Les valeurs des exercices sont générés automatiquement. Si une valeur a un nombre adhérant de chiffres après la virgule, vous pouvez l'arrondir à l'entier le plus proche. + \Block{include "./tpl_optimisation.tex"} \Block{include "./tpl_bassin.tex"} +\Block{include "./tpl_stylos.tex"} \end{document} diff --git a/Complementaire/DM/2105_DM1/tpl_bassin.tex b/Complementaire/DM/2105_DM1/tpl_bassin.tex index 1b304d2..f1c758a 100644 --- a/Complementaire/DM/2105_DM1/tpl_bassin.tex +++ b/Complementaire/DM/2105_DM1/tpl_bassin.tex @@ -1,50 +1,49 @@ +%- set latex = sympy.latex +%- set sqrt = sympy.sqrt +%- set exp = sympy.functions.exp +%- set integrate = sympy.integrate + \begin{exercise}[subtitle={Bassin}] - %- set Vinit = randint(1, 10)*100000 - %- set tx = round((random()+1)/2, 1) - Le clinker est un constituant du ciment qui résulte de la cuisson d'un mélange composé de calcaire et d'argile. La fabrication du clinker nécessite des fours à très haute température qui libèrent dans l'air une grande quantité de dioxyde de carbone (CO$_2$). + %- set a = round(random()*10, 1) + %- set b = round(random()*10, 1) + %- set x = sympy.symbols("x") + %- set f = -(x**2 - a*x + b)*exp(-x) + b + %- set F = integrate(f, x) + Le tour d'un bassin au niveau du sol présente deux axes de symétrie : l’axe des abscisses et la droite d’équation $x=4$. Il est obtenu par symétrie de la courbe $\mathcal{C}_f$ sur $\intFF{0}{4}$ où $f$ est la fonction définie par - Dans une cimenterie, la fabrication du clinker s'effectue de 7 h 30 à 20 h, dans une pièce de volume \np{\Var{Vinit}}~dm$^3$. - - À 20 h, après une journée de travail, le taux volumique de CO$_2$ dans la pièce est de \Var{tx}\,\%. + \[ + f(x) = \Var{latex(f)} + \] + On admet que sur $\intFF{0}{4}$ la fonction $f$ est positive. \begin{enumerate} - %- set v20 = int(Vinit*tx/100) - \item Justifier que le volume de CO$_2$ présent dans cette pièce à 20 h est de \np{\Var{v20}}~dm$^3$ . - %- set q = round(random()/10, 2) - %- set c = randint(20, 60)*10 - %- set v0 = int(v20 - c) - %- set t = sympy.symbols("t") - %- set V = v0*exp(- q*t) + c - %- set Vp = V.diff() - \item On modélise le volume de CO$_2$ présent dans la pièce par une fonction du temps $t$ écoulé après 20h (exprimé en minutes) qui pour formule $V(t) = V_0e^{-\Var{q}t} + \Var{c}$ - \begin{enumerate} - \item Démontrer que $V_0$ est égale à \np{\Var{v0}}. - %- set decal = randint(1, 4) - \item Quel sera, au dm$^3$ près, le volume de CO$_2$ dans cette pièce à \Var{20+decal} h ? - \item Démontrer que $V'(t) = \Var{latex(Vp)}$. - \item Étudier le signe de $V'(t)$ puis en déduire le sens de variation de $V(t)$. - \item Que peut-on dire du volume de CO$_2$ quand $t$ devient grand? - \end{enumerate} + \item Sur un repère, tracer l'allure de la courbe $\mathcal{C}_f$, les axes de symétries puis compléter pour dessiner la forme du bassin. + \item Montrer que la fonction $f$ admet comme primitive sur $\R$ la fonction $F$ définie par + \[ + F(x) = \Var{latex(F) | replace("1.0", "")} + \] + \item Calculer la quantité $\ds \int_0^4 f(x) \; dx$, vous donnerez le résultat sous forme exacte. Interpréter le résultat et reportez cette quantité sur le graphique. + \item On considère que l'échelle de votre graphique est de 1unité pour 15m. Calculer l'aire du bassin. Vous donnerez un résultat arrondi au $m^2$ près. \end{enumerate} \end{exercise} \begin{solution} \begin{enumerate} - \item Volume à 20h: $\Var{Vinit}\times \Var{tx/100} = \Var{v20}$ \item - \begin{enumerate} - \item $t=0$ correspond à 20h. - - Donc $V(0) = \Var{v20} = V_0e^{-\Var{q}\times 0} + \Var{c} = V_0 + \Var{c}$ - - Donc $V_0 = \Var{v20} - \Var{c} = \Var{v0}$ - \item Il faut calculer $V(t)$ pour $t = \Var{decal}$ donc - \[ - V(\Var{decal}) = \Var{round(V.subs(t, str(decal)), 2)} - \] - \item Pas de correction pour cette question. - \item Pas de correction pour cette question. - \item Pas de correction pour cette question. - \end{enumerate} + \begin{tikzpicture}[baseline=(a.north), xscale=1, yscale=0.5] + \tkzInit[xmin=0,xmax=5,xstep=1, + ymin=0,ymax=10,ystep=1] + \tkzGrid + \tkzAxeXY + \tkzFct[domain=0:10,color=red,very thick]% + { \Var{f} }; + \end{tikzpicture} + \item Il faut dériver $F(x)$ et vérifier que $F'(x) = f(x)$. + %- set surf = integrate(f, (x, 0, 4)) + \item $\ds \int_0^4 f(x) \; dx = F(4) - F(0) = \Var{latex(surf)}$ + \item La quantité calculée à la question précédente se retrouve 4fois pour former le bassin. Il faut ensuite prendre en compte l'échelle, comme 1unité de longueur correspond à 15m, une unité d'air correspond à $15\times15 = 225m^2$. Ainsi l'aire du bassin est égale à + \[ + (\Var{latex(surf)})\times 4 \times 15^2 = \Var{round(sympy.N(surf*4*15**2, 10), 0)} + \] \end{enumerate} \end{solution} diff --git a/Complementaire/DM/2105_DM1/tpl_stylos.tex b/Complementaire/DM/2105_DM1/tpl_stylos.tex new file mode 100644 index 0000000..2b14ea1 --- /dev/null +++ b/Complementaire/DM/2105_DM1/tpl_stylos.tex @@ -0,0 +1,195 @@ +\begin{exercise}[subtitle={Stylos}] + %- set pA = round(random(), 2) + %- set pB = round(1 - pA, 2) + %- set pD_A = round(random(), 2) + %- set pD_B = round(random(), 2) + %- set pDB = round(pB*pD_B, 2) + %- set pDA = round(pA*pD_A, 2) + %- set pD = round(pDA + pDB, 2) + \emph{Les parties {\rm A} et {\rm B} de cet exercice sont indépendantes.} + + \bigskip + + \begin{minipage}{0.6\linewidth} + \textbf{Partie A} + + \medskip + + Deux ateliers A et B fabriquent des stylos pour une entreprise. + + L'atelier A fabrique \Var{pA*100 | round(2)}\,\% des stylos, et parmi ceux-là, \Var{pD_A*100 | round(2)}\,\% possèdent un défaut de fabrication. + + De plus, \Var{pDB*100 | round(2)}\,\% des stylos possèdent un défaut de fabrication et sortent de l'atelier B. + + Un stylo est prélevé au hasard dans le stock de l'entreprise. + + On considère les évènements suivants: + + \begin{itemize} + \item A : \og Le stylo a été fabriqué par l'atelier A \fg + \item B : \og Le stylo a été fabriqué par l'atelier B \fg + \item D : \og Le stylo possède un défaut de fabrication \fg + \end{itemize} + \end{minipage} + \begin{minipage}{0.4\linewidth} + \begin{center} + \begin{tikzpicture}[sloped] + \node {.} + child {node {$A$} + child {node {$D$} + edge from parent + node[above] {...} + } + child {node {$\overline{D}$} + edge from parent + node[above] {...} + } + edge from parent + node[above] {...} + } + child[missing] {} + child { node {$B$} + child {node {$D$} + edge from parent + node[above] {...} + } + child {node {$\overline{D}$} + edge from parent + node[above] {...} + } + edge from parent + node[above] {...} + } ; + \end{tikzpicture} + \end{center} + \end{minipage} + + \medskip + + \begin{enumerate} + \item Compléter l'arbre de probabilité ci-contre + \item Interpréter puis donner les probabilités $P(A)$, $P(B)$, $P_A(D)$ et $P(B \cap D)$. + + \item + \begin{enumerate} + \item Calculer la probabilité qu'un stylo provienne de l'atelier A et possède un défaut de fabrication. + \item En déduire que la probabilité qu'un stylo possède un défaut de fabrication est de $\Var{pD}$. + \end{enumerate} + \item On prélève un stylo au hasard avec un défaut. Quelle est la probabilité qu'il vienne de l'atelier A? + \end{enumerate} + + \bigskip + + \textbf{Partie B} + %- set nbr = randint(10, 20) + %- set k = randint(int(nbr/2), nbr) + + \medskip + + Dans cette partie, on suppose que \Var{pD*100 | round(2)}\,\% des stylos possèdent un défaut de fabrication. + + L'entreprise confectionne des paquets contenant chacun $4$~stylos. + + Le fait qu'un stylo possède ou non un défaut de fabrication est indépendant des autres stylos. + + On appelle $X$ la variable aléatoire donnant pour un paquet le nombre de stylos qui possèdent un défaut de fabrication. + + On admet que la variable aléatoire $X$ suit une loi binomiale. + + \medskip + + \begin{enumerate} + \setcounter{enumi}{4} + \item Avec quelle loi peut-on modéliser $X$. Préciser les paramètres. + \item Calculer et interpréter la probabilité $P(X = \Var{k})$. + \item Le directeur de l'entreprise affirme qu'il y a plus d'une chance sur deux qu'un paquet ne comporte aucun stylo défectueux. A-t-il raison ? + \item Combien de stylos peut-on espérer avoir en moyenne? + \end{enumerate} + \pagebreak +\end{exercise} + +\begin{solution} + \begin{enumerate} + \item + \begin{center} + \begin{tikzpicture}[sloped] + \node {.} + child {node {$A$} + child {node {$D$} + edge from parent + node[above] {\Var{pD_A}} + } + child {node {$\overline{D}$} + edge from parent + node[above] {\Var{round(1-pD_A, 2)}} + } + edge from parent + node[above] {\Var{pA}} + } + child[missing] {} + child { node {$B$} + child {node {$D$} + edge from parent + node[above] {\Var{pD_B}} + } + child {node {$\overline{D}$} + edge from parent + node[above] {\Var{round(1-pD_B, 2)}} + } + edge from parent + node[above] {\Var{pB}} + } ; + \end{tikzpicture} + \end{center} + \item + \begin{itemize} + \item Probabilité que le stylo vienne de l'atelier A + \[ + P(A) = \Var{pA} + \] + \item Probabilité que le stylo vienne de l'atelier B + \[ + P(B) = \Var{pB} + \] + \item Probabilité que le stylo ait un défaut sachant qu'il vient de l'atelier A. + \[ + P_A(D) = \Var{pD_A} + \] + \item Probabilité que le stylo vienne de l'atelier B et qu'il ait un défaut. + \[ + P(D \cap D) = \Var{pDB} + \] + \end{itemize} + \item + \begin{enumerate} + \item Probabilité qu'un stylo vienne de l'atelier A et qu'il ait un defaut + \[ + P(A\cap D) = P(A) \times P_A(D) = \Var{pA} \times \Var{pD_A} = \Var{pDA} + \] + \item Probabilité que le stylo ai un défaut de fabrication. + \[ + P(D) = P(A\cap D) + P(B\cap D) = \Var{pDA} + \Var{pDB} = \Var{pD} + \] + \end{enumerate} + \item Probabilité qu'il vienne de l'atelier A sachant qu'il a un defaut + \[ + P_D(A) = \frac{P(A\cap D)}{P(D)} = \frac{\Var{pDA}}{\Var{pD}} = \Var{round(pDA/pD, 2)} + \] + \item $X$ peut être modélisée par une loi binomiale de paramètres $n=\Var{nbr}$ et $p=\Var{pD}$. + \item (\textit{par de correction automatique disponible pour le résultat final} + \[ + P(X = \Var{k}) = \coefBino{\Var{nbr}}{\Var{k}}\times \Var{pD}^{\Var{k}} \times \Var{round(1 - pD, 2)}^{\Var{nbr-k}} + \] + \item (\textit{par de correction automatique disponible pour le résultat final} + + Il faut calculer la probabilité qu'il y ait 0 stylo avec un defaut. + \[ + P(X = 0) = \coefBino{\Var{nbr}}{0}\times \Var{pD}^{0} \times \Var{round(1 - pD, 2)}^{\Var{nbr}} + \] + Puis comparer ce nombre à 0,5. + \item Il faut calculer l'espérance + \[ + E[X] = n\times p = \Var{nbr} \times \Var{pD} = \Var{round(nbr*pD, 2)} + \] + \end{enumerate} +\end{solution}